Εισαγωγη σε διαγωνιστικα μαθηματικα για το γυμνασιο

340
http://www.mathematica.gr/forum/viewtopic.php?f=109&t=30104 Επιμέλεια: xr.tsif Σελίδα 1

description

ΔΙΑΓΩΝΙΣΜΟΙ ΕΜΕ

Transcript of Εισαγωγη σε διαγωνιστικα μαθηματικα για το γυμνασιο

Page 1: Εισαγωγη σε διαγωνιστικα μαθηματικα για το γυμνασιο

http://www.mathematica.gr/forum/viewtopic.php?f=109&t=30104

Επιμέλεια: xr.tsif Σελίδα 1

Page 2: Εισαγωγη σε διαγωνιστικα μαθηματικα για το γυμνασιο

http://www.mathematica.gr/forum/viewtopic.php?f=109&t=30104

Επιμέλεια: xr.tsif Σελίδα 2

Σε αυτή την προσπάθεια πρωτοστάτησε ο ΔΗΜΗΤΡΗΣ ΙΩΑΝΝΟΥ και

έδωσαν το παρόν αρκετοί συνάδελφοι, εξαίρετοι ΜΑΘΗΜΑΤΙΚΟΙ όπως ο

Θανάσης Κοντογιώργης και ο Θάνος Μάγκος και πολλοί ταλαντούχοι μαθητές

προτείνοντας και λύνοντας θέματα.

Όλοι έχουν το ίδιο μικρόβιο, την αγάπη για τα μαθηματικά .

Και φυσικά όταν έχεις το μικρόβιο, είσαι καταδικασμένος να δημιουργήσεις

κάποια στιγμή όμορφα πράγματα.

Ξεκίνησα την αποδελτίωση με την συμπαράσταση του Μιχάλη Νάννου ο

οποίος φιλοτέχνησε το εξώφυλλο. Επειδή όλο και κάτι ξεφεύγει παρακαλώ

στείλτε τις παρατηρήσεις σας στο [email protected]

Αφιερωμένο σε κάθε μαθητή Γυμνασίου που ασχολείται ή πρόκειται να

ασχοληθεί με Μαθηματικούς διαγωνισμούς

Τσιφάκης Χρήστος

Page 3: Εισαγωγη σε διαγωνιστικα μαθηματικα για το γυμνασιο

http://www.mathematica.gr/forum/viewtopic.php?f=109&t=30104

Επιμέλεια: xr.tsif Σελίδα 3

ΕΙΣΑΓΩΓΗ ΣΕ

ΔΙΑΓΩΝΙΣΤΙΚΑ ΜΑΘΗΜΑΤΙΚΑ

ΓΙΑ ΤΟ ΓΥΜΝΑΣΙΟ

Ασκήσεις 1 – 400

Page 4: Εισαγωγη σε διαγωνιστικα μαθηματικα για το γυμνασιο

http://www.mathematica.gr/forum/viewtopic.php?f=109&t=30104

Επιμέλεια: xr.tsif Σελίδα 4

ΘΕΜΑ 1 (ΔΗΜΗΤΡΗΣ ΙΩΑΝΝΟΥ)

Αν x 1 y 3 5 z

2 5 7

και αν 2x 3y 5z 10 , να βρεθούν οι αριθμοί x,y,z .

Λύση:

Αρχικά αφού όλα τα κλάσματα είναι ίσα μεταξύ τους, είναι ίσα με έναν αριθμό

k . Άρα x 1 y 3 5 z

k2 5 7

.

Αντιστοιχίζουμε τους x,y,z με το k .

x 1k 2k x 1 x 2k 1

2

,

y 3k 5k y 3 y 5k 3

5

,

5 zk 7k 5 z z 7k 5

7

.

Τώρα αντικαθιστούμε τους x,y,z με το k .

2x 3y 5z 10 2(2k 1) 3(5k 3) 5( 7k 5) 10

4k 2 15k 9 35k 25 10 4k 15k 35k 10 2 9 25 26 26

46k 26 k k46 46

.

Οπότε τώρα μπορούμε να βρούμε τις τιμές τους.

x 1 2646(x 1) 26 2 46x 46 52 46x 52 46

2 46

98 49x

46 23 .

Page 5: Εισαγωγη σε διαγωνιστικα μαθηματικα για το γυμνασιο

http://www.mathematica.gr/forum/viewtopic.php?f=109&t=30104

Επιμέλεια: xr.tsif Σελίδα 5

y 3 2646(y 3) 5 26 46y 138 130 46y 130 138

5 46

8 446y 8 y

46 23

.

5 z 2646(5 z) 7 26 230 46z 182

7 46

48 48 2446z 182 230 46z 48 z z

46 46 23

.

Άρα 49

x23

, 4

y23

,

24z

23 .

Β τρόπος

x 1 y 3 5 z 2x 2 3y 9 5z 25 2x 2 3y 9 5z 25

2 5 7 4 15 35 4 15 35

2x 3y 5z 36 26 13

46 46 23

.

Έτσι έχουμε

2x 2 13 49x

4 23 23

.

3y 9 13 4y

15 23 23

.

5z 25 13 24z

35 23 23

.

ΘΕΜΑ 2 (Atemlos)

Να συγκριθούν οι αριθμοί 4 3a 3 ·44 και 3 4b 4 ·33 .

Page 6: Εισαγωγη σε διαγωνιστικα μαθηματικα για το γυμνασιο

http://www.mathematica.gr/forum/viewtopic.php?f=109&t=30104

Επιμέλεια: xr.tsif Σελίδα 6

Λύση:

Έστω a b . Τότε : 4 3 3 4 3 4 3 444 333 ·44 4 ·33 ( ) ( ) 11 11

4 3 , που ισχύει, άρα

ισχύει και η αρχική υπόθεση.

ΘΕΜΑ 3 (ΔΗΜΗΤΡΗΣ ΙΩΑΝΝΟΥ)

Αν οι αριθμοί x,yείναι ομόσημοι και ισχύουν οι σχέσεις: x y z

y z w ,

3xyz 64w και x y z 42 , να βρεθούν οι αριθμοί x,y,z,w .

Λύση:

Καταρχήν, αφού όλοι οι λόγοι είναι ίσοι μεταξύ τους, θα είναι ίσοι και με έναν

αριθμό n , δηλαδή x y z

ny z w .

Άρα x

n x nyy ,

και y

n y nzz ,

και z

n z nww .

Συνεπώς: z nw ,

2y nz y n n w y n w και

2 3x ny y n n w n w .

Οπότε 3 3 2 3 6 3 3 6xyz 64w n w n w nw 64w n w 64w n 64

6n 64 n 2 .

Page 7: Εισαγωγη σε διαγωνιστικα μαθηματικα για το γυμνασιο

http://www.mathematica.gr/forum/viewtopic.php?f=109&t=30104

Επιμέλεια: xr.tsif Σελίδα 7

Ο λόγος για τον οποίο ο nείναι θετικός είναι επειδή x

ny

και x,yομόσημοι

αριθμοί.

Τώρα βρίσκουμε τους x,y,z συναρτήσει του w :

Έχουμε x y z 42 και γνωρίζουμε ότι:

3 3x n w 2 w 8w , 2 2y n w 2 w 4w , z nw 2w .

Οπότε αντικαθιστούμε τους x,y,z με τους w τύπους:

42x y z 42 8w 4w 2w 42 14w 42 w w 3

14 .

Tώρα είναι εύκολο να βρούμε τους x,yκαι z .

x 8w 8 3 24 , y 4w 4 3 12 , z 2w 2 3 6 .

Αν θέλουμε να επαληθεύσουμε το αποτέλεσμα μπορούμε να κάνουμε και πάλι

αντικατάσταση.

ΘΕΜΑ 4 (ΔΗΜΗΤΡΗΣ ΙΩΑΝΝΟΥ)

Να βρεθούν τα ψηφία x,yαν γνωρίζουμε ότι ο αριθμός 4x25y διαιρείται με το

2 , με το 3και με το 5και ότι είναι μικρότερος από τον αριθμό 344 10 .

Λύση:

Αρχίζουμε λέγοντας ότι το 34x25y 44 10 4x25y 44000 .

O αριθμός 4x25y είναι άρτιος αφού διαιρείται με το 2και το 5, που σημαίνει

ότι το yμπορεί να πάρει ΜΟΝΟ την τιμή 0. (έτσι ώστε να διαιρείται και με το

5 και με το 2 , και για να ισχύσει αυτό, ο αριθμός πρέπει να λήγει σε 0.

Δεδομένου του ότι ο αριθμός είναι μικρότερος του 44000 , και αφού το

άθροισμα των ψηφίων του αριθμού είναι πολλαπλάσιο του 3, τότε:

4 x 2 5 0 3 , δηλαδή, 11 x 3k x 3k 11 .

Page 8: Εισαγωγη σε διαγωνιστικα μαθηματικα για το γυμνασιο

http://www.mathematica.gr/forum/viewtopic.php?f=109&t=30104

Επιμέλεια: xr.tsif Σελίδα 8

Όμως 0 x 4 , άρα 0 3k 11 4 , όπου ο k είναι ακέραιος.

Οπότε : 11

0 3k 11 3k 11 k3

ΚΑΙ

3k 11 4 3k 4 11 3k 15 k 5 .

Οι κοινές ακέραιες λύσεις είναι το 4και το 5.

Οπότε το xμπορεί να πάρει τις τιμές x 3k 11 x 3 4 11 x 1 ή

x 3k 11 x 3 5 11 x 4 . Και στις δύο περιπτώσεις έχουμε άθροισμα

πολλαπλάσιο του 3, όμως αν πάρουμε το x ως 4 , τότε ξεπερνάμε τον αριθμό

44000 . Οπότε το xπαίρνει την τιμή 1.

Άρα y 0 , x 1 .

Και ο αριθμός είναι 41250 .

ΣΗΜΕΙΩΣΗ

Γράφω το τι μας χρειάζεται από την Α Γυμνασίου:

α) Ένας αριθμός διαιρείται με το 2 , αν είναι άρτιος (δηλ. λήγει σε 0,2,4,6,8.

β) Ένας αριθμός διαιρείται με το 5, αν και μόνο αν λήγει σε 0 ή σε 5.

γ) Ένας αριθμός διαιρείται με το 3, αν και μόνο αν το άθροισμα των ψηφίων

του, είναι πολλαπλάσιο του 3.

ΘΕΜΑ 5 (ΔΗΜΗΤΡΗΣ ΙΩΑΝΝΟΥ)

Έστω n 1 nf (n) 2 3 5 , όπου n Z , (δηλαδή ο n , είναι ακέραιος).

Να αποδείξετε ότι δεν υπάρχει ακέραιος m, έτσι ώστε να είναι:

f (m) 5f(m 2) 4f(m 1) 2f(m 3) .

Page 9: Εισαγωγη σε διαγωνιστικα μαθηματικα για το γυμνασιο

http://www.mathematica.gr/forum/viewtopic.php?f=109&t=30104

Επιμέλεια: xr.tsif Σελίδα 9

ΣΗΜΕΙΩΣΗ: (Για τους μικρούς μαθητές).

Έστω π.χ. ότι x 2f (x) 5 3x .

Τότε , π.χ. το f (3y 4) , θα το βρούμε αν βάλλουμε όπου x , το 3y 4 .

Δηλαδή, 3y 4 2f (3y 4) 5 3(3y 4) .

Επίσης, θυμηθείτε τις ιδιότητες των δυνάμεων και κυρίως:

m n m na a a και m

m n

n

aa

a

με a 0 .

Λύση:

m 1 mf (m) 2 3 5 ,

m 2 1 m 2f (m 2) 2 3 5 ,

m 1 1 m 1f (m 1) 2 3 5 ,

m 3 1 m 3f (m 3) 2 3 5 .

Άμα αντικαταστήσουμε στην εξίσωση, τότε:

m 1 m m 3 m 2 m 2 m 1 m 4 m 32 3 5 5[2 3 5] 4[2 3 5] 2[2 3 5] ,

επομένως:

m m m m m m m

m

3 2 2 4 3

2 2 3 2 3 2 33 5 5 5 25 4 4 20 2 2 10

2 2 3 2 3 2 3 ,

επομένως:

m m m m m m m

m2 2 3 2 3 2 33 5 5 5 25 4 4 20 2 2 10

2 8 9 4 3 16 27 ,

επομένως: m m m m m m m

m2 2 2 2 3 3 35 4 2 3 5 4 2

2 8 4 16 9 3 27 ,

Page 10: Εισαγωγη σε διαγωνιστικα μαθηματικα για το γυμνασιο

http://www.mathematica.gr/forum/viewtopic.php?f=109&t=30104

Επιμέλεια: xr.tsif Σελίδα 10

επομένως: m m1 5 1 5 4 22 ( 1 ) 3 (1 )

2 8 8 9 3 27 ,

επομένως: m m 262 0 3 ( )

27 .

Επειδή η εξίσωση αυτή είναι αδύνατη, σημαίνει ότι δεν υπάρχει ακέραιος m, ο

οποίος θα επαληθεύει την εξίσωση της εκφώνησης.

ΘΕΜΑ 6 (gauss1988)

Σε ένα μαθηματικό διαγωνισμό, υπήρχαν δύο ισοδύναμα βαθμολογικά θέματα.

Στο πρώτο θέμα, απάντησαν σωστά το 40% των μαθητών , στο δεύτερο θέμα

απάντησαν σωστά το 25%, ενώ το 50% των μαθητών δεν απάντησε σωστά σε

κανένα από τα δύο θέματα.

Κάθε σωστή απάντηση παίρνει άριστα και κάθε λανθασμένη, παίρνει μηδέν.

Κάποιος ισχυρίζεται ότι όσοι μαθητές πήραν άριστα είναι τόσοι, όσοι

απάντησαν μόνο στο πρώτο θέμα. Είναι σωστός ο ισχυρισμός;

Λύση:

Αφού το 50% δεν απάντησε σωστά σε κανένα θέμα, άρα όσοι απάντησαν

σωστά σε ένα τουλάχιστον θέμα αποτελούν το υπόλοιπο 50% του συνόλου των

μαθητών. Αν x είναι το

ποσοστό των μαθητών που

απάντησε σωστά μόνο στην

A ερώτηση, w το ποσοστό

όσων απάντησε σωστά μόνο

στην Bερώτηση και yόσων

απάντησαν σωστά και στις

δύο ερωτήσεις, τότε με βάση το πρόβλημα έχουμε: x y 40 , y w 25 ,

x y w 50 .

Προσθέτουμε τις δύο πρώτες εξισώσεις κατά μέλη και ύστερα αφαιρούμε την

Page 11: Εισαγωγη σε διαγωνιστικα μαθηματικα για το γυμνασιο

http://www.mathematica.gr/forum/viewtopic.php?f=109&t=30104

Επιμέλεια: xr.tsif Σελίδα 11

τρίτη, οπότε βρίσκουμε y 15 . Άρα και στα δύο ερωτήματα απάντησε το 15%

των μαθητών, ενώ στο πρώτο θέμα απάντησε το 40% 15% 25% .

Άρα ο ισχυρισμός είναι λάθος.

ΘΕΜΑ 7 (ΔΗΜΗΤΡΗΣ ΙΩΑΝΝΟΥ)

Να βρείτε το ψηφίο των μονάδων των αριθμών:

α) 2002132 .

b) 100 10007 656 .

Λύση:

α) Διαιρούμε το 2002με το 4και βρίσκουμε υπόλοιπο 2 . Άρα, το τελευταίο

ψηφίο θα είναι 2002 2(132 ) (2 ) 4 .

b) Η διαίρεση του 100με το 4 είναι τέλεια και άρα το τελευταίο ψηφίο του 1007 είναι

100 4(7 ) (7 ) 1 .

Το ίδιο συμβαίνει και με το 1000 και άρα: 1000 4(656 ) (6 ) 6 .

Άρα, το τελευταίο ψηφίο του αθροίσματος των δύο αριθμών θα είναι το

άθροισμα των τελευταίων ψηφίων των αριθμών, δηλ. 1 6 7 .

ΘΕΜΑ 8 (ΜΠΑΜΠΗΣ ΣΤΕΡΓΙΟΥ)

Αν 777 75

x  777 78

,

888 85y

888 89

και οι όροι των κλασμάτων έχουν το ίδιο

πλήθος ψηφίων, να αποδείξετε ότι x y .

Λύση:

Ας είναι a 77 75 και b 88 85 .

Τότε, a

xa 3

και b

yb 4

.

Page 12: Εισαγωγη σε διαγωνιστικα μαθηματικα για το γυμνασιο

http://www.mathematica.gr/forum/viewtopic.php?f=109&t=30104

Επιμέλεια: xr.tsif Σελίδα 12

Επειδή x,y 0 , αρκεί να αποδειχθεί ότι 1 1

x y δηλαδή ότι 3b 4a .

Όμως, είναι n n

3b 3·8·11 10 15 24·11 10 15

και n n

4a 4·7·11 10 20 28·11 10 20 .

ΘΕΜΑ 9 (ΓΙΩΡΓΟΣ ΡΙΖΟΣ)

Στο σχήμα όπου η x είναι παράλληλη προς τη

y να υπολογιστεί το άθροισμα των γωνιών

, , ,

.

Θαλής 1998 – 1999 (Β Γυμνασίου)

Λύση:

Κοιτώντας το σχήμα του κ. Ρίζου, παίρνω πάνω στην ευθεία x , ένα τυχαίο

σημείο , και φέρνω από αυτό την κάθετη πάνω στην ημιευθεία y . Τότε

έχουμε δημιουργήσει ένα εξάγωνο . Αν από μια κορυφή αυτού του

εξαγώνου φέρουμε τις διαγώνιες, τότε εμφανίζονται 4 τρίγωνα. Άρα το

άθροισμα των γωνιών του εξαγώνου, είναι o4 180 720 .

Συνεπώς: o o o o720 90 90 540

.

ΘΕΜΑ 10 (ΓΙΩΡΓΟΣ ΡΙΖΟΣ)

Στο παρακάτω σχήμα υπολογίσετε το x σε μοίρες.

Θαλής 2006 – 2007 (Γ Γυμνασίου)

Page 13: Εισαγωγη σε διαγωνιστικα μαθηματικα για το γυμνασιο

http://www.mathematica.gr/forum/viewtopic.php?f=109&t=30104

Επιμέλεια: xr.tsif Σελίδα 13

Λύση:

Από το πρώτο τρίγωνο παίρνουμε ότι: o3x 4x 180 .

Από το δεύτερο

τρίγωνο παίρνουμε ότι:o5x 180 E .

Και από το τρίτο

τρίγωνο, ισχύει ότι:o6x 2x 180 E .

Άρα: o5x C 180 E 6x 2x 5x C 8x 5x 8x 5x 3x .

Και για να βρούμε το E ισχύει ότι: o o5x 180 E 5x E 180 .

Άρα: 5x E 3x 4x E 3x 4x 5x E 2x .

Τέλος, αντικαθιστούμε τις σχέσεις και βρίσκουμε ότι:

o o3x 4x 180 3x 4x 180 3x 4x 3x 180

o o10x 180 x 18 .

Άρα η μόνη τιμή που μπορεί να πάρει το x είναι ο αριθμός o18 .

ΘΕΜΑ 11 (ΓΙΩΡΓΟΣ ΡΙΖΟΣ)

Στο διπλανό σχήμα δίνεται ότι

i. 1 2 3) )( / /( / )/( .

ii. 3

( )

iii. AE E

iv. ˆˆ 30 , 50

Να βρεθούν οι γωνίες του τετραπλεύρου AB .

Page 14: Εισαγωγη σε διαγωνιστικα μαθηματικα για το γυμνασιο

http://www.mathematica.gr/forum/viewtopic.php?f=109&t=30104

Επιμέλεια: xr.tsif Σελίδα 14

Θαλής 1999 – 2000 (Β Γυμνασίου)

Λύση:

(Αναφέρομαι στο σχήμα του κ. Ρίζου)

Επειδή η είναι κάθετη στην 3

( ) , θα είναι

κάθετη και στην 1

( ) . Επομένως το τρίγωνο

A E , είναι ορθογώνιο και ισοσκελές.

Επομένως η γωνία A E

θα είναι ίση με o45 ,

και τόσο θα είναι και η γωνία E

.

H γωνία

, είναι ίση με o o o90 30 60 .

H γωνία

, είναι ίση με o o o50 30 80 .

Τέλος, η γωνία

είναι ίση με o o o 0AE EAB 45 (180 50 ) 175

.

Β τρόπος

Ας ονομάσουμε την ορθή γωνία δίπλα στη

, x . Τότε

o o o o o180 x 180 30 90 60

.

Ονομάζουμε την ορθή γωνία στα αριστερά της 2

( ) , y. Και τη γωνία B

χωρίς

το , z .

o o o o oz 180 y z 180 60 90 z 30

και άρα

o o oB z B 30 50 B 80

.

Η A

ανήκει στο τετράπλευρο και ισούται με:

A 360 90 90 A 360 90 90 50 A 130

.

Page 15: Εισαγωγη σε διαγωνιστικα μαθηματικα για το γυμνασιο

http://www.mathematica.gr/forum/viewtopic.php?f=109&t=30104

Επιμέλεια: xr.tsif Σελίδα 15

Στο τρίγωνο A E , άρα και αφού είναι ορθογώνιο και ισοσκελές η

45

.

Άρα A 130 A 130 45 A 175

.

ΘΕΜΑ 12 (ΓΙΩΡΓΟΣ ΡΙΖΟΣ)

Στο παρακάτω σχήμα δίνονται

α) 1 2) / /( ( ) .

β) το ισοσκελές τρίγωνο AB B με BA 20 .

γ) η B είναι διχοτόμος της γωνίας AB.

δ) Z A .

Να βρείτε τις γωνίες ˆE, AE , .

Να εξηγήσετε γιατί οι ευθείες BE, Z δεν είναι παράλληλες.

Θαλής 2003 – 2004 (Β Γυμνασίου)

Λύση:

o o1AB A 80 B 80 40

2

(διχοτόμος).

o o o o o o o o180 40 80 60 180 180 60 120 .

o40 , εντός εναλλάξ των παραλλήλων 1 2),( ( ) τεμνόμενες από την .

Page 16: Εισαγωγη σε διαγωνιστικα μαθηματικα για το γυμνασιο

http://www.mathematica.gr/forum/viewtopic.php?f=109&t=30104

Επιμέλεια: xr.tsif Σελίδα 16

o10 εντός – εκτός επί τα αυτά των παραλλήλων 1 2),( ( ) τεμνόμενες από

την , ίση με o o oZ H 180 70 10

.

Οι , δεν είναι παράλληλες αφού .

ΘΕΜΑ 13 (ΓΙΩΡΓΟΣ ΡΙΖΟΣ)

Στο διπλανό τραπέζιο AB ,

AB/ / δίνεται ότι

B A

και ότι τα

τρίγωνα AB , A είναι

ισοσκελή με AB A ,

A.

i) Να αποδείξετε ότι η A

διχοτομεί τη γωνία AB .

ii) Να υπολογίσετε τη γωνία . Θαλής 2006 – 2007 (Β Γυμνασίου)

Λύση:

i) Με βάση την υπόθεση, έχουμε ότι x y καιB

.

Επίσης A B x x

. Όμως A y , ως εντός εναλλάξ των

παραλλήλων AB, που τέμνονται από την A .

Άρα x y x x 2x . Άρα A x 2x x x και άρα η

A είναι διχοτόμος της A .

ii) Έχουμε από την υπόθεση ότι AB B 2x

. Τώρα από το τρίγωνο

έχουμε: o o oB x 180 2x 2x x 180 x 36

.

Άρα oB 72 .

Page 17: Εισαγωγη σε διαγωνιστικα μαθηματικα για το γυμνασιο

http://www.mathematica.gr/forum/viewtopic.php?f=109&t=30104

Επιμέλεια: xr.tsif Σελίδα 17

ΘΕΜΑ 14 (ΓΙΩΡΓΟΣ ΡΙΖΟΣ)

Στο παρακάτω σχήμα είναι

AB B και η διχοτόμος x της

γωνίας A

είναι παράλληλη

στην AB . Να υπολογίσετε τις

γωνίες του τριγώνου AB.

Θαλής 2005 – 2006

(Β Γυμνασίου)

Λύση:

Ονομάζω yτην γωνία A x

. Τότε A x

ως εντός εναλλάξ και A B x

διότι

το τρίγωνο AB είναι ισοσκελές. Τώρα η εξωτερική γωνία του τριγώνου AB

που είναι ίση με 2x , θα είναι ίση με το άθροισμα των απέναντι γωνιών. Δηλαδή

2x x

και άρα x

. Τώρα είδαμε ότι το τρίγωνο ABέχει όλες τις

γωνίες ίσες, δηλαδή είναι ισόπλευρο και όλες οι γωνίες του είναι από o60

μοίρες.

ΘΕΜΑ 15 (ΜΠΑΜΠΗΣ ΣΤΕΡΓΙΟΥ)

Στον πίνακα είναι γραμμένοι οι αριθμοί :

1,2,3,4,6,7,8,9,11,12,13,14,16,17,18,19,21.....

α) Να αποδείξετε ότι στην παραπάνω σειρά αριθμών (ακολουθία) δεν ανήκει ο

αριθμός 2010 .

β) Ποιος αριθμός βρίσκεται στη 2010 – ή θέση της ακολουθίας αυτής.

γ) Να βρείτε το άθροισμα των πρώτων 2010 αριθμών της παραπάνω

ακολουθίας.

Page 18: Εισαγωγη σε διαγωνιστικα μαθηματικα για το γυμνασιο

http://www.mathematica.gr/forum/viewtopic.php?f=109&t=30104

Επιμέλεια: xr.tsif Σελίδα 18

Λύση:

α) Παρατηρώ ότι από την παραπάνω σειρά των αριθμών, λείπουν όλα τα

πολλαπλάσια του 5. Αφού ο 2010είναι πολλαπλάσιο του 5, άρα δεν

συμπεριλαμβάνεται .

β) Αν ήσαν γραμμένοι όλοι οι αριθμοί , χωρίς να λείπει κάποιος, τότε θα είχαμε

2010 στο πλήθος αριθμούς. Αφού όμως λείπουν τα πολλαπλάσια του 5 , που

από το 5, μέχρι το 2010 έχουν πλήθος 2010:5 402 , αυτό σημαίνει ότι ο

αριθμός που κατέχει την 2010ή θέση στην σειρά των αριθμών, θα είναι ο

2010 402 2412 .

γ) Το άθροισμα που ζητάμε είναι ίσο με

(1 2 3 4 5 6 ... 2412) (5 10 15 ... 2010) .

Είναι όμως γνωστό, ότι 2412 20413

1 2 3 4 ... 2412 1206 24132

.

Και

5 10 15 ... 2010 1 5 2 5 3 5 ... 402 5 5(1 2 3 ... 402)

402 4035 201 2015

2

.

Άρα το άθροισμά μας είναι ίσο με: 1206 2413 201 2015 .

ΘΕΜΑ 16 (ΔΗΜΗΤΡΗΣ ΙΩΑΝΝΟΥ)

Να λυθεί η εξίσωση:

2 2 2 2

2 2 2 2

x 4x 3 x 4x 3 x 6x 8 x 6x 8

x 4x 3 x 4x 3 x 6x 8 x 6x 8

. Πολυτεχνείο 1954

Λύση:

Το σύνολο ορισμού της εξίσωσης είναι: R 1, 2, 3, 4,1,2,3,4 .

Page 19: Εισαγωγη σε διαγωνιστικα μαθηματικα για το γυμνασιο

http://www.mathematica.gr/forum/viewtopic.php?f=109&t=30104

Επιμέλεια: xr.tsif Σελίδα 19

2 2 2 2

2 2 2 2

x 4x 3 x 4x 3 x 6x 8 x 6x 8

x 4x 3 x 4x 3 x 6x 8 x 6x 8

(1) .

Έστω 2 2

2 2

x 4x 3 x 6x 8a , b

x 4x 3 x 6x 8

.

2 2

2 21 1 a 1 b 1(1) a b ba b ab a

a b a b

2 2ba b ab a 0 ab a b a b 0 a b ab 1 0

a b ή ab=1.

Από 2 2

2 2

x 4x 3 x 6x 8a b

x 4x 3 x 6x 8

(2) .

Θέτουμε 2k x 3 ,2m x 8 .

(2) k 4x m 6x k 4x m 6x

2 2km 6kx 4mx 24x km 6kx 4mx 24x

12kx 8mx x 0 ή 2 2 23k 2m 3x 9 2x 16 x 7 x 7 .

Από 2 2

2 2

x 4x 3 x 6x 8ab 1 1

x 4x 3 x 6x 8

k 4x m 6x k 4x m 6x

2 2km 6kx 4mx 24x km 6kx 4mx 24x 12kx 8mx 0

x 0 ή 2 2 23k= 2m 3x 9 2x 16 x 5 , Αδύνατη.

Άρα ρίζες τελικά είναι οι x 0 , x 7 .

ΘΕΜΑ 17 (ΔΗΜΗΤΡΗΣ ΙΩΑΝΝΟΥ)

Να λυθεί η εξίσωση: 2 2 26x (x 1)(x 4) x (x 1) . Πολυτεχνείο 1949

Page 20: Εισαγωγη σε διαγωνιστικα μαθηματικα για το γυμνασιο

http://www.mathematica.gr/forum/viewtopic.php?f=109&t=30104

Επιμέλεια: xr.tsif Σελίδα 20

Λύση:

Η εξίσωση γράφεται:

2 2 2 2 25(x x 1) (x x) 1 (x x 1)(x x 1)

2 2(x x 1)(x x 4) 0 .

Η άσκηση έχει πρακτικά τελειώσει.

ΘΕΜΑ 18 (ΔΗΜΗΤΡΗΣ ΙΩΑΝΝΟΥ)

Να βρεθεί το σύνολο λύσεων της εξίσωσης: 25x 2x 3 2sinx .

(Με sinx , συμβολίζουμε το ημίτονο της γωνίας x).

Λύση:

Γράφω 21 145(x ) 2sinx

5 5 .

Όμως είναι 1 sinx 1 2 2sinx 2 .

Συνεπώς 14

LHS 25

και RHS 2 .

Συνεπώς η ισότητα δεν πιάνεται πουθενά και η εξίσωση είναι αδύνατη.

ΘΕΜΑ 19 (KARKAR)

Να λυθεί η εξίσωση: 2 x 12x 4x 0

x

.

Λύση:

Πρέπει x 0 .

Η εξίσωση γράφεται: 2 2 x

x x x 4xx

.

Page 21: Εισαγωγη σε διαγωνιστικα μαθηματικα για το γυμνασιο

http://www.mathematica.gr/forum/viewtopic.php?f=109&t=30104

Επιμέλεια: xr.tsif Σελίδα 21

Από ΑΜ – ΓΜ στο αριστερό μέλος προκύπτει ότι L 4x ,άρα πρέπει όλοι οι

όροι του αριστερού μέλους να είναι ίσοι.

Αυτό γίνεται για x 1 . Το μηδέν απορρίπτεται, λόγω των περιορισμών.

Β τρόπος

Η εξίσωση γράφεται: 21x 2x 4x

x . Επειδή x 0 είναι

1x 2

x

(θυμηθείτε 1

2 ), ενώ το τριώνυμο του β' μέλους έχει μέγιστη τιμή το 2

για x 1 (θυμηθείτε την κορυφή της παραβολής που είναι η γραφική

παράσταση του τριωνύμου).

Έτσι το x 1 είναι η μοναδική λύση της εξίσωσης .

ΘΕΜΑ 20 (ΓΙΩΡΓΟΣ ΚΑΛΑΘΑΚΗΣ)

Να βρεθούν όλες οι τριάδες ακεραίων αριθμών x,y,z που ικανοποιούν τη

σχέση xyz 2(xy xz yz) 4(x y z) 7 .

Λύση:

Παρατήρηση:

Το μοναδικό που χρειάζεται να γνωρίζεις για να λύσεις αυτήν την άσκηση είναι

μόνο το τί είναι ένας ακέραιος αριθμός (που νομίζω ότι το γνωρίζεις).

Αρκεί να παρατηρήσουμε ότι :

(x 2)(y 2)(z 2) LHS 8 (x 2)(y 2)(z 2) 1 και αφού x,y,z Z

τα πράγματα είναι απλά.

ΠΑΡΑΤΗΡΗΣΕΙΣ

(x 2)(y 2)(z 2) LHS 8 (x 2)(y 2)(z 2) 1 : αυτό πως το

παρατήρησες;

LHS: left hand side δηλ. εννοεί το αριστερό μέρος της ισότητας.

Page 22: Εισαγωγη σε διαγωνιστικα μαθηματικα για το γυμνασιο

http://www.mathematica.gr/forum/viewtopic.php?f=109&t=30104

Επιμέλεια: xr.tsif Σελίδα 22

RHS:right hand side ,το δεξιό μέρος της ισότητας.

Υπάρχουν δύο τρόποι (που γνωρίζω). Ο ένας είναι να προσπαθήσεις να κάνεις

παραγοντοποίηση με διάφορες προσθαφαιρέσεις.

Εγώ πήγα αλλιώς. Σκέφτηκα ότι στο ανάπτυγμα της παράστασης

(x k)(y k)(z k) εμφανίζονται το άθροισμα, τα διπλάσια γινόμενα των 3

μεταβλητών και το γινόμενο τους και έτσι το πρόβλημα ανάχθηκε στην

αναζήτηση του k που στην προκειμένη είναι πολύ απλό.

" Σκέφτηκα ότι στο ανάπτυγμα της παράστασης (x k)(y k)(z k)

εμφανίζονται το άθροισμα, τα διπλάσια γινόμενα των 3 μεταβλητών και το

γινόμενο τους " : αυτό από που το ξέρει κάποιος;

Δεν είναι να το ξέρεις απέξω.

Είναι να το φανταστείς ή να το έχεις δει κάπου μιας και παραστάσεις αυτής της

μορφής συναντούνται πολύ συχνά στους διαγωνισμούς γυμνασίου.

Κάνω πράξεις:

xyz 2xy 2xz 2yz 4x 4y 4z 7

xy(z 2) 2x(z 2) 2y(z 2) 4z 7

(z 2)(xy 2x 2y) 4z 8 8 7

(z 2)(xy 2x 2y) 4(z 2) 1 (z 2)(xy 2x 2y 4) 1

(z 2)[x(y 2) 2(y 2)] 1 (z 2)(y 2)(x 2) 1 .

Άρα:

z 2 1,y 2 1,x 2 1 ή z 2 1,y 2 1,x 2 1 ή ........ κλπ.

Δηλαδή (z 3,y 3,x 1) ή (z 3,y 1,x 3) ή (z 1,y 3,x 3) ή

(z 1,y 1,x 1) .

Page 23: Εισαγωγη σε διαγωνιστικα μαθηματικα για το γυμνασιο

http://www.mathematica.gr/forum/viewtopic.php?f=109&t=30104

Επιμέλεια: xr.tsif Σελίδα 23

1η τοποθέτηση (δύο σκέψεις)

{Υπάρχουν 2 τρόποι . Ο ένας είναι να προσπαθήσεις να κάνεις

παραγοντοποίηση με διάφορες προσθαφαιρέσεις.

Εγώ πήγα αλλιώς. Σκέφτηκα ότι στο ανάπτυγμα της παράστασης εμφανίζονται

το άθροισμα, τα διπλάσια γινόμενα των 3 μεταβλητών και το γινόμενο τους και

έτσι το πρόβλημα ανάχθηκε στην αναζήτηση του που στην προκειμένη είναι

πολύ απλό."}

2η τοποθέτηση

{Δεν είναι να το ξέρεις απέξω..είναι να το φανταστείς ή να το έχεις δει κάπου

μιας και παραστάσεις αυτής της μορφής συναντούνται πολύ συχνά στους

διαγωνισμούς γυμνασίου.}

3η τοποθέτηση

Η αναλυτική επίλυση του gauss1988 .

Όλα τα παραπάνω είναι αυτά που θα ήθελε να ακούσει και να δει γραμμένα,

ένα παιδί που τώρα ξεκινάει και ασχολείται με τους διαγωνισμούς, όπως

άλλωστε ρητά δηλώνεται και στον τίτλο του συγκεκριμένου φακέλου ,

"Εισαγωγή σε Διαγωνιστικά Μαθηματικά".

Σίγουρα λοιπόν δεν του έφτανε να ξέρει ,"τι είναι ακέραιος για να λύσει την

άσκηση".

ΘΕΜΑ 21 (ΓΙΩΡΓΟΣ ΚΑΛΑΘΑΚΗΣ)

Μια διαδικασία ολοκληρώνεται από τον σε 3 ώρες , από τον σε 10 ώρες

και από τον σε 15 ώρες . Αν εργαστούν και οι τρεις μαζί σε πόσες ώρες θα

την ολοκληρώσουν; (Β Γυμνασίου)

Λύση:

Έστω ότι η διαδικασία θα ολοκληρωθεί σε x ώρες. Τότε σε 1 ώρα , ο , θα

Page 24: Εισαγωγη σε διαγωνιστικα μαθηματικα για το γυμνασιο

http://www.mathematica.gr/forum/viewtopic.php?f=109&t=30104

Επιμέλεια: xr.tsif Σελίδα 24

έχει εκτελέσει τo 1

3 της διαδικασίας, ο τo

1

10 και ο , τo

1

15 της

διαδικασίας. Επομένως, μέσα σε μια ώρα, και οι τρεις μαζί, θα έχουνε εκτελέσει

τα 1 1 1

3 10 15 της διαδικασίας, δηλαδή τα

1

2 της διαδικασίας.

Επομένως, μέσα σε μια ώρα, θα έχει εκτελεστεί η μισή διαδικασία και άρα

ολόκληρη θα εκτελεστεί μέσα σε 2ώρες.

ΘΕΜΑ 22 (freyia)

Μια βρύση, γεμίζει μια δεξαμενή σε 6 ώρες. Μια άλλη σε 9 ώρες. Και οι δύο

μαζί σε πόσες ώρες θα την γεμίσουν;

Λύση:

Έστω ότι η δεξαμενή θα γεμίσει σε x ώρες. Τότε η πρώτη βρύση θα έχει

γεμίσει τα x

6 της δεξαμενής. και η δεύτερη βρύση τα

x

9 αυτής. Άρα για να βρω

το x , θα λύσω την εξίσωση: x x

16 9 , από όπου προκύπτει

18x

5 ώρες.

ΘΕΜΑ 23 (freyia)

Μια βρύση, γεμίζει μια δεξαμενή σε 4ώρες. Μια άλλη, που είναι στο κάτω

μέρος της δεξαμενής, μπορεί να την αδειάσει σε 6 ώρες. Άμα ανοιχτούνε και οι

δύο μαζί, σε πόση ώρα θα γεμίσει η δεξαμενή;

Λύση:

Όμοια αν εργαστούμε όπως και πριν, έχουμε: x x

14 6 , από όπου προκύπτει

x 12 ώρες.

Page 25: Εισαγωγη σε διαγωνιστικα μαθηματικα για το γυμνασιο

http://www.mathematica.gr/forum/viewtopic.php?f=109&t=30104

Επιμέλεια: xr.tsif Σελίδα 25

ΘΕΜΑ 24 (freyia)

Σε κυλιόμενες σκάλες ενός πολυκαταστήματος, ένας πελάτης, άμα σταθεί

ακίνητος, ανεβαίνει σε 2λεπτά.

Άμα όμως έχει κοπεί το ρεύμα, παρατήρησε ότι ανεβαίνει σε 3 λεπτά. Πόσο

χρόνο θα χρειαστεί να ανέβει, όταν και ο ίδιος ανεβαίνει αλλά και οι σκάλες

λειτουργούνε;

Λύση:

Όμοια όπως και με τις παραπάνω, αρκεί να λύσουμε την εξίσωση:

x x1

2 3 , από όπου

6x

5 λεπτά.

ΘΕΜΑ 25 (ΔΗΜΗΤΡΗΣ ΙΩΑΝΝΟΥ)

Αν οι αριθμοί a,b,x,yείναι διάφοροι του μηδενός και αν ay bx , να

υπολογίσετε την αριθμητική τιμή της παράστασης: 2 2

2 2 2 2

x bP

x y a b

.

Μια μικρή υπόδειξη για τους μικρούς μαθητές, αν δεν καταφέρουν να την

λύσουν:

Από την σχέση a x

ay bxb y

. Σε καθένα τώρα από τα κλάσματα της

παράστασης P , προσπαθήστε να εμφανίσετε τους λόγους a x

,b y

.

Λύση:

Έχουμε a x

ay bxb y

.

Τώρα, έχουμε:

Page 26: Εισαγωγη σε διαγωνιστικα μαθηματικα για το γυμνασιο

http://www.mathematica.gr/forum/viewtopic.php?f=109&t=30104

Επιμέλεια: xr.tsif Σελίδα 26

2 22 2

2 2

2 22 2 2 2

2 2

x x xb ( ) ( )1 1y y ybP 1

x a x xx a( ) 1 ( ) 1 ( ) 1 ( ) 11 1y b y yy b

.

ΘΕΜΑ 26 (ΔΗΜΗΤΡΗΣ ΙΩΑΝΝΟΥ)

Σε ένα ισοσκελές τρίγωνο ABC, με

AB AC , θεωρούμε σημείο D στην βάση

BC και σημείο E στην πλευρά AC, έτσι

ώστε η γωνία BAD

να ισούται με το

διπλάσιο της γωνίας CDE

. Να αποδείξετε

ότι AD AE .

Λύση:

Θέλω να αποδείξω ότι AD AE . Επομένως, πρέπει να αποδείξω ότι οι γωνίες

y και w είναι ίσες.

Όμως η γωνία w , είναι εξωτερική στο τρίγωνο DECκαι επομένως, θα ισούται

με το άθροισμα των απέναντι εσωτερικών γωνιών του. Δηλαδή, w x z (1) .

Επίσης, η γωνία ADC

, είναι εξωτερική στο τρίγωνο ABD , και επομένως,

y z x 2z , δηλ. y x z (2) .

Από (1) και (2) , βρίσκω w y , δηλαδή AD AE .

ΘΕΜΑ 27 (ΔΗΜΗΤΡΗΣ ΙΩΑΝΝΟΥ)

Να λυθεί η εξίσωση: 2n 2m n(x 1)(y 1) 2x , με άγνωστο τον πραγματικό

αριθμό x , όταν *m,n N .

Page 27: Εισαγωγη σε διαγωνιστικα μαθηματικα για το γυμνασιο

http://www.mathematica.gr/forum/viewtopic.php?f=109&t=30104

Επιμέλεια: xr.tsif Σελίδα 27

Λύση:

Λόγω του ότι οι αριθμοί 2m 2mx 1, y 1 είναι θετικοί και ισχύει 2m nx 1 2x

έχουμε 2n 2m n 2m(x 1)(y 1) 2x (y 1) δηλαδή

2n n 2m2 2x (y÷ 1) οπότε

2my 0 που ισχύει μόνο αν y 0 .

Για y 0 είναι 2n nx 1 2x δηλαδή n 2(x 1) 0 απ' όπου αν n άρτιος έχουμε

τις λύσεις x 1 ενώ αν n περιττός έχουμε τη λύση x 1 .

Άρα τελικά αν n άρτιος οι λύσεις είναι (x,y) (1,0) και (x,y) ( 1,0) ενώ αν

n περιττός τότε η λύση είναι η (x,y) (1,0) .

ΘΕΜΑ 28 (ΜΠΑΜΠΗΣ ΣΤΕΡΓΙΟΥ)

Αν οι φυσικοί αριθμοί ab,bc,ca είναι τέλειοι κύβοι, να αποδειχθεί ότι και οι

φυσικοί αριθμοί a,b,c είναι τέλειοι κύβοι.

Λύση:

Ας είναι p ένας πρώτος διαιρέτης του γινομένου abc . Ας είναι ακόμα 1ap η

μεγαλύτερη δύναμη του p που διαιρεί το a και αντιστοίχως 1bp η μεγαλύτερη

δύναμη του p που διαιρεί το b και 1cp η μεγαλύτερη δύναμη του p που διαιρεί

το c .

Τότε 1 1a bp είναι η μεγαλύτερη δύναμη του p που διαιρεί το ab . Εφόσον ο ab

είναι τέλειος κύβος, έπεται ότι το άθροισμα 1 1

a b είναι πολλαπλάσιο του 3.

Ομοίως τα αθροίσματα 1 1

b c και 1 1

a c είναι πολλαπλάσια του 3. Από εδώ

εύκολα προκύπτει ότι οι 1 1 1

a ,b ,c είναι όλοι τους πολλαπλάσια του 3.

Επομένως οι a,b,c είναι τέλειοι κύβοι.

ΘΕΜΑ 29 (ΜΠΑΜΠΗΣ ΣΤΕΡΓΙΟΥ)

Έστω a,b ακέραιοι αριθμοί, έτσι ώστε οι αριθμοί ab,(a 1)(b 1) να είναι

Page 28: Εισαγωγη σε διαγωνιστικα μαθηματικα για το γυμνασιο

http://www.mathematica.gr/forum/viewtopic.php?f=109&t=30104

Επιμέλεια: xr.tsif Σελίδα 28

διαδοχικοί ακέραιοι. Να αποδείξετε ότι ο αριθμός ab 1 είναι τέλειο

τετράγωνο.

Λύση:

Θα είναι (a 1) b 1 ab 1 b a 2 και έχουμε ότι :

22ab 1 a a 2 1 a 2a 1 a a 1 a 1 a 1 .

ΘΕΜΑ 30 (ΓΙΩΡΓΟΣ ΚΑΛΑΘΑΚΗΣ)

Στο παρακάτω σχήμα το

τετράγωνο έχει εμβαδόν 1

και το τρίγωνο έχει

εμβαδόν 2

3. Να βρείτε το

μήκος της .

Λύση:

Αφού AB Z 1 AZ AB 1 .

Έστω H x και y το αντίστοιχο ύψος προς την , στο τρίγωνο .

Τότε 1 2 4

xy 3xy 4 x2 3 3y

. Αν EK είναι το αντίστοιχο ύψος προς την

AZ , στο τρίγωνο AEZ , τότε: EK 1 y .

Από την ομοιότητα των τριγώνων AEZ και έχουμε:

2H y x y 4 y3y 4y 4 0

AZ EK 1 1 y 3y 1 y

. Η εξίσωση αυτή δίνει

μία θετική και μία αρνητική ρίζα. Δεκτή είναι η y 2 από την οποία προκύπτει

2H x

3 .

Page 29: Εισαγωγη σε διαγωνιστικα μαθηματικα για το γυμνασιο

http://www.mathematica.gr/forum/viewtopic.php?f=109&t=30104

Επιμέλεια: xr.tsif Σελίδα 29

Β τρόπος

Μια λύση με εμβαδά.

Αφού AB Z 1 AZ AB 1 (1) .

Έστω H x και y το αντίστοιχο ύψος προς την , στο τρίγωνο .

Τότε

1 2 4xy 3xy 4 x

2 3 3y (2) . Αν EK είναι το αντίστοιχο ύψος προς την

AZ , στο τρίγωνο AEZ , τότε: EK 1 y (3) .

Έχουμε (1),(2)

(3)

2 1 AZ H 2AEZ A HZ ·AZ·EK ·AB

3 2 2 3

41

1 2 43y1 1 y 1 3 1 y 3 1 4

2 2 3 3y

243 3y 3 4 3y 4y 4 0

y . Η εξίσωση αυτή δίνει μία θετική και

μία αρνητική ρίζα. Δεκτή είναι η y 2 από την οποία παίρνουμε 2

H x3

.

Γ τρόπος

Μεταφέρω το E "κατακόρυφα" ώστε να γίνει

Page 30: Εισαγωγη σε διαγωνιστικα μαθηματικα για το γυμνασιο

http://www.mathematica.gr/forum/viewtopic.php?f=109&t=30104

Επιμέλεια: xr.tsif Σελίδα 30

συνευθειακό των D,C . Το ύψος CEτου τριγώνου TCEδεν άλλαξε , άρα ούτε η

βάση x . Από την ομοιότητα των TCE , TBAπαίρνω : x

CE1 x

, x 1 .

Συνεπώς : x x 2 2

... x2 1 x 3 3

.

ΘΕΜΑ 31 (ΣΤΡΑΤΗΣ ΑΝΤΩΝΕΑΣ)

Να γράψετε τον αντίστροφο ενός πρώτου αριθμού p , σαν άθροισμα δύο

κλασμάτων, που το καθένα από αυτά να έχει αριθμητή το 1και παρονομαστή

(θετικό) ακέραιο.

Λύση:

1 1 1

p p(p 1) p 1

.

Β τρόπος

Έστω p πρώτος. Τότε: 1 1 1

p 2p 2p .

Μήπως έχει παραληφθεί κάτι από την εκφώνηση; Γιατί δεν είναι απαραίτητο να

είναι ο p πρώτος.

Γ τρόπος

Θέλουμε 1 1 1

p x y και έστω 0 x y αν οι παρονομαστές είναι θετικοί

ακέραιοι ή x y και x,y 0 αν οι παρονομαστές είναι ακέραιοι.

2 21 1 1xy px py xy px py p p

p x y

2(x p)(y p) p 1·p·p 1·( p)( p) ( 1)·( p)·p .

Με εξέταση των διάφορων περιπτώσεων έχουμε:

Page 31: Εισαγωγη σε διαγωνιστικα μαθηματικα για το γυμνασιο

http://www.mathematica.gr/forum/viewtopic.php?f=109&t=30104

Επιμέλεια: xr.tsif Σελίδα 31

1 1 1

p p(p 1) p 1

και

1 1 1

p 2p 2p με παρονομαστή θετικό ακέραιο

και επί πλέον 1 1 1

p p(1 p) p 1

με παρονομαστή ακέραιο.

ΘΕΜΑ 32 (ΔΗΜΗΤΡΗΣ ΙΩΑΝΝΟΥ)

Αν a,b,c είναι ακέραιοι αριθμοί, να αποδείξετε ότι ο αριθμός:

(a b)(b c)(c a) , είναι άρτιος.

Λύση:

Μεταξύ των τριών αυτών αριθμών θα υπάρχουν σίγουρα δύο – ας υποθέσουμε

οι a,b – οι οποίοι είναι είτε άρτιοι είτε περιττοί. Συνεπώς η διαφορά τους

(a b) είναι άρτιος, άρα και το γινόμενο (a b)(b c)(c a) είναι άρτιος.

ΘΕΜΑ 33 (ΔΗΜΗΤΡΗΣ ΙΩΑΝΝΟΥ)

Δείξτε ότι ο αριθμός 2x 2 , με x Z , δεν διαιρείται με το 4 .

Λύση:

Έστω x 4 + με =0,1,2,3 . Τότε:

22 2 2 2 2 2x 2 4 + 2 16 8 2 4(4 2 ) 2 4 2 .

Επομένως:

Για 20 x 2 4 2 .

Για 21 x 2 4 3 .

Για 22 x 2 4 6 4 4 2 4 2 .

Για 23 x 2 4 11 4 8 3 4 3 .

Page 32: Εισαγωγη σε διαγωνιστικα μαθηματικα για το γυμνασιο

http://www.mathematica.gr/forum/viewtopic.php?f=109&t=30104

Επιμέλεια: xr.tsif Σελίδα 32

Βλέπουμε ότι ο αριθμός 2x 2 έχει πάντα τη μορφή 4 2 ή 4 3 και

επομένως δεν διαιρείται με το 4 .

Β τρόπος

Αν x 2k , τότε 2 2x 2 4k 2 , άρα ο 2x 2 , δεν διαιρείται με το 4 , (αφού

αφήνει υπόλοιπο 2 )

Αν πάλι x 2k 1 , τότε 2 2x 2 4k 4k 3 4m 3 , και άρα και πάλι δεν

διαιρείται με το 4 , (αφού δίνει υπόλοιπο 3).

ΘΕΜΑ 34 (ΔΗΜΗΤΡΗΣ ΙΩΑΝΝΟΥ)

Να αποδείξετε ότι ο αριθμός 25, δεν μπορεί να γραφεί ως άθροισμα δέκα

ακεραίων προσθετέων, καθένας από τους οποίους να είναι ίσος με 1 ή 3 ή 5 .

Λύση:

Έστω ότι αυτό γίνεται . Έστω ότι χρησιμοποιούμε kφορές το 1, mφορές το 3,

nφορές το 5. Τότε ισχύει: k m n 10 n 10 k m και

k·1 m·3 n·5 25 k 3m 5(10 k m) 25

4k 2m 25 4k 2m 25 2 2k m 25 2 =25 .

Το τελευταίο είναι προφανώς άτοπο.

ΘΕΜΑ 35 (ΔΗΜΗΤΡΗΣ ΙΩΑΝΝΟΥ)

Αν ο a είναι περιττός ακέραιος, τότε ο αριθμός:

2 2 2A a (a 2) (a 4) 1 , διαιρείται με το 12 .

Λύση:

Έχουμε ότι 2 2 2 2 2 2 2 2a +(a+2) +(a+4) +1=a +a +4a+4+a +8a+16+1=3a +12a+21=3(a +4a+7) .

Στην παρένθεση έχουμε άρτιο αριθμό άρα το αποτέλεσμα θα είναι άρτιο.

Συνεπώς εύκολα αποδεικνύεται ότι 212|3(a +4a+7) ή

24|(a +4a+7) .

Page 33: Εισαγωγη σε διαγωνιστικα μαθηματικα για το γυμνασιο

http://www.mathematica.gr/forum/viewtopic.php?f=109&t=30104

Επιμέλεια: xr.tsif Σελίδα 33

Θεωρώ ότι είναι μια πολύ ωραία άσκηση να γίνει μέσα στην τάξη τώρα που οι

μαθητές της Γ Γυμνασίου, διδάσκονται τις ταυτότητες. Έτσι τους φέρνουμε

(όσοι είναι αρχάριοι) σε μια πρώτη επαφή με τα διαγωνιστικά μαθηματικά.

Και μπορούμε να δώσουμε και για το σπίτι να λύσουν την παρόμοια:

Αν a άρτιος, τότε ο αριθμός 2 2 2A a (a 1) (a 3) 2a 2 , διαιρείται με

το 4 .

Έχουμε ότι αφού ο a είναι περιττός θα έχει μορφή a 2n 1 δηλαδή 2 2 2 2a +4a+7=(2n+1) +4(2n+1)+7=4n +4n+1+8n+4+7=4n +12n+12 .

Παραγοντοποιώντας και πάλι μπορούμε να διώξουμε το τέσσερα και θα έχουμε 24|4(n +3n+3) ή

21|(n +3n+3) που ισχύει αφού το 1 διαιρεί κάθε αριθμό.

ΘΕΜΑ 36 (ΓΙΩΡΓΟΣ ΚΑΛΑΘΑΚΗΣ)

Γνωρίζοντας ότι ο κύκλος έχει ακτίνα 1

και το τόξο είναι τετραπλάσιο

από το , να υπολογίσετε

τη διαφορά 1 2

E E των

έγχρωμων κυκλικών

τμημάτων.

Λύση:

Από την υπόθεση συνεπάγεται ότι A 144 και B 36 .

Αν το κέντρο του κύκλου, τότε OA OB .

Έχουμε 1 2 1 2 1 2E E E E E E

2 2

O.A O.

1 144 1 36 108 3

360 360 360 10

.

Page 34: Εισαγωγη σε διαγωνιστικα μαθηματικα για το γυμνασιο

http://www.mathematica.gr/forum/viewtopic.php?f=109&t=30104

Επιμέλεια: xr.tsif Σελίδα 34

ΘΕΜΑ 37 (ΓΙΩΡΓΟΣ ΚΑΛΑΘΑΚΗΣ)

Να βρείτε το πλήθος των αριθμών από το 1 έως το 2012 που έχουν όλα τα

ψηφία τους διαφορετικά.

Λύση:

Κανένας αριθμός από το 2000 έως το 2012 δεν έχει όλα του τα ψηφία

διαφορετικά.

Οι τετραψήφιοι της μορφής 1abc με όλα τους τα ψηφία διαφορετικά έχουν

πλήθος 9·8·7 .

Οι τριψήφιοι με διαφορετικά ψηφία είναι 9·8·8.

Οι διψήφιοι με διαφορετικά ψηφία είναι 9·9.

Υπάρχουν και 9 μονοψήφιοι.

Συνολικά υπάρχουν 1242τέτοιοι αριθμοί.

ΘΕΜΑ 38 (ΟΡΕΣΤΗΣ ΓΟΤΣΗΣ)

Τετραψήφιου αριθμού το πρώτο ψηφίο είναι ίσο με το δεύτερο και το τρίτο ίσο

με το τέταρτο. Να βρεθεί ο αριθμός αν είναι γνωστό ότι είναι τέλειο τετράγωνο.

Λύση:

Ας είναι ο ζητούμενος αριθμός ο aabb , όπου a,b {0,1,2,3,4,5,6,7,8,9} .

Είναι aabb 1000a 100a 10b b 1100a 11b 11(100a b) .

Αφού είναι τέλειο τετράγωνο, πρέπει το 100a b να είναι πολλαπλάσιο του 11

και επειδή 100a b 99a a b πρέπει το a b να είναι πολλαπλάσιο του 11.

Επειδή a,b {0,1,2,3,4,5,6,7,8,9} πρέπει a b 11 .

Επίσης, αφού ο aabb είναι τέλειο τετράγωνο, το b δεν μπορεί να ισούται με

2,3,7,8. Άρα έχουμε τις εξής περιπτώσεις:

Page 35: Εισαγωγη σε διαγωνιστικα μαθηματικα για το γυμνασιο

http://www.mathematica.gr/forum/viewtopic.php?f=109&t=30104

Επιμέλεια: xr.tsif Σελίδα 35

5,b 6 ,

6,b 5 ,

7,b 4 ,

4,b 7 .

Τώρα, είναι απλό να δούμε ότι μόνο ο αριθμός 7744 είναι τέλειο τετράγωνο

( 27744 88 ).

ΘΕΜΑ 39 (ΔΗΜΗΤΡΗΣ ΙΩΑΝΝΟΥ)

Να βρεθούν οι φυσικοί αριθμοί a,b,c,d , αν είναι μικρότεροι του 3 και αν

27a 9b 3c d 67 .

Λύση:

Η δοσμένη γράφεται 27a 9b 3c d 67 3(9a 3b c) 67 d .

Επομένως ο αριθμός 67 d είναι πολλαπλάσιο του 3.

Για d 0,1,2 παίρνουμε τους67,66,65 , από τους οποίους μόνο το 66 είναι

πολλαπλάσιο του 3.

Έτσι η προηγούμενη σχέση γίνεται 9a 3b c 22 3(3a b) 22 c .

Με όμοιο συλλογισμό βρίσκουμε διαδοχικά c 1 και b 1 , οπότε a 2 .

Β τρόπος

Έχουμε ότι 27a 9b 3c d 67 67 3(9a 3b c) d .

Άρα ο αριθμός 67 διαιρείται με το 3 και δίνει πηλίκο 9a 3b c και υπόλοιπο

d 3 . Αλλά 67 3 22 1 οπότε 22 9a 3b c και d 1 . Όμοια

22 3(3a b) c και επειδή 22 3 7 1 τότε 7 3a b και c 1 .

Όμοια, αφού 7 3 2 1 έχουμε ότι a 2 και b 1 .

Page 36: Εισαγωγη σε διαγωνιστικα μαθηματικα για το γυμνασιο

http://www.mathematica.gr/forum/viewtopic.php?f=109&t=30104

Επιμέλεια: xr.tsif Σελίδα 36

ΘΕΜΑ 40 (ΔΗΜΗΤΡΗΣ ΙΩΑΝΝΟΥ)

Αν a 1 b u , είναι η ευκλείδεια διαίρεση του a 1 με τον b , όπου

*a,b N , να βρεθεί το πηλίκο και το υπόλοιπο της διαίρεσης k k 1(ab 1):b .

Λύση:

Από υπόθεση έχουμε a b u 1,0 u b .

Επομένως: k k 1 k k k 1 kab b (u 1)b ab 1 b (u 1)b 1 .

Για να είναι η πιο πάνω ισότητα ευκλείδεια διαίρεση, (με διαιρέτη το k 1b ), θα

πρέπει να αποδείξω ότι: k k 1(u 1)b 1 b .

Επειδή k k k k 1 k 1u b u 1 b (u 1)b bb (u 1)b 1 b 1 b .

Επομένως έχουμε ευκλείδεια διαίρεση με διαιρετέο τον ka b 1 , διαιρέτη το k 1b πηλίκο το , και υπόλοιπο το

k(u 1)b 1 .

ΘΕΜΑ 41 (KARKAR)

Στο άκρο της διαμέτρου του ημικυκλίου

φέραμε κάθετη . Αν το άκρο C χορδής

BC μήκους x , απέχει από την

κάθετη απόσταση y, πώς

μπορούμε να βρούμε την ακτίνα

του ημικυκλίου ;

Λύση:

Mε το σχήμα:

Αν φέρω την χορδή AC, παρατηρώ ότι τα τρίγωνα ACB και BCC' , είναι

Page 37: Εισαγωγη σε διαγωνιστικα μαθηματικα για το γυμνασιο

http://www.mathematica.gr/forum/viewtopic.php?f=109&t=30104

Επιμέλεια: xr.tsif Σελίδα 37

όμοια επειδή είναι ορθογώνια και έχουν την γωνία CBC' BAC

(γωνία υπό

χορδής και εφαπτομένης).

Επομένως 2

2x y x2Ry x R

2R x 2y .

ΘΕΜΑ 42 (ΔΗΜΗΤΡΗΣ ΙΩΑΝΝΟΥ)

Αν a,b,c είναι περιττοί φυσικοί αριθμοί, δείξτε ότι η εξίσωση: 2ax bx c 0

δεν έχει ρίζες ακέραιες.

Ας αποδειχθεί και το ισχυρότερο, δηλαδή ότι η παραπάνω εξίσωση δεν έχει

ρητές ρίζες.

Λύση:

Έστω μία ρητή ρίζα της παραπάνω εξίσωσης της μορφής p

xq

με p,q N και

(p,q) 1 . Άρα θα ισχύει: 2

2 2

2

p pa b c 0 ap bpq cq 0

q q (1) .

Φυσικά δεν γίνεται να είναι p q(mod2) και με mod2 παίρνουμε άτοπο

στην (1) αφού το δεξί μέλος είναι 0 ενώ το αριστερό είναι πάντα περιττό.

Β τρόπος

Ας δούμε και μια ακόμα λύση στο θέμα αυτό, χωρίς την χρήση των ισοτιμιών.

Αφού οι αριθμοί a,b,c είναι περιττοί φυσικοί αριθμοί, θα υπάρχουν

k,m,n N , ώστε: a 2k 1,b 2m 1,c 2n 1 .

Η διακρίνουσα του τριωνύμου, είναι:

2 2b 4ac (2m 1) 4(2k 1)(2n 1)

24m 4m 16kn 8k 8n 3 4m(m 1) 16kn 8k 8n 3 (1) .

Page 38: Εισαγωγη σε διαγωνιστικα μαθηματικα για το γυμνασιο

http://www.mathematica.gr/forum/viewtopic.php?f=109&t=30104

Επιμέλεια: xr.tsif Σελίδα 38

ΠΡΕΠΕΙ ΟΜΩΣ ΝΑ ΓΝΩΡΙΖΟΥΜΕ ΟΤΙ :

"Το γινόμενο δύο διαδοχικών ακεραίων, είναι πάντοτε πολλαπλάσιο του 2 ".

Άρα m(m 1) 2q,q N . Οπότε η (1) γράφεται:

8q 16kn 8k 8n 3 8(q 2kn k n) 3 8t 3 , όπου t Z .

Πρέπει τώρα να αποδείξουμε ότι η διακρίνουσα δεν μπορεί να είναι τέλειο

τετράγωνο ρητού αριθμού και μάλιστα, επειδή είναι ακέραιος αριθμός, θα

δείξουμε ότι δεν μπορεί να είναι τέλειο τετράγωνο ακεραίου.

ΠΡΕΠΕΙ ΟΜΩΣ ΝΑ ΓΝΩΡΙΖΟΥΜΕ ΟΤΙ :

Το τετράγωνο περιττού αριθμού παίρνει πάντα την μορφή 8v 1 .

(Η απόδειξη της πρότασης αυτής είναι απλή και αφήνεται ως άσκηση)

Άρα, αν η διακρίνουσα ήταν τέλειο τετράγωνο ακεραίου, θα έπρεπε

8v 1,v N .

Άρα θα έχουμε 8t 3 8v 1 8t 8v 4 2t 2v 1 , πράγμα που είναι

άτοπο, αφού το πρώτο μέλος είναι άρτιος, ενώ το δεύτερο περιττός.

Άρα το τριώνυμο, δεν έχει ρίζες ρητές.

ΘΕΜΑ 43 (ΔΗΜΗΤΡΗΣ ΙΩΑΝΝΟΥ)

Σε ένα ισοσκελές τρίγωνο ABC, με βάση την BC, φέρνουμε το ύψος CD. Επί

της CA, παίρνουμε σημείο E , έτσι ώστε CE CD . Φέρνουμε και το

ευθύγραμμο τμήμα DE . Αν η γωνία oA 50

, να υπολογιστούν οι γωνίες B

,

DCB

, EDC

, AED

.

Λύση:

Το τρίγωνο είναι ισοσκελές και άρα B C

. Έxoυμε ότι o o oˆ ˆˆ ˆ ˆA B C 180 2B 1 30 B 65 .

Page 39: Εισαγωγη σε διαγωνιστικα μαθηματικα για το γυμνασιο

http://www.mathematica.gr/forum/viewtopic.php?f=109&t=30104

Επιμέλεια: xr.tsif Σελίδα 39

Εφόσον το CD είναι ύψος, τότε oˆBDC 90 .

Άρα

o o oˆBDC 180 180 15B 5

o25 .

Επομένως, oDCB 25

.

Έχουμε ότι oB C 65

.

Άρα, okCk 40

.

Από την υπόθεση ισχύει ότι το τρίγωνο

DECείναι ισοσκελές. Έτσι:

o o oEDC k 180 2 140 70

. Άρα, oEDC 70

.

Βλέπουμε ότι o o o oAED 180 AED 180 70 AE D 110

.

ΘΕΜΑ 44 (ΔΗΜΗΤΡΗΣ ΙΩΑΝΝΟΥ)

Θεωρούμε έναν τριψήφιο αριθμό του οποίου το ψηφίο των εκατοντάδων είναι

μεγαλύτερο από το ψηφίο των μονάδων του. Στη συνέχεια, θεωρούμε και τον

τριψήφιο αριθμό, ο οποίος έχει τα ψηφία του πρώτου με αντίστροφη σειρά.

Αφαιρούμε τους δύο αυτούς αριθμούς και βρίσκουμε έναν νέο φυσικό αριθμό.

Στον νέο αυτόν αριθμό, προσθέτουμε τον αριθμό που έχει τα ίδια με αυτόν

ψηφία, γραμμένα κατά την αντίστροφη σειρά.

Να δικαιολογήσετε γιατί θα προκύψει σταθερός αριθμός (τον οποίο και να

βρείτε).

Λύση:

Έστω ο αριθμός abc με a c τότε ο αριθμός με τα ψηφία αντεστραμμένα θα

είναι ο cba .

Οπότε: abc 100a 10b c (1) και cba 100c 10b a (2) .

Page 40: Εισαγωγη σε διαγωνιστικα μαθηματικα για το γυμνασιο

http://www.mathematica.gr/forum/viewtopic.php?f=109&t=30104

Επιμέλεια: xr.tsif Σελίδα 40

Αφαιρώντας κατά μέλη έχω :

(1) (2) abc cba 100a 10b c 100c 10b a

abc cba 100 a c c a .

Δηλαδή ο αριθμός είναι: a c 0 c a 100 a c 10·0 c a (3) .

Ο αριθμός ο οποίος έχει αντιστραμμένα τα ψηφία του είναι ο

c a 0 a c 100 c a 10·0 a c (4) .

Προσθέτοντας κατά μέλη (κάθετα) έχω

(3) (4) a c 0 c a c a 0 a c

100 a c 10·0 c a 100 c a 10·0 a c 100·0 10·0 1·0 (5) .

Παρατηρώ ότι το άθροισμα των μονάδων είναι c a a c 0 και επειδή

a c θα ισχύει c a a c 10 , οπότε γράφουμε 0 και 1 κρατούμενο .

Προσθέτω τις δεκάδες 1 κρατούμενο οπότε έχω 0 0 1 1 .

Παρατηρώ ότι το άθροισμα των εκατοντάδων είναι a c c a 0 και επειδή

a c θα ισχύει a c c a 10 .

Οπότε τελικά έχω: 100 a c 10·0 c a 100 c a 10·0 a c 1010 .

ΘΕΜΑ 45 (KARKAR)

Ένας θετικός ακέραιος έχει περισσότερα από ένα ψηφία . Δείξτε ότι

ο αριθμός είναι μεγαλύτερος από το γινόμενο των ψηφίων του .

Λύση:

Έστω ότι ο αριθμός είναι 2 n

0 1 2 nx 10x 10 x ... 10 x , με ένα τουλάχιστον από

τα 1 2 n 1 n

x ,x ,...,x ,x

, διάφορο του μηδενός. Τότε

2 n

0 1 2 n 0 1 2 nx 10x 10 x ... 10 x x x x ...x

n 2 n 1

n 0 1 n 1 0 1 2 n 1x (10 x x ...x ) x 10x 10 x ... 10 x 0

, επειδή:

Page 41: Εισαγωγη σε διαγωνιστικα μαθηματικα για το γυμνασιο

http://www.mathematica.gr/forum/viewtopic.php?f=109&t=30104

Επιμέλεια: xr.tsif Σελίδα 41

010 x

110 x

210 x

..........

.........

.........

n 110 x

και άμα πολλαπλασιάσω κατά μέλη, θα πάρω:

n n

0 1 2 n 1 0 1 2 n 110 x x x ...x 10 x x x ...x 0

.

Επομένως n

n 0 1 2 n 1x (10 x x x ...x ) 0

n 2 n 1

n 0 1 2 n 1 0 1 2 n 1x (10 x x x ...x )) x 10x 10 x ... 10 x 0

, επειδή κάποιο

από τα 1 2 n 1 n

x ,x ,...,x ,x

, είναι διάφορο του μηδέν .

Δηλαδή, απέδειξα ότι: 2 n

0 1 2 n 0 1 2 nx 10x 10 x ... 10 x x x x ...x .

ΘΕΜΑ 46 (ΔΗΜΗΤΡΗΣ ΙΩΑΝΝΟΥ)

Αν υπάρχουν θετικού αριθμοί x,y,z τέτοιοι ώστε να είναι:

x y y z z xxy( z) yz( x) zx( y) 0

2 2 2

, να αποδείξετε ότι x y z .

Λύση:

Υπόδειξη:

Κάνε τις πράξεις (βγάλε παρενθέσεις και κάνε απαλοιφή παρονομαστών) και

κάτι σημαντικό θα παρατηρήσεις (αν το 6xyz , που θα σου εμφανιστεί, το

γράψεις 2xyz 2xyz 2xyz ).

Page 42: Εισαγωγη σε διαγωνιστικα μαθηματικα για το γυμνασιο

http://www.mathematica.gr/forum/viewtopic.php?f=109&t=30104

Επιμέλεια: xr.tsif Σελίδα 42

Έχουμε:

x y y z x zxy z yz x zx y 0

2 2 2

2 2 2 2 2 2x y xy y z zy z x x zxyz xyz xyz 0

2 2 2

2 2 2 2 2 2x y xy 2xyz y z z y 2xyz z x x z 2xyz0

2

2 2 2 2 2 2y(x 2xz z ) x(y 2yz z ) z(y 2xy x ) 0

2 2 2y(x z) x(y z) z(y x) 0 .

Άρα x z y z y x .

ΘΕΜΑ 47 (ΔΗΜΗΤΡΗΣ ΙΩΑΝΝΟΥ)

Αν ο p είναι πρώτος και αν 2012p 2014 δεν διαιρείται με το 5, να βρεθεί ο p .

Λύση:

Αν ο p είναι πρώτος διαφορετικός του 5, τότε από Euler έχουμε

(5) 4 2012 2012p 1mod5 p 1mod5 p 1mod5 p 2014 0mod5 , άτοπο,

άρα p 5 .

Β τρόπος

Ας δώσω την ίδια λύση, χωρίς modulo, μιας και αναφερόμαστε στο Γυμνάσιο.

Αν ο p είναι διάφορος του 2και του 5, τότε θα λήγει σε 1 ή 3 ή 7 ή 9 .

1η ΠΕΡΙΠΤΩΣΗ: Αν λήγει σε 1,

τότε και ο 2012p θα λήγει σε 1 και άρα ο

2012p 2014 , θα λήγει σε 5 και άρα

διαιρείται με το 5 , άτοπο.

Άρα αποκλείεται ο p να λήγει σε 1.

Page 43: Εισαγωγη σε διαγωνιστικα μαθηματικα για το γυμνασιο

http://www.mathematica.gr/forum/viewtopic.php?f=109&t=30104

Επιμέλεια: xr.tsif Σελίδα 43

2η ΠΕΡΙΠΤΩΣΗ: Αν λήγει σε 3 ,

τότε ο 4p θα λήγει σε 1 και άρα και ο

4 503(p ) λήγει σε 1, δηλαδή ο 2012p λήγει

σε 1 και άρα ο 2012p 2014 λήγει σε 5, άτοπο.

Άρα αποκλείεται ο p να λήγει σε 3.

Όμοια, στις περιπτώσεις να λήγει ο p σε 7 ή σε 9, βρίσκουμε άτοπο, οπότε

αποκλείεται ο p να λήγει σε 7 ή σε 9.

Μένει μόνο να δούμε τι γίνεται όταν ο p 2 ή όταν p 5 .

Αν p 2 ,

τότε ο 2012 4 503p (p ) , λήγει σε 6, εφόσον ο

4p , λήγει σε 6. Άρα ο 2012p 2014 ,

λήγει σε μηδέν και άρα διαιρείται με το 5, άτοπο. Άρα αποκλείεται ο p να

είναι ο 2 .

Μένει μόνο να δούμε τι γίνεται όταν p 5 .

Τότε ο 2012p , θα λήγει επίσης σε 5 και άρα ο

2012p 2014 , λήγει σε 9 και άρα

δεν διαιρείται με το 5, όπως θέλαμε.

Άρα η μοναδική τιμή του p είναι η p 5 .

ΘΕΜΑ 48 (ΔΗΜΗΤΡΗΣ ΙΩΑΝΝΟΥ)

Να προσδιορίσετε τους πραγματικούς αριθμούς x,y,z , αν ισχύει:

2 2 2x y z 17 2(2x 3y 2z) .

Λύση:

Με ισοδυναμίες έχουμε

2 2 2 2 2 2x y z 17 2(2x 3y 2z) x y z 17 4x 6y 4z 0

2 2 2 2 2 2(x 2) (y 3) (z 2) 0 (x 2) (y 3) (z 2) 0

Page 44: Εισαγωγη σε διαγωνιστικα μαθηματικα για το γυμνασιο

http://www.mathematica.gr/forum/viewtopic.php?f=109&t=30104

Επιμέλεια: xr.tsif Σελίδα 44

x 2 0 y 3 0 z 2 0 x 2 y 3 z 2 .

ΘΕΜΑ 49 (ΔΗΜΗΤΡΗΣ ΙΩΑΝΝΟΥ)

Αν *a,b,c Q , και αν a b c 0 , να αποδείξετε ότι ο αριθμός

2 2 2

1 1 1A

a b c είναι ρητός.

Λύση:

Ας παρατηρήσουμε ότι

2

2 2 2 2 2 2

1 1 1 1 1 1 a b c 1 1 12

a b c a b c abc a b c( )

,

οπότε 2 2 2

1 1 1 1 1 1Q

a b c a b c| | .

ΘΕΜΑ 50 (ΔΗΜΗΤΡΗΣ ΙΩΑΝΝΟΥ)

Να βρεθούν οι ακέραιοι x,y αν ισχύει ότι: 22x xy y 13 .

Υπόδειξη:

Συχνά σε τέτοιου είδους ασκήσεις, λύνουμε ως προς έναν άγνωστο, (εδώ είναι

εύκολο να λύσουμε ως προς x), και με κατάλληλες προσθαφαιρέσεις και

διασπάσεις, προσπαθούμε να καταλήξουμε σε ένα κλάσμα που να έχει σταθερό

αριθμητή και μεταβλητό παρονομαστή. Τότε, επειδή θέλουμε το κλάσμα αυτό

να είναι ακέραιος, πρέπει ο παρονομαστής να διαιρεί ακριβώς τον αριθμητή.

Μετά η συνέχεια είναι απλή.

Λύση:

Η δοθείσα γράφεται 2x(y 2) y 13 και προφανώς αυτή δεν ικανοποιείται

για y 2 .

Page 45: Εισαγωγη σε διαγωνιστικα μαθηματικα για το γυμνασιο

http://www.mathematica.gr/forum/viewtopic.php?f=109&t=30104

Επιμέλεια: xr.tsif Σελίδα 45

Τότε, έχουμε 2y 13

xy 2

, η οποία γράφεται και ως

2y 4 17x

y 2 y 2

.

Επειδή είναι 2y 4 (y 2)(y 2)

y 2 Zy 2 y 2

,

πρέπει να ισχύει και 17

y 2 1, 1,17, 17y 2

,

οπότε βρίσκουμε y 1 y 3 y 15 y 19 .

ΘΕΜΑ 51 (ΔΗΜΗΤΡΗΣ ΙΩΑΝΝΟΥ)

Αν ο αριθμός xyy είναι τριψήφιος , ο yz διψήφιος και ο yxy επίσης τριψήφιος

και αν xyy yz yxy , να βρεθούν τα ψηφία x,y,z .

Λύση:

Αρχικά θα πρέπει x,y {1,2,3,4,5,6,7,8,9} ενώ ο z {0,1,2,3,4,5,6,7,8,9} .

Σύμφωνα με τη δοθείσα εξίσωση έχουμε:

xyy yz yxy 100x 10y 10y z 100y 10x y .

Κάνοντας τις πράξεις καταλήγουμε ότι: z

9x 8y10

, από όπου λαμβάνουμε

ότι z 0 , που δεν προσκρούει σε κανέναν περιορισμό.

Άρα 8y

9x 8y x9

, το οποίο μας δίνει ότι x 8 και y 9 .

ΘΕΜΑ 52 (vzf)

Δείξτε ότι αν a,b είναι πραγματικοί αριθμοί μεγαλύτεροι του 1

2, τότε

1a 2b 5ab

4 .

Page 46: Εισαγωγη σε διαγωνιστικα μαθηματικα για το γυμνασιο

http://www.mathematica.gr/forum/viewtopic.php?f=109&t=30104

Επιμέλεια: xr.tsif Σελίδα 46

Λύση:

Είναι 1 1

a k,b2 2

για κάποια k, 0 .

Αρκεί: 1 1 1 1 1

k 2 5 k2 2 2 2 4

.

Αρκεί να αποδείξω ότι: 1 1 k 1

k 1 2 5 k2 4 2 2 4

.

Αρκεί να αποδείξω ότι: 3 5 5k 5 1

k 2 5k2 4 2 2 4 .

Αρκεί να αποδείξω ότι: 5k 5

k 2 5k2 2

.

Αρκεί να αποδείξω ότι: 2k 4 5k 5 10k .

Αρκεί να αποδείξω ότι: 3k 10k 0 , το οποίο αληθεύει.

ΘΕΜΑ 53 (vzf)

Έστω a 0 και b 0 . Αποδείξτε ότι 4 4 2 2

a b 1

a b b a 1 a b

.

Λύση:

Η προς απόδειξη γράφεται 5 5 2 2a b ab a b .

Αυτή όμως είναι αληθής αφού

5 5 3 3 3 3 2 2 2 2a b ab ab(a b ) ab ab(a b 1) ab·3ab 3a b a b .

ΘΕΜΑ 54 (vzf)

Αν x,y,z είναι θετικοί αριθμοί, αποδείξτε ότι

Page 47: Εισαγωγη σε διαγωνιστικα μαθηματικα για το γυμνασιο

http://www.mathematica.gr/forum/viewtopic.php?f=109&t=30104

Επιμέλεια: xr.tsif Σελίδα 47

x y y z z x 1 1 13 2(x y z)

z x y x y z

.

Λύση:

Αρκεί να αποδείξουμε ότι: x y y z z x 1 1 1

3 2(x y z)z x y x y z

.

Αρκεί να αποδείξουμε ότι:

2 2 2 2 2 23 x y y x y z z y z x x z 2 x y z xy yz zx .

Αρκεί να αποδείξουμε ότι: 2 2 2 2 2 23x y 3y x 3y z 3z y 3z x 3x z

2 2 2 2 2 22x y 2y x 2y z 2z y 2z x 2x z 6xyz .

Αρκεί να αποδείξουμε ότι: 2 2 2 2 2 2x y y x y z z y z x x z 6xyz .

το οποίο ισχύει αφού από την Α.Μ – G.M έχω:

2 2 2 2 2 2 6 6 66x y y x y z z y z x x z 6 x y z 6xyz .

Β τρόπος

Λίγο διαφορετικά...

Αρκεί να αποδείξουμε ότι x y x y

3 3 2 3y x y x

ή x y

6y x όπου ισχύει λόγω της Ανισότητας AM – GM.

Γ τρόπος

Και λίγο αλλιώς.

Είναι ομογενής άρα με x y z 1 παίρνει τη μορφή:

1 z 1 x 1 y 1 1 1 1 1 13 2 9

z x y x y z x y z

που ισχύει.

Page 48: Εισαγωγη σε διαγωνιστικα μαθηματικα για το γυμνασιο

http://www.mathematica.gr/forum/viewtopic.php?f=109&t=30104

Επιμέλεια: xr.tsif Σελίδα 48

Δ τρόπος

Λίγο διαφορετικά:

Αφού οι τριάδες (x y,y z,z x) και 1 1 1

( , , )z x y

είναι όμοια διατεταγμένες η

ζητούμενη ανισότητα προκύπτει αμέσως από την ανισότητα Chebyshev.

ΘΕΜΑ 55 (ΔΗΜΗΤΡΗΣ ΙΩΑΝΝΟΥ)

Αν 2 2 2x my a m b και 2 2 2mx y m a b , να αποδείξετε ότι

2 2 2 2x y a b .

Λύση:

2 2 2 2(x my) a m b και 2 2 2 2(mx y) m a b .

Θα προσθέσω κατά μέλη και θα βρω:

2 2 2 2 2 2 2 2 2 2 2 2x m y m x y a m b m a b

2 2 2 2 2 2 2 2 2 2(x y ) m (x y ) (a b ) m (a b )

2 2 2 2 2 2 2 2 2 2(x y )(1 m ) (a b )(1 m ) x y a b .

ΘΕΜΑ 56 (ΔΗΜΗΤΡΗΣ ΙΩΑΝΝΟΥ)

Αν m Z , να αποδείξετε ότι ο αριθμός 2 2m 3 m 1

A5 3

δεν είναι ακέραιος.

Λύση:

2 2 2m 3 m 1 8m 4A

5 3 15

.

Άρα έχουμε ότι: 2 2 2A Z 15| (8m 4) 3| (8m 4) 5| (8m 4) γιατί

είναι (5,3) 1 και 3,5είναι πρώτοι αριθμοί.

Page 49: Εισαγωγη σε διαγωνιστικα μαθηματικα για το γυμνασιο

http://www.mathematica.gr/forum/viewtopic.php?f=109&t=30104

Επιμέλεια: xr.tsif Σελίδα 49

Όμως 2 2 2 2 23| (8m 4) 8m 4mod3 2m 1mod3 m 2·2m 2mod3

2m 2mod3 το οποίο είναι άτοπο γιατί 2a 0,1mod3 για κάθε a Z .

(Ομοίως θα καταλήγαμε σε άτοπο αν δουλεύαμε με την σχέση 25| (8m 4) ).

Άρα ο Aδεν είναι ακέραιος.

Β τρόπος

Ας δούμε και έναν ακόμα τρόπο χωρίς ισοτιμίες:

Έχουμε 2

2 28m 4A 8m 4 15A 4(2m 1) 15A

15

.

Άρα 4|15A και αφού (4,15) 1 , θα πρέπει 4| A. Άρα A 4k,k Z .

Τότε έχουμε: 2 28m 4 15 4k 2m 15k 1 . Αν ο k είναι άρτιος, τότε ο

15k 1 θα είναι περιττός και άρα θα έχουμε ότι ένας άρτιος ισούται με έναν

περιττό, πράγμα άτοπο. Άρα θα πρέπει ο k να είναι περιττός. Δηλαδή

k 2n 1,n Z .

Τότε 2 2 22m 15(2n 1) 1 2m 30n 16 m 15n 8 .

Θα δείξουμε ότι η τελευταία σχέση είναι αδύνατη.

Έστω k 2t,t Z . Τότε 2m 30t 8 . Όμως ο αριθμός 30t 8 , δεν μπορεί να

είναι τέλειο τετράγωνο, αφού λήγει σε 8 .

Έστω k 2t 1,t Z . Τότε 2m 30t 23 . Και πάλι όμως έχουμε άτοπο, αφού

ο αριθμός 30t 23 , λήγει σε 3 και άρα δεν μπορεί να είναι τέλειο τετράγωνο.

Συνεπώς ο A , δεν μπορεί να είναι ακέραιος.

ΘΕΜΑ 57 (ΔΗΜΗΤΡΗΣ ΙΩΑΝΝΟΥ)

Αν ένας φυσικός αριθμός ισούται με το άθροισμα των τετραγώνων δύο

φυσικών αριθμών, τότε το ίδιο θα συμβαίνει και με το πενταπλάσιο του

αριθμού αυτού.

Page 50: Εισαγωγη σε διαγωνιστικα μαθηματικα για το γυμνασιο

http://www.mathematica.gr/forum/viewtopic.php?f=109&t=30104

Επιμέλεια: xr.tsif Σελίδα 50

Λύση:

Αν 2 2A 5(a b ) τότε, όπως εύκολα ελέγχουμε,

2 25A (2a b) (a 2b) .

Γενικότερα, στη θέση του 2 25 2 1 μπορούμε να βάλουμε οποιοδήποτε

άθροισμα τετραγώνων 2 2c d αφού 2 2 2 2 2 2(c d )(a b ) (ca db) (da cb) .

Για παράδειγμα το 34Aείναι επίσης άθροισμα τετραγώνων αφού 2 234 3 5 .

Να συμπληρώσω ότι αυτή είναι η ταυτότητα Lagrange, που αποτελεί ειδική

περίπτωση μιας γενικότερης ταυτότητας.

Και να συμπληρώσω ακόμα ότι η παραπάνω ταυτότητα (εννοώ η ειδική

περίπτωση της Lagrange) εμφανίζεται πρώτη φορά στο Liber Quadratorum του

Fibonacci, το 1225, πολύ πριν τον Lagrange. Επίσης υπάρχει σε ισοδύναμη

μορφή στα Αριθμητικά του Διόφαντου.

Β τρόπος

Αν 2 2A x y τότε

2 2 2 2 2 2 2 2 2 25A 5x 5y 4x x 4y y (4x 4xy y ) (4y 4xy x )

2 2(2x y) (2y x) .

Αφού οι αριθμοί x,y είναι φυσικοί, άρα και οι αριθμοί 2 2(2x y) ,(2y x)

είναι επίσης φυσικοί.

ΘΕΜΑ 58 (ΔΗΜΗΤΡΗΣ ΙΩΑΝΝΟΥ)

Αν ο επταψήφιος αριθμός x9y4xy7 , γνωρίζουμε ότι διαιρείται με το 11, να

βρείτε την τιμή του ψηφίου x .

ΣΗΜΕΙΩΣΗ:

Ένας αριθμός n n 1 n 2 1 0

A a a a ...a a

διαιρείται με το 11, αν και μόνο αν ο

αριθμός: n

n n 1 n 2 0a a a ... ( 1) a

, διαιρείται με το 11.

Page 51: Εισαγωγη σε διαγωνιστικα μαθηματικα για το γυμνασιο

http://www.mathematica.gr/forum/viewtopic.php?f=109&t=30104

Επιμέλεια: xr.tsif Σελίδα 51

Λύση:

Με κάποια πειράματα που έκανα με τα πολλαπλάσια του 11, παρατήρησα ότι

ένας αριθμός διαιρείται με το 11 αν η ακολουθία που είναι στο spoiler είναι

πολλαπλάσιο του 11 ή 0. Λοιπόν:

Παρατηρούμε ότι x {1,2,3,4,5,6,7,8,9} .

Άρα, θα πρέπει x 9 y 4 x y 7 0 (1) ή

x 9 y 4 x y 7 o 11 (2) .

Λύνοντας την (1) καταλήγουμε ότι x 3 . Λύνοντας την (2) έχουμε:

2x 6 o 11 , από όπου καταλαβαίνουμε ότι δεν υπάρχει κανένας αριθμός

που να ικανοποιεί τη σχέση x {1,2,3,4,5,6,7,8,9} .

Άρα, μοναδική λύση είναι η x 3 .

ΘΕΜΑ 59 (ΔΗΜΗΤΡΗΣ ΙΩΑΝΝΟΥ)

Αν x,y,z Z και 2 2 2x x | y 2| (z 4) 0 , να βρεθούν όλες οι δυνατές

τιμές των x,y,z .

Λύση:

Επειδή είναι 2 2| y 2| (z 4) 0 πρέπει να ισχύει 2x x 0 0 x 1 .

Όμως x Z άρα x 0 ή x 1 .

Και στις δύο περιπτώσεις είναι 2x x 0 , οπότε η συνθήκη γράφεται

2 2 2| y 2| (z 4) 0 | y 2| z 4 0 y 2 (z 2 z 2)

Τελικά είναι λοιπόν (x 0,y 2,z 2) ή (x 0,y 2,z 2) ή

(x 1,y 2,z 2) ή (x 1,y 2,z 2) .

Page 52: Εισαγωγη σε διαγωνιστικα μαθηματικα για το γυμνασιο

http://www.mathematica.gr/forum/viewtopic.php?f=109&t=30104

Επιμέλεια: xr.tsif Σελίδα 52

ΘΕΜΑ 60 (ΔΗΜΗΤΡΗΣ ΙΩΑΝΝΟΥ)

Αν ένας τριψήφιος αριθμός έχει το ψηφίο των μονάδων , μικρότερο από το

ψηφίο των δεκάδων και το ψηφίο των δεκάδων μικρότερο από αυτό των

εκατοντάδων και αν επί πλέον τα ψηφία του είναι διαδοχικοί φυσικοί αριθμοί,

να αποδείξετε ότι το υπόλοιπο της διαίρεσης αυτού με το 111, είναι το 99.

Λύση:

Έστω ότι ο τριψήφιος αριθμός είναι xyz . Ακόμα, από την υπόθεση θα ισχύει

ότι: y x 1 και z x 2 . Άρα:

xyz 100x 10y z 100x 10(x 1) x 2 111x 12 .

Σύμφωνα με την ταυτότητα της Ευκλείδειας Διαίρεσης θα ισχύει:

xyz 111m με 0 111 .

Έχουμε:

xyz 111m 111x 12 111m 111x 111m 12

12x m

111

.

Το τελευταίο μας δίνει ότι 12 o 111 και επειδή 0 111 , η μόνη τιμή

που μπορεί να πάρει το είναι το 99.

Έτσι, το ζητούμενο αποδείχτηκε.

Από αυτό το σημείο και μετά, θα μπορούσαμε να εργαστούμε και ως εξής:

xyz 111x 12 111x 99 99 12 111x 111 99 111(x 1) 99 και άρα

έχουμε το ζητούμενο.

Page 53: Εισαγωγη σε διαγωνιστικα μαθηματικα για το γυμνασιο

http://www.mathematica.gr/forum/viewtopic.php?f=109&t=30104

Επιμέλεια: xr.tsif Σελίδα 53

ΘΕΜΑ 61 (ΔΗΜΗΤΡΗΣ ΙΩΑΝΝΟΥ)

Να αποδείξετε ότι: k k k k

1 1 1 1 1(1 )(1 )(1 )...(1 )

2 3 4 2012 2 για κάθε k N , με

k 2 .

Λύση:

k 2

k 2

1 1k 2 2 2 1 1

2 2 .

k k k k 2 2 2 2

1 1 1 1 1 1 1 1(1 )(1 )(1 )...(1 ) (1 )(1 )(1 )(1 )

2 3 4 2012 2 3 4 2012

2 1 2 1 3 1 3 1 2012 1 2012 1 2013 1

2 2 3 3 2012 2012 2·2012 2

.

ΘΕΜΑ 62 (ΔΗΜΗΤΡΗΣ ΙΩΑΝΝΟΥ)

Έστω 1 2 3 2012

1 1 2 1 2 2011

1 1 1 1, 1 , 1 ,..., 1

2 1 1 ... 1

.

Δείξτε ότι: 1 2 3 2012

... 1 .

Λύση:

Έχω, με πράξεις πως:

1

10

2 .

1

2

1

01

.

1 2

3

1 2

01

.

....

Page 54: Εισαγωγη σε διαγωνιστικα μαθηματικα για το γυμνασιο

http://www.mathematica.gr/forum/viewtopic.php?f=109&t=30104

Επιμέλεια: xr.tsif Σελίδα 54

....

1 2 2011

2012

1 2 2011

...0

... 1

,

επομένως όλοι θετικοί.

Επιπλέον αν κάνω χιαστί γινόμενα σε κάθε ισότητα, έχω:

1

10

2 .

2 1 2 1 .

3 2 1 3 1 2 .

....

2012 1 2 2011 2012 1 2 2011... ... ,

απ' όπου με πρόσθεση κατά μέλη έχουμε:

2012 1 2 2011 2 3 2012

1... .........

2

2012 1 2 2011 1 2 3 2012... ... 1

1 2 3 2012 2012 1 2 2011... 1 ... 1 ,

αφού το γινόμενο θετικών είναι θετικός.

ΘΕΜΑ 63 (ΔΗΜΗΤΡΗΣ ΙΩΑΝΝΟΥ)

Αν οι αριθμοί x,y,zείναι ανάλογοι προς τους αριθμούς 3,4,2 να αποδείξετε

ότι: x y z 1 1 1

30y z z x x y 4·7 7·10 10·13

.

και μετά βελτιώστε το στο

x y z 111 45 1 1 130

y z z x x y 70 26 4·7 7·10 10·13

.

Page 55: Εισαγωγη σε διαγωνιστικα μαθηματικα για το γυμνασιο

http://www.mathematica.gr/forum/viewtopic.php?f=109&t=30104

Επιμέλεια: xr.tsif Σελίδα 55

Λύση:

Είναι x y z

k x 3k,y 4k,z 2k3 4 2 .

Επομένως x y z 3k 4k 2k 1 4 2 111

y z z x x y 6k 5k 7k 2 5 7 70

.

Όμως, (όπως έχει γράψει ο κ. Λάμπρου), 1 1 1 45

30( )4 7 7 10 10 13 26

.

Επομένως ισχύει αυτό που θέλουμε να αποδείξουμε διότι 111 45

70 26 , επειδή

111 26 70 45 , δηλαδή επειδή 2886 3150 .

ΘΕΜΑ 64 (ΔΗΜΗΤΡΗΣ ΙΩΑΝΝΟΥ)

Αν a,b,x,y Z και b 2a | 2a(x y) , δείξτε ότι b 2a | b(x y) .

Λύση:

Είναι άμεσο από την ισότητα b(x y) (b 2a)(x y) 2a(x y) .

ΘΕΜΑ 65 (ΔΗΜΗΤΡΗΣ ΙΩΑΝΝΟΥ)

Αν 2 2 2x y z

4x 4z 7yy

, να αποδείξετε ότι:

3 3

3 3

3x y xyz 29

x 3y xyz 15

.

Λύση:

Από την υπόθεση έχουμε 2 2 2 2 2 2 2x y z 4xy 4yz 7y x 4yx 8y z 4yz 0 .

2 2 2 216y 4(8y z 4yz) 4(2y z) .

Και αφού πρέπει 0 z 2y .

Page 56: Εισαγωγη σε διαγωνιστικα μαθηματικα για το γυμνασιο

http://www.mathematica.gr/forum/viewtopic.php?f=109&t=30104

Επιμέλεια: xr.tsif Σελίδα 56

Τώρα με z 2y , η υπόθεση γράφεται:

2 2 2

2 2 2 2 2x y 4y4x 8y 7y x 5y 4xy y x 4y 4xy 0

y

2(x 2y) 0 x 2y .

Δείξαμε λοιπόν ότι x z 2y .

Άρα έχουμε: 3 3 3 3 3 3

3 3 3 3 3 3

3x y xyz 24y y 4y 29y 29

x y xyz 8y 3y 4y 15y 15

, (για κάθε y 0 ).

ΘΕΜΑ 66 (ΔΗΜΗΤΡΗΣ ΙΩΑΝΝΟΥ)

Αν 1 1 1

a 2b,b 2c,c 2abc ca ab

, να βρεθούν οι αριθμοί a,b,c.

Λύση:

Πρέπει a,b,c 0 .

1 2b 12b a 1

bc a abc

1 2c 1 2b 2c 2a 12c b 1 1

ca b abc a b c abc

1 2a 12a c 1

ab c abc

.

-------------

2b 2c 2a 2(a b c)2 a b c

a b c (a b c)

.

Άρα 3a 1 a b c 1 που ικανοποιούν το αρχικό σύστημα.

Page 57: Εισαγωγη σε διαγωνιστικα μαθηματικα για το γυμνασιο

http://www.mathematica.gr/forum/viewtopic.php?f=109&t=30104

Επιμέλεια: xr.tsif Σελίδα 57

ΘΕΜΑ 67 (ΔΗΜΗΤΡΗΣ ΙΩΑΝΝΟΥ)

Αν *x,y,z Q

και 3x 4y 5z

4y 5z 5z 3x 3x 4y

, να αποδείξετε ότι ο αριθμός

2 2 2

2 2 2

x y z είναι ρητός.

Λύση:

3x 4y 5z 3x 4y 5z 1

4y 5z 5z 3x 3x 4y 2(3x 4y 5z) 2

6x 4y 5z

8y 5z 3x

10z 3x 4y 

(1)

(2)

(3) 

.

(3) 10z 3x (6x 5z) 5z 3x .

(1) 3x 4y . Άρα *3x 4y 5z a Q .

Τότε 2

2 2 2 2

1 1 1 100 102( ) ( )

x y z a a .

ΘΕΜΑ 68 (ΔΗΜΗΤΡΗΣ ΙΩΑΝΝΟΥ)

Αν a b a b , με *a,b Q

, να αποδείξετε ότι:

α) a b 1

β) Ο αριθμός a b είναι άρρητος.

Λύση:

α) Μετά από ύψωση των μελών στο τετράγωνο:

a b a b a b a b (a b)( a b) a b a b 1 ,

διότι a b 0 , γιατί αλλιώς θα ήταν a b και τότε από την υπόθεση θα ήταν

a a a a a a a a a 0 a 0 . Αλλά δίδεται ότι

a 0 . Άρα έχουμε άτοπο αν δεχθούμε ότι a b .

Page 58: Εισαγωγη σε διαγωνιστικα μαθηματικα για το γυμνασιο

http://www.mathematica.gr/forum/viewtopic.php?f=109&t=30104

Επιμέλεια: xr.tsif Σελίδα 58

β) Έστω ότι ο αριθμός a b είναι ρητός, δηλαδή *a b k Q .Τότε

έχουμε 2 2a b k a b k a k b .

Αφού 2 2a b 1 k b b 1 b b 1 k , άτοπο γιατί το

b b 1 δεν είναι τέλειο τετράγωνο.

ΘΕΜΑ 69 (ΔΗΜΗΤΡΗΣ ΙΩΑΝΝΟΥ)

Να βρεθούν οι πρώτοι αριθμοί a,b,c αν γνωρίζουμε ότι: 5a 6b 90c 670 .

Λύση:

Είναι 5a 6b 90c 670 5a 670 6b 90c ά a 2 .

Με αντικατάσταση στην αρχική έχουμε:

10 6b 90 670 b 15c 110 b 5 22 3c 5| b b 5 γιατί ο b

είναι πρώτος. Άρα για a 2,b 5 η αρχική σχέση μας δίνει:

10 30 90c 670 90c 630 c 7 .

Επομένως οι ζητούμενοι αριθμοί είναι: a 2,b 5,c 7 .

ΘΕΜΑ 70 (ΔΗΜΗΤΡΗΣ ΙΩΑΝΝΟΥ)

Αν n N , να βρείτε υπό ποιες προϋποθέσεις ο αριθμός: 21 n n

a3 2 6

είναι

φυσικός.

Λύση:

2n 3n 2 (n 1)(n 2)a

6 6

.

Διακρίνουμε τις εξής περιπτώσεις:

Αν 6/ n 1 ,

Page 59: Εισαγωγη σε διαγωνιστικα μαθηματικα για το γυμνασιο

http://www.mathematica.gr/forum/viewtopic.php?f=109&t=30104

Επιμέλεια: xr.tsif Σελίδα 59

τότε οποιοσδήποτε αριθμός nτης μορφής n 6k 1 είναι λύση για k 1

ακέραιο.

Αν 6/ n 2 ,

τότε οποιοσδήποτε αριθμός nτης μορφής n 6k 2 είναι λύση για k 1

ακέραιο.

Αν 2/ n 1 και 3/ n 2 ,

τότε οποιοσδήποτε αριθμός που είναι ταυτόχρονα περιττός και αφήνει υπόλοιπο

1 στη διαίρεσή του με το 3 είναι λύση. Συνεπώς ο αριθμός n είναι της μορφής

n 6k 1 για k 0 ακέραιο (για να το δείτε αυτό πάρτε π.χ. τις υπόλοιπες

διαιρέσεις του n με το 6 και δείτε ότι καταλήγετε σε άτοπο).

Αν 2/ n 2 και 3/ n 1 ,

τότε οποιοσδήποτε αριθμός που είναι άρτιος και αφήνει υπόλοιπο 2 στη

διαίρεσή του με το 3 είναι λύση. Συνεπώς ο αριθμός n είναι της μορφής

n 6k 2 για k 0 ακέραιο.

Β τρόπος

Λίγο διαφορετικά:

Έχουμε βρει πιο πάνω ότι (n 1)(n 2)

a6

. O αριθμός (n 1)(n 2) , είναι

πάντα άρτιος ως γινόμενο διαδοχικών αριθμών. Άρα για να διαιρείται με το 6,

αρκεί να διαιρείται με το 3.

Παίρνοντας τώρα τις περιπτώσεις n 3k,n 3k 1,n 3k 2 , με k N ,

εύκολα διαπιστώνουμε ότι ο a είναι ακέραιος, μόνο όταν n 3k 1 ή

n 3k 2 (και η λύση αυτή, είναι προφανώς ισοδύναμη).

ΘΕΜΑ 71 (ΔΗΜΗΤΡΗΣ ΙΩΑΝΝΟΥ)

Δείξτε ότι 1996 2 1995 998 999

1 1 1 1 1 1(1 )(1 ) (1 )(1 ) ... (1 )(1 ) 1000

2 2 2 2 2 2 .

Page 60: Εισαγωγη σε διαγωνιστικα μαθηματικα για το γυμνασιο

http://www.mathematica.gr/forum/viewtopic.php?f=109&t=30104

Επιμέλεια: xr.tsif Σελίδα 60

Λύση:

Αρχικά παρατηρούμε ότι το άθροισμά μας έχει 998 όρους (το πλήθος μας το

μετράει ο εκθέτης του 1

2 στην πρώτη παρένθεση, ξεκινάμε με

1

1 1

2 2

και

φτάνουμε μέχρι το

998

1

2

).

Θέτουμε για ευκολία

1996 2 1995 998 999

1 1 1 1 1 1A 1 1 1 1 ... 1 1

2 2 2 2 2 2

.

Κάνοντας τις επιμεριστικές σε κάθε ζεύγος παρενθέσεων βρίσκουμε ότι:

1996 2 1995 1995 1996 1997

1 1 1 1 1 1 998A 998

2 2 2 2 2 2 2

, (είχαμε 998

άσσους και 998 κλάσματα της μορφής 1997

1

2),

2 3 1995 1996 1997

1 1 1 1 1 998A 998

2 2 2 2 2 2 .

Τώρα παρατηρούμε ότι:

1997 10

998 998 9981

2 2 1024 .

1996

2 3 1995 1996 1996

1 1 1·

1 1 1 1 1 12 2 2 1 112 2 2 2 2 212

, (πρόκειται για

άθροισμα όρων γεωμετρικής προόδου με λόγο 2 ).

Επομένως έχουμε ότι:

2 3 1995 1996 1997

1 1 1 1 1 998A 998 998 1 1 1000

2 2 2 2 2 2

.

Page 61: Εισαγωγη σε διαγωνιστικα μαθηματικα για το γυμνασιο

http://www.mathematica.gr/forum/viewtopic.php?f=109&t=30104

Επιμέλεια: xr.tsif Σελίδα 61

ΘΕΜΑ 72 (ΔΗΜΗΤΡΗΣ ΙΩΑΝΝΟΥ)

α) Να αποδείξετε ότι για κάθε φυσικό αριθμό k , ισχύει: 13 1 1

k(k 13) k k 13

.

β) Να υπολογίσετε το άθροισμα: 1 1 1 1

S ...1 14 14 27 27 40 1990 2003

.

Λύση:

α) Από την εξίσωση 13 1 1

k(k 13) k k 13

παίρνουμε τους περιορισμούς k 0

και k 13 , περιορισμός ο οποίος δεν μας ενδιαφέρει αφού k N . Το

Ελάχιστο Κοινό Πολλαπλάσιο είναι: EK k(k 13) .

Η δοσμένη εξίσωση γίνεται:

13 1 1 13 1 1k(k 13) k(k 13) k(k 13)

k(k 13) k k 13 k(k 13) k k 13

13 k 13 k . Άρα η 13 1 1

k(k 13) k k 13

είναι ταυτότητα για κάθε

*k N .

Β τρόπος (για το α)

α) Είναι, 1 1 k 13 k k 13 k 13

k k 13 k(k 13) k(k 13) k(k 13) k(k 13)

.

β) Από το (α) έχουμε ότι 1 1 1 1

k(k 13) 13 k k 13

. Έτσι βρίσκουμε ότι

1 1 1 1S ...

1 14 14 27 27 40 1990 2003

1 1 1 1 1 1 1 11

13 14 14 27 27 40 1990 2003

Page 62: Εισαγωγη σε διαγωνιστικα μαθηματικα για το γυμνασιο

http://www.mathematica.gr/forum/viewtopic.php?f=109&t=30104

Επιμέλεια: xr.tsif Σελίδα 62

1 1 2002S 1 S

13 2003 13·2003

.

ΘΕΜΑ 73 (ΔΗΜΗΤΡΗΣ ΙΩΑΝΝΟΥ)

Έστω 3 2 4 3 99 100

A ...2 3 3 4 98 99

. Να αποδείξετε ότι ο αριθμός

2 2A

198 , είναι ρητός.

Λύση:

Εύκολα βλέπουμε ότι: n 1 n 1 1

n(n 1) n n 1

, για κάθε *n N .

ΣΗΜΕΙΩΣΗ:

Παλαιότερα έχουμε δει αρκετές χρήσιμες παρόμοιες σχέσεις, όπως:

(a) 1 1 1

n(n 1) n n 1

.

EΦΑΡΜΟΓΗ: Να υπολογιστεί το άθροισμα: 1 1 1

...1 2 2 3 100 101

.

(b) Γενικά: k 1 1

n(n k) n n k

.

EΦΑΡΜΟΓΗ: Να υπολογιστεί το άθροισμα: 3 3 3

...1 4 4 8 298 301

.

(c) n 1 1

1 2 3 ... n (n 1) 1 2 3 ... n 1 2 3 ... n (n 1)

.

EΦΑΡΜΟΓΗ: Να υπολογιστεί το άθροισμα:

1 2 100...

1 2 1 2 3 1 2 3 ... 100 101

.

Page 63: Εισαγωγη σε διαγωνιστικα μαθηματικα για το γυμνασιο

http://www.mathematica.gr/forum/viewtopic.php?f=109&t=30104

Επιμέλεια: xr.tsif Σελίδα 63

(d) 1 1 1 1

n(n 1)(n 2) 2 n(n 1) (n 1)(n 2)

.

EΦΑΡΜΟΓΗ: Να υπολογιστεί το άθροισμα:

1 1 1...

1 2 3 2 3 4 100 101 102

.

Τώρα, για την άσκησή μας, έχουμε:

1 1 1 1 1 1 1 1A ( ) ( ) ... ( )

2 3 3 4 98 99 2 99 .

Άρα: 2 21 1 2 2 1 1

A A2 99 2 99198 198

και άρα ο 2 2

A198

, είναι

ρητός.

ΘΕΜΑ 74 (ΔΗΜΗΤΡΗΣ ΙΩΑΝΝΟΥ)

Αν *x,y R και αν

3 3 2 2x 4y 4xy x y , να αποδείξετε ότι ο αριθμός

5x 8yA

2x 5y

είναι ακέραιος.

Λύση:

Έχω από τη δοθείσα:

3 2 2 3 2 2 2 2x 4xy x y 4y 0 x x 4y y x 4y 0

2 2x y x 4y 0 x y x 2y x 2y .

Σε κάθε περίπτωση:

x y τότε 3y

A 1 Z3y

.

x 2y τότε 18y

A 2 Z9y

.

Page 64: Εισαγωγη σε διαγωνιστικα μαθηματικα για το γυμνασιο

http://www.mathematica.gr/forum/viewtopic.php?f=109&t=30104

Επιμέλεια: xr.tsif Σελίδα 64

x 2y τότε 2y

A 2 Zy

.

ΘΕΜΑ 75 (ΔΗΜΗΤΡΗΣ ΙΩΑΝΝΟΥ)

Αν x,y Z και 6 6 4 2 5 5 2 4x y x y xy x y x y 1 , δείξτε ότι:

22012 x y 2013 .

Λύση:

Θα παραγοντοποιήσουμε την παράσταση και θα έχουμε :

6 4 2 6 2 4 5 5 4 2 2 4 2 2 4 4x x y y x y xy x y x (x y ) y (y x ) xy(x y ) 4 4 2 2 4 4 4 4 2 2(x y )(x y ) xy(x y ) (x y )(x y xy) 2 2 2 2 2 2 2 2 2 2(x y )(x y )(x xy y ) (x y)(x y)(x y )(x xy y ) 1 .

Εφαρμόζοντας τη γνωστή ταυτότητα έχουμε:

2 2 3 3(x y)(x y )(x y ) 1 .

Για να ισχύει αυτό πρέπει και τα 3 αθροίσματα να είναι ίσα με 1 ή το πρώτο

και το τελευταίο να είναι 1 αφού το δεύτερο είναι σίγουρα 1.

Επομένως οι δυνατές τιμές των x,y είναι 0,1, 1 με τους εξής περιορισμούς :

α) Δεν μπορούν να είναι και οι δύο αριθμοί 0 γιατί τότε τα αθροίσματα της

ισότητας θα ήταν 0.

β) Δεν μπορούν και οι δύο αριθμοί να είναι 1 ή και οι δύο 1 αφού και πάλι τα

αθροίσματα δεν επαληθεύουν την ισότητα .

Άρα ο ένας αριθμός είναι 0 και ο άλλος είναι 1 ή 1 .

Τώρα, διασπούμε ξανά την ισότητα και την φτιάχνουμε ως εξής: 2 2 2 2(x y)(x y)(x y )(x xy y ) 1 .

Άρα 2 2(x y ) 1 αφού οι δύο παρενθέσεις απλοποιούνται με το 1 ( xy 0 ).

Page 65: Εισαγωγη σε διαγωνιστικα μαθηματικα για το γυμνασιο

http://www.mathematica.gr/forum/viewtopic.php?f=109&t=30104

Επιμέλεια: xr.tsif Σελίδα 65

Αν y 1 θα έχουμε: 2 2x y 0 1 1 άρα η ισότητα δεν επαληθεύεται .

Οπότε οι πιθανές περιπτώσεις είναι οι εξής:

22012 1 0 2012 1 0 2013 , 22012 0 1 2012 0 1 2013 και 22012 ( 1) 0 2012 1 2013 .

Και όλες οι περιπτώσεις επαληθεύουν την εκφώνηση .

Ωραιότατα.

Ας μου επιτραπεί ένα σχόλιο: Είναι πιο απλό να αφήσεις την παραπάνω ως έχει

(να μη συγχωνεύσεις όρους).

Οπότε, επειδή 2 2 2 2x y y,0 x xy 0 θα είναι ίσα με 1 .

Άρα και x y x y 1 ή x y x y 1 . Και οι δύο περιπτώσεις δίνουν

y 0 , οπότε x 1 , δηλαδή 2x 1 , και τελειώσαμε (με ένα σμπάρο δύο

τρυγόνια).

Επίσης, προσοχή εδώ. Η περίπτωση x 0 δεν υφίσταται και, αν θες, δεν

επαληθεύει την εκφώνηση. Οδηγεί στην 6y 1 .

ΘΕΜΑ 76 (ΔΗΜΗΤΡΗΣ ΙΩΑΝΝΟΥ)

Αν 3a b 5c

b 5c 3a 5c 3a b

, να βρεθεί η τιμή της παράστασης:

1 1 1P ( ) (2a b 5c)

2a b 5c .

Λύση:

Από την ιδιότητα των αναλογιών έχουμε

3a b 5c 3a b 5c 1

b 5c 3a 5c 3a b 10c 2b 6a 2

.

Συνεπώς 5c b 6a , 3a 5c 2b , 3a b 10c b 3a 5c .

Page 66: Εισαγωγη σε διαγωνιστικα μαθηματικα για το γυμνασιο

http://www.mathematica.gr/forum/viewtopic.php?f=109&t=30104

Επιμέλεια: xr.tsif Σελίδα 66

1 1 1 2b 7 8b 28P ( )( 2b)

2b b b 3 2b 3 3

3

.

ΘΕΜΑ 77 (ΔΗΜΗΤΡΗΣ ΙΩΑΝΝΟΥ)

Να βρεθεί ο ακέραιος αριθμός a , αν η παράσταση :

7 4 3 5 2 6 11 6 2A

2a 1

, είναι αριθμός ακέραιος.

Λύση:

4 4 3 3 3 2 3 2 2 9 6 2 2A

2a 1

2 2 2(2 3) ( 3 2) (3 2) 2 3 3 2 3 2

2a 1 2a 1

5

2a 1

.

Πρέπει 2a 1|5 .

Με πράξεις βρίσκουμε ότι οι δυνατές τιμές του a είναι 0,1,3, 2 .

ΘΕΜΑ 78 (ΔΗΜΗΤΡΗΣ ΙΩΑΝΝΟΥ)

Δίνεται το πολυώνυμο 3 2P(x) ax bx cx d , όπου οι αριθμοί a,b,c,d είναι

ακέραιοι. Να αποδείξετε ότι είναι αδύνατον να ισχύουν ταυτόχρονα ότι:

P(19) 1 και P(62) 2 .

Λύση:

Αν P(19) 1 και P(62) 2 ίσχυαν ταυτόχρονα τότε θα μπορούσαμε να έχουμε:

3 2 3 262 a 62 b 62c d 19 a 19 b 19c d 1 , το οποίο γίνεται:

Page 67: Εισαγωγη σε διαγωνιστικα μαθηματικα για το γυμνασιο

http://www.mathematica.gr/forum/viewtopic.php?f=109&t=30104

Επιμέλεια: xr.tsif Σελίδα 67

3 3 2 262 a 19 a 62 b 19 b 62c 19c

3 3 2 2a(62 19 ) b(62 19 ) c(62 19)

2 2a(62 19)(62 62·19 19 ) b(62 19)(62 19) 43c

2 2143a(62 62·19 19 ) 43b(62 19) 43c

2 2143(62 a 62·19a 19 a 62b 19b c) 1 .

Φυσικά όταν όλοι οι αριθμοί της παρένθεσης είναι ακέραιοι το αποτέλεσμά της

δεν μπορεί να είναι 1

43.

Αξίζει να αναφερθεί ότι η άσκηση είναι άμεση συνέπεια της εξής απλής

πρότασης:

Έστω n n-1

0 1 n-1 nP(x)=a x +a x + a x+a ένα πολυώνυμο με συντελεστές

ακεραίους. Αν b,c Z, b c , τότε ισχύει (b c) / P(b) P(c)( ) .

Η απόδειξη είναι απλή εφαρμογή της ταυτότητας

n n n 1 n 2 n 2 n 1x y =(x y)(x +x y+ xy +y ) (ΜΑΓΚΟΣ ΘΑΝΟΣ)

ΘΕΜΑ 79 (ΔΗΜΗΤΡΗΣ ΙΩΑΝΝΟΥ)

Δείξτε ότι x n n 3

n 1 2x n 1

για κάθε *x N , n N .

Λύση:

Η ανίσωση γίνεται ως εξής (x n)(2x n 1) (n 1)(n 3) .

Κάνουμε τις πράξεις ...2 2 2 22x nx x 2nx n n n 4n 32x 3nx 3n x

3x(2x 1) 3n(x 1) 3 .

Ακόμη και αν αντικαταστήσουμε με τις ελάχιστες δυνατές τιμές των

Page 68: Εισαγωγη σε διαγωνιστικα μαθηματικα για το γυμνασιο

http://www.mathematica.gr/forum/viewtopic.php?f=109&t=30104

Επιμέλεια: xr.tsif Σελίδα 68

μεταβλητών το αποτέλεσμα θα είναι: 1(2 1) 3·0(1 1) 3 το οποίο

επαληθεύει τη σχέση x(2x 1) 3n(x 1) 3 .

Με την χρήση μεγαλυτέρων τιμών το αποτέλεσμα αυξάνεται οπότε η σχέση

συνεχίζει να επαληθεύεται .

ΘΕΜΑ 80 (ΔΗΜΗΤΡΗΣ ΙΩΑΝΝΟΥ)

Δείξτε ότι ο αριθμός: 2n 3 2n 3 2n 1 2n 3A 3 4 2 6 είναι τετράγωνος για κάθε

φυσικό αριθμό n .

Λύση:

2n 3 4n 6 2n 1 2n 3 2n 3 2n 3 4n 4 2 2n 4 4n 4 2 n 2 2 n 1A 3 ·2 2 ·2 ·3 3 ·2 (2 1) 3 ·2 (3 ) ·(4 ) ,

άρα είναι γινόμενο δύο τέλειων τετραγώνων άρα είναι τετράγωνος.

ΘΕΜΑ 81 (ΔΗΜΗΤΡΗΣ ΙΩΑΝΝΟΥ)

Δείξτε ότι ο αριθμός a1b a2b ... a9b

Aa5b

με a 0 , είναι τέλειο τετράγωνο.

Σημείωση:

Με xyz , συμβολίζουμε τον τριψήφιο αριθμό που έχει ψηφίο μονάδων z ,

δεκάδων y και εκατοντάδων x .

Λύση:

Ο αριθμητής γίνεται

100a 10 b 100a 20 b .... 100a 90 b 900a 450 9b

9(100a 50 b) .

Ο παρονομαστής γίνεται 100a 50 b .

Οπότε το κλάσμα είναι 9(100a 50 b)

9100a 50 b

, άρα ο αριθμός είναι τέλειο

Page 69: Εισαγωγη σε διαγωνιστικα μαθηματικα για το γυμνασιο

http://www.mathematica.gr/forum/viewtopic.php?f=109&t=30104

Επιμέλεια: xr.tsif Σελίδα 69

τετράγωνο.

ΘΕΜΑ 82 (ΔΗΜΗΤΡΗΣ ΙΩΑΝΝΟΥ)

Να βρείτε το πλήθος των ψηφίων του συνόλου:2010 2011A {x N| 2 x 2 } .

Λύση:

Έχουμε 2010 2010 2010 2010 2010A {2 1,2 2,2 3,...,2 2 } .

Το πλήθος που ζητάμε είναι όσο είναι και το πλήθος των αριθμών 20101,2,3,...,2 ,

δηλαδή το ζητούμενο πλήθος είναι 20102 .

ΘΕΜΑ 83 (ΔΗΜΗΤΡΗΣ ΙΩΑΝΝΟΥ)

Να βρεθεί ο φυσικός αριθμός n , ώστε ο αριθμός:

n n 1 2 n n 2 3 n 1 n 2A 2 15 10 6 5 3 3 10 5 , να τελειώνει σε 2013 μηδενικά.

Λύση:

n n 1 2 n 2 n2 ·15 ·10 30 ·15·10 30 ·30·50 ,

n n 2 3 n 2 36 ·5 ·3 30 ·5 ·3 ,

n 1 n 2 n 23 ·10 ·5 30 ·3·5 .

Άρα ο αριθμός γίνεται :

n n 2 3 n 2 n 2 2A 30 ·50·30 30 ·5 ·3 30 ·3·5 30 ·3·5(10 5·3 5)

n n n 1 n 1A 30 ·3·5(100 45 5) 60·30 ·3·5 10 ·3 ·5·6 .

Το 6·5 λήγει σε 0 άρα το n πρέπει να είναι 2011.

ΘΕΜΑ 84 (ΔΗΜΗΤΡΗΣ ΙΩΑΝΝΟΥ)

Αν n nab 2 a 3 b όπου *a,b N , n N , να βρεθεί ο διψήφιος αριθμός ab .

Page 70: Εισαγωγη σε διαγωνιστικα μαθηματικα για το γυμνασιο

http://www.mathematica.gr/forum/viewtopic.php?f=109&t=30104

Επιμέλεια: xr.tsif Σελίδα 70

Λύση:

n n n nab 2 a 3 b a(10 2 ) b(3 1) .

Άρα το n 1,2,3 .

Για n 1 έχουμε

b 4a δηλαδή a 1,b 4 οπότε ab 14 ή a 2,b 8 οπότε ab 28 .

Για n 2 έχουμε

4b 3a δηλαδή b 3,a 4 οπότε ab 43 ή b 6,a 8 οπότε ab 86 .

Για n 3 έχουμε

a 13b , δεν προχωράμε.

ΘΕΜΑ 85 (ΔΗΜΗΤΡΗΣ ΙΩΑΝΝΟΥ)

Να λυθεί η εξίσωση: 2n m 2n m2 2 2 496 , m,n N .

Λύση:

Η δοθείσα γράφεται 2n m 22 1 2 1 495 3 ·5·11 . Ελέγχοντας τους διαιρέτες

του δεξιού μέλους για όρους της μορφής m2 1 , έχουμε m2 1 5 , m2 1 9 , m2 1 33 . Το n βρίσκεται από τον δεύτερο παράγοντα, από τις

2n m2 1 2 1 99·5 , 2n m2 1 2 1 55·9 , 2n m2 1 2 1 15·33 .

Τελικά (n,m) (2,5) .

ΘΕΜΑ 86 (ΔΗΜΗΤΡΗΣ ΙΩΑΝΝΟΥ)

Θεωρούμε τους φυσικούς αριθμούς: 1 2 2013

a ,a ,...,a . Δείξτε ότι ο αριθμός

1 2 2 3 2013 1A (a a )(a a )...(a a ) είναι άρτιος.

Page 71: Εισαγωγη σε διαγωνιστικα μαθηματικα για το γυμνασιο

http://www.mathematica.gr/forum/viewtopic.php?f=109&t=30104

Επιμέλεια: xr.tsif Σελίδα 71

Λύση:

Αν το γινόμενο 1 2 2 3 2013 1

A (a a )(a a )...(a a ) ήταν περιττός, τότε θα ήταν

περιττός κάθε παράγοντας.

Δηλαδή θα ήσαν περιττοί οι 2013παράγοντες 1 2 2 3 2013 1

a a ,a a ,...,a a (*) .

Επειδή ο 2013 είναι περιττός, θα ήταν περιττό και το άθροισμα των 2013 όρων

(*) . Αλλά αυτό το άθροισμα ισούται με 1 2 2013

2(a a ... a ) άρτιος. Άτοπο,

και λοιπά.

ΘΕΜΑ 87 (Socrates)

Αν οι πραγματικοί αριθμοί x,yείναι τέτοιοι ώστε 2 2x xy y 4 και

4 2 2 4x x y y 8 , να βρείτε τον 6 3 3 6x x y y .

Λύση:

4 2 2 4 4 2 2 4 2 2 2 2 2 2 2x x y y x 2x y y x y (x y ) x y

2 2 2 2(x y xy)(x y xy) 8 .

Έτσι , από τον πρώτο αριθμό καταλαβαίνουμε ότι :

2 2 2 2 2 2 2 2(x y xy)(x y xy) 4(x y xy) 8 x y xy 2 xy 1

και 2 2x y 3 και

4 4x y 7 .

6 6 3 3 6 6 2 2 4 2 2 4x y x y x y 1 (x y )(x x y y ) 1 3(7 1) 1 19 .

ΘΕΜΑ 88 (Socrates)

Βρείτε τους a,b,c αν 2 2 2 2a b 2c,1 a 2ac, c ab .

Λύση:

Με πρόσθεση κατά μέλη, (αφού πρώτα πολλαπλασιάσουμε τα μέλη της τρίτης

εξίσωσης με το 2 , έχουμε:

Page 72: Εισαγωγη σε διαγωνιστικα μαθηματικα για το γυμνασιο

http://www.mathematica.gr/forum/viewtopic.php?f=109&t=30104

Επιμέλεια: xr.tsif Σελίδα 72

2 2 2 2a b 1 a 2c 2c 2ac 2ab

2 2 2 2 2a b 2ab a c 2ac c 1 2c 0

2 2 2(a b) (a c) (c 1) 0 .

Άρα πρέπει: a b,a c,c 1 , δηλαδή: a b 1 , τιμές που επαληθεύουν το

δοσμένο σύστημα.

ΘΕΜΑ 89 (ΔΗΜΗΤΡΗΣ ΙΩΑΝΝΟΥ)

Αν a b c 0 και a,b,c 0 , δείξτε ότι:

2 2 2 2 2 2 2 2 2

2011 2012 2013a b c b c a c a b( ) ( ) ( ) 1

2ab 2bc 2ac

.

Λύση:

a b c ,

b c a ,

c b a .

Για τον αριθμητή του πρώτου κλάσματος έχουμε :

2 2 2 2 2 2a b c a (b c)(b c) a a(b c) a ab ac a(a b c)

a·2b 2ab . Οπότε το κλάσμα γίνεται 20112ab1 1 1

2ab .

Το δεύτερο κλάσμα γίνεται:

2 2 2b (c a)(c a) b b(c a) b bc ab b(b c a) 2bc1

2bc 2bc 2bc 2bc 2bc

20121 1 .

Και το τρίτο:

Page 73: Εισαγωγη σε διαγωνιστικα μαθηματικα για το γυμνασιο

http://www.mathematica.gr/forum/viewtopic.php?f=109&t=30104

Επιμέλεια: xr.tsif Σελίδα 73

2 2 2c (a b)(a b) c ( c)(a b) c ac ab c(c b a) 2ac1

2ac 2ac 2ac 2ac 2ac

2013( 1) 1 .

Άρα A 1 1 1 1 .

ΘΕΜΑ 90 (ΔΗΜΗΤΡΗΣ ΙΩΑΝΝΟΥ)

Να υπολογίσετε το άθροισμα: 2 3 99A 5 4 5 4 5 4 5 ... 4 5 .

(ΣΗΜΕΙΩΣΗ: Δεν πρέπει να χρησιμοποιηθεί ύλη του Λυκείου).

Λύση:

Δουλεύουμε ως εξής :

2 99 2 3 2 100 99A 5 (5 1)·5 (5 1)·5 ... (5 1)·5 5 5 5 5 5 ... 5 5 1005 , από την απλοποίηση των όρων.

ΘΕΜΑ 91 (ΔΗΜΗΤΡΗΣ ΙΩΑΝΝΟΥ)

Να απλοποιηθεί το κλάσμα:2·3 4·6 6·9 ··· 2010·3015

A3·5 6·10 9·15 ··· 3015·5025

.

Λύση:

Το κάθε γινόμενο του αριθμητή μπορεί να γραφτεί ως 23x

2 οπού το x στον

κάθε όρο αυξάνει κατά 2 . Οπότε μπορούμε να γράψουμε τον αριθμητή ως:

2 2 2 2 2 2 2 2 23[x (x 2) (x 4) ... (x 2008) ] 3(2 4 6 8 ... 2010 )

2 2

που όμως ισούται με:

2 2 2 2 2 2 2 2 2 2 2 2 23(2 ·1 2 ·2 2 ·3 2 ·4 .... 2 ·1005 ) 12(1 2 3 4 ... 1005 )

2 2

.

Το κάθε γινόμενο του παρονομαστή μπορεί να γραφεί ως:

Page 74: Εισαγωγη σε διαγωνιστικα μαθηματικα για το γυμνασιο

http://www.mathematica.gr/forum/viewtopic.php?f=109&t=30104

Επιμέλεια: xr.tsif Σελίδα 74

22x 5xx·(x )

3 3 όπου το x στον πρώτο όρο είναι κατά 1 μεγαλύτερο από το

αντίστοιχο του αριθμητή και αυξάνει ανά όρο κατά 3. Επομένως μπορούμε να

εκφράσουμε τον παρονομαστή ως:

2 2 2 2 2 2 2 2 25[(x 1) (x 4) (x 7) ... (x 3013) ] 5(3 6 9 12 .... 3015 )

3 3

το οποίο όμως είναι ίσο με:

2 2 2 2 2 2 2 2 2 2 2 2 25(3 ·1 3 ·2 3 ·3 3 ·4 ... 3 ·1005 ) 45(1 2 3 4 ... 1005 )

3 3

.

Άρα οι δύο παρενθέσεις απλοποιούνται και έχουμε το κλάσμα: 6 2

15 5 το οποίο

είναι και η τιμή της παράστασης.

Β τρόπος

Ο κάθε όρος του αριθμητή είναι τα 2

5 του αντίστοιχου στον παρονομαστή.

Πράγματι, οι μεν είναι 2n·3nκαι οι δε 3n·5n .

Άρα όλο το κλάσμα ισούται με 2

5.

(Κάνουμε την απλοποίηση. Δεν χρειάζεται να κάνουμε τις προσθέσεις σε

αριθμητή και παρονομαστή).

ΘΕΜΑ 92 (ΔΗΜΗΤΡΗΣ ΙΩΑΝΝΟΥ)

Να υπολογίσετε το άθροισμα: 1333 1332 1331 2A 4(3 3 3 ··· 3 3 1) .

Λύση:

Η παράσταση γίνεται 1332 1330 1332 1330 1328 24[3 (3 1) 3 (3 1) ....(3 1)] 8(3 3 3 ... 3 1) .

Μέσα στην παράσταση έχουμε γεωμετρική πρόοδο με 1

a 1 και λόγο 23 .

Page 75: Εισαγωγη σε διαγωνιστικα μαθηματικα για το γυμνασιο

http://www.mathematica.gr/forum/viewtopic.php?f=109&t=30104

Επιμέλεια: xr.tsif Σελίδα 75

Εφαρμόζουμε τον τύπο του αθροίσματος όρων γεωμετρικής προόδου και

έχουμε n 667

667

1

1 9 1A 8a 8 9 1

1 8

.

Περισσότερα για τις προόδους (μιας και δεν είναι στην ύλη του Γυμνασίου)

δείτε εδώ .

Β τρόπος

1333 1332 1331 2A 4 3 3 3 ... 3 3 1

1333 1332 1331 2A3 3 3 ... 3 3 1

4

1332 1330 0 1332 1330 0A A3 3 1 3 3 1 ... 3 3 1 3 3 ... 3

4 8 (1) .

Το δεύτερο μέλος της (1) είναι άθροισμα όρων γεωμετρικής προόδου με λόγο

23 , 0

1a 3 1 και πλήθος όρων :

1332 v 1 1332 2v 2 1332

v 1a 3 a · 3 1·3 3 2v 2 1332 v 667 .

Άρα 667 667

1332 1330 0

v 667 667

3 1 3 13 3 ... 3 S S S

3 1 2

.

Οπότε 667

667

667 667

A 3 1S A 8S A 8 A 4 3 1

8 2

.

ΘΕΜΑ 93 (ΔΗΜΗΤΡΗΣ ΙΩΑΝΝΟΥ)

Με έναν υπολογιστή, τυπώσαμε τους αριθμούς 2004a 2 και 2004b 5 . Πόσα

ψηφία τυπώθηκαν συνολικά;

(ΠΗΓΗ: ΟΛΥΜΠΙΑΔΕΣ ΜΑΘΗΜΑΤΙΚΩΝ, Β ΓΥΜΝΑΣΙΟΥ του Μπάμπη

Στεργίου)

Λύση:

Έστω ότι ο αριθμός 20042 έχει nψηφία και ο αριθμός 20045 έχει mψηφία.

Ζητάμε να βρούμε το n m .

Page 76: Εισαγωγη σε διαγωνιστικα μαθηματικα για το γυμνασιο

http://www.mathematica.gr/forum/viewtopic.php?f=109&t=30104

Επιμέλεια: xr.tsif Σελίδα 76

Έχουμε όμως: n 1 2004 n10 2 10 και m 1 2004 m10 2 10 .

Με πολλαπλασιασμό κατά μέλη, παίρνουμε n m 2 2004 m n10 10 10 . Άρα

n m 2 2004 m n . Όμως μεταξύ των ακεραίων n m 2 , n m ,

υπάρχει μόνο ο ακέραιος n m 1 . Συνεπώς πρέπει

n m 1 2004 n m 2005 .

Άρα ο υπολογιστής, θα τυπώσει 2005ψηφία.

ΘΕΜΑ 94 (Vzf)

Να βρείτε όλους τους θετικούς ακέραιους nγια τους οποίους ο 3n και ο 4n

περιέχουν τα ψηφία 0,1,2,3,4,5,6,7,8,9ακριβώς μια φορά.

Λύση:

Αφού 4 3n n , σημαίνει ότι τα ψηφία του 4n είναι περισσότερα ή ίσα από τα

ψηφία του 3n . Αφού τα ψηφία των αριθμών 3n , 4n είναι στοιχεία του συνόλου

A {0,1,2,3,4,5,6,7,8,9} και μάλιστα από μία μόνο φορά το καθένα, άρα

πρέπει ο 3n να έχει το πολύ 5 ψηφία και ο 4n τουλάχιστον 5 ψηφία (το

άθροισμα των ψηφίων του 3n και του 4n πρέπει να είναι 10).

Θα αποδείξουμε ότι ο 3n δεν μπορεί να έχει 5 ψηφία. Πράγματι, αν ο 3n ήταν

πενταψήφιος, πρέπει n 22 . Ο πιο μικρός λοιπόν n θα ήταν ο 22 και τότε 3n 10648 και 4n 234256 . Στην περίπτωση αυτή, ο 3n θα είχε 5 ψηφία και ο 4n θα είχε 6 ψηφία, πράγμα άτοπο (αφού το πλήθος των ψηφίων των 3n , 4n

πρέπει να είναι 10).

Προφανώς αν ο 4n ήταν μεγαλύτερος του 22 , τότε και πάλι το άθροισμα των

ψηφίων του 3n και του 4n θα ξεπερνούσε το 10.

Άρα ο 3n είναι το πολύ τετραψήφιος, δηλαδή n 21 . Φυσικά αποκλείεται ο n

να είναι μονοψήφιος, διότι ο μεγαλύτερος μονοψήφιος είναι ο 9, οπότε αν

n 9 τότε 3n 729 και 4n 6561 , που είναι άτοπο αφού το άθροισμα των

ψηφίων των αριθμών αυτών είναι μικρότερο του 10.

Καταλαβαίνουμε λοιπόν ότι πρέπει 10 n 21 .

Page 77: Εισαγωγη σε διαγωνιστικα μαθηματικα για το γυμνασιο

http://www.mathematica.gr/forum/viewtopic.php?f=109&t=30104

Επιμέλεια: xr.tsif Σελίδα 77

Στο μεταξύ, αποκλείεται ο n να λήγει σε 0,1,5,6 , διότι τότε οι αριθμοί 3n και

4n θα είχαν ίδιο το τελευταίο ψηφίο τους, που είναι άτοπο από την υπόθεση.

Καταλήξαμε λοιπόν ότι ο nείναι κάποιος από τους αριθμούς 12,13,14,17,18,19.

Με δοκιμές, βρίσκουμε ότι ο ζητούμενος αριθμός είναι ο n 18 , διότι 318 5832 και 418 104976 .

ΘΕΜΑ 95 (Vzf)

Σε ένα πρωτάθλημα κάθε ομάδα κέρδισε τουλάχιστον 5 παιχνίδια όταν έπαιξε

εναντίον άλλων ομάδων. Αποδείξτε ότι κάποια ομάδα έχασε τουλάχιστον 5

παιχνίδια σε αυτό το πρωτάθλημα.

Λύση:

'Έστω ότι έχουμε n ομάδες . Τότε με βάση τα δεδομένα του προβλήματος, θα

έχουν παιχθεί 5n τουλάχιστον παιχνίδια μεταξύ των ομάδων αυτών. Και αφού η

κάθε ομάδα δημιουργεί στις υπόλοιπες τουλάχιστον πέντε ήττες, άρα οι n

ομάδες δημιουργούν συνολικά τουλάχιστον 5n ήττες.

Αν όλες οι ομάδες είχαν μέχρι 4 ήττες η κάθε μία, τότε συνολικά θα είχαμε το

πολύ 4n ήττες, που είναι άτοπο. Άρα θα υπάρχει μία τουλάχιστον ομάδα η

οποία θα έχει τουλάχιστον 5 ήττες.

ΘΕΜΑ 96 (Vzf)

Έχουμε τους αριθμούς 1 2 3 4

a a a a (1) και 1 2 3 4

b b b b (2) . Έστω ότι

2 1a b (3) και

4 4a b (4) . Αν

1 2 3 4 1 2 3 4a a a a b b b b (5) να

βάλετε σε αύξουσα σειρά τους 8αριθμούς.

Λύση:

1 2 1a a b και άρα από (1) και (2) , ο

1a είναι ο μικρότερος. Όμοια, ο

2a είναι

ο δεύτερος μικρότερος.

Από (3) , (4) , (5) , με πρόσθεση κατά μέλη, παίρνουμε 1 3 2 3

a a b b .

Page 78: Εισαγωγη σε διαγωνιστικα μαθηματικα για το γυμνασιο

http://www.mathematica.gr/forum/viewtopic.php?f=109&t=30104

Επιμέλεια: xr.tsif Σελίδα 78

Όμως: 3 1 3 2 3 2

2a a a b b 2b , δηλαδή 3 2 1

a b b . Και πάλι, από τα ήδη

γνωστά αποτελέσματα και τις (1) και (2) ο 1

b είναι ο τρίτος στην σειρά, και ο

2b ο τέταρτος. Επίσης, επειδή

4 4b a , από (1) και (2)παίρνουμε ότι

4b ο

μεγαλύτερος.

Ισχύει ότι 1 1

a b , 2 2

a b , και προσθέτοντας κατά μέλη στην (5) παίρνουμε

3 4 3 4a a b b .

Όμως: 4 3 4 3 4 3

2a a a b b 2b , οπότε 4 3

a b , και άρα ο 4

a είναι ο δεύτερος

απ' το τέλος.

Αν 3 3

a b , τότε από 1 1

a b , 2 2

a b , 4 4

a b θα ήταν

1 2 3 4 1 2 3 4a a a a b b b b , άτοπο από (5) .

Άρα 1 2 1 2 3 3 4 4

a a b b b a a b .

ΘΕΜΑ 97 (ΔΗΜΗΤΡΗΣ ΙΩΑΝΝΟΥ)

Ένας καθηγητής Γυμνασίου επέλεξε και από τις τρεις τάξεις 25 μαθητές για να

εκπροσωπήσουν το σχολείο σε μια εκδήλωση. Να αποδειχθεί ότι ανάμεσά τους

υπάρχουν τουλάχιστον 9 μαθητές οι οποίοι είναι στην ίδια τάξη.

Μια πρώτη επαφή με την ΑΡΧΗ της ΠΕΡΙΣΤΕΡΟΦΩΛΙΑΣ.

Λύση:

25 3 8 1 , οπότε σύμφωνα με την αρχή της περιστεροφωλιάς, μια τάξη θα

έχει τουλάχιστον 8 1 9 μαθητές.

Μια λίγο πιο προχωρημένη με την αρχή της περιστεροφωλιάς.

ΘΕΜΑ 98 (ΔΗΜΗΤΡΗΣ ΤΣΙΝΤΣΙΛΙΔΑΣ)

Σε ένα ισόπλευρο τρίγωνο πλευράς 2cm έχουμε 5 σημεία. Να αποδείξετε ότι η

απόσταση 2 σημείων από τα 5 είναι μικρότερη ή ίση με 1cm.

Γενικεύστε το πρόβλημα για ισόπλευρο πλευράς n , όπου n ακέραιος.

Page 79: Εισαγωγη σε διαγωνιστικα μαθηματικα για το γυμνασιο

http://www.mathematica.gr/forum/viewtopic.php?f=109&t=30104

Επιμέλεια: xr.tsif Σελίδα 79

Λύση:

Παίρνουμε τα μέσα των τριών πλευρών του

ισοπλεύρου τριγώνου και έτσι

σχηματίζουμε 4 ισόπλευρα τρίγωνα

πλευράς 1 το καθένα. Για να

τοποθετήσουμε τα 5 σημεία μέσα στο

τρίγωνο, θα πρέπει τουλάχιστον σε ένα

από τα 4μικρότερα ισόπλευρα τρίγωνα, να

τοποθετηθούν τουλάχιστον 2 σημεία. Η

απόσταση των δύο αυτών σημείων, θα

είναι βέβαια μικρότερη ή ίση του 1, αφού

ανήκουν στο εσωτερικό ισοπλεύρου τριγώνου με πλευρά 1.

Γενικά, αν η πλευρά του ισοπλεύρου τριγώνου είναι ίση με n , τότε από τα 5

σημεία, δύο τουλάχιστον θα έχουν απόσταση μικρότερη ή ίση του n

2.

ΘΕΜΑ 99 (ΔΗΜΗΤΡΗΣ ΤΣΙΝΤΣΙΛΙΔΑΣ)

Δίνεται ορθογώνιο nxk και 2kn 1 σημεία στο εσωτερικό του.

Να αποδείξετε ότι ένα από τα τρίγωνα που σχηματίζουν αυτά τα σημεία έχει

εμβαδόν το πολύ 1

2.

Λύση:

Πριν αποδείξω την άσκηση, παραθέτω μια πρόταση , η οποία θα μου χρειαστεί:

ΠΡΟΤΑΣΗ:

Δίνεται τετράγωνο ABCD πλευράς 1. Να αποδείξετε ότι αν ένα τρίγωνο EFG

Page 80: Εισαγωγη σε διαγωνιστικα μαθηματικα για το γυμνασιο

http://www.mathematica.gr/forum/viewtopic.php?f=109&t=30104

Επιμέλεια: xr.tsif Σελίδα 80

είναι εντός του τετραγώνου αυτού, τότε το εμβαδόν του είναι 1

2 .

ΑΠΟΔΕΙΞΗ:

Θα αποδείξω την πιο πάνω πρόταση, στην

γενική περίπτωση όπου τα σημεία είναι στο

εσωτερικό του τετραγώνου (ομοίως

εργαζόμαστε και όταν ένα ή περισσότερα από

τα σημεία αυτά βρίσκονται πάνω στις πλευρές

του τετραγώνου).

Έστω ότι η ευθεία EG τέμνει τις πλευρές

AD,CBτου τετραγώνου στα σημεία H,I

αντιστοίχως. Έστω επίσης ότι ο φορέας του

ύψους KFτου τριγώνου EFG τέμνει την AB στο J .

Τότε έχουμε:

(EFG) (HIJ) (ABCD) (AHJ) (IJB) (HDCH)

(1 x)z (1 y)(1 z) (x y) 11

2 2 2

1 xz yz x 1 yz x(1 z) 1

2 2 2 2

.

Δείξαμε λοιπόν ότι: 1

(EFG)2

.

Λύση:

Θα αποδείξουμε τώρα τη άσκησή μας:

Χωρίζουμε το αρχικό ορθογώνιο σε kn τετραγωνάκια (όπως στο σχήμα)

πλευράς 1 το καθένα από αυτά. Αν σε όλα τα τετραγωνάκια υπήρχαν το πολύ

από δύο σημεία, τότε συνολικά θα είχαμε το πολύ 2kn σημεία, που είναι

άτοπο, αφού έχουμε 2kn 1 σημεία. Άρα σε ένα τουλάχιστον τετραγωνάκι, θα

Page 81: Εισαγωγη σε διαγωνιστικα μαθηματικα για το γυμνασιο

http://www.mathematica.gr/forum/viewtopic.php?f=109&t=30104

Επιμέλεια: xr.tsif Σελίδα 81

υπάρχουν τουλάχιστον 3 σημεία.

Αν τα σημεία αυτά είναι

συνευθειακά, τότε το

"εκφυλισμένο" τρίγωνο, θα έχει

εμβαδόν μηδέν, δηλ 1

2 .

Αν πάλι τα σημεία δεν είναι

συνευθειακά, τότε δημιουργούν

τρίγωνο, το οποίο με βάση την

προηγούμενη πρόταση που δείξαμε, θα έχει και πάλι εμβαδόν 1

2 .

Άρα το ζητούμενο εδείχθη.

ΘΕΜΑ 100 (ΚΛΕΟΒΟΥΛΟΣ ΚΟΦΟΝΙΚΟΛΑΣ)

Να δειχθεί ότι για κάθε τριάδα x,y,z R ισχύει η ανισότητα: 2

cyc

(x 1) 3 ,

όπου ο συμβολισμός cyc

ή πιο ολοκληρωμένα cyclic

σημαίνει

2 2 2 2

cyc

(x 1) (x 1) (y 1) (z 1) .

Για ποιες τιμές των x,y,z ισχύει η ισότητα;

Λύση:

2 2 2 2 2 2x 1 y 1 z 1 3 (x y z ) 3 με την ισότητα να ισχύει μόνο όταν

x y z 0 .

ΘΕΜΑ 101 (ΔΗΜΗΤΡΗΣ ΙΩΑΝΝΟΥ)

Αν x,y R , με x y 2 , τότε 2 2x y x y .

Page 82: Εισαγωγη σε διαγωνιστικα μαθηματικα για το γυμνασιο

http://www.mathematica.gr/forum/viewtopic.php?f=109&t=30104

Επιμέλεια: xr.tsif Σελίδα 82

Λύση:

Από την ανισότητα των δυνάμεων (ή απλά με πράξεις) προκύπτει :

2 2x y x y

2 2

. Άρα αρκεί να δείξουμε ότι

2(x y)x y

2

που ισχύει λόγω

της συνθήκης.

ΘΕΜΑ 102 (ΚΟΦΟΝΙΚΟΛΑΣ ΚΛΕΟΒΟΥΛΟΣ)

Να αποδείξετε ότι για κάθε +x,y,z R ισχύει ότι: (x+1) 1 .

Για ποιες τιμές των x,y,z αληθεύει η ισότητα;

Λύση:

Υποθέτω ότι η ζητούμενη σχέση είναι η (x 1)(y 1)(z 1) 1 .

Αποδεικνύεται άμεσα αφού και οι τρεις όροι είναι μη αρνητικοί πραγματικοί

άρα ο κάθε όρος της ισότητας είναι μεγαλύτερος της μονάδας. Συνεπώς και το

γινόμενο θα είναι μεγαλύτερο ή ίσο της μονάδας. Η ισότητα ισχύει αν οι

μεταβλητές ισούνται με μηδέν.

ΘΕΜΑ 103 (ΚΟΦΟΝΙΚΟΛΑΣ ΚΛΕΟΒΟΥΛΟΣ)

Να λυθεί η εξίσωση: 2 2 2 2

40 20 8 12+ = 1

x +2x 48 x +9x+8 x +10x x +5x 50

.

Λύση:

Με x 10, 8, 1,0,5,6 έχουμε

40 20 8 12+ = 1

(x+8)(x 6) (x+1)(x+8) (x+10)x (x+10)(x 5)

20 +4 = 1(x+8)(x 6)(x 1) x(x+10)(x 5

2(x 1) (x 6) 2(x 5) 3x

)

= 1

(x 6)(x 1) x(x 5

20 4

)

Page 83: Εισαγωγη σε διαγωνιστικα μαθηματικα για το γυμνασιο

http://www.mathematica.gr/forum/viewtopic.php?f=109&t=30104

Επιμέλεια: xr.tsif Σελίδα 83

x(x 5) 4(x 6)(x 1) x(x 5)(x20 6)(x 1)

2 2 2 2(x 5x) 4(x 5x 6) (x 5x)(x20 5x 6) .

Αν θέσουμε 2x 5x y τότε η εξίσωση γίνεται:

220y 4(y 6) y(y 6) y 10y 24 0 y 4 y 6 .

Για y 4 έχουμε 2 2x 5x 4 x 5x 4 0 x 1 x 4 , δεκτές.

Για y 6 έχουμε 2 2x 5x 6 x 5x 6 0 x 2 x 3 , δεκτές.

ΘΕΜΑ 103β (ΚΟΦΟΝΙΚΟΛΑΣ ΚΛΕΟΒΟΥΛΟΣ)

Έστω A,B,C τρία ενδεχόμενα. Το ενδεχόμενο {πραγματοποιείται μόνο το A}

μπορεί να εκφραστεί ως A B' C' (Το συμπληρωματικό οποιουδήποτε

ενδεχομένου A μπορεί να γραφεί ως A' ). Να βρεθούν εκφράσεις για τα

ενδεχόμενα έτσι ώστε:

(i) Πραγματοποιούνται και τα τρία.

(ii) Πραγματοποιείται τουλάχιστον ένα.

(iii) Δεν πραγματοποιείται κανένα.

(iv) Δεν πραγματοποιούνται περισσότερα από δύο.

Λύση:

Εύκολη (φτιάξτε διάγραμμα του Venn).

ΘΕΜΑ 104 (ΚΟΦΟΝΙΚΟΛΑΣ ΚΛΕΟΒΟΥΛΟΣ)

Με πόσους τρόπους μπορούμε να τοποθετήσουμε nμη επιτιθέμενους πύργους

(δηλαδή, να μη βρίσκονται δύο στην ίδια σειρά ή στήλη) σε μία σκακιέρα

διαστάσεων n n ;

Page 84: Εισαγωγη σε διαγωνιστικα μαθηματικα για το γυμνασιο

http://www.mathematica.gr/forum/viewtopic.php?f=109&t=30104

Επιμέλεια: xr.tsif Σελίδα 84

Λύση:

Με n! τρόπους. Η εξήγηση είναι η εξής απλή: Ο πρώτος πύργος θα έχει n

επιλογές τοποθέτησης σε μια σειρά. Όμως ο δεύτερος θα έχει n 1 επιλογές σε

κάθε σειρά, ο τρίτος θα έχει n 2 σε κάθε σειρά....ο n– οστός 1 επιλογή.

ΘΕΜΑ 105 (ΔΗΜΗΤΡΗΣ ΙΩΑΝΝΟΥ)

Έστω *n N , 22 n n 4

a2

και

22 n n 4b

2

.

α) Δείξτε ότι: 1 1

1a b

β) Δείξτε ότι οι αριθμοί a,b είναι άρρητοι.

Λύση:

α)

2 2 2 2

1 1 1 1 2 2

a b 2 n n 4 2 n n 4 2 n n 4 2 n n 4

2 2

2 2 2 2

2 2 2 2 2

2(2 n n 4) 2(2 n n 4) 4 2n 2 n 4 4 2n 2 n 4

(2 n 4 n)(2 n 4 n) (2 n 4) n

2 2

2 2 2 2

8 4 n 4 8 4 n 41

4 4 n 4 n 4 n 8 4 n 4

.

Άρα, το ζητούμενο αποδείχτηκε.

β) Οι αριθμοί a,b ( ) διότι 2n +4 ( ) , διότι το άθροισμα δύο

διαδοχικών τετραγώνων δεν είναι ποτέ τετράγωνο άλλου αριθμού (σε αυτήν την

περίπτωση μιλάω για διαδοχικούς αριθμούς διότι η υπόρριζος ποσότητα είναι

n 4 και για να ισχύσει η διαδοχικότητα πρέπει n 1 . Παρόλα αυτά ούτε

αυτήν η τιμή μας δίνει τετράγωνο ακεραίου και άρα δεν φεύγει η ρίζα. Για

οποιοδήποτε άλλη τιμή εξακολουθούμε να έχουμε άρρητο αριθμό) , με

Page 85: Εισαγωγη σε διαγωνιστικα μαθηματικα για το γυμνασιο

http://www.mathematica.gr/forum/viewtopic.php?f=109&t=30104

Επιμέλεια: xr.tsif Σελίδα 85

εξαίρεση τους διαδοχικούς αριθμούς 3,4,5. Συνεπώς θα έπρεπε να έχουμε

2w= n +16 ώστε w .

Έκανα επίσης την παρατήρηση ότι κάθε τετράγωνο αριθμού απέχει από το

τετράγωνο του αμέσως προηγουμένου κατά i

p +2 με i

p πάντα περιττό. Δηλαδή

22 4 και 21 1 η διαφορά μεταξύ τους είναι 3, δηλαδή i

p +2 με i

p 1 . Ο

επόμενος αριθμός (9) θα απέχει από τον προηγούμενο (4 ) κατά 5, δηλαδή τον

προηγούμενο αριθμό που προσθέσαμε συν 2 κάθε φορά κ.ο.κ.

ΘΕΜΑ 106 (ΔΗΜΗΤΡΗΣ ΙΩΑΝΝΟΥ)

Αν x 0 και 0 y 1 , να αποδείξετε ότι: x 1 x 1 y y .

Λύση:

Ολοκληρώνω την απόδειξη μετά την υπόδειξη του Γιώργου:

2 2 221 y+ x 1 y 1 + x y + 1 y = 1+x .

ΘΕΜΑ 107 (ΔΗΜΗΤΡΗΣ ΙΩΑΝΝΟΥ)

α) Να αποδείξετε ότι a b

2ab

, για κάθε a,b 0 . Πότε ισχύει η ισότητα;

β) Για κάθε x 0 , να αποδείξετε ότι: 2 2

2x 1 2x 24

x x x 2x

.

Λύση:

α) Πολλαπλασιάζουμε και τα δύο μέλη με το ab και έχουμε: a+b 2 ab .

Η απόδειξη: 2 2 2(a b) 0 (a+b) 4ab 0 (a+b) 4ab a+b 2 ab και

το ζητούμενο αποδείχθηκε.

Η ισότητα αληθεύει όταν a=b=1.

β) Σύμφωνα με το (α) ισχύουν:

Page 86: Εισαγωγη σε διαγωνιστικα μαθηματικα για το γυμνασιο

http://www.mathematica.gr/forum/viewtopic.php?f=109&t=30104

Επιμέλεια: xr.tsif Σελίδα 86

2

2x 1 x (x 1)2

x(x 1)x x

(1) .

2

2x 2 x (x 2)2

x(x 2)x 2x

(2) .

Με πρόσθεση κατά μέλη των (1) και (2) έχουμε το ζητούμενο.

Η ισότητα δεν ισχύει επειδή x x 1 .

ΘΕΜΑ 108 (ΔΗΜΗΤΡΗΣ ΙΩΑΝΝΟΥ)

Δείξτε ότι: 7 30 10 24 4

36 11 3

.

Λύση:

Ωραία άσκηση και πολύ οριακή η διαφορά.

Υποθέτουμε ότι 7 30 24 4

6 3

(1) .

Θα πρέπει 49 30 48 32

06

που ισχύει.

Άρα πρέπει να αποδείξουμε ότι 10 100

3 31111

που ισχύει επειδή 11·9 99 .

Με πρόσθεση αυτής της σχέσης και της (1) προκύπτει το ζητούμενο.

ΘΕΜΑ 109 (ΔΗΜΗΤΡΗΣ ΙΩΑΝΝΟΥ)

Να βρεθούν (αν υπάρχουν) οι ακέραιοι αριθμοί x,y ώστε να ισχύει:

a) 2 2x y 2015 .

b) 2 2x y 2013 .

Page 87: Εισαγωγη σε διαγωνιστικα μαθηματικα για το γυμνασιο

http://www.mathematica.gr/forum/viewtopic.php?f=109&t=30104

Επιμέλεια: xr.tsif Σελίδα 87

Λύση:

a) Η απάντηση είναι ότι δεν υπάρχουν, και η αιτιολόγηση είναι απλή.

Αφού το άθροισμα είναι περιττό, ο ένας όρος θα είναι άρτιος κι ο άλλος

περιττός. Ας υποθέσουμε ότι ο x είναι άρτιος και ο y περιττός (ξέρουμε

βέβαια

ότι τετράγωνο άρτιου είναι άρτιος και περιττού είναι περιττός και αντίστροφα).

Θα ήταν 2 2 2 2 2 2(2k) (2a 1) 2015 4k 4a 4a 1 2015 4(k a a) 2014

που είναι άτοπο επειδή το 2014 δεν είναι πολλαπλάσιο του 4 .

b) Αφού οι x,y έχουν περιττό άθροισμα, τότε ο ένας θα είναι περιττός και ο

άλλος άρτιος. Έστω λοιπόν x 2k (άρτιος) και y 2n 1 (περιττός). Οπότε

έχουμε:

2 2 2 2 2 2(2k) +(2n+1) =2013 4k +4n +4n+1=2013 k +n +n=503

2 2n +n+(k 503)=0 . Η δοσμένη εξίσωση είναι δευτεροβάθμια ως προς n .

Οπότε θα έχει διακρίνουσα 2 2=b 4ac=1 4(k 503) .

Για να έχουμε λύσεις θα απαιτήσω 21503+ k

4 (δείτε και παρακάτω). Στην

περίπτωση ισότητας έχουμε k= 503 . Άτοπο, αφού x,y Z .

Συνεπώς 21

503+ >k4

αλλά αφού k Z έχουμε 2503 k . Άρα k [ 22,22] .

Με χρήση των πιθανών τιμών βρίσκουμε ότι για κάθε τιμή του kη ρίζα της

διακρίνουσας θα είναι άρρητος αριθμός. Συνεπώς δεν έχουμε ακέραιες λύσεις.

Ο κύριος Δημήτρης επισήμανε ότι υπάρχουν και άλλοι τρόποι λύσης

x=3m,y=3n

Page 88: Εισαγωγη σε διαγωνιστικα μαθηματικα για το γυμνασιο

http://www.mathematica.gr/forum/viewtopic.php?f=109&t=30104

Επιμέλεια: xr.tsif Σελίδα 88

x=3m,y=3n+1

x=3m,y=3n+2

x=3m+1,y=3n+1

x=3m+1,y=3n+2

x=3m+2,y=3n+2

Όλοι όμως οδηγούν σε άτοπο.

Βασικά για να μειώσουμε τις δοκιμές μπορούμε να πούμε το εξής:

Ο ένας όρος, αφού θα είναι άρτιος και τέλειο τετράγωνο θα λήγει σε 0,4,6 και

ο δεύτερος ως περιττό τετράγωνο σε 1,5,9. Απ' όλους τους συνδυασμούς μόνο

ο 4,9 δίνει τελευταίο ψηφίο 3.

Συνεπώς πρέπει να κάνουμε, για τον άρτιο όρο τις δοκιμές

2,12,22,32,8,18,28,38 (οι αντίστοιχοι αρνητικοί δίνουν ίσα τετράγωνα) και να

δούμε αν ο περιττός όρος είναι τέλειο τετράγωνο περιττού.

β) Λίγο συντομότερα, με χρήση της βασικής ιδιότητας

2 23| (x y ) (3| x 3| y) .

Επειδή 3| 2013 είναι 2 23| (x y ) x 3a, y 3b , άρα 2 29a 9b 2013 ,

άτοπο, αφού το 9 δεν διαιρεί το 2013

Δίνω και μία ακόμα λύση, για ποικιλία:

Παίρνουμε τις παρακάτω περιπτώσεις:

1η ΠΕΡΙΠΤΩΣΗ: x=3m,y=3n .

Τότε η εξίσωση γράφεται: 2 29m 9n 2013 9| 2013 , που είναι άτοπο

2η ΠΕΡΙΠΤΩΣΗ: x=3m,y=3n+1.

Τότε η εξίσωση γράφεται: 2 2 2 29m 9n 6n 1 2013 9m 9n 6n 2012

Άρα 3| 2012 , που είναι άτοπο.

Page 89: Εισαγωγη σε διαγωνιστικα μαθηματικα για το γυμνασιο

http://www.mathematica.gr/forum/viewtopic.php?f=109&t=30104

Επιμέλεια: xr.tsif Σελίδα 89

Ακριβώς με τον ίδιο τρόπο καταλήγουμε σε άτοπο, και στις υπόλοιπες

περιπτώσεις, οι οποίες είναι:

x=3m,y=3n+2

x=3m+1,y=3n+1

x=3m+1,y=3n+2

x=3m+2,y=3n+2 .

ΘΕΜΑ 110 (ΔΗΜΗΤΡΗΣ ΙΩΑΝΝΟΥ)

Να αποδείξετε ότι:

α) 1 1 1

n(n 1) n n 1

, για κάθε *n N .

β) 2013 2013 2013 2013

1 1 1 1... 1

4 9 16 2025 .

Λύση:

α) Πολλαπλασιάζοντας και τα δύο μέλη της ισότητας με το n(n+1)

(περιορισμοί δεν χρειάζονται αφού *n N ) το ζητούμενο έπεται.

β) Έχουμε ότι

2 2013 2 2013 2 2013 2 2013

2013 2013 2013 2013

1 1 1 1 1 1 1 1+ + +...+ =[( ) ] +[( ) ] +[( ) ] +...+[( ) ]

4 9 16 2025 2 3 4 45.

Το ζητούμενο έπεται.

Β τρόπος

α) Μια ακόμη αντιμετώπιση είναι η εξής.

1 (n 1) n n 1 n 1 1

n(n 1) n(n 1) n(n 1) n(n 1) n n 1

.

β) Νομίζω είναι πολύ προφανές ότι ισχύει αυτό.

Page 90: Εισαγωγη σε διαγωνιστικα μαθηματικα για το γυμνασιο

http://www.mathematica.gr/forum/viewtopic.php?f=109&t=30104

Επιμέλεια: xr.tsif Σελίδα 90

Η σχέση γίνεται 4026 4026 4026 4026 4026

1 1 1 1 1.... 44· 1

2 3 4 45 2 .

ΘΕΜΑ 111 (ΔΗΜΗΤΡΗΣ ΙΩΑΝΝΟΥ)

Αν a,b είναι αριθμοί πρώτοι και c N , και αν επί πλέον ισχύει ότι:

5 a 7 b c 2 , να βρεθούν οι αριθμοί a,b,c.

Λύση:

Υψώνοντας τα δύο μέλη στο τετράγωνο έχουμε 225a+70 ab+49b=2c . Για να

έχουμε ότι c πρέπει a b ώστε να διώξουμε τη ρίζα. Έχουμε λοιπόν ότι 2c =72a . Οι μοναδικές λύσεις της εξίσωσης είναι (a,b,c)=(2,2,12) .

ΘΕΜΑ 112 (ΔΗΜΗΤΡΗΣ ΤΣΙΝΤΣΙΛΙΔΑΣ)

α) Να αποδείξετε ότι 2 2 2x y (x y)

a b a b

, αν a,b 0 .

β) Με τη βοήθεια του α) να αποδείξετε την γενικευμένη ανισότητα:

2 2 2 2

1 2 n 1 2 n

1 2 n 1 2 n

x x x (x x ... x )...

a a a a a ...a

, όπου

1 2 na ,a ,...,a 0 .

γ) Να αποδείξετε ότι για x,y,z 0 ισχύει: x y z 3

y z z x x y 2

.

Λύση:

Είναι,

22 2 2 2 2x yx y b(a b)x a(a b)y ab(x y)

a b (a b) ab(a b)

22 2 2 2 bx ayb x 2abxy a y

ab(a b) ab(a b)

.

Page 91: Εισαγωγη σε διαγωνιστικα μαθηματικα για το γυμνασιο

http://www.mathematica.gr/forum/viewtopic.php?f=109&t=30104

Επιμέλεια: xr.tsif Σελίδα 91

Συνεπώς, για να ισχύει η ανισότητα που θέλουμε, θα πρέπει

ab(a b) 0 ab 0 a b 0 ab 0 a b 0 .

Διακρίνουμε τις εξής περιπτώσεις.

1) a 0,b 0 .

Τότε, η συνθήκη είναι αληθής.

2) a 0,b 0 .

Για να είναι αληθής η παραπάνω συνθήκη, θα πρέπει a b 0 }

3) a 0,b 0 .

Θα πρέπει να απαιτήσουμε πάλι a b 0

4) a 0,b 0 .

Η συνθήκη δεν αληθεύει.

Η άσκηση ίσως έπρεπε να είχε δοθεί ως εξής

Εκδοχή 1

Για τους θετικούς αριθμούς a και b , να δειχθεί ότι 2 2 2x y (x y)

a b a b

.

Εκδοχή 2

Για τους ετερόσημους αριθμούς a και b , με την ιδιότητα a b 0 ,

να δείξετε ότι 2 2 2x y (x y)

a b a b

.

το (β) είναι άμεση συνέπεια της Ανισότητας Cauchy – Buniakowski ή

Andreescu.

Β τρόπος

Το (α) ερώτημα έχει ήδη απαντηθεί σε προηγούμενη δημοσίευση.

Για το (β) εργαζόμαστε επαγωγικά:

Page 92: Εισαγωγη σε διαγωνιστικα μαθηματικα για το γυμνασιο

http://www.mathematica.gr/forum/viewtopic.php?f=109&t=30104

Επιμέλεια: xr.tsif Σελίδα 92

Για n 2 , το ζητούμενο ισχύει (λόγω του (α)).

Υποθέτουμε ότι το ζητούμενο ισχύει για n k και θα αποδείξουμε ότι θα

ισχύει και για n k 1 .

Πράγματι, από την υπόθεση της επαγωγής, έχουμε ότι:

2 2 2 2

1 2 k 1 2 k

1 2 k 1 2 k

x x x (x x ... x )...

a a a a a ... a

2 2 2 2 2 2 2

1 2 k k 1 1 2 k k 1 1 2 k k 1

1 2 k k 1 1 2 k k 1 1 2 k k 1

x x x x (x x ... x ) x (x x ... x x )...

a a a a a a ... a a a a ... a a

,

(λόγω του ερωτήματος (α)).

Άρα το ζητούμενο εδείχθη.

Το (γ) ερώτημα είναι πολύ γνωστή άσκηση, την οποία πολλές φορές έχουμε δει

και εδώ στο mathematica.

Έχουμε: 2 2 2 2x y z x y z (x y z)

y z x z x y xy yz xy yz xz yz 2(xy yz zx)

.

Αρκεί λοιπόν να αποδείξουμε ότι: 2(x y z) 3

2(xy yz zx) 2

,

ή ότι 2(x y z) 3(xy yz zx) , ή

2 2 2x y z xy yz zx ,

ή 2 2 22x 2y 2z 2xy 2yz 2zx , ή

2 2 2 2 2 2x x y y z z 2xy 2yz 2zx 0 ,

ή 2 2 2(x y) (y z) (z x) 0 , το οποίο είναι αληθές.

ΘΕΜΑ 113 (ΔΗΜΗΤΡΗΣ ΙΩΑΝΝΟΥ)

Δείξτε ότι: 2 2 2a 6a 25 b 8b 41 c 10c 34 12 .

Page 93: Εισαγωγη σε διαγωνιστικα μαθηματικα για το γυμνασιο

http://www.mathematica.gr/forum/viewtopic.php?f=109&t=30104

Επιμέλεια: xr.tsif Σελίδα 93

Λύση:

Λοιπόν:

2 2 2a 6a 25 a 6a 9 16 (a 3) 16 16 .

2 2 2b 8b 41 b 8b 16 25 (b 4) 25 25 .

2 2 2c 10c 34 c 10c 25 9 (c 5) 9 9 .

Άρα: 2 2 2a 6a 25 (a 3) 16 16 (a 3) 6 4 1 (1) .

2 2 2b 8b 41 (b 4) 25 25 (b 4) 5 5 2 (2) .

2 2 2c 10c 34 (c 5) 9 9 (c 5) 9 3 (3) .

Με πρόσθεση κατά μέλη των (1) , (2) και (3) προκύπτει το ζητούμενο.

ΘΕΜΑ 114 (ΔΗΜΗΤΡΗΣ ΙΩΑΝΝΟΥ)

Αν ο αριθμός Aείναι ρητός και αν και ο αριθμός xείναι επίσης ρητός, και επί

πλέον ισχύει:18 8 2 x 2 2 2

A 1 17 12 25 3 15

, να αποδείξετε ότι

6A

5 .

Λύση:

Έχουμε: 217 12 12 (3 2 2) |3 2 | 3 2 .

Ακόμη 218 8 2 (4 2) | 4 2 | 4 2 .

Συνεπώς

45 30 2 12 3 2 5x 2 2 2 18 5 2(x 5)A 1 A

15 15

.

Page 94: Εισαγωγη σε διαγωνιστικα μαθηματικα για το γυμνασιο

http://www.mathematica.gr/forum/viewtopic.php?f=109&t=30104

Επιμέλεια: xr.tsif Σελίδα 94

Όπως ξέρουμε γινόμενο ρητού επί άρρητο είναι άρρητος άρα 5 2(x 5)

εκτός αν x 5 0 x 5 το οποίο θα ισχύει επειδή αλλιώς ο Aθα είναι

άρρητος ως πηλίκο αρρήτου προς ρητό. Συνεπώς 18 6

A15 5

.

ΘΕΜΑ 115 (ΔΗΜΗΤΡΗΣ ΙΩΑΝΝΟΥ)

Αν 2 2 33a b 3a 5b 0

4 , να αποδείξετε ότι 3 a b 5 .

Λύση:

Μετατρέπουμε την ισότητα σε 2 2

2 29 25 1 3 5 1a 3a b 5b a b

4 4 4 2 2 4

.

Συνεπώς βλέπουμε ότι 3 1

a a 22 2

και 5 1

b b 32 2

. Συνεπώς

a b 5 με πρόσθεση κατά μέλη των δύο παραπάνω σχέσεων.

Ακόμη πρέπει 3 1

a a 12 2

και 5 1

b b 22 2

. Συνεπώς a b 3

με πρόσθεση κατά μέλη των παραπάνω σχέσεων και συνεπώς έχουμε αποδείξει

τα ζητούμενα.

ΘΕΜΑ 116 (ΔΗΜΗΤΡΗΣ ΙΩΑΝΝΟΥ)

Δείξτε ότι υπάρχουν ακριβώς δύο ακέραιες τιμές για τον αριθμό a , ώστε η

εξίσωση: 2 2 2a x 21 x 10ax 4x , να έχει ως προς x δύο ρίζες ακέραιες.

Λύση:

Λοιπόν:

2 2 2 2 2 2a x 21 x 10ax 4x a x x 10ax 4x 21 0

2 2x (a 1) x(10a 4) 21 0 .

Page 95: Εισαγωγη σε διαγωνιστικα μαθηματικα για το γυμνασιο

http://www.mathematica.gr/forum/viewtopic.php?f=109&t=30104

Επιμέλεια: xr.tsif Σελίδα 95

Η εξίσωση έχει διακρίνουσα:

2 2 2 2 24 (10a 4) 4(a 1)·21 100a 80a 16 84a 84

2 216a 80a 100 (4a 10) .

Η εξίσωση έχει ρίζες: 1,2

x2a

.

H πρώτη μας δίνει (αφού εκτελέσουμε τις πράξεις ): 6

2(a 1)

, από όπου

λαμβάνουμε ότι 2(a 1) | 6 και παίρνουμε τις τιμές: a 2,a 4,a 2,a 0 .

Κάνοντας τις πράξεις και στη δεύτερη ρίζα καταλήγουμε: 14

2(a 1)

, από όπου

λαμβάνουμε ότι 2(a 1) | 14 .

Οι μόνες τιμές που το επαληθεύουν είναι οι a 2 και a 0 . Έτσι, αποδείξαμε

το ζητούμενο.

ΘΕΜΑ 117 (ΔΗΜΗΤΡΗΣ ΙΩΑΝΝΟΥ)

Αν a,b,c,d 0 και a b d c 2013 , να αποδείξετε ότι:

ab ac ad ab bc bd ac bc cd ad bd cd 4016 .

Λύση:

Μετατρέπουμε το πρώτο μέλος σε

a(2013 a) b(2013 b) c(2013 c) d(2013 d) και

διπλασιάζοντας τα δύο μέλη έχουμε να αποδείξουμε ότι

2 a(2013 a) 2 b(2013 b) 2 c(2013 c) 2 d(2013 d) 8032 .

Σύμφωνα με την απλή ανισότητα 2 ab a b έχουμε

2 a(2013 a) 2013

Page 96: Εισαγωγη σε διαγωνιστικα μαθηματικα για το γυμνασιο

http://www.mathematica.gr/forum/viewtopic.php?f=109&t=30104

Επιμέλεια: xr.tsif Σελίδα 96

2 b(2013 b) 2013

2 c(2013 c) 2013

2 d(2013 d) 2013

και με πρόσθεση κατά μέλη προκύπτει το ζητούμενο.

ΘΕΜΑ 118 (ΔΗΜΗΤΡΗΣ ΙΩΑΝΝΟΥ)

Θεωρούμε την συνάρτηση n n 2f (x) ( 1) ( 1) 2013 . Να υπολογίσετε το

άθροισμα: f (1) f (2) ... f (2014) .

Λύση:

Στην περίπτωση που n=2k f(n)=2014 και αν n=2k+1 f(n)= 2014 .

Άρα έχουμε ότι 2014

n=1

f (n)=0 .

ΘΕΜΑ 119 (ΔΗΜΗΤΡΗΣ ΙΩΑΝΝΟΥ)

Αν x 7y 5 και 16 x 5 , να βρεθεί η αριθμητική τιμή της παράστασης:

2 2 2 2A (x 5) 4y (x 16) 4(y 3) .

Λύση:

Ξέρουμε ότι x 7y 5 άρα η μέγιστη τιμή του y είναι 0 για x 5 και η

ελάχιστη τιμή το 3 για x 16 .

Η παράσταση με βάση την ισότητα που δίνεται γίνεται

2 245y (x 10 2y)(x 22 2y) 3| y | 5 45(y 3)

3| y | 5 3| y 3| 5 .

Αφού x 22 7y 27 x 10 7y 15 .

Page 97: Εισαγωγη σε διαγωνιστικα μαθηματικα για το γυμνασιο

http://www.mathematica.gr/forum/viewtopic.php?f=109&t=30104

Επιμέλεια: xr.tsif Σελίδα 97

Αν y 0 η παράσταση παίρνει την τιμή 9 5 όπως και για y [ 3,0)

(διαχώρισα τις περιπτώσεις λόγω της απόλυτης τιμής – το y 3 δεν είναι

αρνητικό οπότε μπορούσαμε να απαλείψουμε το σύμβολο της απόλυτης τιμής).

Άρα A 9 5 .

ΘΕΜΑ 120 (ΔΗΜΗΤΡΗΣ ΙΩΑΝΝΟΥ)

Να βρεθούν οι πραγματικοί αριθμοί x,y με x y για τους οποίους ισχύει ότι:

2 1x y x y

2 .

ΥΠΟΔΕΙΞΗ:

Προσπαθήστε να καταλήξετε σε άθροισμα δύο τετραγώνων ίσο με το μηδέν.

Χρειάζεται πρώτα να προσθέσετε και να αφαιρέσετε κάτι.

Λύση:

Έχουμε

2 21 1 1 1x y x x x y 0 x x x y x y 0

4 4 4 4

2 2

1 1x x y 0

2 2

. Άρα

1x

2 και

1 1x y y

4 4 .

ΘΕΜΑ 121 (ΔΗΜΗΤΡΗΣ ΙΩΑΝΝΟΥ)

Να λυθεί το σύστημα:

2

2

2

x 8y 23 0

y 24z 272 0

z 6x 126 0

.

Λύση:

Προσθέτουμε τα τις τρεις σχέσεις κατά μέλη και έχουμε

Page 98: Εισαγωγη σε διαγωνιστικα μαθηματικα για το γυμνασιο

http://www.mathematica.gr/forum/viewtopic.php?f=109&t=30104

Επιμέλεια: xr.tsif Σελίδα 98

2 2 2(x 6x 9) (y 8y 16) (z 24y 144) 126 126 0

2 2 2(x 3) (y 4) (z 12) 0 . Επομένως x 3, y 4 , z 12 .

ΘΕΜΑ 122 (ΔΗΜΗΤΡΗΣ ΙΩΑΝΝΟΥ)

Αν οι αριθμοί x,y είναι ακέραιοι και αν ισχύει η σχέση:

2 28x 5y 4xy 4x 12y 7 0 , να βρείτε τους αριθμούς x και y .

Λύση:

Η δοσμένη γράφεται ως 2 2 2(2x+y) + (2x+1) + 4y 12y + 6 = 0 . Προσθέτοντας

και στα δύο μέλη τον αριθμό 3 η δοσμένη γράφεται ως: 2 2 2(2x+y) + (2x+1) + (2y 3) =3 . Αφού x,y Z τότε ο κάθε όρος θα ισούται με

1. Έτσι οι λύσεις είναι (x,y)=(0,1) .

ΘΕΜΑ 123 (ΔΗΜΗΤΡΗΣ ΙΩΑΝΝΟΥ)

Αν x,y 0 , δείξτε ότι: 21 1 2

(1 )(1 ) (1 )x y x y

.

Λύση:

Η δοσμένη γράφεται ως: 2

x+y+1 4(x+y)+4

xy (x+y) .

Αφού από την υπόθεση x,y 0 πολλαπλασιάζω τα δύο μέλη με το 2(x+y) xy.

Μετά από πράξεις στα δύο μέλη έχουμε ότι 3 3 2 2 2 2x +y +x +y +2x y+2xy 0 , που

πάλι προφανώς ισχύει λόγω της υπόθεσης.

Β τρόπος

Από την Cauchy – Schwarz έχουμε να αποδείξουμε ότι

Page 99: Εισαγωγη σε διαγωνιστικα μαθηματικα για το γυμνασιο

http://www.mathematica.gr/forum/viewtopic.php?f=109&t=30104

Επιμέλεια: xr.tsif Σελίδα 99

2 2

1 21 1

x yxy

.

Και αφού μιλάμε μόνο για θετικούς μπορούμε απλά να δείξουμε ότι

1 2 1 21 1

x y x yxy xy

, όπου πολλαπλασιάζοντας χιαστί έπεται το

ζητούμενο (λόγω της x y 2 xy ).

ΘΕΜΑ 124 (ΔΗΜΗΤΡΗΣ ΙΩΑΝΝΟΥ)

Δείξτε ότι δεν υπάρχει φυσικός αριθμός n , ώστε ο αριθμός:

k n 2013 n 2014 να είναι επίσης φυσικός.

Λύση:

Θέτουμε για ευκολία m n 2013 .

Έστω ότι υπάρχει τέτοιος φυσικός αριθμός n .

Υψώνοντας εις την τετάρτη και κάνοντας μερικές πράξεις καταλήγουμε στην 4k 2m 1 2 m(m 1) .

Αλλά 2 2m m(m 1) (m 1) , και άρα καταλήγουμε σε άτοπο.

Β τρόπος

Ακόμα μια λύση: 2k n 2003 n 2014 .

k N n 2003, n 2014 N .

Δηλαδή n 2003 x, n 2014 y με *x,y N .

Άρα 2 2n 2003 x ,n 2004 y (με αφαίρεση κατά μέλη)

2 21 y x (y x)(y x) .

Page 100: Εισαγωγη σε διαγωνιστικα μαθηματικα για το γυμνασιο

http://www.mathematica.gr/forum/viewtopic.php?f=109&t=30104

Επιμέλεια: xr.tsif Σελίδα 100

Είναι y x y x 0 . Άρα πρέπει y x 1 και y x 1 . Με πρόσθεση κατά

μέλη 2y 0 y 0 , αδύνατον, διότι *y N .

ΘΕΜΑ 125 (ΔΗΜΗΤΡΗΣ ΙΩΑΝΝΟΥ)

Δίνεται ότι τα σημεία: A(k m,1) και B( 3,k m) , ανήκουν στην γραφική

παράσταση της συνάρτησης 2y ax , όπου a 0 . Αν επί πλέον είναι γνωστό,

ότι τα σημεία αυτά είναι συμμετρικά ως προς τον άξονα y΄y , να βρείτε την

εξίσωση της ευθείας ( ) , που έχει κλίση a και διέρχεται από το σημείο A . Στη

συνέχεια να βρείτε το δεύτερο σημείο τομής της ευθείας αυτής με την

παραβολή.

Λύση:

α) Τα σημεία έχουν ίσες τετμημένες και αντίθετες τεταγμένες. Συνεπώς

λύνοντας το σύστημα k m 3

k m 1

, έχουμε k 2 , m 1 και

1a

9 .

Η ευθεία θα έχει εξίσωση y ax b . Αντικαθιστώντας στην εξίσωση τις

συντεταγμένες του σημείου καθώς και την τιμή του a έχουμε 2

b3

. Συνεπώς η

ευθεία είναι η 1 2

y x9 3

.

β) Θα ισχύει 2x 2 x

(x 3)(x 2) 09 3 9 , συνεπώς το δεύτερο σημείο

τομής των δύο γραφικών παραστάσεων θα έχει τετμημένη x 2 ,

αντικαθιστώντας την τιμή αυτή στην εξίσωση της παραβολής(ή της ευθείας)

έχουμε ότι 4

y9

.

Επομένως το σημείο αυτό είναι το 4

K( 2, )9

.

Page 101: Εισαγωγη σε διαγωνιστικα μαθηματικα για το γυμνασιο

http://www.mathematica.gr/forum/viewtopic.php?f=109&t=30104

Επιμέλεια: xr.tsif Σελίδα 101

ΘΕΜΑ 126 (ΔΗΜΗΤΡΗΣ ΙΩΑΝΝΟΥ)

Αν x y z 0 , να αποδείξετε ότι:

α) 2

2 2 zx y

2 .

b) 4

4 4 zx y

8 .

Λύση:

α) 2 2z (x y)

2 2

. Μένει να αποδείξουμε ότι

2

2 2 2(x y)x y (x y) 0

2

.

b) 4 2

2 2 2 2z 1 z 1( ) (x y )

8 2 2 2 .

Όμως 2 2 2 4 4 2 2 21(x y ) x y (x y ) 0

2 .

ΘΕΜΑ 127 (ΔΗΜΗΤΡΗΣ ΙΩΑΝΝΟΥ)

Αν x,y,z 0 και x y z 7 και αν:1 1 1

1x 1 y 1 z 1

, να αποδείξετε ότι:

(x y)(z 1) 12 .

Λύση:

Η δοσμένη εξίσωση γράφεται:

(y 1)(z 1) (x 1)(z 1) (x 1)(y 1) (x 1)(y 1)(z 1)

yz y z 1 xz x z 1 xy x y 1

xyz xy xz x yz y z 1 xyz x y z 2 xyz 9 (1) , αφού

x y z 7 .

Τώρα, από ΑΜ – ΓΜ παίρνω:

Page 102: Εισαγωγη σε διαγωνιστικα μαθηματικα για το γυμνασιο

http://www.mathematica.gr/forum/viewtopic.php?f=109&t=30104

Επιμέλεια: xr.tsif Σελίδα 102

x y 2 xy (2) .

z 1 2 z (3) .

Πολλαπλασιάζοντας κατά μέλη τις (2) και (3) έχω:

(1)2 2(x y)(z 1) 4 xyz (x y)(z 1) 12 a b .

Έτσι, το ζητούμενο αποδείχθηκε.

ΘΕΜΑ 128 (ΔΗΜΗΤΡΗΣ ΙΩΑΝΝΟΥ)

Αν 1 2 3 9

0 a a a ... a , να αποδείξετε ότι: 1 2 3 9

3 6 9

a a a ... a3

a a a

.

Λύση:

Πολλαπλασιάζω και τα δύο μέλη της δοσμένης με το 3 6 9

a +a +a και αφού όλα

είναι θετικά η φορά δεν αλλάζει. Η δοσμένη γράφεται ως :

1 2 4 5 7 8 3 6 9a +a +a +a +a +a 2a +2a +2a .

Εργάζομαι λέγοντας ότι 1 3

a a και 2 3

a a .

Προσθέτοντας κατά μέλη έχω 1 2 3

a a 2a . Εργαζόμενος ομοίως με τα

υπόλοιπα σετ η ανισότητα αποδεικνύεται.

ΘΕΜΑ 129 (ΔΗΜΗΤΡΗΣ ΙΩΑΝΝΟΥ)

Αν | a | 1 και | b| 1 , να αποδείξετε ότι | a b| |1 ab| .

Λύση:

2 2 2 2 2 2a b 1 ab a b 1 ab a b 2ab 1 2ab a b

2 2 2 2 2 2 2 2 2a b 1 a b 0 a 1 b 1 b 0 a 1 1 b 0 .

Αρκεί λοιπόν να αποδειχθεί η τελευταία ανισότητα.

Page 103: Εισαγωγη σε διαγωνιστικα μαθηματικα για το γυμνασιο

http://www.mathematica.gr/forum/viewtopic.php?f=109&t=30104

Επιμέλεια: xr.tsif Σελίδα 103

Έχουμε,

2 2a 1 a 1 a 1 0 .

2 2b 1 b 1 1 b 0 .

Είναι τώρα εμφανές ότι η ζητούμενη αληθεύει.

ΘΕΩΡΙΑ 1 – ΑΝΙΣΟΤΗΤΕΣ

1) 2 2 2 2 2 2 2(a b c )(x y z ) (ax by cz) για κάθε a,b,c .

(Ανισότητα Cauchy – Schwarz)

2) 2 2 2 2 2 2 2

1 2 n 1 2 n 1 1 2 2 n n(a a ... a )(b b ... b ) (a b a b ... a b ) .

(Ανισότητα Cauchy – Schwarz – Buniakowski ή B – C – S)

3) 3a b c 3· abc (Ανισότητα του Cauchy)

4) Εάν 1 2 3 n

a ,a ,a ,...,a 0 , τότε

1 2 3 n n1 2 3 n

1 2 n

a a a ... a na a a ...a

1 1 1n ...a a a

(Ανισότητα των μέσων, γνωστή ως ΑΜ – ΓΜ)

5) Αν a,b,c 0 και x,y,z , τότε 2 2 2 2x y z (x y z)

a b c a b c

(Ανισότητα Andreescu).

Page 104: Εισαγωγη σε διαγωνιστικα μαθηματικα για το γυμνασιο

http://www.mathematica.gr/forum/viewtopic.php?f=109&t=30104

Επιμέλεια: xr.tsif Σελίδα 104

ΘΕΜΑ 130 (ΔΗΜΗΤΡΗΣ ΙΩΑΝΝΟΥ)

Αν b 0,x b,y b , να αποδείξετε ότι: xy 4b (x b)(y b) .

Λύση:

xy (b x b)(b y b) 2 b(x b)·2 b(y b) 4b (x b)(y b) , από

την ανισότητα ΑΜ – ΓΜ.

Β τρόπος

Έστω x b a,y b c x a b,y b c . Άρα αρκεί να αποδείξουμε

(a b)(b c) 4b ac .

Είναι γνωστό ότι a b 2 ab και ότι b c 2 bc .

Με πολλαπλασιασμό κατά μέλη (επιτρέπεται γιατί τα μέλη είναι θετικά),

βρίσκουμε ότι (a b)(b c) 4b ac .

ΘΕΜΑ 131 (ΔΗΜΗΤΡΗΣ ΙΩΑΝΝΟΥ)

Αν x,y Z και 1 x

1 0x x y

, να βρεθούν οι αριθμοί x,y .

Λύση:

Η δοσμένη σχέση γράφεται: 2y(x 1) (2x x) (1) , όπου είναι x 0,x y .

Αν ήταν x 1 , τότε η σχέση (1) θα έδινε 0 1 , που είναι άτοπο.

Άρα x 1 , οπότε η σχέση (1) γράφεται:

2 22x x 2x 2x x 2x(x 1) x xy 2x

x 1 x 1 x 1 x 1

Page 105: Εισαγωγη σε διαγωνιστικα μαθηματικα για το γυμνασιο

http://www.mathematica.gr/forum/viewtopic.php?f=109&t=30104

Επιμέλεια: xr.tsif Σελίδα 105

x 1 1 12x 2x 1

x 1 x 1

.

Αφού x,y Z πρέπει x 1|1 . Άρα x 1 1 , δηλαδή x 0 ή x 2 . Αφού

λοιπόν x 0 , άρα x 2 , από όπου προκύπτει y 6 .

ΘΕΜΑ 132 (ΔΗΜΗΤΡΗΣ ΙΩΑΝΝΟΥ)

Αν 2013 x 2013 y 2 2013 z , να αποδείξετε ότι: x y 2z .

Λύση:

Υψώνουμε στο τετράγωνο και έχουμε

x y 4026 2 (x 2013)(y 2013) 4z 4·2013

x y 2 (x 2013)(y 2013) 4z 4026 .

Από τη γνωστή ανισότητα 2 xy x y έχουμε

2 (x 2013)(y 2013) x y 4026 .

Επομένως x y x y 4026 4026 4z 2(x y) 4z x y 2z .

ΘΕΜΑ 133 (ΔΗΜΗΤΡΗΣ ΙΩΑΝΝΟΥ)

α) Να παραγοντοποιηθεί η παράσταση: 2 2A 20x xy y .

b) Αν είναι 2x y 1 και x y 2 , να αποδείξετε ότι: A 20 .

Λύση:

α) Θεωρούμε την παράσταση ως τριώνυμο με άγνωστο το x .

Τότε 2 2

y81y (9y) ,

1,2

y 9y y yx ,

40 4 5

. Άρα

y yA 20(x )(x )

4 5 .

b) Προσθέτοντας κατά μέλη τις ανισώσεις που μας δίνονται έχουμε x 1 .

Page 106: Εισαγωγη σε διαγωνιστικα μαθηματικα για το γυμνασιο

http://www.mathematica.gr/forum/viewtopic.php?f=109&t=30104

Επιμέλεια: xr.tsif Σελίδα 106

A (4x y)(5x y) (x y) 3x (2x y) 3x (2 3)(1 3) 20 .

Για το β) λίγο διαφορετικά:

Έχει βρεθεί από το (α) ότι A (4x y)(5x y) .

Από την υπόθεση δίνεται

2x y 1,x y 2 4x 2y 2,x y 2 5x y 4 . (Πρόσθεση κατά

μέλη).

Ακόμα: 2x y 1,x y 2 2x y 1,2x 2y 4 4x y 5 . ( Πρόσθεση

κατά μέλη).

Πολλαπλασιάζουμε κατά μέλη (επιτρέπεται, διότι τα μέλη είναι θετικά) και

βρίσκουμε (5x y)(4x y) 20 A 20 .

Ακόμα ένας τρόπος για το β) που δεν απαιτείται η παραγοντοποίηση της A .

Έστω 2x y 1 a,x y 2 b . Λύνουμε το σύστημα αυτό και βρίσκουμε

a b a 2bx 1,y 1

3 3

.

Αντικαθιστούμε στην A :

2 2a b a b a 2b a 2bA 20( 1) ( 1)( 1) ( 1)

3 3 3 3

και ύστερα από τις πράξεις: 2 218a 18b 45ab 117b 126a 180

A9

2 218a 18b 45ab 117b 126a20 20

9

,

διότι εξ υποθέσεως είναι a,b 0 .

ΘΕΜΑ 134 (ΔΗΜΗΤΡΗΣ ΙΩΑΝΝΟΥ)

α) Να παραγοντοποιηθεί η παράσταση: 2 2A 9ab 15bc 27ac 9a 20c .

β) Αν a 2b c και a b 2c , δείξτε ότι A 0 .

Page 107: Εισαγωγη σε διαγωνιστικα μαθηματικα για το γυμνασιο

http://www.mathematica.gr/forum/viewtopic.php?f=109&t=30104

Επιμέλεια: xr.tsif Σελίδα 107

Λύση:

α) 2 2A 9ab 15bc 12ac 15ac 9a 20c 3a(3b 4c 3a) 5c(3b 4c 3a)

(3b 4c 3a)(3a 5c) .

β) Θέτω a 2b c k 0 και a b 2c 0 .

Άρα 2 k 5c

a3

και

k cb

3

.

Άρα A (3b 4c 3a)(3a 5c) ..... (3k 3l)(2l k) 0 .

ΘΕΜΑ 135 (ΔΗΜΗΤΡΗΣ ΙΩΑΝΝΟΥ)

Αν x 2y z και 3x 4y 2w z , να αποδείξετε ότι: 2 2 2 2x y 11z 2w 0 .

Πότε ισχύει η ισότητα;

Λύση:

Αντικατέστησα z x 2y,w 2x 3y (προέκυψε μετά από πρόσθεση κατά

μέλη): 2 2 2 2x y 11(x 2y) 2(2x 3y) ...

2 2 2 28x 25y 20xy (2x) (2x 5y) ...

με την ισότητα να ισχύει αν και μόνο αν x y 0 .

ΘΕΜΑ 136 (ΚΩΣΤΑΣ Broly)

Να λυθεί η εξίσωση : 3 3 3x 1 x x 1 0 στους πραγματικούς .

Σημείωση:

Αν ο λύτης δυσκολεύεται να την λύσει μπορεί να κοιτάξει την ταυτότητα:

3 3 3 2 2 21a b c 3abc (a b c)[(a b) (b c) (c a) ]

2 .

Page 108: Εισαγωγη σε διαγωνιστικα μαθηματικα για το γυμνασιο

http://www.mathematica.gr/forum/viewtopic.php?f=109&t=30104

Επιμέλεια: xr.tsif Σελίδα 108

Λύση:

Ξέρουμε ότι αν a b c 0 ισχύει 3 3 3a b c 3abc .

Συνεπώς

2 2 3 33 3x 1 x x 1 3 x(x 1) x x(x 1) x x x x 0 .

ΘΕΜΑ 137 (ΔΗΜΗΤΡΗΣ ΙΩΑΝΝΟΥ)

Να υπολογίσετε το άθροισμα όλων των διψήφιων αριθμών που αν διαιρεθούν

με το 4 , δίνουν υπόλοιπο 1.

Λύση:

Οι αριθμοί είναι οι 13,17,21,...,97.

Έχουμε δηλαδή μια αριθμητική πρόοδο με διαφορά 4 .

1a 13 ,

1a a

1 21 22

.

Συνεπώς 1a a

S · 55·22 12102

.

ΘΕΜΑ 138 (ΔΗΜΗΤΡΗΣ ΙΩΑΝΝΟΥ)

Να υπολογίσετε το άθροισμα των γωνιών A,B,C,D,E, του παρακάτω

σχήματος:

Λύση:

Σχημάτισα το πεντάγωνο ADBEC. Κάθε μια από τις γωνίες A,B,C,D,E,

χωρίζεται σε τρία μέρη και πηγαίνοντας με την αντίθετη φορά των δεικτών του

ωρολογίου ονομάζω τις γωνίες αυτές

1 2 3 1 2 3 1 2 3 1 2 3 1 2 3A ,A ,A ,D ,D ,D ,B ,B ,B ,E ,E ,E ,C ,C ,C .

Page 109: Εισαγωγη σε διαγωνιστικα μαθηματικα για το γυμνασιο

http://www.mathematica.gr/forum/viewtopic.php?f=109&t=30104

Επιμέλεια: xr.tsif Σελίδα 109

Από το τρίγωνο ABE , έχουμε 3 1 2 2 2

B E 180 (A B E ) .

Από το τρίγωνο CDE , o

1 3 2 2 2C E 180 (C D E ) .

Από το τρίγωνο ABC, o

1 3 2 2 2A C 180 (A B C ) .

Από το τρίγωνο ADE, o

1 3 2 2 2D A 180 (A D E ) .

Τέλος από το τρίγωνο BCD, o

1 3 2 2 2B D 180 (B C D ) .

Από το πεντάγωνο όμως ADBEC, έχουμε :

oA B C D E 540

o

1 2 3 1 2 3 1 2 3 1 2 3 1 2 3A A A B B B C C C D D D E E E 540 .

Σε αυτό το σημείο αν αντικαταστήσουμε τις παραπάνω σχέσεις στην τελευταία

, κατέληξα στην:o

2 2 2 2 2A B C D E 180 , το άθροισμα που ψάχναμε.

Β τρόπος

Φέρνω το τμήμα AZ .Τότε AZεξωτερική

του τριγώνου AZB, άρα 1AZ A B .

Ομοίως, 2AZ A E .

Άρα Z A B E .

Στο τρίγωνο Z έχουμε

oˆˆZ 180 .

Δηλαδή oˆˆA B E 180 .

Page 110: Εισαγωγη σε διαγωνιστικα μαθηματικα για το γυμνασιο

http://www.mathematica.gr/forum/viewtopic.php?f=109&t=30104

Επιμέλεια: xr.tsif Σελίδα 110

ΘΕΜΑ 139 (ΔΗΜΗΤΡΗΣ ΙΩΑΝΝΟΥ)

α) Να αποδείξετε ότι:

2 2 2 22(x y z) 6(xy yz zx) (x y) (y z) (z x) .

β) Αν οι θετικοί πραγματικοί αριθμοί x,y,z είναι τέτοιοι ώστε xy yz zx

σταθερό, τότε το άθροισμα: 2 2 2x y z γίνεται ελάχιστο, αν και μόνο αν

x y z .

γ) Από όλα τα ορθογώνια παραλληλεπίπεδα που έχουν σταθερό το άθροισμα

των τριών διαστάσεών τους, ποιο είναι εκείνο που έχει την μεγαλύτερη

επιφάνεια;

Λύση:

α) Η παράσταση γράφεται ως

2 2 22x 2y 2z 4xy 4yz 4zx 6xy 6yz 6zx

2 2 2 2 2 2(x 2xy y ) (y 2yz z ) (z 2zx x )

2 2 2(x y) (y z) (z x) .

β) Στην ανισότητα 2 2 2x y z xy yz zx , ξέρουμε ότι η ισότητα ισχύει για

x y z άρα το άθροισμα γίνεται ελάχιστο όταν οι τιμές των μεταβλητών

είναι ίσες. (μπορούμε να το δείξουμε πιο αναλυτικά με την απόδειξη της

ανισότητας ή τη χρήση του α)).

γ) Ξέρουμε(και από το α) μπορούμε να το συμπεράνουμε) ότι 2(x y z) 3(xy zx zx) με ισότητα για x y z . Αφού η επιφάνεια του

παραλληλεπιπέδου είναι ίση με 2(xy yz zx) μεγιστοποιείται (γίνεται ίση με

22(x y z)

3

) για x y z . Άρα το παραλληλεπίπεδο πρέπει να είναι κύβος.

Page 111: Εισαγωγη σε διαγωνιστικα μαθηματικα για το γυμνασιο

http://www.mathematica.gr/forum/viewtopic.php?f=109&t=30104

Επιμέλεια: xr.tsif Σελίδα 111

** Γενικά τα θεωρήματα επί των μεγίστων και των ελαχίστων έχουν

αρκετό υλικό για να εξασκηθεί κάποιος.

Γενικά πάντως ισχύουν:

α) Το γινόμενο δύο μεταβλητών, των οποίων το άθροισμα είναι σταθερό,

γίνεται μέγιστο όταν οι παράγοντες είναι ίσοι.

β) Όταν το γινόμενο δύο μεταβλητών(θετικών) είναι σταθερό, το άθροισμά

τους γίνεται ελάχιστο όταν οι παράγοντες είναι ίσοι.

Οι δύο προτάσεις ισχύουν για κάθε αριθμό μεταβλητών.

ΘΕΜΑ 140 (ΔΗΜΗΤΡΗΣ ΙΩΑΝΝΟΥ)

Αν 2(x y) 6(x y) (x y) 9 0 , να αποδειχθεί ότι: | x y | 3 .

Λύση:

Είναι, 2(x y) 6(x y) (x y) 9 0

2 2 2x y 6 x y · x y 9 x y 9 x y 0

2

2x y 3 x y 9 x y 0 .

Από την τελευταία σχέση, έπεται ότι,

x y 3 x y και x y 0 .

Έχουμε, 2 2 2x y x y 1 x y 1 και άρα

2 2x y 9 x y 9 x y 3 .

Page 112: Εισαγωγη σε διαγωνιστικα μαθηματικα για το γυμνασιο

http://www.mathematica.gr/forum/viewtopic.php?f=109&t=30104

Επιμέλεια: xr.tsif Σελίδα 112

ΘΕΜΑ 141 (ΔΗΜΗΤΡΗΣ ΙΩΑΝΝΟΥ)

Αν σε τρίγωνο AB, ισχύει: 3 4 3 4B B

2013 2013 2013 2013

,

να αποδείξετε ότι το τρίγωνο αυτό είναι ισοσκελές.

Λύση:

Θέτουμε για ευκολία: B C

a sin ,b sin2013 2013

.

Η παράσταση τώρα γίνεται : 3 2 2 3 2 2a (1 b ) b (1 a ) όπου μετά από τις

απαραίτητες πράξεις καταλήγουμε στην: 2 2 2 2 3 3(a b)(a ab b 2a b a b ) 0 .

a b μας δίνει το ζητούμενο μιας και τότε θα έχουμε B C

, δηλαδή το AB

είναι ισοσκελές.

Επίσης 2 24 (| ab | 1) (ab 1) , άρα

2 2 2 2 3 3 2 2 2 2a ab b 2a b a b (a b) ab(ab 1) (a b) 4ab (a b) 0

και τελειώσαμε.

Με την παρατήρηση ότι οι αριθμοί a,b είναι θετικοί (εφόσον οι γωνίες

B,

2013 2012

είναι οξείες), σωστά έχει αντιμετωπίσει ο Κώστας (Broly) την

άσκηση 141.

Ωστόσο, υπάρχει και άλλος τρόπος, ο οποίος μας επιτρέπει π.χ. να

αντιμετωπίσουμε και την παρακάτω άσκηση:

ΘΕΜΑ 142 (ΔΗΜΗΤΡΗΣ ΙΩΑΝΝΟΥ)

Αν B,

είναι γωνίες τριγώνου και αν:

n m n mB B

2013 2013 2013 2013

, (με m,n N ), δείξτε ότι το

Page 113: Εισαγωγη σε διαγωνιστικα μαθηματικα για το γυμνασιο

http://www.mathematica.gr/forum/viewtopic.php?f=109&t=30104

Επιμέλεια: xr.tsif Σελίδα 113

τρίγωνο AB είναι ισοσκελές.

Λύση:

Η ισότητα μπορεί να γραφεί ως εξής :

n m

B B

2013 2013C C

2013 2013

.

Ας δεχθούμε αρχικά ότι B

.

Τότε επειδή έχουμε να κάνουμε με οξείες γωνίες : B

2013 2013

, τότε έχουμε

B

2013 2013

και

B

2013 2013

, άρα ο λόγος που βρίσκεται στο

αριστερό μέλος της παραπάνω ισότητας είναι μικρότερος του 1 με τα ημίτονα

ενώ ο λόγος στο δεξί μέλος με τα συνημίτονα είναι μεγαλύτερος του 1.

Παρόμοια καταλήγουμε σε άτοπο και για την περίπτωση B

.

Άρα B

και το τρίγωνο είναι ισοσκελές.

ΘΕΜΑ 143 (Broly)

Να βρεθεί το 2 2minmax(a b,b a) , όπου a,b πραγματικοί αριθμοί.

Λύση:

Είναι x y | x y|

max{x,y}2

.

Επομένως θέλουμε να ελαχιστοποιηθεί η παράσταση

2 2 2 2a +b +a+b+|a +b b a|f (a,b)=

2

.

Επειδή 2 1

a +a4

, είναι φανερό ότι 1

minf(a,b)=4

όταν 1

a b2

.

Page 114: Εισαγωγη σε διαγωνιστικα μαθηματικα για το γυμνασιο

http://www.mathematica.gr/forum/viewtopic.php?f=109&t=30104

Επιμέλεια: xr.tsif Σελίδα 114

Β τρόπος

α) Παίρνω πρώτα την περίπτωση να είναι 2 2a b b a (a b)(a b 1) 0

(a b 0 και a b 1 0 ) ή (a b 0 και a b 1 0 ).

Τότε όπως παραπάνω έχω βρει, το ελάχιστο που ζητείται είναι το 3

4.

b) Ομοίως και όταν 2 2b a a b , το ελάχιστο είναι το 3

4.

c) Aν είναι, 2 2a b b a , τότε (a b)(a b 1) 0 a b ή a b 1 .

Aν a b τότε ζητείται το 2min(a a) , το οποίο επιτυγχάνεται για

1a

2 ,

δηλαδή για 1

a b2

και το ελάχιστο είναι το 2 1a a

4 .

Αν a b 1 τότε b 1 a και ζητείται το ελάχιστο της 2a b δηλαδή της

2a a 1 , το οποίο επιτυγχάνεται όταν 1

a2

και είναι το 3

4.

Από τις παραπάνω περιπτώσεις, βγαίνει το συμπέρασμα ότι το ελάχιστο που

ζητείται, είναι το 1

4 .

ΘΕΜΑ 144 (ΔΗΜΗΤΡΗΣ ΙΩΑΝΝΟΥ)

Δίνονται τα σύνολα των διαδοχικών φυσικών αριθμών:

1 2 3 4A {1},A {2,3},A {4,5,6},A {7,8,9,10},... . Να βρεθεί το άθροισμα

των στοιχείων του συνόλου 2013

A .

Λύση:

Τα πρώτα 2012 σύνολα περιέχουν

Page 115: Εισαγωγη σε διαγωνιστικα μαθηματικα για το γυμνασιο

http://www.mathematica.gr/forum/viewtopic.php?f=109&t=30104

Επιμέλεια: xr.tsif Σελίδα 115

n(n 1) 2012·20131 2 3 4 ... 2012 1006·2013 2.025.078

2 2

στοιχεία.

Άρα το σύνολο 2013

A περιέχει 2013 στοιχεία τα οποία είναι τα

2.025.079,2.025.080,.....,2.027.091.

Οι αριθμοί αυτοί αποτελούν αριθμητική πρόοδο με

1 na 2.025.079, a 2.027.091, n 2013, 1 .

Άρα το άθροισμά τους είναι ίσο με

1 n

2013

2013(a a ) 2013·4.052.170S 4.078.509.105

2 2

.

ΘΕΜΑ 145 (ΚΛΕΟΒΟΥΛΟΣ ΚΟΦΟΝΙΚΟΛΑΣ)

Έστω δύο παράλληλες ευθείες 1

( ) και 2

( ) . Στην ευθεία 1

( ) ανήκουνε 10

σημεία, ενώ στην ευθεία 2

( ) ανήκουνε 11 σημεία.

Πόσα τρίγωνα μπορούμε να σχηματίσουμε συνολικά;

Λύση:

Βρίσκουμε το πλήθος ευθυγράμμων τμημάτων σε κάθε ευθεία και

πολλαπλασιάζουμε το αποτέλεσμα με τον αριθμό σημείων της απέναντι

ευθείας. Δηλαδή στην πρώτη ευθεία υπάρχουν 10

2

ευθύγραμμα τμήματα ενώ

στην δεύτερη 11

2

ευθύγραμμα τμήματα. Δουλεύοντας όπως περιέγραψα

παραπάνω, σχηματίζονται 10

112

τρίγωνα με τις βάσεις τους στην πρώτη

ευθεία ενώ σχηματίζονται 11

102

τρίγωνα με τις βάσεις τους στην δεύτερη

ευθεία. Συνολικά 10 11

11 102 2

τρίγωνα.

Page 116: Εισαγωγη σε διαγωνιστικα μαθηματικα για το γυμνασιο

http://www.mathematica.gr/forum/viewtopic.php?f=109&t=30104

Επιμέλεια: xr.tsif Σελίδα 116

ΘΕΜΑ 146 (ΚΛΕΟΒΟΥΛΟΣ ΚΟΦΟΝΙΚΟΛΑΣ)

Έστω τρίγωνοABC και το ορθόκεντρό του. Παίρνουμε το συμμετρικό του

ως προς BC το οποίο ονομάζουμε H

K . Να αποδειχθεί ότι τα σημεία

HA,B,C,K είναι ομοκυκλικά.

Λύση:

Αρκεί να αποδείξουμε ότι το τετράπλευρο H

ABCK είναι εγγράψιμο.

Έστω το σημείο τομής της AHμε την BC.

Το τρίγωνο H

CHK , έχει την CMως ύψος και ως διάμεσος, άρα και ως

διχοτόμος. Οπότε

H H HHCM HCK CHM CK M 90 HCM 90 HCK

.

Η γωνία όμως ABC

έχει κάθετες πλευρές με την CHM

, άρα

HABC CHM CK M

, και το τετράπλευρο είναι εγγράψιμο.

Β τρόπος

Δίνω μία πιο απλή προσέγγιση.

3 2AH H H,

1 3BH HH ,

2 1HH CH εγγράψιμα τετράπλευρα (

1 2 3H ,H ,H ύψη).

HHBC BK C και oK H 180 A

και αποδείχθηκε ότι H

ABK C

εγγράψιμο άρα τα σημεία H

A,B,C,K είναι ομοκυκλικά.

ΘΕΜΑ 147 (ΚΛΕΟΒΟΥΛΟΣ ΚΟΦΟΝΙΚΟΛΑΣ)

Σ' ένα χωριό οι γυναίκες βάφουν τα νύχια τους κόκκινα όλες με διαφορετικό

τρόπο, χωρίς να υπάρχει γυναίκα με όλα τα νύχια άβαφα. Ποιος είναι ο αριθμός

γυναικών στο χωριό;

Page 117: Εισαγωγη σε διαγωνιστικα μαθηματικα για το γυμνασιο

http://www.mathematica.gr/forum/viewtopic.php?f=109&t=30104

Επιμέλεια: xr.tsif Σελίδα 117

Λύση:

Από την πολλαπλασιαστική αρχή είναι 102·2·2·...·2 2 . Όμως ένας συνδυασμός

περιλαμβάνει δέκα άβαφα νύχια άρα σύνολο 102 1 1024 1 1023

ΘΕΜΑ 148 (ΚΛΕΟΒΟΥΛΟΣ ΚΟΦΟΝΙΚΟΛΑΣ)

Αν abc=1, +a,b,c τότε να αποδειχθεί ότι 2 2 2a +b +c 3 . Για ποιες τιμές των

a,b,c αληθεύει η ισότητα;

Λύση:

Από ΑΜ – ΓΜ ισχύει ότι 2 2 2 23a b c 3 (abc) 3 .

Η ισότητα ισχύει για a=b=c=1.

ΘΕΜΑ 149 (ΔΗΜΗΤΡΗΣ ΙΩΑΝΝΟΥ)

Θεωρούμε το πολυώνυμο: 3 2P(x) 4x bx 4a 1 , όπου a,b R και

1 a 1 . Δίνεται επί πλέον ότι το P(x) έχει παράγοντα το x 1 .

α) Να εκφράσετε το b σαν συνάρτηση του a .

β) Να βρείτε την ελάχιστη τιμή του b .

γ) Για την ελάχιστη τιμή του b που βρήκατε παραπάνω, να λύσετε την

εξίσωση: P(x) 0 .

δ) Να βρείτε όλα τα ζεύγη (a,b) με b ακέραιο, για τα οποία η εξίσωση

P(x) 0 , έχει και άλλες ακέραιες ρίζες (εκτός την x 1 ), τις οποίες και να

βρείτε.

Λύση:

α) Αρχικά δοκίμασα με Vieta να βρω την σχέση μεταξύ a,b αλλά τελικά

έβγαινε πιο απλά.

Page 118: Εισαγωγη σε διαγωνιστικα μαθηματικα για το γυμνασιο

http://www.mathematica.gr/forum/viewtopic.php?f=109&t=30104

Επιμέλεια: xr.tsif Σελίδα 118

Έστω λοιπόν ότι 3 2 2P(x) 4x bx 4a 1 4(x 1)(cx dx e) όπου c,d,e

πραγματικοί αριθμοί.

Κάνοντας τις πράξεις και εξισώνοντας τους συντελεστές καταλήγουμε στις

παρακάτω σχέσεις :

2c 1,4(d c) 0,4(e d) b, 4e 4a 1

2c 1,d c,b 4 4e,b 4a 3 .

Β τρόπος

Πιο απλά, P(1) 0 οπότε 24 b 4a 1 0 και λοιπά.

β) 2b 4a 3 3 , άρα η ελάχιστη τιμή του b είναι 3 και επιτυγχάνεται όταν

a 0 .

γ) Για a 0 , b 3 , η εξίσωση γράφεται: 34x 3x 1 0 .

Όμως : 3 34x 3x 1 4x 4x x 1 4x(x 1)(x 1) (x 1)

2 2(x 1)(4x 4x 1) (x 1)(2x 1) ,

άρα οι λύσεις της εξίσωσης είναι : 1

x 1,x2

.

δ) Από την σχέση 2b 4a 3 , η εξίσωση γράφεται: 34x bx b 4 0 .

Όμως 3 2 24x bx 4 4(x 1)(x x 1) b(x 1) (x 1)(4x 4x 4 b) .

Επειδή b Z , θα πρέπει το πολυώνυμο 2Q(x) 4x 4x 4 b 0 , να έχει

διακρίνουσα τέλειο τετράγωνο.

Έχουμε 2 2 2

b4 4 (4 b) 4 (b 3) . Άρα θα πρέπει το b 3 να είναι τέλειο

τετράγωνο. Όμως 2 24a b 3 (2a) b 3 ,

άρα και ο 2a πρέπει να είναι ακέραιος.

Από την σχέση 2b 4a 3 , μπορούμε να πάμε καλύτερα μέσω a , όπου τότε η

Page 119: Εισαγωγη σε διαγωνιστικα μαθηματικα για το γυμνασιο

http://www.mathematica.gr/forum/viewtopic.php?f=109&t=30104

Επιμέλεια: xr.tsif Σελίδα 119

εξίσωση γράφεται : 2 24x 4x 4a 1 0 , με διακρίνουσα 2

a(8a) , από όπου

παίρνουμε τις ρίζες 1 2

1 1x a ,x a

2 2 .

Για να έχουμε ακέραιες ρίζες μιας και 1 a 1 , θα πρέπει το a να παίρνει τις

τιμές 1 1

,2 2 , που συμβαδίζει και με το γεγονός ότι ο 2a είναι ακέραιος και

συνεπώς ο a ρητός.

Για 1 1

a ,2 2

, έχουμε 2b 4a 3 1 3 4 , και η αρχική εξίσωση τώρα

γράφεται :

3P(x) 4x 4x 0 x(x 1)(x 1) 0 . Άρα έχουμε τις ακέραιες ρίζες

1,0,1 οι οποίες επιτυγχάνονται για τα ζεύγη 1 1

(a,b) ( ,4),( ,4)2 2

.

ΘΕΜΑ 150 (ΔΗΜΗΤΡΗΣ ΙΩΑΝΝΟΥ)

Δίνεται η εξίσωση 2ax bx c 0 , όπου οι αριθμοί a,b,c είναι περιττοί

ακέραιοι. Να αποδείξετε ότι η εξίσωση αυτή, δεν έχει ακέραια ρίζα.

Λύση:

Θα αποδείξω κάτι ισχυρότερο. Η εξίσωση αυτή δεν έχει ρητή ρίζα.

Θα υποθέσω ότι y

xz

είναι μία ρητή ρίζα της δοσμένης, με (y,z) 1 . Τότε θα

ισχύει 2 2ay byz cz 0 . Αφού (y,z) 1 , τότε δεν μπορεί και οι δύο να είναι

άρτιοι.

Παίρνουμε τις ακόλουθες περιπτώσεις:

Να είναι και οι δύο περιττοί.

2 2ay byz cz 0 που είναι άτοπο διότι 2 2ay byz cz περιττός και 0 άρτιος.

Να είναι ο z άρτιος και ο y περιττός,

Page 120: Εισαγωγη σε διαγωνιστικα μαθηματικα για το γυμνασιο

http://www.mathematica.gr/forum/viewtopic.php?f=109&t=30104

Επιμέλεια: xr.tsif Σελίδα 120

τότε 2ay περιττός και

2byz,cz άρτιοι.

Από τη θεωρία αριθμών άρτιος + περιττός = περιττός. Άτοπο.

όμοια και αν ο y άρτιος και ο z περιττός.

Άρα η δοσμένη δεν έχει ρητές (συνεπώς και όχι ακέραιες που είναι υποσύνολο

των ρητών αριθμών) ρίζες.

ΘΕΜΑ 151 (ΔΗΜΗΤΡΗΣ ΙΩΑΝΝΟΥ)

Αν x,y N , να αποδείξετε ότι η εξίσωση: x x x y , δεν έχει άλλη

ακέραια λύση, εκτός από την (x,y) (0,0) .

Λύση:

Ύστερα από 2 τετραγωνισμούς, προκύπτει 2x m ,m N και 2 2m m , N .

Όμως 2 2 2m m m (m ) για m 0 .

ΘΕΜΑ 152 (ΔΗΜΗΤΡΗΣ ΙΩΑΝΝΟΥ)

Να λυθεί η εξίσωση: 2 2 2(x 2x) (x 1) 55 .

Λύση:

Για x R έχουμε,

22 2 2 2 2(x 2x) (x 1) 55 x 2x x 2x 1 55 0

2

22 2 2 1 1x 2x x 2x 56 0 x 2x 56 0

2 4

2

2 2 21 225 1 15 1 15x 2x x 2x x 2x

2 4 2 2 2 2

.

2 22 2x 2x 8 x 2x 7 x 1 9 x 1 6

Page 121: Εισαγωγη σε διαγωνιστικα μαθηματικα για το γυμνασιο

http://www.mathematica.gr/forum/viewtopic.php?f=109&t=30104

Επιμέλεια: xr.tsif Σελίδα 121

x 2 x 4 x x 2 x 4 .

Β τρόπος

Μια λίγο πιο διαφορετική λύση

2 2 2 2 2(x 2x) (x 1) [x(x 2)] (x 1) . Θέτουμε u x 1 .

Οπότε η εξίσωση γράφεται

2 2 2 2 2 4 2[(u 1)(u 1)] u 55 (u 1) u u 3u 54 0 ,

με 2225 15 και καταλήγουμε πάλι ότι 2u 9... .

ΘΕΜΑ 153 (ΔΗΜΗΤΡΗΣ ΙΩΑΝΝΟΥ)

Να λυθεί η εξίσωση: (5 x) 5 x (x 3) x 3 2( 5 x x 3) .

Λύση:

Εννοείται ότι 5 x 3 .

Η ισότητα γράφεται

3 3(5 x) (x 3) 2( 5 x x 3)

( 5 x x 3)(5 x (5 x)(x 3) x 3) 2( 5 x x 3) .

Πολλαπλασιάζουμε με τη συζυγή του δεύτερου παράγοντα του δεύτερου

μέλους και έχουμε ( (x 3)(5 x) 1)( (x 3)(5 x) 2) (8 2x) .

LHS 2 και RHS 2 . Οπότε καταλαβαίνουμε ότι 8 2x 2 x 3 .

ΘΕΜΑ 154 (ΔΗΜΗΤΡΗΣ ΙΩΑΝΝΟΥ)

Αν a,b 0 να αποδείξετε ότι: 10 10 11a b(1 ) (1 ) 2

b a .

Όσοι μπορείτε, αποδείξτε και την γενίκευση:

Page 122: Εισαγωγη σε διαγωνιστικα μαθηματικα για το γυμνασιο

http://www.mathematica.gr/forum/viewtopic.php?f=109&t=30104

Επιμέλεια: xr.tsif Σελίδα 122

n n n 1a b(1 ) (1 ) 2

b a

, για κάθε n N .

Λύση:

Για την γενίκευση, θα εργαστούμε με Αρχή Μαθηματικής Επαγωγής.

Θα χρειαστούμε επίσης και την βασική ανισότητα 1

x 2,x 0x

.

Έστω a,b 0 .

i) n 1 ,

Είναι, 1 1a b a b1 1 2 2 2 4 2

b a b a

.

ii) n 2 ,

2 2 2 2

2 1

2 2

a b a b a b1 1 2 2 2 2 2·2 2

b a b a b a

.

iii) Έστω ότι

n n

n 1a b1 1 2 ,n 2

b a

.

iv) Θέτουμε όπου n n 1 ,

n 1 n 1 n n 1

a b a a b b1 1 1 1 1 1

b a b b a a

n 1 k n 1 k

n 1 n 1n 1 n 2

k 0 k 0

n 1 n 1a b2 2 1 1 2

k kb a

.

Συνεπώς, από Αρχή Μαθηματικής Επαγωγής, είναι

n n

n 1a b1 1 2 n N

a,

b

.

Για n 10 έχουμε και την πρώτη ανισότητα.

Page 123: Εισαγωγη σε διαγωνιστικα μαθηματικα για το γυμνασιο

http://www.mathematica.gr/forum/viewtopic.php?f=109&t=30104

Επιμέλεια: xr.tsif Σελίδα 123

Β τρόπος για το α

Θα δώσω και μια λύση για το (α) χωρίς επαγωγή, και έτσι θα δοθεί και ο

δρόμος για την απόδειξη της γενίκευσης επίσης χωρίς επαγωγή:

Θα χρησιμοποιηθούν δύο βασικές και χιλιοειπωμένες ανισότητες:

α) 2 2x y 2xy για κάθε x,y R .

b) x y

2y x , για κάθε x,y ομόσημους.

Έχουμε λοιπόν:

10 10 5 2 5 2 5 5a b a b a b(1 ) (1 ) [(1 ) ] [(1 ) ] 2(1 ) (1 )

b a b a b a

5 5 5 5a b b a a b2[(1 )(1 )] 2(1 1) 2(2 ) 2(2 2)

b a a b b a

5 10 112 4 2 2 2 .

Άλλος τρόπος

α) Το βρήκα διαφορετικά:

Από την ανισότητα αριθμητικού – γεωμετρικού μέσου η δοσμένη γράφεται ως

10 10 5a b a b(1 ) (1 ) 2[(1 )(1 )]

b a b a ,

όμως επειδή a b

2b a και μετά από κατάλληλες πράξεις έχουμε το ζητούμενο.

β) Για τη γενίκευση θα θέσω όπου είχα πριν το 5 το n

2 και με την ίδια μέθοδο

βγαίνει.

Page 124: Εισαγωγη σε διαγωνιστικα μαθηματικα για το γυμνασιο

http://www.mathematica.gr/forum/viewtopic.php?f=109&t=30104

Επιμέλεια: xr.tsif Σελίδα 124

ΘΕΜΑ 155 (ΔΗΜΗΤΡΗΣ ΙΩΑΝΝΟΥ)

Αν x,y,z 0 και αν xyz 1 και 1 1 1

x y zx y z

, να αποδείξετε ότι οι

αριθμοί x,y,z είναι διάφοροι της μονάδας.

Λύση:

Έστω ότι x 1 . Θα είναι 1 1

y z y z yz(y z)y z

. Όμως έχουμε

yz 1 άρα yz(y z) y z , άρα καταλήγουμε σε άτοπο.

Με τον ίδιο τρόπο κανένας εκ των τριών αριθμών δε γίνεται να είναι ίσος της

μονάδας.

ΘΕΜΑ 156 (ΔΗΜΗΤΡΗΣ ΙΩΑΝΝΟΥ)

Αν η ισότητα: 2n 2 2n 2

n n n

(sinx) (cosx) 1

a b (a b)

, όπου a,b 0 , αληθεύει για

n 1 , να αποδείξετε ότι θα αληθεύει και για κάθε *n N .

Λύση:

Αφού ισχύει για n 1 έχουμε 4 4sin x cos x 1

a b a b

(1) .

Όμως, από την ανισότητα Cauchy – Schwarz ισχύει

4 4 2 2 2sin x cos x (sin x cos x) 1

a b a b a b

.

Επομένως, η (1) μας λέει ότι ισχύει η ισότητα στην Cauchy – Schwarz. Αυτό

συμβαίνει αν και μόνο αν

2 2 2 2 2 2sin x cos x sin x cos x sin x cos x 1

a b a b a b a b

Page 125: Εισαγωγη σε διαγωνιστικα μαθηματικα για το γυμνασιο

http://www.mathematica.gr/forum/viewtopic.php?f=109&t=30104

Επιμέλεια: xr.tsif Σελίδα 125

2 2a bsin x ,cos x

a b a b

.

Τότε, με απλή αντικατάσταση, διαπιστώνουμε ότι η αρχική ισχύει για

κάθε n 1 .

Β τρόπος

Για n 1 , έχουμε 4 4sin x cos x 1

a b a b

4 4 2 2 4b(a b)sin x a(a b)cos x ab b(a b)(1 cos x) a(a b)cos x ab

4 2 4b(a b)(1 cos x 2cos x) a(a b)cos x ab

4 2 4b(a b) b(a b)cos x 2b(a b)cos x a(a b)cos x ab

4 2[b(a b) a(a b)]cos x 2b(a b)cos x b(a b) ab 0

2 4 2 2 2 2 b(a b) cos x 2b(a b) b 0 [(a b)cos x b] 0 cos x

a b

.

Kαι αφού 2 2 2 b asin x 1 cos x sin x 1

a b a b

.

Στη συνέχεια, όπως πιο πάνω έχει γράψει ο Θάνος, με απλή αντικατάσταση,

έχουμε:

n 1 n 1

2n 2 2n 2 2 n 1 2 n 1 n 1 n 1

n n n n n n

a b

(sinx) (cosx) (sin x) (cos x) (a b) (a b)

a b a b a b

n 1 n 1 n 1 n

a b a b 1

(a b) (a b) (a b) (a b)

.

ΘΕΜΑ 157 (ΔΗΜΗΤΡΗΣ ΙΩΑΝΝΟΥ)

Να εξετάσετε αν υπάρχουν ακέραιοι x,yτέτοιοι ώστε να είναι: 2 22x y 220 .

Page 126: Εισαγωγη σε διαγωνιστικα μαθηματικα για το γυμνασιο

http://www.mathematica.gr/forum/viewtopic.php?f=109&t=30104

Επιμέλεια: xr.tsif Σελίδα 126

Λύση:

Η δοθείσα εξίσωση γράφεται 2 2 2 2y 220 2x y 2(110 x ) (1) .

H σχέση (1) μας δίνει ότι ο 2y είναι άρτιος.

Άρα, θα είναι άρτιο τέλειο τετράγωνο μικρότερο από το 220 . Τα τέλεια

τετράγωνα τα οποία είναι άρτια κάτω από το 220 είναι

4,16,36,64,100,144,196 .

Διακρίνουμε τις περιπτώσεις:

22 24 220 2x 4 x 1 8y 0 , άτοπο.

22 216 220 2x 16 1 2y x 0 , άτοπο.

22 236 220 2x 36 x 2y 9 , άτοπο.

22 264 220 2x 64 x 8y 7 , άτοπο.

22 2100 220 2x 100 x 6y 0 , άτοπο.

22 2144 220 2x 144 x 3y 8 , άτοπο.

22 2196 220 2x 196 x 2y 11 , άτοπο.

Παρατηρούμε ότι σε όλες τις παραπάνω περιπτώσεις παίρνουμε άτοπο και άρα

δεν υπάρχουν ακέραιοι x,yπου να ικανοποιούν την εξίσωση της υπόθεσης.

Β τρόπος

Αν x,y Z με 2 22x y 220 , τότε επειδή το τετράγωνο ενός ακεραίου είναι

ισοδύναμο με 0 ή 1 mod4 , κι αφού 220 0 mod4 , θα είναι

2 2x y 0 mod4 , όποτε οι x,y είναι και οι δυο άρτιοι.

Συνεπώς, 2 2

1 12x y 55 , όπου

1

xx

2 και

1

yy

2 .

Δουλεύοντας mod3 βλέπουμε ότι αφού 55 1 mod3 και 1

y περιττός,

θα πρέπει 2

1x 0 mod3 , κι άρα 1

x 0 mod3 .

Page 127: Εισαγωγη σε διαγωνιστικα μαθηματικα για το γυμνασιο

http://www.mathematica.gr/forum/viewtopic.php?f=109&t=30104

Επιμέλεια: xr.tsif Σελίδα 127

Αν 1 2

x 3x η εξίσωση γίνεται 2 2

2 118x y 55 , οπότε 2

2

55 1x 3 4

18 18 .

Εξετάζοντας τις δυο περιπτώσεις όπου 2

2x 0 και

2

2x 1 , αντίστοιχα, βλέπουμε

ότι η εξίσωση είναι αδύνατη.

Γ τρόπος

Από την υπόθεση, είναι φανερό ότι ο y είναι άρτιος. Άρα υπάρχει k Z ώστε

y 2k . Έτσι η εξίσωση γράφεται:

2 2 2 22x 4k 220 x 2k 110 x άρτιος. Άρα υπάρχει 1

k Z ώστε

1x 2k .

Άρα 2 2 2 2

1 14k 2k 110 2k k 55 k περιττός. Άρα υπάρχει

2k Z ώστε

2k 2k 1 .

Άρα 2 2 2 2 2 2

1 2 1 2 2 1 2 22k (2k 1) 55 2k 4k 4k 54 k 2k 2k 27 .

Άρα 1

k περιττός. Άρα υπάρχει 3

k Z ώστε 1 3

k 2k 1 .

Άρα 2 2 2 2 2

3 2 2 3 3 2 2(2k 1) 2k 2k 27 4k 4k 2k 2k 26

2 2 2

3 3 2 2 3 3 2 22k 2k k k 13 2k 2k k (k 1) 13 .

Η εξίσωση αυτή είναι αδύνατη, διότι το πρώτο της μέλος είναι άρτιος (αφού

2 2k (k 1) άρτιος) και το δεύτερο περιττός.

ΘΕΜΑ 158 (ΔΗΜΗΤΡΗΣ ΙΩΑΝΝΟΥ)

Θεωρούμε την εξίσωση: 2x (k 2)x k 2 0 , με k 2 .

α) Να αποδείξετε ότι έχει δύο ρίζες πραγματικές και άνισες.

β) Αν m,n είναι οι ρίζες της πιο πάνω εξίσωσης, να αποδείξετε ότι: 3 3 3 3m n m n 0 .

Page 128: Εισαγωγη σε διαγωνιστικα μαθηματικα για το γυμνασιο

http://www.mathematica.gr/forum/viewtopic.php?f=109&t=30104

Επιμέλεια: xr.tsif Σελίδα 128

Λύση:

α) 2 2(k 2) 4( k 2) k 4 (k 2)(k 2) 0 , αφού k 2 , και άρα η

εξίσωση έχει 2 ρίζες πραγματικές και άνισες.

β) Από Vieta , S m n k 2,P mn 2 k .

Άρα 3 3 3 2 3m n (mn) (m n)[(m n) 3mn] (mn)

2 3 2(k 2)[(k 2) 3(2 k)] (2 k) ... 3(k 2) 0 .

ΘΕΜΑ 159 (ΔΗΜΗΤΡΗΣ ΙΩΑΝΝΟΥ)

Δίνεται το πολυώνυμο 3 2P(x) x 5x 8x m , m R και k Z .

Αν γνωρίζουμε ότι το P(x) έχει για παράγοντα το 2(x k) ,

α) Να αποδείξετε ότι ο k είναι ρίζα της εξίσωσης 23x 10x 8 0 .

b) Να δείξετε ότι ο αριθμός: 100 100

50

m k

m 1

, είναι τέλειο τετράγωνο ακεραίου.

Λύση:

α) Εφ' όσον το πολυώνυμο P(x) έχει για παράγοντα το 2(x k) ,

υπάρχει πολυώνυμο Q(x) ax b,a 0,b R ώστε 2

P(x) x k ax b .

Τότε, ισχύει ότι

3 2 3 2 2 2x 5x 8x m ax b 2ak x ak 2bk x bk και συνεπώς

2

2

a 1

b 2k 5

k 2bk 8

bk m

.

Από την δεύτερη εξίσωση έχουμε b 2k 5 και έτσι η τρίτη δίνει

Page 129: Εισαγωγη σε διαγωνιστικα μαθηματικα για το γυμνασιο

http://www.mathematica.gr/forum/viewtopic.php?f=109&t=30104

Επιμέλεια: xr.tsif Σελίδα 129

2 2 2 2k 2 2k 5 k 8 0 k 4k 10k 8 0 3k 10k 8 0

23k 10k 8 0 , γεγονός που αποδεικνύει ότι ο αριθμός k k είναι ρίζα της

εξίσωσης 23x 10x 8 0 .

β) Για x R είναι,

2

2 2 10 8 5 1 43x 10x 8 0 x x x x x 2

3 3 3 9 3

.

Επειδή k Z , συνάγεται ότι k 2 .

Για k 2 παίρνουμε b 2k 5 4 5 1 και 2m bk 4 .

Έτσι,

100 100100 100 100 100

2100 50

50 50 100

2 2 1m k 4 22 2

m 1 4 1 2 1

, με

502 N .

Β τρόπος

α) Το πολυώνυμο έχει διπλή λύση τον αριθμό k . Ως πολυώνυμο τρίτου βαθμού

θα έχει μια ακόμη λύση. Ας την ονομάσουμε .

Χρησιμοποιούμε τύπους Vieta και έχουμε 2

2k 5

2k k 8

, απ' όπου λαμβάνουμε

23k 10k 8 0 κι έτσι αποδείξαμε το ζητούμενο.

β) Βρίσκουμε ότι k 2 , m 4 . Συνεπώς πρέπει να αποδείξουμε ότι ο αριθμός

100 100

50

4 2A

4 1

είναι τέλειο τετράγωνο ακεραίου.

100 100

100 50 2

100

2 (2 1)A 2 (2 )

2 1

.

Σημείωση:

Αν δεν ξέρει κάποιος τύπους Vieta μπορεί να πει ότι 2P(x) (x k) (x ) και

να κάνει τις πράξεις. Θα καταλήξει στις ίδιες σχέσεις.

Page 130: Εισαγωγη σε διαγωνιστικα μαθηματικα για το γυμνασιο

http://www.mathematica.gr/forum/viewtopic.php?f=109&t=30104

Επιμέλεια: xr.tsif Σελίδα 130

ΘΕΜΑ 160 (ΔΗΜΗΤΡΗΣ ΙΩΑΝΝΟΥ)

Να λυθεί η εξίσωση: 2 3 27(x 1) 11(x 1) 4(x x 1) 0 .

Λύση:

Μιας και δεν αναφέρεται το σύνολο στο οποίο πρέπει να λυθεί θα υποθέσω ότι

είναι το R .

2 3 27(x 1) 11(x 1) 4(x x 1) 0

2 2 27(x 1) 11(x 1)(x x 1) 4(x x 1) 0 2 2 2 27(x 1) 7(x 1)(x x 1) 4(x 1)(x x 1) 4(x x 1) 0

2 2 27(x 1)(x 2x) 4(x x 1)x 0 7x(x 1)(x 2) 4(x x 1)x 0 2x[7(x 1)(x 2) 4(x x 1)] 0 . Άρα προκύπτει ότι x 0 ή

2 2 27(x x 2) 4(x x 1) 0 11x 11x 10 0

1x (11 561)

22 ή

1x (11 561)

22 .

ΘΕΜΑ 161 (ΔΗΜΗΤΡΗΣ ΙΩΑΝΝΟΥ)

Να βρεθεί ο αριθμός a , ώστε η ελάχιστη τιμή της παράστασης:

2 2y a(1 x) (1 a)x να γίνει η μέγιστη δυνατή.

Λύση:

Κάνοντας πράξεις έχω:

22 2 2 2y x 1 2x a y x 2ax a x a a a a a .

Επομένως η ελάχιστη τιμή της παράστασης y είναι 2a a και λαμβάνεται για

x a . Τώρα έχω

2

2 1 1 1a a a

2 4 4

δηλ έχει μέγιστη τιμή

1

4 και την

"πιάνει" για 1

a2

.

Page 131: Εισαγωγη σε διαγωνιστικα μαθηματικα για το γυμνασιο

http://www.mathematica.gr/forum/viewtopic.php?f=109&t=30104

Επιμέλεια: xr.tsif Σελίδα 131

ΘΕΜΑ 162 (ΔΗΜΗΤΡΗΣ ΙΩΑΝΝΟΥ)

Αν a,b,c,d R και αν abcd 1 , να αποδείξετε ότι: 2 2 2 2(a c )(b d ) 4 .

Λύση:

Βγαίνει και με Cauchy – Schwarz αλλά και με AM – GM.

2 2 2 2 2 2 2 2 2 2 2 2 2 2 2 2 44 4LHS a b a d b c c d 4 (a b )(a d )(b c )(c d ) 4 (abcd)

4| abcd| 4 .

Β τρόπος

Αν δεν κάνω λάθος στην παραπάνω έγινε χρήση της ΑΜ – ΓΜ.

Δίνω λοιπόν λύση με την Cauchy – Schwarz.

2 2 2 2 2(a b )(c d ) (ac bd) 4 .

ΠΕΡΙΠΤΩΣΗ 1η : Αν ac,bd R ,

το δεύτερο μέλος είναι μεγαλύτερο ή ίσο του 4διότι ac bd 2 ως άθροισμα

αντιστρόφων και 22 4 .

ΠΕΡΙΠΤΩΣΗ 2η : Αν ac,bd R ,

το δεύτερο μέλος είναι 4 διότι ac bd 2 ως άθροισμα αρνητικών

αντιστρόφων και 2( 2) 4 .

ΘΕΜΑ 163 (ΔΗΜΗΤΡΗΣ ΙΩΑΝΝΟΥ)

Αν z 0 , (a x)(a y) 0 και αν x y z

a2

, να αποδείξετε ότι x a και

y a , (όπου x,y,z,a R ).

Λύση:

Υποθέτουμε ότι x a και y a .

Page 132: Εισαγωγη σε διαγωνιστικα μαθηματικα για το γυμνασιο

http://www.mathematica.gr/forum/viewtopic.php?f=109&t=30104

Επιμέλεια: xr.tsif Σελίδα 132

Τότε, επειδή z 0 , θα ισχύει x y z x y a a

a a2 2 2

, άτοπο.

Αν ίσχυε ότι x a και y a , τότε a x a y 0 , άτοπο.

Με το ίδιο σκεπτικό, καταλήγουμε σε άτοπο, αν υποθέσουμε ότι x a και

y a .

Συνεπώς, x a και y a .

Β τρόπος

Αφού (a x)(a y) 0 (x a y a) (x a y a) .

Όμως x y z

a 2a x y z (a x) a y) z 02

.

Αν ήταν x a και y a , άτοπο. Άρα x a και y a .

ΘΕΜΑ 164 (ΚΑΛΑΘΑΚΗΣ ΓΙΩΡΓΗΣ)

Ένας ακριβώς από τους παρακάτω αριθμούς

999973,999983,999991,1000001,7999973 είναι πρώτος. Ποιος ;

Λύση:

3 3 2999973 1.000.000 27 100 3 (100 3)(100 300 9) .

6 2 2999991 10 9 1000 3 (1000 3)(1000 3) .

6 3 3 21000001 10 1 100 1 (100 1)(100 100 1) .

3 3 27999973 8.000.000 27 200 3 (200 3)(200 600 9) .

Άρα ο 999983 είναι πρώτος αφού οι υπόλοιποι είναι σύνθετοι.

Page 133: Εισαγωγη σε διαγωνιστικα μαθηματικα για το γυμνασιο

http://www.mathematica.gr/forum/viewtopic.php?f=109&t=30104

Επιμέλεια: xr.tsif Σελίδα 133

ΘΕΜΑ 165 (ΔΗΜΗΤΡΗΣ ΙΩΑΝΝΟΥ)

Δίνεται το πολυώνυμο 3 2f (x) ax bx cx d , με a 0 και a,b,c,d Z . Αν

f (0)f (1) είναι περιττός αριθμός, δείξτε ότι το f (x) , δεν έχει ρίζα ακέραιο

αριθμό.

Λύση:

Αρκεί να δείξουμε ότι ο f (x) είναι περιττός για κάθε x Z

Οι f (0),f (1) είναι περιττοί.

Αν ο x είναι άρτιος τότε 2f (x) f(0) x ax bx c άρτιος, οπότε ο f (x)

είναι περιττός.

Αν ο x είναι περιττός τότε 3 2f(x) f(1) a x 1 b(x 1) c(x 1) άρτιος,

οπότε ο f (x) είναι περιττός.

Β τρόπος

Έστω ότι το πολυώνυμο έχει ρίζα p Z . Τότε f (x) (x p)H(x) , όπου το

H(x) είναι πολυώνυμο δευτέρου βαθμού με ακέραιους συντελεστές.

Τώρα f (0)f (1) pH(0)(1 p)H(1) p(p 1)H(0)H(1) άρτιος, αφού ο αριθμός

p(p 1) , είναι άρτιος ως γνωστόν, (και οι αριθμοί H(0),H(1) , είναι ακέραιοι).

Καταλήξαμε λοιπόν σε άτοπο και η απόδειξη ολοκληρώθηκε.

ΘΕΜΑ 166 (ΔΗΜΗΤΡΗΣ ΙΩΑΝΝΟΥ)

Να προσδιορίσετε πολυώνυμο P(x) δευτέρου βαθμού τέτοιο ώστε P(0) 0

και P(x) P(x 2) 4x , για κάθε x R .

Λύση:

Έστω 2P(x) ax bx c,a 0,x R το ζητούμενο πολυώνυμο.

Page 134: Εισαγωγη σε διαγωνιστικα μαθηματικα για το γυμνασιο

http://www.mathematica.gr/forum/viewtopic.php?f=109&t=30104

Επιμέλεια: xr.tsif Σελίδα 134

2P(0) 0 c 0 P(x) ax bx,x R .

Για κάθε x R έχουμε

22P(x) P(x 2) 4x ax bx a x 2 b x 2 4x

2 2ax bx ax ax 4a bx 2b 4x 4ax 2b 4a 4x .

Για x 0 παίρνουμε, 2b 4a 0 b 2a , ενώ για x 1 ,

4a 2b 4a 4 2b 4 b 2 .

Για b 2 , βρίσκουμε 2a 2 a 1 .

Τότε, 2P(x) x 2x,x R και για κάθε x R είναι,

22P(x) P(x 2) x 2x x 2 2 x 2

2 2x 2x x 4x 4 2x 4 4x .

Συνεπώς, το ζητούμενο πολυώνυμο είναι το 2P(x) x 2x,x R .

ΘΕΜΑ 167 (ΔΗΜΗΤΡΗΣ ΙΩΑΝΝΟΥ)

Να βρεθούν οι κοινές λύσεις των εξισώσεων:

3 22x x 2x 1 0 και 2013 20142(2x 1) (4x 1) 1 4x .

Λύση:

Για x R έχουμε,

3 2 3 2 2 22x x 2x 1 0 2x 2x x 1 0 2x x 1 x 1 0

2 1x 1 2x 1 0 x 1 2x 1 x 1 0 x 1 x x 1

2 .

Για x 1 είναι 2013 2014 2013 2014

2 2x 1 4x 1 1 4x 2 3 5 5 0 .

Page 135: Εισαγωγη σε διαγωνιστικα μαθηματικα για το γυμνασιο

http://www.mathematica.gr/forum/viewtopic.php?f=109&t=30104

Επιμέλεια: xr.tsif Σελίδα 135

Για 1

x2

είναι 2013 2014

2 2x 1 4x 1 1 4x 0 1 1 2 0 .

Για x 1 είναι 2013 2014 2014 20142 2x 1 4x 1 1 4x 2 3 1 4 3 1 0 .

Συνεπώς, η μοναδική κοινή λύση των δύο εξισώσεων είναι η 1

x2

.

ΘΕΜΑ 168 (ΔΗΜΗΤΡΗΣ ΙΩΑΝΝΟΥ)

Αν *a,b,c R

και αν ab bc ca 1 , να αποδείξετε ότι :

2 2 2 2 2 2

2 2 2

(b 1)(c 1) a 1)(c 1) (a 1)(b 1)a b c 2

a 1 b 1 c 1

.

Λύση:

Είναι,

2 2a 1 a ab bc ca a a b c a b a b a c ,

2 2b 1 b ab bc ca b a b c a b a b b c ,

2 2c 1 c ab bc ca c a c b a c a c b c .

Χρησιμοποιώντας και το γεγονός ότι a,b,c 0 λαμβάνουμε,

2 22

2

b 1 c 1a a b c ab ca

a 1

,

2 22

2

a 1 c 1b b a c ab bc

b 1

,

2 22

2

a 1 b 1c c a b bc ca

c 1

.

Προσθέτουμε κατά μέλη και παίρνουμε

Page 136: Εισαγωγη σε διαγωνιστικα μαθηματικα για το γυμνασιο

http://www.mathematica.gr/forum/viewtopic.php?f=109&t=30104

Επιμέλεια: xr.tsif Σελίδα 136

2 2 2 2 2 2

2 2 2

(b 1 c 1 a 1 c 1 a 1 b 1a b c

a 1 b 1 c 1

2 ab bc ca 2 ,

που είναι το ζητούμενο.

Β τρόπος

Ας το δούμε και τριγωνομετρικά:

Υπάρχει τρίγωνο ABC ώστε a cotA,b cotB,c cotC .

Λόγω της ταυτότητας 2

2

11 cot x

sin x το αριστερό μέλος γράφεται

sinA cosA cos(B C)cotA

sinBsinC sinBsinC sinBsinC

cotBcotC 1 1 3 2( ) .

ΘΕΜΑ 169 (ΔΗΜΗΤΡΗΣ ΙΩΑΝΝΟΥ)

Έστω p πρώτος. Να αποδείξετε ότι το υπόλοιπο της διαίρεσης του pμε τον 30,

ή θα είναι ίσο με την μονάδα ή θα είναι επίσης πρώτος αριθμός.

Λύση:

Έστω λοιπόν p 30k u,u 30 . Καταρχάς αν ο u είναι άρτιος ,u 2m , τότε

p 30k 2m 2(15k m) , p σύνθετος, άτοπο.

Τώρα αν ο u 30 είναι περιττός και μη πρώτος , καταλήγουμε στο ότι ο p θα

είναι είτε πολλαπλάσιο του 3, είτε πολλαπλάσιο του 5.

Πράγματι για u 9,21,27 , ο pείναι πολλαπλάσιο του 3, ενώ για u 15,25 ο

p είναι πολλαπλάσιο του 5.

Page 137: Εισαγωγη σε διαγωνιστικα μαθηματικα για το γυμνασιο

http://www.mathematica.gr/forum/viewtopic.php?f=109&t=30104

Επιμέλεια: xr.tsif Σελίδα 137

Ας δούμε και μερικές ασκησούλες του αγαπημένου μαθηματικού κλάδου,

της Συνδυαστικής.

ΘΕΜΑ 170 (ΚΛΕΟΒΟΥΛΟΣ ΚΟΦΟΝΙΚΟΛΑΣ)

Μια ομάδα μπάσκετ έχει 10 παίκτες, από τους οποίους ο μικρότερος είναι 20

ετών και ο μεγαλύτερος 28. Να εξετάσετε αν υπάρχουν δύο παίκτες με την ίδια

ηλικία.

Λύση:

Αν είχαν όλοι διαφορετική ηλικία, θα είχαν τις ηλικίες

20,21,22,23,24,25,26,27,28, δηλαδή θα ήταν 9 άτομα.

Άρα αφού είναι 10, δύο τουλάχιστον θα έχουν την ίδια ηλικία.

ΘΕΜΑ 171 (ΚΛΕΟΒΟΥΛΟΣ ΚΟΦΟΝΙΚΟΛΑΣ)

Ένας διευθυντής Γυμνασίου επέλεξε και από τις τρεις τάξεις συνολικά 25

μαθητές για να εκπροσωπήσουν το σχολείο σε μια εκδήλωση. Να αποδείξετε

ότι ανάμεσά τους υπάρχουν τουλάχιστον 9 μαθητές της ίδιας τάξης.

Λύση:

Στο ίδιο στυλ και η άσκηση αυτή. Αν υπάρχουν το πολύ 8 μαθητές από κάθε

τάξη, τότε οι επιλεχθέντες μαθητές θα ήτανε 24 . Αφού όμως είναι 25, άρα

τουλάχιστον 9 μαθητές της ίδιας τάξης θα υπάρχουν.

Με παρόμοιο τρόπο λύνονται και οι υπόλοιπες.

Page 138: Εισαγωγη σε διαγωνιστικα μαθηματικα για το γυμνασιο

http://www.mathematica.gr/forum/viewtopic.php?f=109&t=30104

Επιμέλεια: xr.tsif Σελίδα 138

ΘΕΜΑ 172 (ΚΛΕΟΒΟΥΛΟΣ ΚΟΦΟΝΙΚΟΛΑΣ)

Στις αθλητικές εκδηλώσεις ενός Γυμνασίου συμμετέχουν συνολικά 31μαθητές

και από τις τρεις τάξεις. Αν κάθε τάξη έχει δύο τμήματα, να αποδείξετε ότι

ανάμεσα στους μαθητές αυτούς υπάρχουνε τουλάχιστον 6 συμμαθητές, δηλαδή

6 μαθητές του ίδιου τμήματος.

Λύση:

Αφού κάθε τάξη έχει 2 τμήματα, όλα τα τμήματα του σχολείου είναι 2 3 6 .

Αν επέλεγε από κάθε τμήμα 5 μαθητές τότε θα είχε συνολικά 5 6 30 . Αφού

όμως έχει 31 από ένα τμήμα επέλεξε 6 μαθητές.

ΘΕΜΑ 173 (ΚΛΕΟΒΟΥΛΟΣ ΚΟΦΟΝΙΚΟΛΑΣ)

Σε ένα τμήμα της Α' Λυκείου υπάρχουν 28 μαθητές. Σε ένα τεστ του πρώτου

τριμήνου έγραψαν όλοι οι μαθητές από 10 έως 18. Να αποδείξετε ότι

τουλάχιστον τέσσερις μαθητές έγραψαν τον ίδιο βαθμό.

Υπόδειξη: Αρχή του Dirichlet.

Λύση:

Όλοι οι βαθμοί είναι 9.

Όμως 28 3 9 1 και άρα οι μαθητές που έγραψαν τον ίδιο βαθμό είναι

3 1 4 .

Page 139: Εισαγωγη σε διαγωνιστικα μαθηματικα για το γυμνασιο

http://www.mathematica.gr/forum/viewtopic.php?f=109&t=30104

Επιμέλεια: xr.tsif Σελίδα 139

Ας δώσουμε και τον ορισμό της Αρχής της Περιστεροφωλιάς /Αρχής του

Dirichlet για μία άλλη προσέγγιση. Θεωρητικά είναι ο ίδιος τρόπος με λιγότερο

γράψιμο.

ΘΕΩΡΙΑ : ΑΡΧΗ ΤΗΣ ΠΕΡΙΣΤΕΡΟΦΩΛΙΑΣ

Αν nk 1 περιστέρια καθίσουν σε k φωλιές, τότε σε κάποια φωλιά θα

καθίσουν τουλάχιστον n 1 περιστέρια.

Δηλαδή το πρόβλημα με βάση της αρχής της περιστεροφωλιάς μπορεί να λυθεί

ως:

Από την αρχή της περιστεροφωλιάς ισχύει ότι 31 6 5 1 . Άρα ένα τμήμα είχε

τουλάχιστον 6 μαθητές.

Αν θες να δεις εφαρμογή της Αρχής της Περιστεροφωλιάς σε συνθετότερα

θέματα δες το blog με θέματα από ΒΜΟΝ/ΒΜΟ/ΔΜΟ εδώ:

http://problsolving.blogspot.gr/

ΘΕΜΑ 174 (ΔΗΜΗΤΡΗΣ ΙΩΑΝΝΟΥ)

a) Αν οι αριθμοί p και 28p 1 είναι πρώτοι, δείξτε ότι υποχρεωτικά θα είναι

p 3 .

b) Αν οι αριθμοί p,p 10,p 14 είναι πρώτοι, τότε p 3 .

Λύση:

Θα απαντήσω μόνο για το (α). Λοιπόν:

Έστω p 3 .

Αν p 2 , τότε 28p 1 33 , που είναι σύνθετος, άτοπο. Άρα p 3 .

Page 140: Εισαγωγη σε διαγωνιστικα μαθηματικα για το γυμνασιο

http://www.mathematica.gr/forum/viewtopic.php?f=109&t=30104

Επιμέλεια: xr.tsif Σελίδα 140

Δίνουμε στον pτις μορφές p 3k 1 και p 3k 2 .

Διακρίνουμε τις περιπτώσεις:

Αν p 3k 1 έχουμε:

2 2 2 2 28p 1 8(3k 1) 1 8(9k 6k 1) 1 72k 48k 9 3(24k 16k 3)

που είναι σύνθετος, άτοπο.

Αν p 3k 2 έχουμε:

2 2 2 2 28p 1 8(3k 2) 1 8(9k 12k 4) 1 72k 96k 33 3(24k 32k 11)

που είναι σύνθετος, άτοπο. Άρα αναγκαστικά p 3 .

Με παρόμοιο σκεπτικό λύνεται και το (β).

Έστω ότι p 3 .

Αν p 2 παίρνουμε ότι p 10 2 10 12 , που είναι σύνθετος, άτοπο.

Η διαίρεση του p με τον 3 μας δίνει υπόλοιπο 1 ή 2 . Άρα p 3k 1 ή

p 3k 2 .

Διακρίνουμε τις περιπτώσεις:

Αν p 3k 1 ,

τότε p 14 3k 15 3(k 3) , που είναι σύνθετος, άτοπο.

Αν p 3k 2 ,

τότε p 10 3k 12 3(k 4) , που είναι σύνθετος, άτοπο.

Άρα, αναγκαστικά p 3 .

ΘΕΜΑ 175 (ΔΗΜΗΤΡΗΣ ΙΩΑΝΝΟΥ)

Αν x y z

a b b c c a

, όπου (a b)(b c)(c a) 0 , να αποδείξετε ότι

x y z 0 .

Page 141: Εισαγωγη σε διαγωνιστικα μαθηματικα για το γυμνασιο

http://www.mathematica.gr/forum/viewtopic.php?f=109&t=30104

Επιμέλεια: xr.tsif Σελίδα 141

Λύση:

Έστω ότι όλα τα κλάσματα είναι ίσα με έναν σταθερό αριθμό k . Τότε θα

ισχύει:

x k(a b),y k(b c),z k(c a) και άρα

x y z ka kb kb kc kc ka 0 . Αποδείχθηκε.

ΘΕΜΑ 176 (ΔΗΜΗΤΡΗΣ ΙΩΑΝΝΟΥ)

Αν *a,b,x,y R και αν a y,b x , να αποδείξετε ότι:

2 2 2 2

2 2 2 2

b bx x a ay y x y

b bx x a ay y b a

ή

x b

a y .

Λύση:

Για ευκολία ας θέσουμε x

cb

και y

da

. Τότε

2 2 2 2 2 2

2 2 2 2 2 2

b bx x a ay y c c 1 d d 1

b bx x a ay y c c 1 d d 1

2 2

2 2 2 2

c c 1 d d 1 2c 2d1 1

c c 1 d d 1 c c 1 d d 1

x y

c d cd 1 0b a

ή x b

a y .

ΘΕΜΑ 177 (ΔΗΜΗΤΡΗΣ ΙΩΑΝΝΟΥ)

Αν a b c 0 , να αποδείξετε ότι:

n 3 n 3 n 3 n n n 2 2 2 n 1 n 1 n 11a b c abc(a b c ) (a b c )(a b c )

2

για κάθε

n N .

Page 142: Εισαγωγη σε διαγωνιστικα μαθηματικα για το γυμνασιο

http://www.mathematica.gr/forum/viewtopic.php?f=109&t=30104

Επιμέλεια: xr.tsif Σελίδα 142

Υπόδειξη

Χρησιμοποιούμε την ταυτότητα:

3 2(x a)(x b)(x c) x (a b c)x (ab ac bc)x abc , για κάθε x R .

Λύση:

Ας είναι 3 2x (a b c)x (ab bc ca)x abc 0 , η εξίσωση τρίτου βαθμού

με ρίζες τις a,b,c

Είναι a b c 0 άρα και 2 2 2a b c

ab bc ca2

οπότε η εξίσωση

γράφεται 2 2 2

3 a b cx x abc 0

2

.

Το a είναι ρίζα της εξίσωσης, άρα

2 2 2 2 2 2

3 n 3 n 1 na b c a b ca a abc 0 a a abca 0

2 2

Γράφουμε ακόμα δύο τέτοιες σχέσεις, μια για το b και μια για το c .

Προσθέτουμε τις τρεις αυτές ισότητες και προκύπτει η ζητούμενη.

ΘΕΜΑ 178 (ΔΗΜΗΤΡΗΣ ΙΩΑΝΝΟΥ)

Να βρεθεί ο μικρότερος φυσικός αριθμός n 0 , για τον οποίο το άθροισμα:

17 17 17 17(1 1) (1 2) (1 3) ... (1 n)

76 76 76 76 , είναι αριθμός ακέραιος.

Λύση:

Το άθροισμα μπορεί να γραφτεί:

17 17 n(n 1) 17S (1 1 1 ... 1) (1 2 3 ... n) n n n(n 1)

76 76 2 152

Page 143: Εισαγωγη σε διαγωνιστικα μαθηματικα για το γυμνασιο

http://www.mathematica.gr/forum/viewtopic.php?f=109&t=30104

Επιμέλεια: xr.tsif Σελίδα 143

Το S είναι ακέραιος, αν ο n(n 1) διαιρείται με το 152και ζητούμε την

ελάχιστη τιμή του nγια να συμβαίνει αυτό.

Παρατηρούμε ότι 152 1 152 2 76 4 38 8 19 .

Πρέπει να βρούμε λοιπόν θετικούς ακεραίους k,c και μάλιστα με γινόμενο το

ελάχιστο δυνατό, έτσι ώστε ο αριθμός (1k) (152c) ή ο (2k) (76c) ή ο

(4k) (38c) ή ο (8k) (19c) να είναι γινόμενο δύο διαδοχικών θετικών

ακεραίων.

Οι περιπτώσεις (2k) (76c) και (4k) (38c) , αποκλείονται, διότι είναι γινόμενα

αρτίων και άρα δεν μπορούν να είναι διαδοχικοί.

Μας μένουν οι περιπτώσεις (1k) (152c) και (8k) (19c) .

Για την πρώτη περίπτωση, θέλουμε να είναι 1k n,152c n 1 ή

1k n 1,152c n . Δηλαδή 152c k 1 ή 152c k 1 , δηλαδή k 152c 1 ή

k 152c 1 και τότε οι ελάχιστες τιμές των k,c είναι (k,c) (151,1) ή

(k,c) (153,1) .

Για την δεύτερη περίπτωση, θέλουμε να είναι 8k n,19c n 1 ή

8k n 1,19c n .

Δηλαδή 19c 8k 1 ή 19c 8k 1 , δηλαδή 19c 1

k8

ή

19c 1k

8

.

Δηλαδή 3c 1

k 2c8

ή

3c 1k 2c

8

και τότε οι ελάχιστες τιμές των k,c

είναι (k,c) (7,3) ή (k,c) (12,5) .

Από τα ζεύγη (151,1),(153,1),(7,3),(12,5) , ελάχιστο γινόμενο παίρνουμε με το

ζεύγος (7,3) και άρα πρέπει k 7,c 3 .

Άρα η ελάχιστη τιμή για το n είναι η n 8k 8 7 56 .

Page 144: Εισαγωγη σε διαγωνιστικα μαθηματικα για το γυμνασιο

http://www.mathematica.gr/forum/viewtopic.php?f=109&t=30104

Επιμέλεια: xr.tsif Σελίδα 144

ΘΕΜΑ 179 (ΔΗΜΗΤΡΗΣ ΙΩΑΝΝΟΥ)

Δίνεται το πολυώνυμο: 3 2P(x) x x 2x 1 . Αν P(n) 0 , να αποδείξετε ότι

και ο αριθμός 22 n , είναι επίσης ρίζα της εξίσωσης P(x) 0 .

Λύση:

3 2P(n) 0 n n 2n 1 0 (I) .

Επίσης, για κάθε x έχουμε,

2P(x) x n ax bx c

3 2 3 2x x 2x 1 ax b an x c bn x cn .

Έτσι, προκύπτει, το ακόλουθο σύστημα

a 1

b n 1

c bn 2

cn 1

.

Από την δεύτερη εξίσωση έχουμε b n 1 και από την τρίτη, με τη βοήθεια

της δεύτερης, 2c n n 2 .

Η τελευταία εξίσωση ικανοποιείται μιας και

(I)

2 3 2cn n n 2 n n n 2n 1 .

Επομένως, P(x) (x n)Q(x) , όπου 2 2Q(x) x n 1 x n n 2 .

Είναι,

22 2 2 2Q(2 n ) 2 n n 1 2 n n n 2

2 4 3 2 2 4 3 24 4n n 2n n 2 n n n 2 n n 2n n

(I)

3 2n n n 2n 1 0

Page 145: Εισαγωγη σε διαγωνιστικα μαθηματικα για το γυμνασιο

http://www.mathematica.gr/forum/viewtopic.php?f=109&t=30104

Επιμέλεια: xr.tsif Σελίδα 145

και άρα 2 2 2P 2 n 2 n n Q(2 n ) 0 , όπως θέλαμε.

ΘΕΜΑ 180 (ΣΩΚΡΑΤΗΣ ΛΥΡΑΣ)

Αν x,y,z 0 , να αποδείξετε ότι: 3 2 2 2 6 33 (x y) (y z) (z x) 2 xyz(x y z) .

Λύση:

Λόγω ομογένειας μπορούμε να υποθέσουμε ότι x y z 1

Τότε, η αποδεικτέα γράφεται 227(xy yz zx xyz) 64xyz .

Επειδή ισχύει xy yz zx 3xyz(x y z) 3xyz είναι αρκετό να

αποδείξουμε ότι 227( 3xyz xyz) 64xyz (1) .

Όμως, είναι 3(x y z) 1

xyz27 27

.

Δηλαδή έχουμε να αποδείξουμε ότι αν 1

0 a27

, τότε 227( 3a a) 64a .

Πράγματι, 2

2 227( 3a a) 27 27 1( 3 a) 3 1

64a 64 64 3 3( )

.

Β τρόπος

Είναι άμεση συνέπεια της ανισότητας Popoviciu στην κοίλη συνάρτηση

f (t) ln t, t 0 .

Απλώς θεωρώ ότι η συγκεκριμένη ανισότητα ξεφεύγει αρκετά από τις

γυμνασιακές ασκήσεις.

Όπως και ναναι, η ανισότητα Popoviciu λέει:

Page 146: Εισαγωγη σε διαγωνιστικα μαθηματικα για το γυμνασιο

http://www.mathematica.gr/forum/viewtopic.php?f=109&t=30104

Επιμέλεια: xr.tsif Σελίδα 146

Θεώρημα: (Ανισότητα Popoviciu)

Έστω I ένα διάστημα και f μια συνάρτηση, η οποία είναι κοίλη στο I .

Τότε, για οποιαδήποτε a,b,c I ισχύει

a+b b+c c+a a+b+c2 f +f +f f (a)+f(b)+f(c)+3f

2 2 2 3

.

Αν η f είναι κυρτή στο I , η ανισότητα αντιστρέφεται.

Για μια απόδειξη του θεωρήματος αυτού μπορεί κανείς να δει το βιβλίο

Mathematical Miniatures, S.Savchev, T. Andreescu, MAA, 2003, σελ. 19.

ΘΕΜΑ 181 (ΜΑΓΚΟΣ ΘΑΝΟΣ)

Δίνονται οι πραγματικοί αριθμοί 1 2 49

x ,x ,...,x , για τους οποίους ισχύει

2 2 2

1 2 49x 2x 49x 1 . Να βρείτε τη μέγιστη τιμή της παράστασης

1 2 3 49x 2x 3x 49x .

Λύση:

Έστω Aη παράσταση της οποίας ζητείται το μέγιστο.

2 2 2 2

1 2 49(1 2 ... 49)(x 2x ... 49x ) A A 7·5 35 .

Η ισότητα ισχύει όταν 1 2 49

x x ... x .

Β τρόπος

Είναι ευκαιρία να δώσουμε μια αναλυτική παρουσίαση της ανισότητας C – S

που χρησιμοποίησε πιο πάνω ο Σωκράτης, ώστε να μπορέσουν να κατανοήσουν

την λύση οι αρχάριοι σε διαγωνιστικά μαθηματικά, μαθητές.

Page 147: Εισαγωγη σε διαγωνιστικα μαθηματικα για το γυμνασιο

http://www.mathematica.gr/forum/viewtopic.php?f=109&t=30104

Επιμέλεια: xr.tsif Σελίδα 147

Θεωρούμε τις ν – άδες: 1 2 3 n

x ,x ,x ,...,x και 1 2 3 n

a ,a ,a ,...,a .

Τότε ισχύει η εξής ανισότητα:

2 2 2 2 2 2 2 2 2

1 2 3 n 1 2 3 n 1 1 2 2 n n(x x x ... x )(a a a ... a ) (x a x a ... x a ) .

(Η ισότητα, ισχύει όταν 1 2 n

1 2 n

x x x...

a a a ).

Έτσι, για να λύσουμε την πιο πάνω άσκηση, θεωρήσαμε τους αριθμούς:

1, 2,..., 49 και 1 2 49

1x , 2x ,..., 49x .

Αν λοιπόν εφαρμόσουμε την ανισότητα C – S, θα έχουμε:

2 2 2 2 2 2

1 2 49[( 1) ( 2) ... ( 49) ][( 1x ) ( 2x ) ... ( 49x ) ]

2

1 2 49( 1 1x 2 2x ... 49 49x ) .

Άρα: 2 2 2 2

1 2 49 1 2 49(1 2 ... 49)(x 2x ... 49x ) (x 2x ... 49x ) .

Άρα: 2

1 2 49(x 2x ... 49x ) (1 2 ... 49) 1 .

Άρα: 2

1 2 49

(1 49) 49(x 2x ... 49x )

2

.

( Διότι έχουμε πολλές φορές και στο παρελθόν αναφέρει, ότι

(1 n)n1 2 3 ... n

2

).

Συνεπώς: 2

1 2 49(x 2x ... 49x ) 25 49 και άρα:

1 2 49x 2x ... 49x 35 .

Άρα το μέγιστο που ζητάμε είναι το 35.

(Η ισότητα επιτυγχάνεται όταν 1 2 491x 2x 49x

...1 2 49 ,

δηλαδή όταν 1 2 49

x x ... x .

Και αφού είναι από την υπόθεση 2 2 2

1 2 49x 2x ... 49x 1 , θα έχουμε:

Page 148: Εισαγωγη σε διαγωνιστικα μαθηματικα για το γυμνασιο

http://www.mathematica.gr/forum/viewtopic.php?f=109&t=30104

Επιμέλεια: xr.tsif Σελίδα 148

2 2 2 2

1 1 1 1 1

1x 2x ... 49x 1 (1 2 ... 49)x 1 | x |

35 .

Άρα , η ισότητα επιτυγχάνεται αν π.χ πάρουμε 1 2 49

1x x ... x

35 ).

Γράφω μερικά στοιχεία για τις ΟΜΟΓΕΝΕΙΣ ΑΝΙΣΟΤΗΤΕΣ

Α) Μια ανισότητα με μεταβλητές πχ τις x,y,z λέγεται ομογενής, αν δεν

μεταβάλλεται όταν στην θέση των x,y,z βάλλουμε τα kx,ky,kz , όπου k είναι

ένας θετικός αριθμός.

Έτσι, η πιο πάνω ανίσωση, αν θέσουμε όπου x το kx , όπου y το ky και όπου

z το kz , γράφεται:

3 2 2 2 6 33 (kx ky) (ky kz) (kz kx) 2 kxkykz(kx ky kz)

3 6 2 2 2 6 6 33 k (x y) (y z) (z x) 2 k (x y z) ,

και με απλοποίηση του θετικού αριθμού k , ξαναγυρίζουμε στην αρχική

ανίσωση. Άρα πράγματι είναι ομογενής.

Μπορούμε συχνά να απλοποιήσουμε την λύση μιας ομογενούς ανισότητας,

γιατί μπορούμε να θεωρήσουμε ότι οι μεταβλητές της, υπακούουν σε κάποια

κατάλληλη συνθήκη.

Έτσι για παράδειγμα, σε μια ομογενή ανισότητα με μεταβλητές x,y,z ,

μπορούμε να θεωρήσουμε ότι x y z 1 , ή ότι x y z 3 ή ότι xyz 1

κλπ.

Page 149: Εισαγωγη σε διαγωνιστικα μαθηματικα για το γυμνασιο

http://www.mathematica.gr/forum/viewtopic.php?f=109&t=30104

Επιμέλεια: xr.tsif Σελίδα 149

Β) ΟΜΟΓΕΝΟΠΟΙΗΣΗ:

Όταν μας δίνουν να αποδείξουμε μια ανισότητα και υπάρχει κάποια συνθήκη,

τότε είναι δυνατόν να αντικαταστήσουμε κατάλληλα την συνθήκη στην

δοσμένη ανισότητα και έτσι αυτή να γίνει ομογενής, οπότε ίσως να μπορεί να

λυθεί εύκολα.

ΠΑΡΑΔΕΙΓΜΑ

Αν x y z 1 , να αποδείξετε ότι: 2 2 2x y z 1 4(xy yz zx) .

ΛΥΣΗ

Παρατηρούμε ότι η ανισότητα αυτή δεν είναι ομογενής (μας χαλάει την

ομογένεια ο αριθμός 1, διότι αν θέσουμε όπου x,y,z τα kx,ky,kz , δεν θα

μπορέσουμε να απλοποιήσουμε το k ).

Επίσης παρατηρούμε ότι τα πολυώνυμα 2 2 2x y z ,4(xy yz zx) , τα οποία

εμπλέκονται στην ανισότητα, είναι δευτέρου βαθμού.

Γι αυτό θα αντικαταστήσουμε τον αριθμό 1, που δημιουργεί το πρόβλημα, με

το 2(x y z) .

Έτσι, αρκεί να αποδείξουμε ότι:

2 2 2 2x y z (x y z) 4(xy yz zx) ή αρκεί:

2 2 22x 2y 2z 2xy 2yz 2zx 0 ή αρκεί:

2 2 2 2 2 2x 2xy y y 2yz z z 2zx x 0 ή αρκεί:

2 2 2(x y) (y z) (z x) 0 , το οποίο είναι αληθές.

Page 150: Εισαγωγη σε διαγωνιστικα μαθηματικα για το γυμνασιο

http://www.mathematica.gr/forum/viewtopic.php?f=109&t=30104

Επιμέλεια: xr.tsif Σελίδα 150

ΘΕΜΑ 182 (ΔΗΜΗΤΡΗΣ ΙΩΑΝΝΟΥ)

Αν a,b,x,y 0 , να αποδείξετε ότι : 2 2 2 2 42(1 a )(1 b )(x y ) ( x aby) .

Yπόδειξη:

Να εφαρμόσετε πάνω από μια φορά την ανισότητα C – S για κατάλληλες

δυάδες αριθμών.

Λύση:

Για τους αριθμούς 1,και x,yέχουμε:

2 2 2 2 2 2 2 2 2(1 a )(x y ) (1 x a y) (1 a )(x y ) (x ay) (1) .

Για τους αριθμούς 1,1 και 1,b έχουμε:

2 2 2 2 2 2 2(1 1 )(1 b ) (1 1 1 b) 2(1 b ) (1 b) (2) .

Από τις (1) , (2) με πολλαπλασιασμό κατά μέλη, έπεται

2 2 2 2 2 22(1 a )(1 b )(x y ) (x ay) (1 b) .

Αρκεί επομένως να αποδειχθεί ότι 2 2 4(x ay) (1 b) ( x aby) .

Παίρνω τους αριθμούς x, ay και 1, b .

Άρα 2 2 2 2 2[( x) ( ay) ][1 ( b) ] (1 x b ay)

2 2 2 4(x ay)(1 b) ( x aby) (x ay) (1 b) ( x aby) και

τελειώσαμε.

Page 151: Εισαγωγη σε διαγωνιστικα μαθηματικα για το γυμνασιο

http://www.mathematica.gr/forum/viewtopic.php?f=109&t=30104

Επιμέλεια: xr.tsif Σελίδα 151

Μετά και από την παραπάνω απόδειξη ας επισημανθεί ότι αυτή ανισότητα είναι

ειδική περίπτωση της εξής γενικότερης:

• Αν 1 2 n 1 2 n

x ,x ,...,x ,y ,y ,...,y 0 , ισχύει

nn n1 1 2 2 n n 1 2 n 1 2 n

(x +y )(x +y ) (x +y ) x x x + y y y( ) .

Η ανισότητα αυτή αποδεικνύεται π.χ. με ΑΜ – ΓΜ ή Hölder.

ΘΕΩΡΗΤΙΚΑ ΣΤΟΙΧΕΙΑ

(Α) ΑΝΙΣΟΤΗΤΑ ΑΜ – ΓΜ

Έστω 1 2 3 n

x ,x ,x ,...,x 0 . Τότε ισχύει η εξής ανισότητα:

1 2 3 n n1 2 3 n

x x x ... xx x x ...x

n

.

Η ισότητα ισχύει μόνο όταν 1 2 n

x x ... x .

B) ΑΝΙΣΟΤΗΤΑ ΤΟΥ HOLDER

1η ΜΟΡΦΗ:

Αν a,b,c,x,y,z,k,m,n 0 , τότε 1 1

2 2 2 22 2(a b ) (x y ) ax by και επίσης

1 1 13 3 3 2 2 2 3 3 33 3 3(a b c ) (k m n ) (x y z ) akx bmy cnz .

Για την πρώτη ανισότητα, η ισότητα ισχύει όταν a b

x y .

Για την δεύτερη ανισότητα, η ισότητα ισχύει όταν a b c

k m n και

a b c

x y z .

Page 152: Εισαγωγη σε διαγωνιστικα μαθηματικα για το γυμνασιο

http://www.mathematica.gr/forum/viewtopic.php?f=109&t=30104

Επιμέλεια: xr.tsif Σελίδα 152

2η ΜΟΡΦΗ:

Αν a,b,c,x,y,z 0 και *m,n R

και αν 1 1

1m n , τότε:

1 1m m n nm n(a b ) (x y ) ax by ,

με την ισότητα να ισχύει όταν a b

x y .

Επίσης:

1 1m m m n n nm n(a b c ) (x y z ) ax by cz .

με την ισότητα να ισχύει όταν a b c

x y z .

3η ΜΟΡΦΗ:

Αν 1 2 n 1 2 n 1 2 n

a ,a ,...,a ,b ,b ,...,b ,...,x ,x ,...,x 0 και αν

1 2 nk ,k ,...,k 0 με

1 2 nk k ... k 1 , τότε:

1 2 nk k k

1 2 n 1 2 n 1 2 n(a a ... a ) (b b ... b ) ... (x x ... x )

1 2 n 1 2 n 1 2 nk k k k k k k k k

1 1 1 2 2 2 n n na b ... x a b ... x ... a b ... x .

και η ισότητα ισχύει όταν 1 2 n 1 2 n

1 2 n 1 2 n

a a a a a a... ,..., ...

b b b x x x .

ΘΕΜΑ 183 (ΔΗΜΗΤΡΗΣ ΙΩΑΝΝΟΥ)

Αν για τους θετικούς αριθμούς x,y,z ισχύει ότι xy yz zx 1 , να αποδείξετε

ότι: 3xyz(x y z) 1 .

Page 153: Εισαγωγη σε διαγωνιστικα μαθηματικα για το γυμνασιο

http://www.mathematica.gr/forum/viewtopic.php?f=109&t=30104

Επιμέλεια: xr.tsif Σελίδα 153

Υπόδειξη

Ένας τρόπος λύσης είναι με την μέθοδο της ομογενοποίησης (υπάρχει σε

προηγούμενη ανάρτηση).

Παρατηρείστε ότι το πρώτο μέλος είναι τετάρτου βαθμού. Άρα μην

αντικαταστήσετε το 1 με το xy yz zx , που είναι δευτέρου βαθμού.

Σκεφτείτε το πως θα το αντικαταστήσετε.

Λύση:

Αρκεί να αποδείξουμε ότι 23xyz(x y z) (xy yz zx) .

(Η ομογενοποίηση έγινε, αφού και στα δύο μέλη έχουμε πολυώνυμα τετάρτου

βαθμού).

Άρα αρκεί: 2 2 2 2 2 2 2 2 2 2 2 23x yz 3xy z 3xyz x y y z z x 2xy z 2x yz 2xyz .

Άρα αρκεί: 2 2 2 2 2 2 2 2 2x y y z z x x yz xy z xyz 0 .

H ανισότητα τώρα αυτή ανάγεται σε στοιχειώδη ανισότητα, με την

αντικατάσταση που ο Σωκράτης προτείνει.

Βέβαια, μπορούμε και να συνεχίσουμε και ως εξής (κάνοντας ουσιαστικά μια

απόδειξη της στοιχειώδους ανισότητας που αναφέραμε)

Έτσι, αρκεί : 2 2 2 2 2 2 2 2 22x y 2y z 2z x 2x yz 2xy z 2xyz 0 ή:

2 2 2 2 2 2 2 2 2 2 2 2 2 2 2(x y 2x yz x z ) (x y 2xy z y z ) (y z 2xyz x z ) 0 , ή :

2 2 2(xy xz) (xy yz) (yz zx) 0 , το οποίο είναι αληθές.

Page 154: Εισαγωγη σε διαγωνιστικα μαθηματικα για το γυμνασιο

http://www.mathematica.gr/forum/viewtopic.php?f=109&t=30104

Επιμέλεια: xr.tsif Σελίδα 154

ΘΕΩΡΗΤΙΚΑ ΣΤΟΙΧΕΙΑ

Η ανισότητα Andreescu.

Aν 1 2 3 n

x ,x ,x ,...,x R και 1 2 3 n

a ,a ,a ,...,a 0 , τότε:

2 2 2 2 2

1 2 3 n 1 2 3 n

1 2 3 n 1 2 3 n

x x x x (x x x ... x )...

a a a a a a a ... a

.

Η ισότητα ισχύει όταν 1 2 3 n

1 2 3 n

x x x x...

a a a a .

ΘΕΜΑ 184 (ΚΛΕΟΒΟΥΛΟΣ ΚΟΦΟΝΙΚΟΛΑΣ)

Για κάθε +a,b,c,d R με άθροισμα τη μονάδα (a+b+c+d=1) να αποδείξετε ότι

ισχύει η παρακάτω ανισοϊσότητα: 2 2 2 2a b c d 1

+ + +a+b b+c c+d d+a 2

.

Λύση:

Από την ανισότητα Andreescu έχουμε:

2 2 2 2 2a b c d (a b c d) 1 1

a b b c c d d a 2a 2b 2c 2d 2(a b c d) 2

.

ΘΕΜΑ 185 (ΚΛΕΟΒΟΥΛΟΣ ΚΟΦΟΝΙΚΟΛΑΣ)

Αν 0

a,b,c R

να αποδειχθεί ότι 3 3 3 2 2 2

2

2(a +b +c ) 6(a +b +c )+ 8

abc (a+b+c) .

Λύση:

Από την βασική ανισότητα: 2 2 2a b c ab bc ac , έχουμε:

Page 155: Εισαγωγη σε διαγωνιστικα μαθηματικα για το γυμνασιο

http://www.mathematica.gr/forum/viewtopic.php?f=109&t=30104

Επιμέλεια: xr.tsif Σελίδα 155

2 2 22a 2b 2c 2ab 2bc 2ac 2 2 2 2 2 2 2 2 22a 2b 2c a b c a b c 2ab 2ac 2bc 2 2 2 2 2 2 2 23a 3b 3c (a b c) 3(a b c ) (a b c)

2 2 2 2 2 2

2 2

3(a b c ) 6(a b c )1 2

(a b c) (a b c)

(1) .

Στη συνέχεια, από την ΤΑΥΤΟΤΗΤΑ ΤΟΥ EULER έχουμε:

3 3 3 2 2 21a b c 3abc (a b c)[(a b) (a c) (c b) ]

2 .

Σημείωση:

(Την ταυτότητα αυτή δεν την αναφέρουν τα σχολικά βιβλία του Γυμνασίου,

καλό όμως είναι να απομνημονευθεί, διότι έχει πολλές εφαρμογές).

Επειδή τώρα από την υπόθεση έχουμε ότι a,b,c 0 , έπεται από τη παραπάνω

ταυτότητα, ότι: 3 3 3

3 3 3 3 3 3 a b ca b c 3abc 0 a b c 3abc 3

abc

.

Άρα: 3 3 32(a b c )

6abc

(2) .

Με πρόσθεση κατά μέλη των σχέσεων (1) και (2) έχουμε το ζητούμενο.

ΘΕΜΑ 186 (ΔΗΜΗΤΡΗΣ ΙΩΑΝΝΟΥ)

Αν x y z 2 , να αποδείξετε ότι 2 2 2x y z 4 4(xy yz zx) .

Υπόδειξη

Ένας τρόπος λύσης είναι με ομογενοποίηση (βλ. προηγούμενη δημοσίευση).

Λύση:

Αφού x y z 2 τότε η δοσμένη μπορεί να γραφεί με ομογενοποίηση ως

εξής:

Page 156: Εισαγωγη σε διαγωνιστικα μαθηματικα για το γυμνασιο

http://www.mathematica.gr/forum/viewtopic.php?f=109&t=30104

Επιμέλεια: xr.tsif Σελίδα 156

2 2 2 2 2 2 2x +y +z +4 4(xy+yz+zx) x +y +z +(x+y+z) 4xy 4yz 4zx 0

2 2 2 2 2 22x +2y +2z 2xy 2yz 2zx 0 (x y) +(y z) +(z x) 0 ,

που ισχύει ως άθροισμα τετραγώνων.

ΘΕΜΑ 187 (ΔΗΜΗΤΡΗΣ ΙΩΑΝΝΟΥ)

Αν x,y,z 0 δείξτε ότι: 3 3 3 3 3 3x y y z z x xy yz zx

6xyzz x y z x y .

Υπόδειξη

Ένας τρόπος λύσης με την ανισότητα Andreescu , αφού πρώτα γράψουμε: 3 4 2x y x y

z xyz , κλπ .(βλ. προηγούμενη δημοσίευση)

Λύση:

Είναι άμεση συνέπεια της ανισότητας ΑΜ – ΓΜ: 6a+b+c+d+e+f 6 abcdef .

Β τρόπος

Πολύ ωραία και απλούστατη λύση. Ας το γράψω αναλυτικά για να το

καταλάβουνε και οι νέοι (χαίρομαι που μπορώ να λέω πως δεν είμαι πια νέος)

στα διαγωνιστικά μαθηματικά.

Από την ανισότητα αριθμητικού – γεωμετρικού μέσου θα ισχύει ότι:

4 2 4 2 4 2 4 2 4 2 2 4 12 12 12 2 2 2

66 6 6

x y y x y z z y z x z x x y z x y z+ + + + + 6 =6 =6xyz

xyz xyz xyz xyz xyz xyz x y z xyz .

ΘΕΜΑ 188 (ΔΗΜΗΤΡΗΣ ΙΩΑΝΝΟΥ)

Αν a,b 1 , να αποδείξετε ότι 2 2a b

8b 1 a 1

. (Από Ρωσικό διαγωνισμό)

Page 157: Εισαγωγη σε διαγωνιστικα μαθηματικα για το γυμνασιο

http://www.mathematica.gr/forum/viewtopic.php?f=109&t=30104

Επιμέλεια: xr.tsif Σελίδα 157

Λύση:

Θέτω a 1 c 0 και b 1 d 0 . Τότε αρκεί να δείξω ότι:

2 2

c 1 d 18

d c

.

Όμως, από την Cauchy έχουμε:

2 2 2

c 1 d 1 c d 2

d c c d

.

Αρκεί να δείξω ότι:

2

c d 28

c d

.

Αρκεί να δείξω ότι: 2

c d 2 8 c d .

Αρκεί να δείξω ότι: 2 2c d 4 4c 4d 2cd 8c 8d .

Αρκεί να δείξω ότι: 2 2c d 4 4c 4d 2cd 0 .

που ισχύει αφού δίνει: 2

c d 2 0 .

Η ισότητα επιτυγχάνεται όταν c d 1 δηλαδή όταν a b 2 .

Β τρόπος

Αρκεί να αποδείξουμε την ισχύ της

2 2a b

b a 2 8 b a 2b 1 a 1

αρκεί ,

2 2

a b 8 a b 16 0 a b 4 0 , που ισχύει.

Γ τρόπος

Από την ανισότητα Andreescu (η οποία στην ουσία προκύπτει άμεσα από την

ανισότητα Cauchy ), έχουμε: 2 2 2a b (a b)

b 1 a 1 a b 2

.

Αρκεί λοιπόν να αποδείξουμε ότι 2(a b)

8a b 2

ή αρκεί:

Page 158: Εισαγωγη σε διαγωνιστικα μαθηματικα για το γυμνασιο

http://www.mathematica.gr/forum/viewtopic.php?f=109&t=30104

Επιμέλεια: xr.tsif Σελίδα 158

2(a b) 8(a b) 16 0 ή αρκεί: 2(a b 4) 0 , η οποία είναι αληθής.

ΘΕΜΑ 189 (ΔΗΜΗΤΡΗΣ ΙΩΑΝΝΟΥ)

Αν a,b,c 0 , να αποδείξετε ότι: a b c 3

b c c a a b 2

.

(Aνισότητα του Nesbitt)

(Υπάρχουν διάφοροι τρόποι απόδειξής της. Μιας και τις τελευταίες μέρες

ασχολούμαστε με την εμπέδωση της ανισότητας Andreescu (ή Cauchy), ας

λυθεί με την χρήση αυτής)

(Υπόδειξη

Πρέπει να εμφανίσουμε στους αριθμητές των κλασμάτων του πρώτου μέλους,

τετράγωνα.

Λύση:

Ένας τρόπος:

Η ανισότητα είναι ισοδύναμη με την

a b c b c a b c c a a b c a b 3

b c b c c a c a a b a b 2

1 1 1 9(a b c)

b c c a a b 2

1 1 1(2a 2b 2c) 9

b c c a a b

1 1 1

(b c) (c a) (a b) 9b c c a a b

, που ισχύει από C S .

Β τρόπος

Αν a b c από την ανισότητα της αναδιάταξης ( διπλή εφαρμογή ) για τις

Page 159: Εισαγωγη σε διαγωνιστικα μαθηματικα για το γυμνασιο

http://www.mathematica.gr/forum/viewtopic.php?f=109&t=30104

Επιμέλεια: xr.tsif Σελίδα 159

τριάδες (a,b,c) και 1 1 1

, ,b c c a a b

και προκύπτει το ζητούμενο με

πρόσθεση κατά μέλη.

Αλλιώς υποθέτουμε ότι a b c 1 και μετά jensen στην κυρτή x

f(x)1 x

.

Και υπάρχουν και πολλές άλλες αποδείξεις.

Γ τρόπος

Μια ακόμα απόδειξη με Andreescu:

2 2 2 2a b c a b c (a b c)

b c a c a b a(b c) b(a c) c(a b) 2(ab ac bc)

.

Αρκεί λοιπόν να αποδειχθεί ότι:

2(a b c) 3

2(ab ac bc) 2

ή αρκεί:

2(a b c) 3(ab ac bc) ,ή : 2 2 2a b c ab ac bc , η οποία είναι

αληθής (την έχουμε δει πολλές φορές σε προηγούμενες δημοσιεύσεις).

Δ τρόπος

Μία «χειροκίνητη» διαπραγμάτευση, αλλά που περνά μέθοδο (για να φέρουμε

"τα κάτω πάνω" απλοποιώντας τα κάτω απελευθερώνοντας έτσι πράξεις, όπως

διασπάσεις κλασμάτων κ.τ.λ.).

Θεωρούμε:

b c 2x, c a 2y, a b 2z a b c x y z

a y z x, b z x y, c x y z , οπότε αρκεί να αποδείξουμε:

y z x x z y y x z x y y z z x3 6

x y z y x z y x z

, που

προφανώς ισχύει.

Page 160: Εισαγωγη σε διαγωνιστικα μαθηματικα για το γυμνασιο

http://www.mathematica.gr/forum/viewtopic.php?f=109&t=30104

Επιμέλεια: xr.tsif Σελίδα 160

ΘΕΜΑ 190 (ΔΗΜΗΤΡΗΣ ΙΩΑΝΝΟΥ)

Αν a,b,c 0 και 2 2 2a b c 3 , να αποδείξετε ότι:

1 1 1 3

1 ab 1 bc 1 ca 2

.

(ΠΗΓΗ: "ΑΛΓΕΒΡΙΚΕΣ ΑΝΙΣΟΤΗΤΕΣ" των Μπάμπη Στεργίου και Νίκου

Σκομπρή)

Λύση:

2 2 21 1 1 1 1 1 9 9 3

1 ab 1 bc 1 ca 1 ab 1 bc 1 ca 3 ab bc ca 6 2

αφού 2 2 2a b c ab bc ca 6 3 ab bc ca .

ΘΕΜΑ 191 (ΚΛΕΟΒΟΥΛΟΣ ΚΟΦΟΝΙΚΟΛΑΣ)

Αν 0

a,b,c R

και ισχύει ότι a+b+c=1 να αποδειχθεί ότι:

2 2 2(a+b) (b+c) (c+a)+ + 1

c+1 a+1 b+1 .

Λύση:

Η λύση με Andreescu είναι άμεση.

Θα δώσω μια λύση διαφορετική με ύλη για Seniors.

Είναι a b 1 c , a c 1 b , c b 1 a , άρα η ανισότητα γράφεται:

2 2 2(1 c) (1 a) (1 b)1

c 1 a 1 b 1

.

Έστω 2(1 x)

f (x) , x 0x 1

.

Page 161: Εισαγωγη σε διαγωνιστικα μαθηματικα για το γυμνασιο

http://www.mathematica.gr/forum/viewtopic.php?f=109&t=30104

Επιμέλεια: xr.tsif Σελίδα 161

Είναι 3

8f (x) 0

(x 1)

, άρα η f είναι κυρτή.

Επομένως f (a) f (b) f (c) a b c

f3 3

2 2 2 2 2 2(1 a) (1 b) (1 c) 1 (1 a) (1 b) (1 c)3f 1

a 1 b 1 c 1 3 a 1 b 1 c 1

.

Β τρόπος

Λοιπόν: 2 2 2 2(a b) (b c) (c a) [2(a b c)] 4

1c 1 a 1 b 1 a b c 3 4

.

ΘΕΩΡΗΜΑ:

Οι Ανισοϊσότητες Αριθμητικού – Γεωμετρικού Μέσου, Αριθμητικού –

Αρμονικού Μέσου, Γεωμετρικού – Αρμονικού Μέσου.

Αν θεωρήσουμε ένα πλήθος nθετικών πραγματικών αριθμών 1 2 n

a ,a ,...,a , τότε:

Ο αριθμητικός μέσος τους θα είναι ίσος με το άθροισμα των αριθμών αυτών δια

το πλήθος τους:

*Αν +a,b R , τότε ο αριθμητικός τους μέσος θα είναι ο a+b

2.

*Αν +a,b,c R , τότε ο αριθμητικός τους μέσος θα είναι ο a+b+c

3.

*Αν +a,b,c,d R , τότε ο αριθμητικός τους μέσος θα είναι ο a+b+c+d

4.

Page 162: Εισαγωγη σε διαγωνιστικα μαθηματικα για το γυμνασιο

http://www.mathematica.gr/forum/viewtopic.php?f=109&t=30104

Επιμέλεια: xr.tsif Σελίδα 162

ΓΕΝΙΚΑ:

Αν 1 2 n

a ,a ,...,a R , τότε ο αριθμητικός τους μέσος θα είναι ο 1 2 na +a +...+a

n.

Τον Αριθμητικό Μέσο θα τον συμβολίζουμε ως ΑΜ.

Ο γεωμετρικός μέσος τους θα είναι ίσος με το είδος της ρίζας που αντιστοιχεί

στο πλήθος τους του γινομένου τους:

*Αν +a,b R , τότε ο γεωμετρικός τους μέσος θα είναι ο ab .

*Αν +a,b,c R , τότε ο γεωμετρικός τους μέσος θα είναι ο 3 abc .

*Αν +a,b,c,d R , τότε ο γεωμετρικός τους μέσος θα είναι ο 4 abcd .

ΓΕΝΙΚΑ:

Αν 1 2 n

a ,a ,...,a R , τότε ο γεωμετρικός τους μέσος θα είναι ο n1 2 n

a a ...a .

Τον Γεωμετρικό Μέσο θα τον συμβολίζουμε ως GΜ.

Ο αρμονικός μέσος τους θα είναι ίσος με το πηλίκο του πλήθους των αριθμών

προς το άθροισμα των αντιστρόφων τους:

*Αν +a,b R , τότε ο αρμονικός τους μέσος θα είναι ο 2

1 1+

a b

.

*Αν +a,b,c R , τότε ο αρμονικός τους μέσος θα είναι ο 3

1 1 1+ +

a b c

.

Page 163: Εισαγωγη σε διαγωνιστικα μαθηματικα για το γυμνασιο

http://www.mathematica.gr/forum/viewtopic.php?f=109&t=30104

Επιμέλεια: xr.tsif Σελίδα 163

*Αν +a,b,c,d R , τότε ο αρμονικός τους μέσος θα είναι ο 4

1 1 1 1+ + +

a b c d

.

ΓΕΝΙΚΑ:

Αν 1 2 n

a ,a ,...,a R , τότε ο αρμονικός τους μέσος θα είναι ο

1 2 n

n

1 1 1+ +...+

a a a

.

Τον Γεωμετρικό Μέσο θα τον συμβολίζουμε ως HΜ.

Για τους τρεις μέσους θα ισχύει η ακόλουθη σχέση:

AM GM HM .

ΘΕΜΑ 192 (ΚΛΕΟΒΟΥΛΟΣ ΚΟΦΟΝΙΚΟΛΑΣ)

Δίνονται 0

a,b,c R

που ικανοποιούν τη συνθήκη abc 1 . Να αποδείξετε ότι:

2 2 2a +b +c 3 . Πότε ισχύει η ισότητα;

Λύση:

Από ΑΜ – GM έχουμε: 2 2 2 23a b c 3 (abc) 3 .

Η ισότητα ισχύει όταν a b c 1 .

ΘΕΜΑ 193 (ΚΛΕΟΒΟΥΛΟΣ ΚΟΦΟΝΙΚΟΛΑΣ)

Δίνονται +a,b,c,d R , να αποδείξετε ότι: 1 1 1 1

(a+b+c+d)( + + + ) 16a b c d

.

Δύο λύσεις:

α) Ανισότητα Αριθμητικού – Αρμονικού.

β) Ανισότητα Buniakowsky – Cauchy – Schwarz.

Page 164: Εισαγωγη σε διαγωνιστικα μαθηματικα για το γυμνασιο

http://www.mathematica.gr/forum/viewtopic.php?f=109&t=30104

Επιμέλεια: xr.tsif Σελίδα 164

Λύση:

Μιας που ασχολούμαστε με ΑΜ – GM θα δώσω με αυτήν λύση... Λοιπόν:

4a b c d 4 abcd .

41 1 1 1 1

4a b c d abcd .

Με πολλαπλασιασμό κατά μέλη παίρνουμε:

4 41 1 1 1 1

(a b c d)( ) 4 abcd·4a b c d abcd

1 1 1 1(a b c d)( ) 16

a b c d .

Διότι 44 4 41 1

abcd· ·abcd 1 1abcd abcd

.

ΘΕΜΑ 194 (ΜΑΓΚΟΣ ΘΑΝΟΣ)

Αν a,b,c 0 να αποδείξετε ότι

3 3 33 33

a+b+c a+1+ b+1+ c+1+1 (a+1)(b+1)(c+1) abc+1

3 3( ) .

Λύση:

Ας κάνω μία προσπάθεια:

Από την ανισότητα αριθμητικού – γεωμετρικού μέσου έχω:

3a+b+c (a+1)+(b+1)+(c+1)

+1= (a+1)(b+1)(c+1)3 3

.

Μετά πάλι από AM – ΓΜ:

Page 165: Εισαγωγη σε διαγωνιστικα μαθηματικα για το γυμνασιο

http://www.mathematica.gr/forum/viewtopic.php?f=109&t=30104

Επιμέλεια: xr.tsif Σελίδα 165

3 3 3

3 3a+1+ b+1+ c+1

(a+1)(b+1)(c+1)3

και υψώνοντας στον κύβο πιάνω ισότητα και ούτω καθεξής...

Από την ανισότητα ΑΜ – ΓΜ έχω:

3(a+1)+(b+1)+(c+1)

(a+1)(b+1)(c+1)3

και επίσης

3 3 3

3 3a+1+ b+1+ c+1

(a+1)(b+1)(c+1)3

.

Με ύψωση στον κύβο πιάνω ισότητα και αποδεικνύεται κατά τον ίδιο τρόπο.

Υψώνοντας στον κύβο έχω ότι: 23 3(a 1)(b 1)(c 1) abc 3 abc 3 (abc) 1

που ισχύει για τον λόγο ότι: 3a b c 3 abc και 23ab bc ca 3 (abc) ,

που με πρόσθεση κατά μέλη μας δίνουν τη ζητούμενη.

Αποδεικνύω την πρώτη ανισότητα:

Έχουμε:

3 3 33 3 3a b c (a 1) (b 1) (c 1) ( a 1) ( b 1) ( c 1)1

3 3 3 3 3

3 3 3 3 3 33

3 2

a 1 b 1 c 1 a 1 b 1 c 1( )

3 (3 3 3) 3

.

(Χρησιμοποιήσαμε την γενικευμένη ανισότητα Andreescu).

Δίνω και την απόδειξη σε αυτήν την ανισότητα:

Στην ανισότητα του HOLDER,

Αν x,t,e,y,n,k,z,q,r 0 , τότε:

1 1 13 3 3 3 3 3 3 3 33 3 3(x t e ) (y n k ) (z q r ) xyz tnq ckr

Page 166: Εισαγωγη σε διαγωνιστικα μαθηματικα για το γυμνασιο

http://www.mathematica.gr/forum/viewtopic.php?f=109&t=30104

Επιμέλεια: xr.tsif Σελίδα 166

(όπου η ανισότητα αυτή ισχύει και όταν e k r 0 ) .

Αν θέσουμε 3 3 3x a,y b,z c,t n q 1,e k r 0 , παίρνουμε:

1 1 1

3 3 33 3 3(a 1 0) (b 1 0) (c 1 0) a b c 1 1 1 0 , και άρα το ζητούμενο.

ΘΕΜΑ 195 (ΜΑΓΚΟΣ ΘΑΝΟΣ)

Αν 5 3a=x x +x , να αποδείξετε ότι 6x 2a 1 .

Λύση:

Έχουμε 4 2a x x x 1 (1) .

Επίσης (1)

6 2 3 2 4 2 4 2x 2a 1 (x ) 1 2a (x 1)(x x 1) 2x(x x 1)

4 2 2 4 2 2(x x 1)(x 2x 1) 0 (x x 1)(x 1) 0 , που ισχύει γιατί

4 2 4 2 2 2 2 2x x 1 x 2x 1 x (x 1) x 0 και 2(x 1) 0 .

ΘΕΜΑ 196 (ΔΗΜΗΤΡΗΣ ΙΩΑΝΝΟΥ)

Αν k,m,n,r είναι οι θετικοί αριθμοί a,b,c,d γραμμένοι με τυχαία σειρά, (δηλ.

ο k είναι κάποιος από τους a,b,c,d , o mεπίσης κάποιος από τους υπόλοιπους

κλπ), να αποδείξετε ότι: k m n r

4a b c d .

Λύση:

Με χρήση της ανισοϊσότητας αριθμητικού – γεωμετρικού μέσου έχω ότι:

4 4k m n r k m n r kmnr

+ + + 4 · · · =4a b c d a b c d abcd

.

Όμως από την υπόθεση ισχύει ότι abcd=kmnr άρα:

Page 167: Εισαγωγη σε διαγωνιστικα μαθηματικα για το γυμνασιο

http://www.mathematica.gr/forum/viewtopic.php?f=109&t=30104

Επιμέλεια: xr.tsif Σελίδα 167

44 4k m n r k m n r kmnr

+ + + 4 · · · =4 =4 1=4a b c d a b c d abcd

.

ΘΕΜΑ 197 (ΑΡΓΥΡΗΣ ΚΑΡΑΝΙΚΟΛΑΟΥ)

Έστω 1 2 n

a ,a ,...,a θετικοί πραγματικοί αριθμοί τέτοιοι ώστε 1 2 n

a a ...a 1 .

Να δειχθεί ότι n

1 2 n(1 a )(1 a )...(1 a ) 2 .

Λύση:

Από ΑΜ – ΓΜ θα ισχύει ότι:

1 1a 1 2 a

2 2a 1 2 a

3 3a 1 2 a

……

n na 1 2 a .

Με πολλαπλασιασμό των σχέσεων κατά μέλη έχω την ανισοϊσότητα: n n

1 2 3 n 1 2 3 n(a 1)(a 1)(a 1) ... (a 1) 2 a a a ...a 2 , αφού

1 2 na a ...a 1 .

ΘΕΜΑ 198 (ΣΩΚΡΑΤΗΣ ΛΥΡΑΣ)

Αν a,b,c 0 και a+b+c=1, να αποδείξετε ότι: ab bc ca 1

1 c 1 a 1 b 4

.

Λύση:

Μια λύση με AM – GM, ύστερα από αρκετή προσπάθεια

Έχουμε: c 1 c a b c (a c) (b c) 2 (a c)(b c)

Page 168: Εισαγωγη σε διαγωνιστικα μαθηματικα για το γυμνασιο

http://www.mathematica.gr/forum/viewtopic.php?f=109&t=30104

Επιμέλεια: xr.tsif Σελίδα 168

2 c 1 (a c)(b c) 4 c 1(c 1) 4(a c)(b c)

4 c 1 c 1 (a c)(b c)

4ab ab(c a b c) 4ab 1 1ab

c 1 (a c)(b c) c 1 b c a c

.

Άρα: 4ab ab ab

c 1 b c a c

(1) .

Ομοίως βρίσκουμε ότι: 4bc bc bc

a 1 b a c a

(2) ,

και 4ca ca ca

b 1 c b a b

(3) .

Με πρόσθεση κατά μέλη των (1) , (2) , (3) έχουμε:

4ab 4bc 4ca ab ca ab bc bc caa b c 1

c 1 a 1 b 1 b c a c b a

.

Άρα: ab bc ca 1

1 c 1 a 1 b 4

.

Β τρόπος

Είναι

ab bc ca 1 1 1+ + =abc =abc =

c+1 a+1 b+1 c(c+1) c c+1( )

1 9=ab+bc+ca abc ab+bc+ca abc

c+1 4 , λόγω της

1 1 1 9 9

a 1 b 1 c 1 a b c 3 4

.

Πλέον αρκεί να αποδειχθεί ότι 9 1

ab bc ca abc4 4

, δηλαδή ότι

3(a b c) 9abc 4(a b c)(ab bc ca) , η οποία είναι η Schur!

Page 169: Εισαγωγη σε διαγωνιστικα μαθηματικα για το γυμνασιο

http://www.mathematica.gr/forum/viewtopic.php?f=109&t=30104

Επιμέλεια: xr.tsif Σελίδα 169

ΘΕΜΑ 199 (ΘΑΝΟΣ ΜΑΓΚΟΣ)

Να βρείτε τη μέγιστη τιμή της συνάρτησης

2 2f (a,b)=sin a+sin b+cosa+cosb+cosacosb , a,b R .

Λύση:

2 2 2 2sin a+cos b=1 f(a,b)=2 cos a cos b+cosa+cosb+cosa·cosb=P .

Αν x=cosa και y=cosb τότε 2 2P=2 x y +x+y+xy και λόγω των

2 2 22(x +y ) (x+y) και 24xy (x+y) , προκύπτει ότι

2sP s +2=g(s)

4 , όταν s=x+y και με μελέτη της g προκύπτει ότι Pmax 3

όταν cosa=cosb=1.

ΘΕΜΑ 200 (ΑΡΓΥΡΗΣ ΚΑΡΑΝΙΚΟΛΑΟΥ)

Έστω a,b,c θετικοί πραγματικοί αριθμοί τέτοιοι ώστε a b c 3 . Δείξτε ότι b c aa b c 1 .

Λύση:

Είναι b c a

b c a33 3 3b c a ab+bc+ca

a +b +c a b c a b c3 3 3 3

,

Η απόδειξη ολοκληρώνεται αν θυμηθούμε πως 2(a+b+c)

ab+bc+ca3

.

ΘΕΜΑ 201 (ΑΡΓΥΡΗΣ ΚΑΡΑΝΙΚΟΛΑΟΥ)

Αν x,y,z 0 , να αποδείξετε ότι: 8 2 42 xy z (x 2y z) .

Λύση:

24x+2y+z=x+y+y+z 4 xy z . Άρα 4 4 2 8 2(x+2y+z) 4 xy z=2 xy z , όπως θέλαμε.

Page 170: Εισαγωγη σε διαγωνιστικα μαθηματικα για το γυμνασιο

http://www.mathematica.gr/forum/viewtopic.php?f=109&t=30104

Επιμέλεια: xr.tsif Σελίδα 170

ΘΕΜΑ 202 (ΚΩΣΤΑΣ ΖΕΡΒΟΣ)

Αν 1 2 n

a ,a , ,a θετικοί αριθμοί με γινόμενο 1, τότε να αποδειχτεί ότι

n 1 n 1 n 1 n 1

1 2 2 3 n 1(a a ) (a a ) (a a ) n2 , *n N .

Λύση:

Με εφαρμογή της ΑΜ – ΓΜ παίρνουμε :

n 1 n 1 n 1 n 1n1 2 2 3 n 1 1 2 n 1

(a a ) (a a ) (a a ) n( (a a ) ... (a a )) .

Αλλά: x y 2 xy .

Την ανισότητα αυτή εφαρμόζουμε σε όλους τους παράγοντες του υπόριζου.

Επειδή οι αριθμοί 1 2 n

a ,a , ,a έχουν γινόμενο 1, μετά την τελείως βασική

απλοποίηση , παίρνουμε τη ζητούμενη.

ΘΕΜΑ 203 (ΣΩΤΗΡΗΣ ΛΟΥΡΙΔΑΣ)

Υπολογίστε τον μέγιστο στοιχείο του συνόλου:

4 n3A 1, 2, 3, 4,..., n,... με n 2,3,... .

Λύση:

Πιστεύω ότι το μέγιστο στοιχείο του δοσμένου συνόλου είναι το 3 3 .

Για την πρώτη τριάδα αριθμών ισχύει το ακόλουθο: 31 2 3 . Για την

δεύτερη τριάδα ισχύει ότι 4 5 64 5 6 . Όμως 4 4= 2 άρα η προηγούμενη

τριάδα είναι επίσης μικρότερη από το 3 3 . Η ίδια συνθήκη ισχύει και για τα

υπόλοιπα στοιχεία του συνόλου. Έχουμε ως μέγιστο στοιχείο το 3 3 και ως

ελάχιστο το 1, διότι κάθε ρίζα της μορφής n n 1 όσο και αν αυξάνονται τα

ψηφία μετά την υποδιαστολή.

Page 171: Εισαγωγη σε διαγωνιστικα μαθηματικα για το γυμνασιο

http://www.mathematica.gr/forum/viewtopic.php?f=109&t=30104

Επιμέλεια: xr.tsif Σελίδα 171

Β τρόπος

Για να την κλείσουμε αυτή, για n 4 εφαρμόζουμε την ανισότητα ΑΜ – ΓΜ

στα 1 2 3 4 5 n

nx n,x x ,x 4,x x 1

2 για να πάρουμε

n

n 3

nn 2 4 (n 4)

n n23 n 4 nn 2 2

.

Άρα n 3n 3 για κάθε n 4 και είναι απλό μετά να ελεγχθεί ότι το μέγιστο

στοιχείο του συνόλου είναι το 3 3 .

Η ανισότητα 3 nn 3 για n 4 μπορεί να αποδειχθεί και επαγωγικά.

Για όφελος των μαθητών μας, ας το δούμε:

Το επαγωγικό βήμα με υπόθεση 3 NN 3 , όπου N 4 δοθείς, είναι

3 3 2 3 2 2 2 3 2(N 1) N 3N 3N 1 N 3N 3N N N 7N

3 2 3 2 3 N N 1N 2·4·N N 2·N·N 3N 3·3 3 .

Γ τρόπος

Ας την δούμε και έτσι και μόνο χάριν της Μαθηματικής πολυφωνίας:

Πράγματι, κατ’ αρχάς έχουμε: 3 3max 1, 2, 3 3 .

Γνωρίζουμε ότι η ακολουθία

n

n

1a 1

n

έχει τις εξής ιδιότητες:

n n(a , a 3) .

Άρα παίρνουμε:

n n 1 nnn n 1n 3 n 1 3n n 3 n 1 n n 1 n .

Page 172: Εισαγωγη σε διαγωνιστικα μαθηματικα για το γυμνασιο

http://www.mathematica.gr/forum/viewtopic.php?f=109&t=30104

Επιμέλεια: xr.tsif Σελίδα 172

Συνεπώς το ζητούμενο μέγιστο είναι το 3 3 .

Δ τρόπος

Δίνω ακόμα μια λύση εκτός φακέλου.

Θεωρώ την συνάρτηση 1 1

lnxx xf (x) x e με x 0 .

Είναι 1

x

2

1f (x) x (1 lnx)

x .

Βρίσκουμε με πρόσημο παραγώγου πως η f είναι γν. αύξουσα στο (0,e] και γν.

φθίνουσα στο [e, ) .

Άρα f (1) f (2) f (e),f (n) f (3) f (e),n 4 .

Αρκεί να συγκρίνουμε λοιπόν το f (2) 2 και το 3f (3) 3 .

Έστω 3 26 63 6 62 3 2 3 8 9 που προφανώς ισχύει , άρα ισχύει και

η αρχική υπόθεση.

Άρα ο 3 3 είναι ο μεγαλύτερος από του δοσμένους αριθμούς.

ΘΕΜΑ 204 (ΣΩΤΗΡΗΣ ΛΟΥΡΙΔΑΣ)

Αποδείξτε την αλήθεια της συνεπαγωγής:

a b a b* 2 2 a ba,b / a b a b a b

.

Λύση:

Η ανισότητα γράφεται :

a+b2 2

a ba +ba ·b

a+b

.

Ο αριθμητής του LHS γράφεται a+a+...+a+b+b+...+b όπου το πλήθος των a

είναι a και των b είναι b .

Και τώρα απλά ΑΜ – ΓΜ.

Page 173: Εισαγωγη σε διαγωνιστικα μαθηματικα για το γυμνασιο

http://www.mathematica.gr/forum/viewtopic.php?f=109&t=30104

Επιμέλεια: xr.tsif Σελίδα 173

Υπέθεσα ότι a,b N .

Σωκράτη, ουσιαστικά η κεντρική ιδέα της λύσης είναι αυτή που έγραψες.

Υπάρχει μόνο το κενό , στο ότι θεώρησες τους αριθμούς ,a b ως φυσικούς,

πράγμα που η εκφώνηση το αποκλείει.

Η λύση που έδωσες σώζεται, αν θέσεις m

an

και k

br

, με m,n,k,r N .

Συγκεκριμένα, πρέπει τότε να αποδείξουμε ότι

mr kn2 2 2 2 m knr

n rm r n k m k

( ) ( )(mr kn)nr n r

.

Όμως,

2 2 2 2m r n k (mr)(mr) (nk)(nk) mr mr ... mr nk nk ... nk

mr nkmr nk mr nk(mr nk) mr mr ... mr nk nk ... nk (mr nk) (mr) (nk) .

Άρα έχουμε: 2 2 2 2 mr nk mr nk mr nk(m r n k ) (mr nk) (mr) (nk) .

Άρα 2 2 2 2

mr nk mr mr nk nkm r n k( ) m r n k

mr nk

,

και άρα:

mr nk2 2 2 2 mr nk

nk nk mr mr mr nk

m r n k 1 m k

mr nk n r n r n r

.

Άρα

mr nk2 2 2 2

mr nk

mr nk

m r n k 1 m k( ) ( )

mr nk (nr) n r

και η απόδειξη ολοκληρώθηκε.

Η ανισότητα γενικεύεται και όταν οι a,b είναι θετικοί πραγματικοί.

Πράγματι έστω ότι a b r . Τότε a b

1r r οπότε από την ανισότητα των

βαρών έχουμε: a b

r ra b

a b a br r

, δηλαδή

a b2 2

a ba ba b

a b

που είναι και το

ζητούμενο.

Page 174: Εισαγωγη σε διαγωνιστικα μαθηματικα για το γυμνασιο

http://www.mathematica.gr/forum/viewtopic.php?f=109&t=30104

Επιμέλεια: xr.tsif Σελίδα 174

Ουσιαστικά αποτελεί παρόμοιο θέμα με το θέμα του 3 Ευκλείδης 2006 της Β

Λυκείου και η γενίκευσή του βρίσκεται εδώ.

ΘΕΜΑ 205 (ΚΛΕΟΒΟΥΛΟΣ ΚΟΦΟΝΙΚΟΛΑΣ)

Αν x,y,z,w 0 να αποδείξετε ότι: (x+y)(y+z)(z+w)(w+x) 16xyzw .

Λύση:

x y 2 xy ,

y z 2 yz ,

z w 2 zw ,

x w 2 xw .

Πολλαπλασιάζοντας κατά μέλη έχουμε ότι

2(x y)(y z)(z w)(w x) 16 (xyzw)

(x y)(y z)(z w)(w x) 16xyzw .

ΘΕΩΡΗΤΙΚΑ

Η ανισότητα B – C – S (Buniakowsky – Cauchy – Schwarz)

Αν και έχει αναφερθεί σε άλλη δημοσίευση, υπενθυμίζουμε την πολύ χρήσιμη

αυτή ανισότητα:

Θεωρούμε τους πραγματικούς αριθμούς: 1 2 3 n 1 2 3 n

a ,a ,a ,...,a x ,x ,x ,...,x ,.

Τότε : 2 2 2 2 2 2 2

1 2 n 1 2 n 1 1 2 2 n n(a a ... a )(x x ... x ) (a x a x ... a x ) .

Η ισότητα ισχύει όταν 1 2 n

1 2 n

a a a...

x x x .

Page 175: Εισαγωγη σε διαγωνιστικα μαθηματικα για το γυμνασιο

http://www.mathematica.gr/forum/viewtopic.php?f=109&t=30104

Επιμέλεια: xr.tsif Σελίδα 175

ΠΑΡΑΔΕΙΓΜΑ

Δείξτε ότι: 2 2 2

2 2 2 2 2 2 2 2 2 2

2 2 2

x y z(x y y z z x )( ) (x y z )

y z x .

AΠΟΔΕΙΞΗ

Θεωρούμε τους αριθμούς: x y z

xy,yz,zx, , ,y z x

.

Τότε από την ανισότητα B – C – S έχουμε:

2

2 2 2 2 2 2x y z x y z(xy) (yz) (zx) ( ) ( ) ( ) xy yz zx

y z x y z x

.

Και άρα: 2 2 2

2 2 2 2 2 2 2 2 2 2

2 2 2

x y z(x y y z z x )( ) (x y z )

y z x .

ΘΕΜΑ 206 (ΔΗΜΗΤΡΗΣ ΙΩΑΝΝΟΥ)

Να αποδείξετε ότι για τους θετικούς αριθμούς a,b,c, ισχύει:

14(a b c) a 4b 9c .

Λύση:

Υψώνουμε και τα δύο μέλη στο τετράγωνο και πρέπει να δείξουμε ότι

2

214(a b c) ( a 2 b 3 c) (1 4 9)(a b c) a 2 b 3 c ,

το οποίο ισχύει από την προαναφερθείσα ανισότητα.

ΘΕΜΑ 207 (ΔΗΜΗΤΡΗΣ ΙΩΑΝΝΟΥ)

Αν για τους θετικούς αριθμούς a,b,c ισχύει ότι: a b c 7

b c a 2 , να

αποδείξετε ότι: 1 1 1

4(a b c)( ) 49a b c

.

Page 176: Εισαγωγη σε διαγωνιστικα μαθηματικα για το γυμνασιο

http://www.mathematica.gr/forum/viewtopic.php?f=109&t=30104

Επιμέλεια: xr.tsif Σελίδα 176

Λύση:

Όμορφη. 1 1 1 1 1 1

(a+b+c)( + + )=(a+b+c)( + + )a b c b c a

.

Για τις τριάδες ( a, b, c) και 1 1 1

( , , )b c a

,

από την ανισότητα Buniakowski – Cauchy – Schwarz ισχύει ότι:

2 21 1 1 a b c 7 49(a+b+c)( + + ) ( + + ) =( ) =

b c a b c a 2 4 . Πολλαπλασιάζω και τα δύο

μέλη με τον αριθμό 4και έτσι προκύπτει η ζητούμενη.

ΘΕΜΑ 208 (ΔΗΜΗΤΡΗΣ ΙΩΑΝΝΟΥ)

Αν a,b,c είναι θετικοί πραγματικοί αριθμοί, να βρεθεί η ελάχιστη τιμή της

παράστασης: 2 2 2 2

2 2 2 2

a a b cA (1 )(1 )

b c a a , καθώς και οι τιμές των a,b,c για

τις οποίες αυτό επιτυγχάνεται.

Λύση:

Από την ανισοϊσότητα Buniakowski – Cauchy – Schwarz για τις τριάδες

a a(1, , )

b c

και b c

(1, , )a a

ισχύει ότι: 2 2a·b a·cA (1+ + ) =3 =9

b·a c·a .

Η ελάχιστη τιμή θα επιτυγχάνεται κατά την ισότητα, δηλαδή όταν A 9 . Είναι

γνωστό όμως ότι η ισότητα ισχύει όταν θα ισχύει ότι: 4 4

4 4

a a= =1 a=b=c

b c .

Page 177: Εισαγωγη σε διαγωνιστικα μαθηματικα για το γυμνασιο

http://www.mathematica.gr/forum/viewtopic.php?f=109&t=30104

Επιμέλεια: xr.tsif Σελίδα 177

Β τρόπος

Θεωρούμε τους αριθμούς 1 2 3 1 2 3

a a b ca 1,a ,a ,x 1,x ,x

b c a a .

Από την ανισότητα B C S , έχουμε

22 2 2 2 2 2

1 2 3 1 2 3 1 2 2 2 3 3a a a x x x a x a x a x A 9 .

Η ισότητα επιτυγχάνεται, όταν a b c 0 και άρα min

A 9 .

Γ τρόπος

Κάνοντας τις πράξεις στην παράσταση του A , βρίσκουμε

2 2 2 2 2 2

a b a c c bA 3

b a c a b c

.

Όμως, για κάθε x 0 έχουμε

22 x 11 x 2x 1 1x 2 0 x 2

x x x x

,

με την ισότητα, να επιτυγχάνεται, αν, και μόνο αν, x 1 .

Εφαρμόζοντας αυτήν διαδοχικά, για a a b

x 0,x 0,x 0b c c

, προκύπτει

A 3 2 2 2 9 .

Η ισότητα ισχύει, αν , και μόνο αν, a a b

1 a b c 0b c c .

Συνεπώς, min

A 9 .

Δ τρόπος

Απλούστερη είναι η λύση με ΑΜ – ΓΜ.

Είναι 4 2 2

3 32 2 4

a b cA 3 ·3 9

b c a .

Η ισότητα ισχύει, αν και μόνο αν, a b c .

Page 178: Εισαγωγη σε διαγωνιστικα μαθηματικα για το γυμνασιο

http://www.mathematica.gr/forum/viewtopic.php?f=109&t=30104

Επιμέλεια: xr.tsif Σελίδα 178

ΘΕΜΑ 209 (ΔΗΜΗΤΡΗΣ ΙΩΑΝΝΟΥ)

Αν a,b 0 , να αποδείξετε ότι:

2 2 4 4 2 23 (a b 1)(a b 2) (a b 1)(a b 2) .

Λύση:

Από Cauchy – Schwarz έχουμε

2 2 2(a b 1)(1 1 1) (a b 1) ή 2 23(a b 1) (a b 1) .

4 4 2 2 2(a b 2)(1 1 1) (a b 2) ή 4 4 2 23(a b 2) (a b 2) .

Με πολλαπλασιασμό κατά μέλη των δύο παραπάνω, το ζητούμενο έπεται.

Σημείωση:

Η συνθήκη ότι πρέπει να είναι μη αρνητικά δεν είναι περιττή; Αφού το απόλυτο

οποιουδήποτε αριθμού είναι μεγαλύτερο από τον αριθμό.

ΘΕΜΑ 210 (ΔΗΜΗΤΡΗΣ ΙΩΑΝΝΟΥ)

Αν 2 2 2a b c 1 τότε: 2 2 2 4 2 2 2 2a b c a (a c b a c b) .

Λύση:

Έχουμε ότι 2 2 2a b c 1 ή 2 2 2b c 1 a .

Από Cauchy – Schwarz έχουμε

2 2 2 2 2 2 2 2 2 2((ac) (ba) (bc) )(a b c ) (a c b a c b) ή

2 2 2 2 2 2 2(ac) (ba) (bc) (a c b a c b) (*) .

Όμως,

2 2 2 2 2 2 2 2 2 2 2 2 2 4 2 2(ac) (ba) (bc) a (b c ) b c a (1 a ) b c a a b c (**) .

Από τις (*) και (**) , το ζητούμενο έπεται.

Page 179: Εισαγωγη σε διαγωνιστικα μαθηματικα για το γυμνασιο

http://www.mathematica.gr/forum/viewtopic.php?f=109&t=30104

Επιμέλεια: xr.tsif Σελίδα 179

ΘΕΜΑ 211 (ΔΗΜΗΤΡΗΣ ΙΩΑΝΝΟΥ)

Να αποδειχθεί ότι αν x,y,z 0 τότε: x y z 3

y z z x x y 2

.

(Πηγή: Αλγεβρικές Ανισότητες , των ΜΠΑΜΠΗ ΣΤΕΡΓΙΟΥ και ΝΙΚΟΥ

ΣΚΟΜΠΡΗ)

ΣΗΜΕΙΩΣΗ

Η άσκηση αυτή, αντιμετωπίζεται εύκολα π.χ με την Andreescu.

Εδώ, ζητάμε μια απόδειξη με την χρήση της B – C – S.

Λύση:

Πρόκειται για την ανισότητα Nesbitt. Υπάρχουν πολλές αποδείξεις, αλλά ας

δώσω μία που μοιάζει περίεργη σε όσους δεν έχουν ασχοληθεί με ανισότητες.

Λόγω ομογένειας μπορούμε να υποθέσουμε x y z 1 οπότε η αποδεικτέα

γράφεται x y z 3

1 x 1 y 1 z 2

.

Τώρα, εύκολα δείχνουμε ότι x 9x 1

1 x 4

(αν κάνουμε τις πράξεις ισοδυναμεί

με την αληθή 2(3x 1) 0 ).

Όμοια για τις άλλες μεταβλητές, y 9y 1 z 9z 1

,1 y 4 1 z 4

.

Προσθέτουμε κατά μέλη τις τρεις θα βρούμε

x y z 9(x y z) 3 9·1 3 3

1 x 1 y 1 z 4 4 2

, όπως θέλαμε.

Β τρόπος

Έχουν ήδη δοθεί πολλές λύσεις για την ανισότητα αυτή.

Για εξάσκηση στην B – C – S, γράφω και μία ακόμα:

Page 180: Εισαγωγη σε διαγωνιστικα μαθηματικα για το γυμνασιο

http://www.mathematica.gr/forum/viewtopic.php?f=109&t=30104

Επιμέλεια: xr.tsif Σελίδα 180

Θεωρούμε τις τριάδες: x y z

, ,y z x z x y

και

x(y z), y(x z), z(x y) .

Τότε:

2x y z( )[x(y z) y(x z) z(x y)] (x y z)y z z x x y

.

Άρα: 2x y z

( )(2xy 2yz 2xz) (x y z)y z x z x y

.

Άρα: 2x y z (x y z)

y z x z x y 2(xy yz zx)

.

Αρκεί λοιπόν να αποδείξουμε ότι: 2(x y z) 3

2(xy yz zx) 2

,

ή ότι: 2(x y z) 3(xy yz zx) , η οποία είναι προφανής.

Γ Τρόπος

Πράγματι, υπάρχουν πολλές αποδείξεις αυτής της ανισότητας.

Μια ακόμα με Cauchy – Schwarz (ή αν θέλει κανείς, με ανισότητα Αριθμητικού

– Αρμονικού Μέσου) είναι η εξής:

x y z x y z x+y+z x+y+z x+y+z+ + = +1+ +1+ +1 3= + + 3=

y+z z+x x+y y+z z+x x+y y+z z+x x+y

1 1 1 9 9 3=(x+y+z) + + 3 (x+y+z) 3= 3=

x+y y+z z+x x+y+y+z+z+x 2 2( ) .

Δ τρόπος

Ιδού και μία ουρανοκατέβατη λύση με χρήση της ταυτότητας

2 2 2x y z 3 1 (x y) (x z) (y z)

y z z x x y 2 2 (x z)(y z) (x y)(y z) (x y)(x z)( )

.

Page 181: Εισαγωγη σε διαγωνιστικα μαθηματικα για το γυμνασιο

http://www.mathematica.gr/forum/viewtopic.php?f=109&t=30104

Επιμέλεια: xr.tsif Σελίδα 181

Πραγματικά, η ανισότητα αποδεικνύεται άμεσα!

Αυτό που με προβληματίζει με τέτοιες λύσεις είναι πως καταλήγουμε σε τέτοιες

ταυτότητες, γιατί όπως φαίνεται αποτελούν πανίσχυρο εργαλείο επίλυσης

ανισοτήτων. Προφανώς, έχει να κάνει με συμμετρικά πολυώνυμα, αλλά μου

είναι ακόμα αόριστο το πως και το γιατί.

Ε τρόπος

Και αλλιώς.

Γράφω τον παρακάτω τρόπο γιατί δεν θέλει σχεδόν καθόλου φαντασία αλλά

μόνο πράξεις ρουτίνας. Σχολιάζω ότι καμιά φορά ξεχνάμε τα απλά και

ψάχνουμε τα επιτηδευμένα:

Πολλαπλασιάζουμε με τους παρονομαστές και τα φέρνουμε όλα στο πρώτο

μέλος. Η αποδεικτέα γίνεται ισοδύναμα

3 3 3 2 2 2 2 2 22x 2y 2z x y xy y z zy x zx x y 0 , δηλαδή

3 3 2 2 3 3 2 2 3 3 2 2(x y x y xy ) (y z y z zy ) (z x z x xz ) 0 .

Αλλά αυτό είναι απλό. Π.χ. ο πρώτος προσθετέος ισούται 2 2 2x (x y) y (x y) (x y) (x y) 0 , και λοιπά.

Στ τρόπος

Ένα τέτοιο μη επιτηδευμένο είναι και η μέθοδος της απαγωγής σε άτοπο, η

οποία καλό θα είναι να "κυκλοφορεί" στην σκέψη μας.

Ας εφαρμόσουμε λοιπόν την (B – C – S) ως έξης:

Αν 3 x 9 1

x y z2 y z 2 y z

Πρόσθεση και στα δύο μέλη του

3 1 1 1 και εξαγωγή του a b c ως κοινού παράγοντα έχουμε

(B C S)1

9 2 x y z 9 9y z

, που είναι άτοπο.

Page 182: Εισαγωγη σε διαγωνιστικα μαθηματικα για το γυμνασιο

http://www.mathematica.gr/forum/viewtopic.php?f=109&t=30104

Επιμέλεια: xr.tsif Σελίδα 182

Συνεπώς ισχύει: 3 x

2 y z .

Ζ τρόπος

Να και μία αντιμετώπιση με την ανισότητα της αναδιάταξης.

Θεωρούμε τη συνάρτηση x y z

f (x,y,z)y z z x y x

.

Παρατηρούμε ότι

f (x,y,z) f (y,z,x) f (z,x,y) f (x,z,y) f (y,x,z) f (z,y,z) .

Επομένως, πρόκειται περί συμμετρικής συνάρτησης και μπορούμε δίχως βλάβη

της γενικότητας να υποθέσουμε ότι

x y z ή x y x z y z ή 1 1 1

x y x z y z

.

Εφαρμόζοντας την ανισότητα της αναδιάταξης στις αύξουσες ακολουθίες

1 1 1(x,y,z), , ,

y z x z x y( )

έχουμε

x y z z x y

y z x z x y y z x z x y

,

x y z y z x

y z x z x y y z x z x y

, με τις αναδιατάξεις

s(1) (z,x,y),s(2) (y,z,x) , αντίστοιχα.

Με πρόσθεση κατά μέλη προκύπτει η ζητούμενη.

Η τρόπος

Να και μία λύση με Jensen που μου έδωσε ο αδερφός μου που ασχολείται με

optimization problems.

Οι μικρότεροι να μην δώσετε σημασία σε αυτή τη λύση.

Έστω τυχαίος t 0 . Παρατηρούμε ότι για την συνάρτηση f (x,y,z) ισχύει

Page 183: Εισαγωγη σε διαγωνιστικα μαθηματικα για το γυμνασιο

http://www.mathematica.gr/forum/viewtopic.php?f=109&t=30104

Επιμέλεια: xr.tsif Σελίδα 183

of (tx,ty,tz) t f (x,y,z) .

Συνεπώς, η f είναι ομογενής συνάρτηση μηδενικού βαθμού και επομένως δίχως

βλάβη της γενικότητας μπορούμε να υποθέσουμε ότι x y z 1 .

Έτσι, η συνάρτηση γράφεται x y z

f (x,y,z)1 x 1 y 1 z

.

2

2 3

d f 20

da (1 a)

, αφού x,y,z (0,1) , όπου a μία από τις μεταβλητές x,y,z .

Έτσι, η συνάρτηση gπου ορίζεται από τη σχέση a

g(a)1 a

είναι κυρτή στο

πεδίο ορισμού της D(g) (0,1) και επομένως εφαρμόζεται η ανισότητα Jensen.

Άρα,

x y z g(x) g(y) g(z)g( )

3 3

ή

1 f (x,y,z)g( )

3 3 ,

και αφού 1 1

g( )3 2 , το ζητούμενο έπεται.

ΘΕΜΑ 212 (ΚΛΕΟΒΟΥΛΟΣ ΚΟΦΟΝΙΚΟΛΑΣ)

Δίνονται θετικοί πραγματικοί αριθμοί a,b,c,d με άθροισμα ίσο με τη μονάδα.

Να αποδείξετε, με χρήση της ανισότητας Βuniacowski – Cauchy – Schwarz ότι

ισχύει ότι: 2 2 2 2a b c d 1

a b b c c d d a 2

.

Λύση:

Με B – C – S για τις τετράδες ( a b, b c, c d, d a) και

a b c d( , , , )

a b b c c d d a .

Πιο αναλυτικά έχω :

Page 184: Εισαγωγη σε διαγωνιστικα μαθηματικα για το γυμνασιο

http://www.mathematica.gr/forum/viewtopic.php?f=109&t=30104

Επιμέλεια: xr.tsif Σελίδα 184

2 2 2 2( a b) ( b c) ( c d) ( d a)

2 2 2 2 2a b c d( ) ( ) ( ) ( ) (a b c d)

a b b c c d d a

2 2 2 2a b c d 12(a b c d)( ) 1 L.H.S

a b b c c d d a 2

.

ΘΕΜΑ 213 (ΔΗΜΗΤΡΗΣ ΙΩΑΝΝΟΥ)

Αν a 5,b 5 , να αποδείξετε ότι: 5a 25 5b 25 ab .

Λύση:

Θέτουμε a 5 x και b 5 y . Από C – S έχουμε

2(x 5)(y 5) ( 5x 5x) ή (x 5)(y 5) 5x 5y , και το ζητούμενο

έπεται.

ΘΕΜΑ 214 (ΚΛΕΟΒΟΥΛΟΣ ΚΟΦΟΝΙΚΟΛΑΣ)

Να αποδείξετε ότι για κάθε a,b,c,d,x,y,z,w R ισχύει ότι:

2 2 2

2 2 2 2 2 2y z w(a +2b +3c +4d )(x + + + ) (ax+by+cz+dw)

2 3 4 .

Λύση:

Για τις τετράδες των αριθμών y z w

(a, 2b, 3c,2d),(x, , , )22 3

,

εφαρμόζουμε την ανισότητα B – C – S και έπεται.

ΘΕΜΑ 215 (ΚΛΕΟΒΟΥΛΟΣ ΚΟΦΟΝΙΚΟΛΑΣ)

Αν a,b,c,d R να δειχθεί ότι: 1 1 1 1(a b c d)(a b c d ) 16 .

με την ανισοϊσότητα Βuniacowski – Cauchy – Schwarz . (AM – GM είδαμε).

Page 185: Εισαγωγη σε διαγωνιστικα μαθηματικα για το γυμνασιο

http://www.mathematica.gr/forum/viewtopic.php?f=109&t=30104

Επιμέλεια: xr.tsif Σελίδα 185

Λύση:

Θεωρούμε τις τετράδες ( a, b, c, d)και 1 1 1 1( a , b , c , d )

Από την ανισότητα B – C – S έχουμε:

2 2 2 2 1 2 1 2 1 2 1 2( a) ( b) ( c) ( d) ( a ) ( b ) ( c ) ( d )

2

1 1 1 1a a b b c c d d .

Άρα: 1 1 1 1 2(a b c d)(a b c d ] (1 1 1 1) και άρα

1 1 1 1(a b c d)(a b c d ) 16 .

ΘΕΜΑ 216 (Αλγεβριστής)

Για τους πραγματικούς a,b,c να αποδείξετε ότι 2 2 2a b c ab bc ca .

Να την λύσετε με C – S, με ΑΜ – ΓΜ, με πράξεις και με αναδιάταξη.

Λύση:

1ος τρόπος: (Με B – C – S)

Θεωρούμε τις τριάδες: a,b,c και b,c,a .

Τότε: 2 2 2 2 2 2 2 2 2 2 2 2(a b c )(b c a ) (ab bc ca) (a b c ) (ab bc ca)

2 2 2a b c ab bc ca ab bc ca ,

(διότι κάθε αριθμός είναι μικρότερος ή ίσος από την απόλυτη τιμή του).

2ος τρόπος: (Με απλές πράξεις)

Αρκεί να αποδείξουμε ότι:

2 2 22a 2b 2c 2ab 2bc 2ca ή αρκεί:

Page 186: Εισαγωγη σε διαγωνιστικα μαθηματικα για το γυμνασιο

http://www.mathematica.gr/forum/viewtopic.php?f=109&t=30104

Επιμέλεια: xr.tsif Σελίδα 186

2 2 2 2 2 2a a b b c c 2ab 2bc 2ca 0 .

Άρα αρκεί: 2 2 2(a b) (b c) (c a) 0 , το οποίο είναι αληθές.

3ος τρόπος: (Με απλές ανισότητες)

Έχουμε: 2 2a b 2ab (πολύ απλή αλλά χρήσιμη ανισότητα).

Επίσης: 2 2b c 2bc ,

και : 2 2c a 2ca .

Με πρόσθεση κατά μέλη, βρίσκουμε το ζητούμενο

4ος τρόπος: (Με άλλη απλή ανισότητα)

Έχουμε : 2 2 22(a b ) (a b) .

(Πολύ απλή στην απόδειξή της, αλλά και πολύ χρήσιμη σε απόδειξη άλλων

ανισοτήτων. Πρέπει να την απομνημονεύσουμε. )

Ομοίως: 2 2 22(b c ) (b c) .

Και: 2 2 22(c a ) (c a) .

Με πρόσθεση κατά μέλη, παίρνουμε: 2 2 2 2 2 2 2 2 24a 4b 4c 2a 2b 2c 2ab 2bc 2ca a b c ab bc ca .

Φυσικά, μπορούμε να βρούμε και πολλούς ακόμα τρόπους.

5ος τρόπος

Δίνω μάλλον την πιο απλή λύση.

Πολλαπλασιάζοντας με το 2και τα δυο μέλη έχω:

2 2 2 2 2 2a a b b c c 2ab 2bc 2ca

2 2 2 2 2 2(a 2ac b ) (b 2bc c ) (a 2ca c ) 0 2 2 2(a b) (b c) (a c) 0 , που προφανώς ισχύει.

6ος τρόπος Ας τη δούμε και με αναδιάταξη.

Page 187: Εισαγωγη σε διαγωνιστικα μαθηματικα για το γυμνασιο

http://www.mathematica.gr/forum/viewtopic.php?f=109&t=30104

Επιμέλεια: xr.tsif Σελίδα 187

Θεωρούμε τη συνάρτηση 2 2 2f (a,b,c) a b c ab bc ca με a,b,c R .

Παρατηρούμε ότι οι τιμές της f και για τις 3!το πλήθος μεταθέσεις των a,b,c

είναι ίσες. Συνεπώς, η f είναι συμμετρική ως προς τα a,b,c και συνεπώς, δίχως

βλάβη της γενικότητας μπορούμε να υποθέσουμε ότι a b c .

(Πιο συγκεκριμένα είναι συμμετρικό πολυώνυμο).

Έτσι, εφαρμόζοντας την ανισότητα της αναδιάταξης για τις ακολουθίες (a,b,c) ,

(a,b,c) έχουμε 2 2 2a b c ab bc ca , όπου s(1) (b,c,a) η μετάθεση, που

είναι και η ζητούμενη.

7ος τρόπος Με τριώνυμο

Θεωρούμε το τριώνυμο ως προς a .

Η αποδεικτέα γράφεται 2 2 2a (b c)a b c bc .

Η διακρίνουσα, είναι 2 2 2 2 2 2 2 2D (b c) 4(b c bc) b c 2bc 4b 4c 4bc 3(b c) 0 .

Αφού η διακρίνουσα είναι αρνητική το δευτεροβάθμιο πολυώνυμο έχει το

πρόσημο του μεγιστοβάθμιου όρου. Στην περίπτωση μας είναι θετικό πράγμα,

που αποδεικνύει και την ζητούμενη.

Η μικρή διακρίνουσα είναι πολύ πιο ισχυρή από ότι φαίνεται!

Λησμόνησα την περίπτωση b c , που όμως είναι αρκετά στοιχειώδης, αφού

αντικαθιστώντας συμπληρώνουμε το τετράγωνο.

ΘΕΜΑ 217 (ΔΗΜΗΤΡΗΣ ΙΩΑΝΝΟΥ)

Αν x,y,z 0 και x y z 2013 , να αποδείξετε ότι: 1 1 1 3

x y z 671 .

Λύση:

Με χρήση της ανισοϊσότητας Andreescu βγάζω ότι:

Page 188: Εισαγωγη σε διαγωνιστικα μαθηματικα για το γυμνασιο

http://www.mathematica.gr/forum/viewtopic.php?f=109&t=30104

Επιμέλεια: xr.tsif Σελίδα 188

1 1 1 9 9 3+ + =

x y z x+y+z 2013 671 .

ΘΕΜΑ 218 (ΔΗΜΗΤΡΗΣ ΙΩΑΝΝΟΥ)

Αν x,y,z 0 και xyz 1 , να αποδείξετε ότι

481(x y)(y z)(z x) 8(x y z) .

Λύση:

Από την ανισοϊσότητα αριθμητικού γεωμετρικού μέσου ισχύει ότι:

x y 2 xy ,

y z 2 yz ,

z x 2 zx .

Με πολλαπλασιασμό των σχέσεων κατά μέλη έχουμε:

2(x y)(y z)(z x) 8 (xyz) 8xyz 8 .

Άρα το πρώτο μέλος της αρχικής ανισότητας είναι μεγαλύτερο ή ίσο του 81·8.

Το δεύτερο μέλος γίνεται πάλι μέσω της ανισοϊσότητας αριθμητικού –

γεωμετρικού μέσου 3x y z 3 xyz 3 , δηλαδή είναι μεγαλύτερο ή ίσο του

48·3 81·8 . Άρα η δοσμένη ισχύει.

Β τρόπος

Από την ανισοϊσότητα αριθμητικού – γεωμετρικού μέσου ισχύει ότι:

3(x y) (y z) (z x) 3 (x y)(y z)(z x)

332(x y z) 3 (x y)(y z)(z x) 8(x y z) 27(x y)(y z)(z x)

και επίσης 3x y z xyz 3 . Με πολλαπλασιασμό κατά μέλη προκύπτει το

ζητούμενο.

Page 189: Εισαγωγη σε διαγωνιστικα μαθηματικα για το γυμνασιο

http://www.mathematica.gr/forum/viewtopic.php?f=109&t=30104

Επιμέλεια: xr.tsif Σελίδα 189

ΘΕΩΡΗΤΙΚΑ

Η ΜΕΘΟΔΟΣ ΤΗΣ ΑΝΑΔΙΑΤΑΞΗΣ

Θεωρούμε τις νι – άδες των αριθμών: 1 2 3 n

x ,x ,x ,...,x και 1 2 3 n

y ,y ,y ,...,y οι

οποίες έχουν την ίδια διάταξη, δηλαδή αν είναι 1 2 n

x x ... x , τότε θα είναι

και

1 2 3 ny y y ... y (ή με αντίστροφη φορά των ανισοτήτων).

Επίσης θεωρούμε και τους αριθμούς 1 2 3 n

a ,a ,a ,...,a , οι οποίοι στην ουσία είναι

οι αριθμοί 1 2 3 n

y ,y ,y ,...,y , με οποιαδήποτε σειρά , δηλαδή:

1 2 3 n 1 2 3 n{a ,a ,a ,...,a } {y ,y ,y ,...,y } .

Ακριβέστερα τα 1 2 3 n

a ,a ,a ,...,a είναι μια μετάθεση των 1 2 3 n

y ,y ,y ,...,y

Για παράδειγμα αν 1 2 3

y 2,y 2,y 1 , τότε τα 1 2 3

a ,a ,a , μπορούν να επιλεγούν

ως εξής: 1 2 3

a 2,a 1,a 2 και όχι 1 2 3

a 1,a 1,a 2 , παρά το γεγονός ότι και

στις δύο επιλογές τα σύνολα 1 2 3

{y ,y ,y }και 1 2 3

{a ,a ,a }είναι ίσα.

Τότε ισχύει η εξής ανισότητα:

1 1 2 2 3 3 n n 1 1 2 2 3 3 n nx y x y x y ... x y a x a x a y ... a x .

Αν όμως οι αρχικοί αριθμοί έχουν αντίθετη διάταξη, δηλαδή αν π.x

1 2 nx x ... x , τότε θα είναι

1 2 ny y ... y , (ή με αντίστροφες τις φορές

των ανισοτήτων), τότε:

1 1 2 2 3 3 n n 1 1 2 2 3 3 n nx y x y x y ... x y a x a x a y ... a x .

Η ισότητα και στις δύο περιπτώσεις, θα ισχύει όταν 1 2 n

x x ... x , ή όταν

1 2 ny y ... y .

Έτσι, στην λύση που έδωσε παραπάνω το μέλος μας ο "Αλγεβριστής", με

δεδομένο ότι θεωρήσαμε a b , παίρνουμε τις τριάδες: a,b,c και a,b,c

Page 190: Εισαγωγη σε διαγωνιστικα μαθηματικα για το γυμνασιο

http://www.mathematica.gr/forum/viewtopic.php?f=109&t=30104

Επιμέλεια: xr.tsif Σελίδα 190

οι οποίες έχουν την ίδια διάταξη (a b c και a b c ), οπότε αν πάρουμε

τους αριθμούς k,m,n , ώστε να είναι {k,m,n} {a,b,c} , θα ισχύει ότι:

a a b b c c ka mb nc .

Θα επιλέξουμε τα k,m,n με κατάλληλο τρόπο, ώστε να μας βγει το ζητούμενο.

Έτσι, επιλέγουμε: k b,m c,n a και θα έχουμε: 2 2 2a b c ba cb ac .

ΘΕΜΑ 219 (ΔΗΜΗΤΡΗΣ ΙΩΑΝΝΟΥ)

Αν x,y,z 0 και 1

x y z2

, να αποδείξετε ότι: xy yz zx 18xyz .

Λύση:

Αν ένας από τους x,y,z είναι ίσος με μηδέν, τότε η ανισότητα ισχύει προφανώς

(για παράδειγμα, αν z 0 , τότε η ανισότητα γράφεται xy 0 , που ισχύει).

Υποθέτουμε, λοιπόν, ότι xyz 0 . Τότε, η αποδεικτέα ανισότητα γράφεται

ισοδύναμα: xy yz zx 1 1 1

18 18xyz x y z

(1) .

Αλλά από την ανισότητα αριθμητικού – αρμονικού μέσου προκύπτει ότι

x y z 3

1 1 13

x y z

και άρα

13 1 1 12 18

1 1 13 x y z

x y z

, που είναι η (1) .

Β τρόπος

Αρκεί να αποδείξουμε ότι (x y z)(xy yz zx) 9xyz .

Όμως, αυτό είναι άμεσο από ΑΜ – ΓΜ:

Page 191: Εισαγωγη σε διαγωνιστικα μαθηματικα για το γυμνασιο

http://www.mathematica.gr/forum/viewtopic.php?f=109&t=30104

Επιμέλεια: xr.tsif Σελίδα 191

233(x y z)(xy yz zx) 3 xyz·3 (xyz) 9xyz .

Γ τρόπος

ας δούμε και μια ακόμα με χρήση της AM – GM

2 2 23xy yz zx 3 x y z .

Αρκεί λοιπόν να δείξουμε ότι: 2 2 233 x y z 18xyz ή αρκεί: 2 2 2 3 3 3 3x y z 6 x y z .

Αν xyz 0 , είναι φανερό ότι η σχέση αυτή είναι αληθής.

Με xyz 0 , αρκεί να αποδείξουμε ότι: 3

1xyz

6 (1) .

Όμως, 3 33

1 1x y z 3 xyz 3 xyz xyz

2 6 και άρα η (1) είναι αληθής.

ΘΕΜΑ 220 (ΔΗΜΗΤΡΗΣ ΙΩΑΝΝΟΥ)

Αν x,y,z 0 , να αποδείξετε ότι: 2 2 2

4 4 4

4 4 4x y z 9

x y z .

Λύση:

Ο τρόπος με τον οποίο την έλυσα, (χρησιμοποιώντας ΑΜ – GM), είναι λίγο

δύσκολος. Ωστόσο, ίσως δούμε και απλές λύσεις από τα ταλαντούχα μας μέλη

Β τρόπος

Για a 0 ισχύει 2 2 2 2

23

4 4 4

4 a a 4 a a 4a 3 · · 3

a 2 2 a 2 2 a .

Συνεπώς,

2 2 2 2 2 2

4 4 4 4 4 4

4 4 4 4 4 4x y z x y z 9

x y z x y z

Με το ίσον να ισχύει αν και μόνο αν 2 2 2x y z 2 .

Page 192: Εισαγωγη σε διαγωνιστικα μαθηματικα για το γυμνασιο

http://www.mathematica.gr/forum/viewtopic.php?f=109&t=30104

Επιμέλεια: xr.tsif Σελίδα 192

Γ τρόπος

Αλλιώς, χωρίς ΑΜ – ΓΜ:

4 2 2 2 4 2

2 2 2

4 4 4 4

4 4 a (a 2)(a 2) (a 2)(a a 2)a 3 a 2 a 2

a a a a

2 2 2

4

(a 2) (a 1)0

a

. Η συνέχεια η ίδια.

Δ τρόπος

Οι μικρότεροι να μην δώσετε σημασία σε αυτή τη λύση.

Και μία αντιμετώπιση με Γ' Λυκείου!

Αρκεί να την αποδείξουμε για x 0 (γιατί;). Έτσι στη συνέχεια μπορούμε να

βάλουμε όπου x το x .

Θεωρούμε τη συνάρτηση 2

4

4f(x) x

x με D(f) (0, ) η οποία είναι

παραγωγίσιμη με 5

df 162x

dx x , D(f ) (0, ) .

Λύνουμε την εξίσωση df

0dx

, από την οποία προκύπτουν τα κρίσιμα σημεία

x 2 ή x 2 . Το δεύτερο, επειδή είναι εκτός πεδίο ορισμού

απορρίπτεται.

Έχουμε, 2

2 6

d f 16 52 0

dx x

, D(f ) (0, ) .

Και συνεπώς, από το κριτήριο δεύτερης παραγώγου το x 2 είναι τοπικό

ελάχιστο και μάλιστα ολικό.

Έτσι, f (x) f ( 2) 3 και συνεπώς κυκλικά έχουμε cyc

f (x) 9 ,

Page 193: Εισαγωγη σε διαγωνιστικα μαθηματικα για το γυμνασιο

http://www.mathematica.gr/forum/viewtopic.php?f=109&t=30104

Επιμέλεια: xr.tsif Σελίδα 193

που είναι και η ζητούμενη.

Τα σημεία στα οποία ισχύει η ισότητα: όλες οι τριάδες (x,y,z) με

2 2 2x y z 2 .

Στη συνέχεια μπορούμε να βάλουμε όπου x το x και να αποδείξουμε την

σχέση για x 0 .

ΘΕΜΑ 221 (ΔΗΜΗΤΡΗΣ ΙΩΑΝΝΟΥ)

Αν x,y,z 0 να αποδείξετε ότι : 3 3 3 2 2 2x y z x y y z z x .

Λύση:

Υπόδειξη:

Μπορεί να λυθεί και με την ανισότητα της αναδιάταξης που αναπτύχθηκε πιο

πάνω, αν μας δίνουν ότι x y z , παίρνοντας τις τριάδες των αριθμών x,y,z

και 2 2 2x ,y ,z με την παρατήρηση ότι αν θεωρήσουμε ότι x y z τότε οι

τριάδες αυτές έχουν την ίδια διάταξη.

(Πράγματι, αφού x,y,z 0 τότε από την σχέση: 2 2 2x y z x y z ).

Β τρόπος

Υπάρχουν πολλοί τρόποι αντιμετώπισης. Βάζω μία με κάποια τεχνική που είχα

ανακαλύψει κάποτε (αλλά δεν αμφιβάλλω ότι είναι κάτι γνωστό). Πιστέψτε με

ότι η τεχνική αυτή λειτουργεί σε πολλές ανισότητες, και είναι ευκαιρία να την

περιγράψω στο εδώ παράδειγμα.

Θα κάνουμε διπλή χρήση της C – S.

Ονομάζουμε Dτο δεξί μέλος και Aτο αριστερό, οπότε θέλουμε D A .

Έχουμε

22 2 2 2 2 3/2 1/2 3/2 1/2 3/2 1/2D (x y y z z x) x ·x y y ·y z z ·z x

3 3 3 2 2 2 1/2 3/2 1/2 3/2 1/2 3/2x y z xy yz zx A xy ·y yz ·z zx ·x

Page 194: Εισαγωγη σε διαγωνιστικα μαθηματικα για το γυμνασιο

http://www.mathematica.gr/forum/viewtopic.php?f=109&t=30104

Επιμέλεια: xr.tsif Σελίδα 194

1/2 1/22 2 2 3 3 3 1/2 1/2A x y y z z x y z x AD A .

Άρα 3/2 3/2D A , από όπου το ζητούμενο.

Γ τρόπος

Με αναδιάταξη

Η συνάρτηση 3 3 3 2 2 2f (x,y,z) x y z x y y z z x είναι συμμετρική ως

προς τα x,y,z . Συνεπώς, χωρίς βλάβη της γενικότητας μπορούμε να

υποθέσουμε ότι x y z ή 2 2 2x y z . Θεωρούμε την μετάθεση

s(1) (y,z,x) . Τώρα εφαρμόζοντας την ανισότητα της αναδιάταξης στις

τριάδες 2 2 2(x,y,z),(x ,y ,z ) και τις

2 2 2(x ,y ,z ),s(1) παίρνουμε

2 2 2 2 2 2x x y y z z x y y z z x ή ισοδύναμα

3 3 3 2 2 2x y z x y y z z x , που είναι και η ζητούμενη.

Δ τρόπος

Προσοχή!

Η συνάρτηση f (x,y,z) δεν είναι συμμετρική! Είναι f (x,y,z) f (y,x,z) .

Βέβαια, η εν λόγω ανισότητα αποδεικνύεται με την ανισότητα της αναδιάταξης

παρατηρώντας ότι οι τριάδες 2 2 2(x,y,z),(x ,y ,z ) είναι της ίδιας "διάταξης", σε

αντίθεση με τις 2 2 2(x ,y ,z ),(y,z,x) .

Ωστόσο, πιστεύω ότι το θέμα αυτό προτάθηκε εδώ για να δούμε την εξής

απόδειξη:

3 3 3 3 3 3 23x y z 3 x x y 3x y .

Ομοίως βρίσκουμε 3 3 3 2 3 3 3 2y y z 3y z, z z x 3z x .

Με πρόσθεση αυτών προκύπτει η ζητούμενη.

Ε τρόπος

Έχουμε και την εξής άποψη και μόνο για λόγους πολυφωνίας:

Page 195: Εισαγωγη σε διαγωνιστικα μαθηματικα για το γυμνασιο

http://www.mathematica.gr/forum/viewtopic.php?f=109&t=30104

Επιμέλεια: xr.tsif Σελίδα 195

3 3 2 2x y z 0 x y x y xy (1) .

2 2 2 2 2 2y z x z y x 0 xy yz zx x y y z z x

2 2 2 2 2 2 2 2 2xy yz zx x y y z z x 2x y 2y z 2z x (2) .

Από τις σχέσεις (1) , (2)παίρνουμε άμεσα την ζητούμενη.

ΘΕΜΑ 222 (ΑΡΓΥΡΗΣ ΚΑΡΑΝΙΚΟΛΑΟΥ)

) Έστω x,y,z θετικοί πραγματικοί αριθμοί. Αποδείξτε ότι

x y z 3

x y y z z x 2

. (Vasile Cirtoaje, Samin Riasat)

Λύση:

Η δοθείσα γράφεται

3x(y z)(z x) y(z x)(x y) z(x y)(y z) (x y)(y z)(z x)

2

Από την Cauchy – Schwarz είναι

x(y z)(z x) y(z x)(x y) z(x y)(y z)

x(y z) y(z x) z(x y) x y y z z x 2 (xy yz zx)(x y z)

Αρκεί λοιπόν, να αποδειχθεί ότι

32 (xy yz zx)(x y z) (x y)(y z)(z x)

2 .

Ισχύει (x y)(y z)(z x) (x y z)(xy yz zx) xyz και

(x y)(y z)(z x) 8xyz , άρα

2 (xy yz zx)(x y z) 2 (x y)(y z)(z x) xyz

Page 196: Εισαγωγη σε διαγωνιστικα μαθηματικα για το γυμνασιο

http://www.mathematica.gr/forum/viewtopic.php?f=109&t=30104

Επιμέλεια: xr.tsif Σελίδα 196

(x y)(y z)(z x) 32 (x y)(y z)(z x) (x y)(y z)(z x)

8 2

.

Β τρόπος

Αρκεί να αποδείξουμε ότι: 1 1 1 3

x z x 21 1 1y y z

.

Θέτουμε: 2 2 2y z x

a , b , cx y z , όπου a,b,c 0 .

Τότε αρκεί να αποδείξουμε ότι: 2 2 2

1 1 1 3

1 a 1 b 1 c 2

.

Η νέα αυτή ανισότητα είναι συμμετρική (αφού δεν μεταβάλλεται με κυκλική

εναλλαγή των γραμμάτων της) και άρα μπορούμε να θεωρήσουμε ότι

a b c (και άρα και 2 2 2a b c , λόγω του ότι a,b,c 0 ).

Επίσης παρατηρούμε ότι 2 2 2a b c 1 (και άρα και 2 2 2a b c 1 ).

Από την βασική ανισότητα 2 2 2(p q) 2(p q ) , παίρνουμε:

2

2 2 2 2

1 1 1 12

1 a 1 b 1 a 1 b

2 2 2 2 2 2 2 2

2 2 2 2 2 2

a b 1 1 (1 a )(1 b ) 1 a b 1 2a b2 2 2 1

(1 a )(1 b ) (1 a )(1 b ) (1 a )(1 b )

,

Δείξαμε λοιπόν ότι:

22 2

2 2 2 2

1 1 1 2a b2 1

1 a 1 b (1 a )(1 b )

(1) .

Όμως:

2 2 2 2 2 2 2 2 2 2(1 a )(1 b ) 1 b a a b 1 a b 2ab 2ab a b

2 2 2(1 ab) (a b) (1 ab) .

Page 197: Εισαγωγη σε διαγωνιστικα μαθηματικα για το γυμνασιο

http://www.mathematica.gr/forum/viewtopic.php?f=109&t=30104

Επιμέλεια: xr.tsif Σελίδα 197

Άρα 2 2 2

1 1

(1 a )(1 b ) (1 ab)

(2) .

Θα αποδείξουμε ότι 2 21 a b 0 .

Πράγματι, έστω ότι 2 2 2 2 2 2 2 2 21 a b 0 a b 1 a b c c 1 c .

Όμως 2 2a c και 2 2b c , οπότε με πολλαπλασιασμό έχουμε

2 2 2 2 2 24 6 3a b c a b c c (c ) 1 1 1 , άτοπο.

Τώρα με δεδομένο ότι είναι 2 21 a b 0 , η (2) γράφεται:

2 2 2 2 2 2

2 2 2 2 2

1 a b 1 a b 1 a b 1 ab 21 1

(1 a )(1 b ) (1 ab) (1 a )(1 b ) (1 ab) 1 ab

.

Άρα: 2 2

2 2

1 a b 42 1

(1 a )(1 b ) 1 ab

.

Έτσι από την (1) έχουμε:

2

2 2 2 2

1 1 4 1 1 2 2 2 c

1 a 1 b 1 ab 1 a 1 b 1 ab 1 1 c1

c

.

Άρα: 2 2

1 1 2 c

1 a 1 b 1 c

(3) .

Όμως 2 2 2 2 2 2(c 1) 0 c 2c 1 0 c 2c 1 c c c 2c 1

2 2 2 2

2

1 22c 2 c 2c 1 2(c 1) (c 1)

1 c c 1

2

2 c 1 2 2 c

1 c 1 cc 1 1 c

.

Θα δείξουμε ότι: 2 2 c 3

1 c 1 c 2

.

Page 198: Εισαγωγη σε διαγωνιστικα μαθηματικα για το γυμνασιο

http://www.mathematica.gr/forum/viewtopic.php?f=109&t=30104

Επιμέλεια: xr.tsif Σελίδα 198

Αρκεί να δείξω ότι: 2 2 2c(1 c) 3 3c ή αρκεί:

3c 1 2 2c(1 c) 0 ή αρκεί: 2c 1 c 2 2c(1 c) 0 ή αρκεί:

2( 2c 1 c) 0 , το οποίο είναι αληθές.

Δείξαμε λοιπόν ότι: 2

2 c 1 3

1 cc 1 2

(4) .

Με πρόσθεση κατά μέλη των σχέσεων (3) και (4) έχουμε το ζητούμενο.

Δείτε και από εδώ και κάτω.

ΘΕΜΑ 223 (ΣΩΤΗΡΗΣ ΛΟΥΡΙΔΑΣ)

) Αν έχουμε x,y 0 , αποδείξτε τη ισχύ της ανισότητας:

x y y z z x

3 2x y z

.

Λύση:

Θέτουμε 4 4 4x a ,y b ,z c και από ΑΜ - ΓΜ έχουμε

4 4 4 4 4 4 2 2 2 2 2 2

4 4 4 4 4 4

a b b c c a 2a b 2b c 2c a

a b c a b c

2 2 2

ab bc ca b c a2 2 3 2

a b c a b c( ) ( ) , και τελειώσαμε.

Page 199: Εισαγωγη σε διαγωνιστικα μαθηματικα για το γυμνασιο

http://www.mathematica.gr/forum/viewtopic.php?f=109&t=30104

Επιμέλεια: xr.tsif Σελίδα 199

ΘΕΩΡΗΤΙΚΑ

Η ΑΝΙΣΟΤΗΤΑ CHEBYSHEV

ΘΕΩΡΗΜΑ (Chebyshev)

Έστω δύο αύξουσες ακολουθίες πραγματικών αριθμών

1 2 n 1 2 na a ... a ,b b ... b , όπου n 1 φυσικός αριθμός.

Για τους όρους των ακολουθιών αυτών ισχύει η παρακάτω ανισότητα:

1 1 2 2 n n 1 2 n 1 2 na b a b ... a b a a ... a b b ... b

n n n

1 n 2 n 1 n 1 2 n 1a b a b ... a b a b

n

.

Παράδειγμα 1

Για τους πραγματικούς a,b,c να αποδείξετε ότι 2 2 2 23(a b c ) (a b c) .

Απόδειξη

Παρατηρούμε ότι η αποδεικτέα είναι συμμετρική και συνεπώς χωρίς βλάβη της

γενικότητας, υποθέτουμε ότι a b c .

Εφαρμόζοντας το πρώτο σκέλος της ανισότητας του Chebyshev για τις τριάδες

a,b,c και a,b,c έχουμε

a a b b c c a b c a b c

3 3 3

ή ισοδύναμα

2 2 2 23(a b c ) (a b c) , που είναι και η ζητούμενη.

Page 200: Εισαγωγη σε διαγωνιστικα μαθηματικα για το γυμνασιο

http://www.mathematica.gr/forum/viewtopic.php?f=109&t=30104

Επιμέλεια: xr.tsif Σελίδα 200

Παράδειγμα 2

Για τους θετικούς a,b,c 0 , να αποδείξετε την ισχύ της παρακάτω ανισότητας:

8 8 8

3 3 3

a b c 1 1 1

a b c a b c

.

Απόδειξη

Παρατηρούμε ότι η αποδεικτέα είναι συμμετρική και συνεπώς χωρίς βλάβη της

γενικότητας μπορούμε να υποθέσουμε ότι

a b c ή 6 6 6a b c ή 2 2 2a b c .

Στα παρακάτω πέρα της Chebyshev θα εφαρμόσουμε και AM – ΓΜ στο 6 6 6a b c και την βασική ανισότητα 2 2 2a b c ab bc ca .

Από την Ανισότητα Chebyshev (πρώτο σκέλος) για τις τριάδες 6 6 6 2 2 2(a ,b ,c ),(a ,b ,c ) αποφαινόμαστε ότι

8 8 8 6 6 6 2 2 2a b c (a b c ) (a b c )

3 3 3

ή ισοδύναμα

8 8 8 6 6 6 2 2 2 2 2 2 2 2 23(a b c ) (a b c )(a b c ) 3a b c (a b c )

2 2 2 3 3 2 2 3 3 3 2 3 3 3 3 1 1 13a b c (ab bc ca) 3a b c 3a b c 3a b c 3a b c

c a b( ) ,

ή ισοδύναμα 8 8 8 3 3 3 1 1 13(a b c ) 3a b c

c a b( ) ,

το οποίο γίνεται 8 8 8

3 3 3

a b c 1 1 1

a b c a b c

, που είναι και η ζητούμενη.

Σημειώσεις – Παρατηρήσεις

Η ανισότητα Chebyshev είναι δυνατό εργαλείο επίλυσης ανισοτήτων και έχει

Page 201: Εισαγωγη σε διαγωνιστικα μαθηματικα για το γυμνασιο

http://www.mathematica.gr/forum/viewtopic.php?f=109&t=30104

Επιμέλεια: xr.tsif Σελίδα 201

ως πλεονέκτημα το ότι οι όροι των ακολουθιών δεν χρειάζεται να είναι θετικοί.

Το μειονέκτημα είναι ότι πρέπει να αποδείξουμε την ιδιότητα της διάταξης πριν

την εφαρμόσουμε.

Όπως, παρατηρείται επιλύει μόνο συμμετρικές ανισότητες, όπως και η

ανισότητα της αναδιάταξης, αφού άλλωστε αποδεικνύεται από αυτήν. Για να

δώσουμε την ιδιότητα της διάταξης, χωρίς βλάβη της γενικότητας, στις

μεταβλητές, αρκεί να αποδείξουμε ότι πρόκειται περί συμμετρικής ανισότητας.

Αυτό σημαίνει, ότι αν αλλάξετε όλες τις δυνατές θέσεις των μεταβλητών, πάντα

θα παίρνουμε την ίδια ανισότητα. Αν έχουμε nμεταβλητές τότε έχουμε n!

μεταθέσεις.

Π.χ. Η ανισότητα 2 2a b 2ab είναι συμμετρική, αφού αν βάλουμε όπου a το

b και όπου bτο a θα πάρουμε 2 2b a 2ba , δηλαδή πάλι την ίδια!

Συνεπώς, μπορούμε να υποθέσουμε χωρίς βλάβη της γενικότητας ότι a b με

στόχο την απόδειξή της.

ΘΕΜΑ 224 (ΑΛΓΕΒΡΙΣΤΗΣ)

) Για τους θετικούς a,b,c 0 , να αποδείξετε ότι

3 3 3 2 2 2 2 2 2a b c ab bc ca 2(a b b c c a) .

Λύση:

Από ΑΜ – ΓΜ έχουμε

3 2 2a ab 2a b ,

3 2 2b bc 2b c ,

3 2 2c ca 2c a .

Με πρόσθεση κατά μέλη των παραπάνω προκύπτει η ζητούμενη.

Page 202: Εισαγωγη σε διαγωνιστικα μαθηματικα για το γυμνασιο

http://www.mathematica.gr/forum/viewtopic.php?f=109&t=30104

Επιμέλεια: xr.tsif Σελίδα 202

ΘΕΜΑ 225 (ΑΛΓΕΒΡΙΣΤΗΣ)

) Για τους θετικούς x,y,z 0 να αποδείξετε ότι:

3 3 3

2 2 2 2 2 2

x y z x y z

x xy y y yz z z zx x 3

.

Λύση:

Έχουμε ότι 3 3 3 3 3

2 2 2 2 2 2 2 2 2 2

x y z y z

x xy y y yz z z zx x x xy y y yz z

3 3 3 3 3 3 3

2 2 2 2 2 2 2 2

x x y y z z x

z zx x x xy y y yz z z zx x

x y y z z x 0 .

Άρα αρκεί να δείξουμε ότι: 3 3 3 3 3 3

2 2 2 2 2 2

x y y z z x 2(x y z)

x xy y y yz z z zx x 3

.

Όμως με χρήση της ανισότητας: 3 3 2 2x y x y y z , ( απόδειξη ;), έχουμε:

3 3 2 2 2 2

2 2 2 2 2 2

x y (x y)(x xy y 2xy) x y xyx y 2

x xy y x xy y x xy y

3 3

2 2

x yx y 2

x xy y

.

Από το οποίο προκύπτει ότι: 3 3

2 2

x y x y

x xy y 3

.

Και ομοίως:

3 3

2 2

y z y z

y yz z 3

,

Page 203: Εισαγωγη σε διαγωνιστικα μαθηματικα για το γυμνασιο

http://www.mathematica.gr/forum/viewtopic.php?f=109&t=30104

Επιμέλεια: xr.tsif Σελίδα 203

3 3

2 3

z x z x

z zx x 3

.

Από τα οποία με πρόσθεση κατά μέλη προκύπτει η ζητούμενη.

ΘΕΜΑ 226 (ΑΛΓΕΒΡΙΣΤΗΣ)

) Έστω οι θετικοί x,y 0 τέτοιοι ώστε

3 3x y x y . Να αποδείξετε ότι

2 2x y 1 .

Λύση:

3 3x x y y 0 άρα 3x x , οπότε x 1 . Από C – S

2 2 3/2 1/2 3/2 1/2 3 3x y x x y y x y x y x y x y

2 2 2x y x x 1 .

Β τρόπος

Έχουμε 3 3x y x y 0 άρα x y . Συνεπώς,

2 2 3 3 3 3 3 3(x y )(x y) (x y ) xy(x y) x y x y x y ,

και αφού x y 0 , έχουμε 2 2x y 1 .

ΘΕΜΑ 227 (ΑΛΓΕΒΡΙΣΤΗΣ)

) Για τους θετικούς a,b,c 0 για τους οποίους ισχύει (a 1)(b 1)(c 1) 8 ,

να αποδείξετε ότι ισχύει abc 1 .

Λύση:

8 (a 1)(b 1)(c 1) (2 a)(2 b)(2 c) 8 abc , άρα 1 abc , από όπου το

ζητούμενο.

Page 204: Εισαγωγη σε διαγωνιστικα μαθηματικα για το γυμνασιο

http://www.mathematica.gr/forum/viewtopic.php?f=109&t=30104

Επιμέλεια: xr.tsif Σελίδα 204

ΘΕΜΑ 228 (ΑΛΓΕΒΡΙΣΤΗΣ)

) Για τους θετικούς x,y,z,w 0 , να αποδείξετε ότι

4 4 4 4 4(x 1)(y 1)(z 1)(w 1) (1 xyzw) .

Λύση:

Από την ανισοϊσότητα Buniakowski – Cauchy – Schwarz ισχύει ότι 4 4 2 2 2(x 1)(y 1) (x y 1) και ότι

4 4 2 2 2(z 1)(w 1) (z w 1) .

Με πολλαπλασιασμό κατά μέλη ισχύει ότι 4 4 4 4 2 2 2 2 2 2 2(x 1)(y 1)(z 1)(w 1) [(x y 1)(z w 1)] [(xyzw 1) ]

4(xyzw 1) .

Β τρόπος

Για αυτήν εδώ ουσιαστικά κάποιος μπορεί να χρησιμοποιήσει την ανισότητα

του Holder και το αποτέλεσμα είναι άμεσο.

1 1 1 1 1 1 1 14 4 4 4 4 4 4 44 4 4 4 4 4 4 4(x 1) (y 1) (z 1) (w 1) (x ) (y ) (z ) (w ) 1 ...

Με ύψωση εις την τετάρτη βγαίνει το ζητούμενο.

Περιληπτικά η ανισότητα Holder λέει :

Holder's inequality

Έστω 1 2 n 1 2 n 1 2 n

a ,a ,...a b ,b ,...b z ,z ,., ..z, είναι ακολουθίες μη αρνητικών

πραγματικών αριθμών και έστω a b z, ,..., θετικοί πραγματικοί με άθροισμα

1. Τότε ισχύει:

a b a b a bz z z

1 2 n 1 2 n 1 2 n 1 1 1 n n n(a a ...a ) (b b ...b ) ... (z z ...z ) a b ...z ... a b ...z

Ουσιαστικά αποτελεί γενίκευση της Cauchy.

Page 205: Εισαγωγη σε διαγωνιστικα μαθηματικα για το γυμνασιο

http://www.mathematica.gr/forum/viewtopic.php?f=109&t=30104

Επιμέλεια: xr.tsif Σελίδα 205

ΘΕΜΑ 229 (ΑΛΓΕΒΡΙΣΤΗΣ)

) Για τους θετικούς a,b,c 0 να αποδείξετε ότι

3 3 3 2 2 2

2

2(a b c ) 6(a b c )8

abc (a b c)

.

Λύση:

3 3 3a b c abc2 2 3 6

abc abc

(1) .

2 2 2 2

2 2

a b c (a b c)2 3 2 2

(a b c) (a b c)

(2) .

Με πρόσθεση των (1) και (2) προκύπτει το ζητούμενο.

ΘΕΜΑ 230 (ΑΛΓΕΒΡΙΣΤΗΣ)

) Για τους θετικούς a,b,c 0 να αποδείξετε ότι (a b)(a c) 2 abc(a b c) .

Λύση:

Η ανισότητα για a b c 1 γίνεται ισοδύναμη με την

(1 b)(1 c) 2 bc(1 b c) 1 b c+bc 2 bc(1 b c) ,

που ισχύει από την x y 2 xy .

ΘΕΜΑ 231 (ΑΛΓΕΒΡΙΣΤΗΣ)

) Αν x,y 0 θετικοί πραγματικοί αριθμοί με x y 2 , να αποδείξετε ότι

2 2 2 2x y (x y ) 2 .

Λύση:

Πράγματι, σύμφωνα με την Ανισότητα AM – GM θα ισχύει

Page 206: Εισαγωγη σε διαγωνιστικα μαθηματικα για το γυμνασιο

http://www.mathematica.gr/forum/viewtopic.php?f=109&t=30104

Επιμέλεια: xr.tsif Σελίδα 206

2 2 2 2 4

2 2 2 2 2 2 2 2

3

(x y) [2xy (x y )] 2x y (x y ) ·xy(x y ) xy(x y ) 2

4 2·4 2

.

ΘΕΜΑ 232 (ΑΛΓΕΒΡΙΣΤΗΣ)

) Αν x,y,z 0 θετικοί πραγματικοί αριθμοί με xyz 1 και

7 7 7x y z 3 , να

υπολογισθεί η παράσταση 35 63 84A 2x 3y 4z .

Πηγή: USA Mathematical Talent Search.

Λύση:

7 7 7 7 7 7 7/333 x y z 3 x y z 3(xyz) 3 , άρα ισότητα παντού. Αυτό σημαίνει,

επιπρόσθετα, ότι πετυχαίνουμε την ισότητα όταν x y z και άρα

(από την xyz 1 ) όταν x y z 1 . Άρα A 2 3 4 9 .

Β τρόπος

Από ΑΜ – ΓΜ: 7 7 7 7 7 7x +y +z 3 x +y +z =3 x=y=z=1 .

ΘΕΜΑ 233 (ΑΛΓΕΒΡΙΣΤΗΣ)

) Για τους θετικούς a,b,c 0 να αποδείξετε ότι 4 4 4 2 2 2a b c a bc ab c abc .

Λύση:

Θα χρησιμοποιήσω την βασική ανισότητα ΑΜ – ΓΜ:

4x y z w 4 xyzw (1) .

Οπότε έχω 4 4 4 4x a ,y b ,z c ,w a .

4 4 4 4 2 4 4 4 2(1) a b c a 4a bc 2a b c 4a bc .

Στη συνέχεια αντικαθιστούμε όπου 4w b και 4w c και με πρόσθεση των

ανισοτήτων που προκύπτουν αποδεικνύεται η ζητούμενη ανισότητα.

Page 207: Εισαγωγη σε διαγωνιστικα μαθηματικα για το γυμνασιο

http://www.mathematica.gr/forum/viewtopic.php?f=109&t=30104

Επιμέλεια: xr.tsif Σελίδα 207

Β τρόπος

Θέτω a x,b y,c z .

Ξέρουμε ότι ισχύει ότι:

4 4 2 2x y 2x y .

4 4 2 2y z 2y z .

4 4 2 2z x 2z x .

Με πρόσθεση κατά μέλη μένει να αποδείξω ότι 2 2 2 2 2 2 2 2 2x y y z z x x yz xy z xyz .

Όμως:

2 2x y 2xy , πολλαπλασιάζω με το 2z .

2 2y z 2yz , πολλαπλασιάζω με το 2x .

2 2z x 2zx , πολλαπλασιάζω με το 2y .

Μετά από πρόσθεση των σχέσεων κατά μέλη προκύπτει το ζητούμενο.

ΘΕΜΑ 234 (ΑΛΓΕΒΡΙΣΤΗΣ)

) Για τους θετικούς x,y,z 0 να αποδείξετε ότι

2 2 2 1 1 1(x y z ) 3(x y z)

x y z( ) .

Πηγή: Wisconsin Mathematical Talent Search

Λύση:

Βλέποντας το L.H.S παρατηρώ ότι 2

2 2 2 (x y z)x y z

3

. Άρα προκύπτει

Page 208: Εισαγωγη σε διαγωνιστικα μαθηματικα για το γυμνασιο

http://www.mathematica.gr/forum/viewtopic.php?f=109&t=30104

Επιμέλεια: xr.tsif Σελίδα 208

ότι 2(x y z) 1 1 1 (x y z) 1 1 1( ) 3(x y z) ( ) 3

3 x y z 3 x y z

1 1 1(x y z)( ) 9

x y z , που ισχύει.

Β τρόπος

Από την Andreescu 2

2 2 2 (x y z)x y z

1 1 1

.

Πάλι από την Andreescu 21 1 1 (1 1 1)

x y z x y z

.

Πολλαπλασιάζω κατά μέλη: 2 2 2 1 1 1

(x y z ) 3(x y z)x y z

( ) .

ΘΕΜΑ 235 (ΑΛΓΕΒΡΙΣΤΗΣ)

) Για τους μη αρνητικούς a,b,c,d,e 0 , να αποδείξετε ότι

2 2 2 2 2 3 3 3 3 3(a b c d e )(a b c d e ) 25abcde .

Πηγή: Russian Mathematical Olympiad

Λύση:

Έχουμε:

2 2 2 2 2 2 2 2 2 25a b c d e 5 a b c d e .

3 3 3 3 3 3 3 3 3 35a b c d e 5 a b c d e .

Άρα, με πολλαπλασιασμό κατά μέλη:

2 2 2 2 2 3 3 3 3 3 5 5 5 5 55(a b c d e )(a b c d e ) 25 a b c d e 25abcde .

Page 209: Εισαγωγη σε διαγωνιστικα μαθηματικα για το γυμνασιο

http://www.mathematica.gr/forum/viewtopic.php?f=109&t=30104

Επιμέλεια: xr.tsif Σελίδα 209

ΘΕΜΑ 236 (ΑΛΓΕΒΡΙΣΤΗΣ)

) Για τους θετικούς a,b,c 0 να αποδείξετε ότι:

8 8 8 4 2 2 2 4 2 2 2 4a b c a b c a b c a b c .

Λύση:

Θεωρούμε πρώτα τις τριάδες: 4 4 4 2 2 2 2 2 2(c ,b ,a ),(a b ,a c ,b c ) , οι οποίες έχουν

αντίθετη διάταξη.

(Πράγματι, αν 4 4 4 2 2 2c b a c b a c b a και άρα:

από 2 2 2 2 2 2 2 2 2 2c b a c a b a b a c .

Επίσης από 2 2 2 2 2 2 2 2 2 2b a b c a c a c b c .

Δείξαμε λοιπόν ότι πράγματι 2 2 2 2a c b c ).

Άρα από την ανισότητα της αναδιάταξης έχουμε ότι: 4 2 2 4 2 2 4 2 2 6 2 6 2 6 2c a b b a c a b c c a b c a b (1) .

Θεωρούμε τώρα τις τριάδες: 6 6 6 2 2 2(c ,b ,a ),(c ,b ,a ) , οι οποίες προφανώς έχουν

την ίδια διάταξη. Άρα: 8 8 8 6 2 6 2 6 2c b a c a b c a b (2) .

Από τις σχέσεις (1) και (2) έπεται το ζητούμενο.

Β τρόπος

Ισχύει από τη βασική 2 2 2x y z xy yz zx .

8 4 4 2 2 2 2 4 2 2a a b a b b c a b c .

Page 210: Εισαγωγη σε διαγωνιστικα μαθηματικα για το γυμνασιο

http://www.mathematica.gr/forum/viewtopic.php?f=109&t=30104

Επιμέλεια: xr.tsif Σελίδα 210

ΘΕΜΑ 237 (ΑΛΓΕΒΡΙΣΤΗΣ)

) Να αποδείξετε για τους θετικούς x,y,z,w 0 ισχύει

12 1 1 1 1 1 1 3 1 1 1 1

x y z w w x w y w z x y x z y z 4 x y z w( )

Πηγή: British Mathematical Olympiad.

Λύση:

Έχουμε λοιπόν από την γνωστή πλέον ανισότητα του Andreescu:

1 1 4

x y x y

,

1 1 4

x z x z

,

1 1 4

x w x w

,

1 1 1

y z y z

,

1 1 1

y w y w

,

1 1 1

z w z w

.

Με πρόσθεση όλων των πιο πάνω ανισοτήτων κατά μέλη, παίρνουμε :

1 1 1 1 1 1 1 1 1 13( ) 4( )

x y z w x y x z x w y z y w z w

,

και άρα το ζητούμενο.

Page 211: Εισαγωγη σε διαγωνιστικα μαθηματικα για το γυμνασιο

http://www.mathematica.gr/forum/viewtopic.php?f=109&t=30104

Επιμέλεια: xr.tsif Σελίδα 211

ΣΗΜΕΙΩΣΗ

Η αριστερή πλευρά της ανισότητας έχει απαντηθεί σε προηγούμενη

δημοσίευση.

ΘΕΜΑ 238 (ΑΛΓΕΒΡΙΣΤΗΣ)

)

Για τους θετικούς a,b,c 0 να αποδείξετε ότι b c a

a b c 8ca ba bc

( )( )( ) .

Λύση:

Από την ανισοϊσότητα αριθμητικού – γεωμετρικού μέσου ισχύει ότι:

b ba 2

ca c (1) .

c cb 2

ab a (2) .

a ac 2

bc b (3) .

Με πολλαπλασιασμό κατά μέλη προκύπτει η δοθείσα.

ΘΕΜΑ 239 (ΑΛΓΕΒΡΙΣΤΗΣ)

)

Για τους θετικούς a,b,c 0 να αποδείξετε ότι 3 3 3a b c

a b cbc ca ab

.

Λύση:

3 3 3 2 2 2 2 2 2

2 2 2 2 2

2 2 2

a b c a b b c c a( ) 3( ) 3(a b c ) (a b c) ...bc ca ab c a b

.

Β τρόπος

Μπορούμε επίσης να πολλαπλασιάσουμε με abc 0 και τότε είναι ίδια με την

233 και μπορεί να λυθεί με ΑΜ – ΓΜ.

Page 212: Εισαγωγη σε διαγωνιστικα μαθηματικα για το γυμνασιο

http://www.mathematica.gr/forum/viewtopic.php?f=109&t=30104

Επιμέλεια: xr.tsif Σελίδα 212

Γ τρόπος

Ακόμα θα μπορούσαμε να κάνουμε ΑΜ – ΓΜ στα παρακάτω και να

προσθέσουμε κατά μέλη.

3ab c 3a

bc .

3bc a 3b

ca .

3cb a 3c

ab .

ΘΕΜΑ 240 (ΑΛΓΕΒΡΙΣΤΗΣ)

) Για τους θετικούς x,y,z 0 να αποδείξετε ότι

3 3 3

2 2 2 2 2 2

y z z x x y 3

x (xy z ) y (zy x ) z (xz y ) 2

.

Λύση:

Έχουμε 3 3 3

2 2 2 2 2 2

y z z x x y(x,y,z)

x (xy z ) y (zy x ) z (xz y )

ή ισοδύναμα 3 3

2 2cyc cyc 3

y z y(x,y,z)

y zx (xy z ) x ( )z x

.

Θέτουμε x

ky ,

ym

z και

zn

x , όπου k,m,n 0 θετικοί πραγματικοί

αριθμοί.

Οι παραπάνω μετατροπές, έχουν ως αποτέλεσμα, να τεθεί ο περιορισμός

kmn 1 στις μεταβλητές.

Συνεπώς,

Page 213: Εισαγωγη σε διαγωνιστικα μαθηματικα για το γυμνασιο

http://www.mathematica.gr/forum/viewtopic.php?f=109&t=30104

Επιμέλεια: xr.tsif Σελίδα 213

23 3 3 3 2

3cyc cyc cyc cyc cyc

11 m n km n (mn) k

1 1 1 1 1 1 1 1k (m n)

kn km n m n m n m

( )

.

Στην τελευταία, εφαρμόζοντας την ανισότητα Cauchy – Schwarz στη μορφή

Engel παίρνουμε

2 2

cyc

1 1 1 1( ) ( )

1 1 1 1 3k k m n1 1 1 1 1 2 k m n 22( )n m k m n

( )

.

Έτσι, από την τελευταία το ζητούμενο έπεται.

Β τρόπος

Μια λίγο διαφορετική λύση:

Παρατηρούμε ότι η ανισότητα είναι ομογενής οπότε δίχως βλάβη της

γενικότητας μπορούμε να υποθέσουμε ότι xyz 1 .

Άρα έχουμε:

3 3 3 4 2 4 2 4 2

2 2 2 2 2 2 2 2 2 2 2 2

y z z x x y y z z x x y

x (xy z ) y (yz x ) z (zx y ) x y z x y z x y z x y z

2 2 2 2 2 2 2

2 2 2

(x y y z z x) x y y z z x 3 3xyz

2(x y y z z x) 2 2 2

.

Για τις ανισότητες χρησιμοποιήθηκαν αντίστοιχα οι C – S και AM – GM.

Γ τρόπος

Και μια ακόμα λύση:

3 3 3

2 2 2 2 2 2

y z z x x y

x (xy z ) y (zy x ) z (xz y

4 2 4 2 4 2

2 2 2 2 2 2

y z z x x y

yzx (xy z ) zxy (zy x ) xyz (xz y )

Page 214: Εισαγωγη σε διαγωνιστικα μαθηματικα για το γυμνασιο

http://www.mathematica.gr/forum/viewtopic.php?f=109&t=30104

Επιμέλεια: xr.tsif Σελίδα 214

2 2 2 2 2 2 2

2 2 2 2 2 2

(y z z x x y) x y y z z x

xyz(x y xz y z yx z x zy ) 2xyz

.

Αρκεί λοιπόν να αποδείξουμε ότι: 2 2 2x y y z z x 3

2xyz 2

, ή αρκεί:

2 2 2x y y z z x 3xyz (1) .

Όμως από την ανισότητα ΑΜ – GM έχουμε:

2 2 2 2 2 2 3 3 33 3x y y z z x 3 x yy zz x 3 x y z .

Άρα 2 2 2x y y z z x 3xyz και άρα η (1) είναι αληθής.

Δ τρόπος

Μια ακόμα απόδειξη:

Λόγω ομογένειας ας είναι xyz 1 . Τότε, υπάρχουν θετικοί a,b,c 0 ώστε

a b cx ,y ,z

b c a .

Είναι

5 5 5 6 6 6

3 3 3 3 3 3 3 3 3 3 3 3

a b c a b cL.H.S= + + = + +

bc(b +c ) ca(c +a ) ab(a +b ) abc(b +c ) abc(c +a ) abc(a +b )

3 3 3 2 3 3 3

3 3 3

(a b c ) 1 a b c 3

2abc(a b c ) 2 abc 2

.

ΘΕΜΑ 241 (ΑΛΓΕΒΡΙΣΤΗΣ)

) Για τους θετικούς x,y,z 0 με xyz 1 να αποδείξετε ότι

1 1 11

1 x y 1 y z 1 x z

.

Λύση:

Θέτουμε 3 3 3x a ,y b ,z c οπότε a,b,c 0 και abc 1 .

Page 215: Εισαγωγη σε διαγωνιστικα μαθηματικα για το γυμνασιο

http://www.mathematica.gr/forum/viewtopic.php?f=109&t=30104

Επιμέλεια: xr.tsif Σελίδα 215

Χρησιμοποιούμε την 3 3A B AB(A B) με A,B 0 .

Είναι

3 3 3 3 3 3

1 1 1 1 1 1

1 x y 1 y z 1 x z 1 a b 1 b c 1 a b

1 1 1

1 ab(a b) 1 bc(b c) 1 ca(c a)

c a b1

c abc(a b) a abc(b c) b abc(c a)

.

ΘΕΜΑ 242 (ΔΗΜΗΤΡΗΣ ΙΩΑΝΝΟΥ)

) Να βρεθούν αν υπάρχουν οι ακέραιοι αριθμοί x,yπου είναι τέτοιοι ώστε:

2 2x y 2015 .

Λύση:

Επειδή το άθροισμα των 2 2x y είναι περιττός, ο ένας εκ των

2 2x ,y είναι

άρτιος και ο άλλος περιττός. Άρα ένας εκ των x,y είναι άρτιος και ο άλλος

περιττός.

Ας είναι x 2a, y 2b 1, x,y Z

Η 2 2x y 2015 γράφεται 2 24a 4b 4b 2014 .

Όμως, το αριστερό μέλος της προηγούμενης σχέσης είναι πολλαπλάσιο του 4,

ενώ το δεξί όχι.

Άρα η εξίσωση 2 2x y 2015 είναι αδύνατη στους ακεραίους.

ΘΕΜΑ 243 (ΘΑΝΟΣ ΜΑΓΚΟΣ)

) Να λυθούν οι εξισώσεις:

α) 2 2x y 2010 .

Page 216: Εισαγωγη σε διαγωνιστικα μαθηματικα για το γυμνασιο

http://www.mathematica.gr/forum/viewtopic.php?f=109&t=30104

Επιμέλεια: xr.tsif Σελίδα 216

β) 2 2x y 2016 .

Λύση:

α) Κλασική εφαρμογή της ιδιότητας 2 23| a b 3| a, 3| b .

Σύμφωνα με την παραπάνω ιδιότητα a 3k , b 3 και 2 2 2 2a b 9k 9 .

Όμως 2 2 0 09 19k 2 είναι άτοπο αφού το 9 δε διαιρεί το 2010 .

Επομένως η εξίσωση δεν έχει λύση στο Z.

β) Όταν δύο αριθμοί έχουνε άθροισμα άρτιο αριθμό τότε είναι και οι δύο άρτιοι

ή και οι δύο περιττοί.

Περίπτωση Α

Έστω ότι και οι δύο εκ των x,yείναι άρτιοι, δηλαδή x 2a και y 2b . Τότε η

εξίσωση θα μετασχηματιστεί ως εξής:

2 2 2 2 2 2x y 2016 4a 4b 2016 a b 504 . Στη νέα διοφαντική

εξίσωση που προκύπτει θα πάρω άλλες δύο υποπεριπτώσεις:

Υποπερίπτωση Α

Όπως και πριν, αφού οι a,b έχουνε άρτιο άθροισμα, τότε θα είναι είτε και οι

δύο άρτιοι είτε και οι δύο περιττοί. Έστω λοιπόν ότι είναι και οι δύο άρτιοι,

δηλαδή a 2w και b 2z , οπότε έχουμε την εξίσωση 2 2w z 126 . Με

δοκιμές αν μπορούν τετράγωνα να έχουνε άθροισμα βρίσκουμε ότι η

προαναφερθείσα είναι αδύνατη στο σύνολο των ακεραίων.

Υποπερίπτωση Β

Έστω τώρα ότι και οι δύο αριθμοί είναι περιττοί, δηλαδή a 2w 1 και

b 2z 1 οπότε και η εξίσωση μετασχηματίζεται ως εξής: 2 2 2 24w 4w 1 4z 4z 1 504 4(w w z z) 502 που προφανώς

και είναι αδύνατη διότι 502 o 4 .

Page 217: Εισαγωγη σε διαγωνιστικα μαθηματικα για το γυμνασιο

http://www.mathematica.gr/forum/viewtopic.php?f=109&t=30104

Επιμέλεια: xr.tsif Σελίδα 217

Περίπτωση B

Έστω ότι και οι δύο εκ των x,yείναι περιττοί, δηλαδή x 2a 1 και y 2b 1

. Τότε η εξίσωση θα μετασχηματιστεί ως εξής:2 2 2 24a 4a 1 4b 4b 1 2016 2(a a b b) 1007 , που προφανώς

και είναι αδύνατη.

Άρα η δοσμένη εξίσωση δεν έχει λύσεις στο σύνολο των ακεραίων.

ΘΕΜΑ 244 (ΔΗΜΗΤΡΗΣ ΙΩΑΝΝΟΥ)

)

Να αποδείξετε ότι ο αριθμός: n 1 n 1100 2 1000 8

3 9

είναι ακέραιος.

Υπόδειξη

Μπορείτε να χρησιμοποιήσετε την παρακάτω γνωστή πρόταση:

n n(a b) a b , ειδικά: n(1 a) 1 a .

Λύση:

Άμα ισχύει n 1 n 1(a b) a o b , τότε έχω λύση... Λοιπόν:

Έστω n 1 n 1100 2 1000 8

A3 9

n 1 n 1100 2 (99 1) 2 1 99k 2 3(33k 1)33k 1

3 3 3 3

n 1 n 11000 8 (999 1) 8 1 999m 8 9(111m 1)111m 1

9 9 9 9

.

Άρα: A 33k 1 111m 1 111m 33k 2 Z .

ΘΕΜΑ 245 (ΔΗΜΗΤΡΗΣ ΙΩΑΝΝΟΥ)

)

Αν ο αριθμός 18n 9 9 4n 1

A2

με n N είναι ρητός, να αποδείξετε ότι

και ο αριθμός A είναι επίσης ρητός.

Page 218: Εισαγωγη σε διαγωνιστικα μαθηματικα για το γυμνασιο

http://www.mathematica.gr/forum/viewtopic.php?f=109&t=30104

Επιμέλεια: xr.tsif Σελίδα 218

Λύση:

Αφού ο αριθμός Aείναι ρητός άρα η υπόριζη ποσότητα πρέπει να είναι τέλειο

τετράγωνο ακεραίου (*) και μάλιστα περιττού αριθμού.

Άρα 24n 1 (2k 1) , για κάποιο ακέραιο αριθμό kδηλαδή 2n k k .

Αντικαθιστώντας στο Aπαίρνουμε:

2

218(k k) 9 9(2k 1)A 9k

2

. Συνεπώς A 3k Z .

(*) : Αφού ο Aείναι ρητός άρα ο 4n 1 είναι ρητός ως πηλίκο ρητών. Άρα ο

4n 1 που είναι ακέραιος είναι τετράγωνο ρητού και άρα τελικά ο 4n 1 είναι

τετράγωνο ακεραίου ( δώστε μια αυστηρή απόδειξη).

Β τρόπος

Έχω:

18n 9 9 4n 1A 2A 18n 9(1 4n 1) ...

2

2A 18n4n 1 1 Q

9

(1) .

Τώρα:

18n 9 9 4n 1 9 9 9A A 9n 4n 1 A 2n 1 4n 1

2 2 2 2

9 9A 4n 2 2 4n 1 A 4n 1 2 4n 1 1

4 4

29 9 9

A 4n 1 1 A 4n 1 1 A 4n 1 14 2 2

.

(αφού 4n 1 1 για κάθε n N .

Είναι προφανές πως λόγω του (1) η A είναι ρητός αριθμός.

Page 219: Εισαγωγη σε διαγωνιστικα μαθηματικα για το γυμνασιο

http://www.mathematica.gr/forum/viewtopic.php?f=109&t=30104

Επιμέλεια: xr.tsif Σελίδα 219

ΘΕΜΑ 246 (ΔΗΜΗΤΡΗΣ ΙΩΑΝΝΟΥ)

) Από όλα τα ορθογώνια παραλληλεπίπεδα με τον ίδιο όγκο, να βρεθεί αυτό που

έχει την μικρότερη επιφάνεια.

ΣΗΜΕΙΩΣΗ

Μπορείτε να εφαρμόσετε μια πολύ γνωστή πλέον ανισότητα, από αυτές που

αναφέρθηκαν σε προηγούμενη δημοσίευση.

Λύση:

Αν οι διαστάσεις του παραλληλεπιπέδου είναι οι a,b,c 0 , ο όγκος του είναι

ίσος με abc k και η επιφάνειά του είναι ίση με 2(ab bc ca) .

Από την ανισότητα ΑΜ – ΓΜ ξέρουμε ότι 23ab bc ca 3 (abc) .

Στο συγκεκριμένο λοιπόν παραλληλεπίπεδο ισχύει

2 233ab bc ca 3 (abc) k .

Κι επειδή στην ΑΜ – ΓΜ η ισότητα ισχύει όταν οι μεταβλητές είναι ίσες

μεταξύ τους, έχουμε ab bc ca a b c .

Το παραλληλεπίπεδο λοιπόν με τη συγκεκριμένη ιδιότητα είναι ο κύβος.

ΘΕΜΑ 247 (ΔΗΜΗΤΡΗΣ ΙΩΑΝΝΟΥ)

) Αν 2x y 6 , να βρείτε τις τιμές των x,y ώστε η παράσταση

2A x 2xy

να παίρνει την μέγιστη τιμή.

Λύση:

y 2(3 x) .

Επομένως 2 2 2 2A x 4x(3 x) x 12x 4x 3x 12x .

Το τριώνυμο αυτό παίρνει μέγιστη τιμή ίση με 144

124a 12

για

Page 220: Εισαγωγη σε διαγωνιστικα μαθηματικα για το γυμνασιο

http://www.mathematica.gr/forum/viewtopic.php?f=109&t=30104

Επιμέλεια: xr.tsif Σελίδα 220

b 12x 2

2a 6

και για y 2 .

ΘΕΜΑ 248 (ΔΗΜΗΤΡΗΣ ΙΩΑΝΝΟΥ)

) Δίνεται το πολυώνυμο:

n 2 nP(x) 3x 5x 8 , *n N .

α) Αν έχει ρίζα το x 1 , να δειχθεί ότι ο φυσικός αριθμός nείναι περιττός.

β) Αν ισχύει P(3) 104 , να παραγοντοποιηθεί το πολυώνυμο P(x) .

Λύση:

α) Είναι n 2 n3·( 1) 5·( 1) 8 0 . Αν ο nείναι άρτιος θα είναι 3·1 5·1 8 0 ,

άτοπο. Επομένως ο nείναι περιττός.

β) n 3 n n 3P(3) 3 5·3 96 3 96 n 1 .

Επομένως 3 3P(x) 3x 5x 8 3x 3x 8x 8 3x(x 1)(x 1) 8(x 1)

2(x 1)(3x 3x 8) .

ΘΕΜΑ 249 (ΔΗΜΗΤΡΗΣ ΙΩΑΝΝΟΥ)

) Να βρεθεί τριψήφιος θετικός αριθμός, ο οποίος να λήγει σε 7 και αν

μετακινήσουμε το τελευταίο ψηφίο του στην πρώτη από τα αριστερά θέση, ο

νέος αριθμός είναι πενταπλάσιος από τον αρχικό.

Λύση:

Αν δεν έχω κάνει λάθος στη λύση μου, τότε δεν υπάρχει ο ζητούμενος αριθμός.

Ο πρώτος αριθμός που δίνεται θα παίρνει τη μορφή xy7 100x 10y 7 ενώ

με την μετάθεση ψηφίων που ζητείται παίρνουμε τον αριθμό

7xy 700 10x y .

Επίσης από την εκφώνηση ισχύει ότι

7xy 5·xy7 700 10x y 500x 50y 35 49(10x y) 665 που όμως

Page 221: Εισαγωγη σε διαγωνιστικα μαθηματικα για το γυμνασιο

http://www.mathematica.gr/forum/viewtopic.php?f=109&t=30104

Επιμέλεια: xr.tsif Σελίδα 221

είναι άτοπο αφού το 665 49 .

Β τρόπος

Αφού:

490x 49y 665 70x 7y 95 7y 95 70x 7y 5(19 14x) .

Από την παραπάνω σχέση καταλήγουμε ότι 5| 7y 5| y , διότι (5,7) 1 .

Άρα y 5 ή y 0 . Και στις δύο όμως περιπτώσεις καταλήγουμε σε άτοπο.

ΘΕΜΑ 250 (ΔΗΜΗΤΡΗΣ ΙΩΑΝΝΟΥ)

) Σε ένα τεστ με άριστα το 100, οι επιδόσεις των μαθητών A,B,C,D,E είναι 5

διαφορετικοί ακέραιοι, όλοι μεγαλύτεροι του 91. Γνωρίζουμε ότι ο μέσος όρος

των επιδόσεων των A,B,C, είναι 95, ενώ ο μέσος όρος των επιδόσεων των

B,C,D είναι 94. Αν ο Aέχει την καλύτερη επίδοση (δηλαδή είναι πρώτος

στην σειρά), ο E είναι τρίτος στην σειρά με επίδοση 96, ποια θα είναι η σειρά

του D ;

Λύση:

Σύμφωνα με την υπόθεση είναι:

A B C95 A B C 285

3

(1) .

B C D94 B C D 282

3

(2) .

Είναι ακόμα: 96 A 100 (3) .

Πολλαπλασιάζοντας τα μέλη της εξίσωσης (2) με το 1 και προσθέτοντας

κατά μέλη παίρνουμε: A D 3 D A 3 .

Αντικαθιστώντας στην (3) παίρνουμε: 93 D 97 (4) .

Υποθέτουμε χωρίς βλάβη της γενικότητας ότι B C .

Page 222: Εισαγωγη σε διαγωνιστικα μαθηματικα για το γυμνασιο

http://www.mathematica.gr/forum/viewtopic.php?f=109&t=30104

Επιμέλεια: xr.tsif Σελίδα 222

Διακρίνουμε τις περιπτώσεις:

D 97

Τότε D 94 ή D 95 .

Από την σχέση (2) παίρνουμε: B C 187 ή B C 188 .

Όμως 96 B A και C 91 , άτοπο σύμφωνα με την υπόθεση.

D 97 .

Από τη σχέση (2) παίρνουμε:

B C 185 . Εύκολα βλέπουμε ότι ισχύει για τις τιμές B 93 και C 92 .

Άρα: A 100,D 97,E 96,B 93,C 92 και άρα ο Dείναι 2 .

ΘΕΜΑ 251 (ΔΗΜΗΤΡΗΣ ΙΩΑΝΝΟΥ)

) Αν η εξίσωση 2x x 3ab 1 έχει ρίζα τον αριθμό a b , να βρείτε τις ρίζες

της εξίσωσης.

Λύση:

2 2x x 3ab 1 x x 3ab 1 0 (1) . Αν a b ρίζα της (1) , τότε

2 2 2a b a b 3ab 1 0 a b 1 a b b 1 0 (2) με

2

a3 b 1 , δηλαδή για να υπάρχει a R ο οποίος να ικανοποιεί την (2)

θα πρέπει b 1 0 b 1 και η (2) δίνει 2a 2a 1 0 a 1 και η αρχική

εξίσωση γίνεται 2x x 2 0 με άλλη ρίζα (εκτός της 1

x a b 2 ) την

2x 1 .

ΘΕΜΑ 252 (Socrates)

)

Λύστε το σύστημα:

x y z

x y z

x y z

2 3 5 540

3 2 5 360

5 3 2 1350

.

Page 223: Εισαγωγη σε διαγωνιστικα μαθηματικα για το γυμνασιο

http://www.mathematica.gr/forum/viewtopic.php?f=109&t=30104

Επιμέλεια: xr.tsif Σελίδα 223

Λύση:

Ας δούμε μία λύση στους ακεραίους...

Είναι:

2 3 1540 2 ·3 ·5

2 3 1360 3 ·2 ·5

2 3 11350 5 ·3 ·2

Άρα: (x,y,z) (2,3,1) .

Β τρόπος

Είναι: x y z 2 3

x y z 2 3

2 3 5 2 ·3 ·5

3 2 5 3 ·2 ·5

και με διαίρεση κατά μέλη παίρνουμε:

x y 1 x y 1

2 2 2 2 2x y 1 y x 1

3 3 3 3 3

(1) .

Επίσης: x y z 2 3

x y z 2 3

2 3 5 2 ·3 ·5

5 3 2 5 ·3 ·2

και με διαίρεση κατά μέλη παίρνουμε:

x z 1

2 2x z 1 x z 1

5 5

(2) .

και y z 2 (3) .

Τελικά:

x y z 2 3 z 2 z z 2 3 z2 3 5 2 ·3 ·5 2·2 ·3 ·3 5 2 ·3 ·5 30 30 z 1....x 2....y 3 .

Άρα προκύπτει η τριάδα: (x,y,z) (2,3,1) που είναι και η μοναδική.

ΘΕΜΑ 253 (ΔΗΜΗΤΡΗΣ ΙΩΑΝΝΟΥ)

) Να λυθεί στο σύνολο των ρητών αριθμών, η εξίσωση:

Page 224: Εισαγωγη σε διαγωνιστικα μαθηματικα για το γυμνασιο

http://www.mathematica.gr/forum/viewtopic.php?f=109&t=30104

Επιμέλεια: xr.tsif Σελίδα 224

2 3 3 x 3 y 3 .

Λύση:

Υψώνοντας στο τετράγωνο έχουμε 2 3 3 (x y) 3 2 3xy ή

2 3 x y 2 xy ή 2 x y 3 2 xy (*) .

Υψώνοντας την (*) στο τετράγωνο έχουμε 2(2 x y) 3 4xy 4 3xy

οπότε ο αριθμός 3xy d είναι ρητός.

Επομένως, η (*) γράφεται 3 2d

2 x y3

.

Αν 2 x y 0 τότε 3 2d

3 Q2 x y

, άτοπο. Άρα 2 x y 3 2d 0 .

Έτσι έχουμε το σύστημα 3

x y 2, xy4

με λύσεις 3 1

,2 2

και 1 3

,2 2

.

Όμως είναι x 3 y 3 0 x y , οπότε κρατάμε την 3 1

,2 2

.

ΘΕΜΑ 254 (ΚΛΕΟΒΟΥΛΟΣ ΚΟΦΟΝΙΚΟΛΑΣ)

) Βρείτε όλα τα διατεταγμένα ζεύγη (a,b)θετικών ακεραίων τέτοια ώστε οι

αριθμοί 3a b 1

a 1

και

3b a 1

b 1

, να είναι και οι δύο θετικοί ακέραιοι.

Λύση:

Είναι 3 2a 1| a b a a(a b 1) . Κι επειδή *a N θα είναι

2a 1| a b 1 a 1| a(ab 1) a 1| ab 1 a 1| b 1 (1) .

Ομοίως είναι 3 2b 1| b a b b(ab 1) . Επειδή ο b είναι θετικός ακέραιος θα

Page 225: Εισαγωγη σε διαγωνιστικα μαθηματικα για το γυμνασιο

http://www.mathematica.gr/forum/viewtopic.php?f=109&t=30104

Επιμέλεια: xr.tsif Σελίδα 225

είναι 2b 1| ab 1 b 1| b(ab 1) b 1| a 1 .

Από τη σχέση (1) θα ισχύει b 1| b 1 b 1| 2 .

Συνεπώς υπάρχουν δύο περιπτώσεις.

b 2 .

Σ' αυτήν την περίπτωση είναι a 1|3 a 2 , απ' όπου παίρνουμε τη λύση

(a,b) (2,2) .

b 3 .

Εδώ θα είναι a 1| 4 , απ' όπου παίρνουμε και τις λύσεις (a,b) (1,3),(3,3) .

ΘΕΜΑ 255 (Socrates)

) Προσδιορίστε όλα τα ζεύγη (a,b)θετικών ρητών αριθμών ώστε

a b 4 7 .

Λύση:

Υψώνουμε στο τετράγωνο και έχουμε a b 2 ab 4 7 .

Οπότε ουσιαστικά λύνουμε το σύστημα

7ab

4

a b 4

, το οποίο λύσεις

1 7 7 1(a,b) , , ,

2 2 2 2

.

ΘΕΜΑ 256 (Θεοχάρης Μαλαμίδης)

)

Αν a,b,c 0 τότε να δείξετε ότι: 3 3 3 2

a b c 27

b c a (a b c)

.

Page 226: Εισαγωγη σε διαγωνιστικα μαθηματικα για το γυμνασιο

http://www.mathematica.gr/forum/viewtopic.php?f=109&t=30104

Επιμέλεια: xr.tsif Σελίδα 226

Λύση:

Σύμφωνα με ΑΜ – ΓΜ: 32

1LHS 3

(abc) .

Οπότε αρκεί 3

6 3 2

2 6

1 9(a b c) 9 (abc)

(abc) (a b c)

.

Πάλι με 6 3 23LHS (3 abc) 9 (abc) RHS , άρα αποδείξαμε το ζητούμενο.

ΘΕΜΑ 257 (ΔΗΜΗΤΡΗΣ ΙΩΑΝΝΟΥ)

) Αν a b c 0 , a,b,c ακέραιοι, να αποδείξετε ότι ο αριθμός 4 4 42a 2b 2c

είναι τέλειο τετράγωνο ακεραίου.

ΣΗΜΕΙΩΣΗ

Η άσκηση αυτή είχε προταθεί από τον δικό μας Αλέξανδρο Συγκελάκη, όταν

ήταν μαθητής, σε παλαιό τεύχος του "ΕΥΚΛΕΙΔΗ Β".

Λύση:

http://www.mathematica.gr/forum/viewtopic.php?f=58&t=33926#p156977

ΘΕΜΑ 258 (ΔΗΜΗΤΡΗΣ ΙΩΑΝΝΟΥ)

)

Αν *a,b,c R και αν 1 a 1 b 1 c

b c a

, να αποδείξετε ότι a b c .

Λύση:

Θέτω:1 a 1 b 1 c

kb c a

.

Άρα: 1 a kb (1) .

Page 227: Εισαγωγη σε διαγωνιστικα μαθηματικα για το γυμνασιο

http://www.mathematica.gr/forum/viewtopic.php?f=109&t=30104

Επιμέλεια: xr.tsif Σελίδα 227

1 b kc (2) .

1 c ka (3) .

Παίρνοντας τη (1) και (2) έχω: a b k(b c) (4) .

Ομοίως για την (1) και (3) και έχω: a c k(b a) (5) .

Διαιρώντας κατά μέλη τις (4) και (5) έχω:

2 2 2a c b aa b c ab bc ca 0

a b b c

2 2 2 2 2 22a 2b 2c 2ab 2bc 2ac 0 (a b) (b c) (c a) 0 .

Άρα:

a b 0 a b .

b c 0 b c .

c a 0 c a .

Από τις τελευταίες παίρνουμε a b c και έτσι αποδείξαμε το ζητούμενο.

ΘΕΜΑ 259 (Socrates)

)

Λύστε το σύστημα:

1 12

y x z y

12 x

6 z

x y z 6

.

Λύση:

Θέτω a y x,b z y,c 6 z . Τότε a,b,c 0 και a b c x 6 . Η

πρώτη ανισότητα δίνει 1 1

2a b και η δεύτερη

12 x

c .

Page 228: Εισαγωγη σε διαγωνιστικα μαθηματικα για το γυμνασιο

http://www.mathematica.gr/forum/viewtopic.php?f=109&t=30104

Επιμέλεια: xr.tsif Σελίδα 228

Οπότε 1 1 1

xa b c .

Όμως για z 0 είναι 1

z 2z

με ισότητα αν και μόνο αν z 1 .

Οπότε 1 1 1

6 a b c x a b c 6a b c

και επειδή έχουμε ισότητα

παίρνουμε a b c 1 και άρα x 3 . Οπότε είναι x 3,y 4,z 5 .

ΘΕΜΑ 260 (ΔΗΜΗΤΡΗΣ ΙΩΑΝΝΟΥ)

) Αν a,b,c είναι τα μήκη των πλευρών ενός τριγώνου ABC, και αν το τριώνυμο:

2a b 2cx x

b c a είναι τέλειο τετράγωνο πρωτοβάθμιου πολυωνύμου, να

αποδείξετε ότι: a

1 3c

. (Γ Γυμνασίου)

Λύση:

Για να είναι το πολυώνυμο αυτό τέλειο τετράγωνο πρωτοβάθμιου πολυωνύμου

σημαίνει ότι 2

2

b 8c0 0 b 2c

c b .

Από την τριγωνική ανισότητα τώρα ισχύει a

b c a 3c a 3c

και

aa c b a c 1

c . Συνεπώς

a1 3

c .

ΘΕΜΑ 261 (ΔΗΜΗΤΡΗΣ ΙΩΑΝΝΟΥ)

) Δίνεται η συνάρτηση f (x) x 1 . Αν a 0 , να αποδείξετε ότι:

f (a) f (a 1)1 2

f (a 1) f (a 2)

. (Γ Γυμνασίου)

Page 229: Εισαγωγη σε διαγωνιστικα μαθηματικα για το γυμνασιο

http://www.mathematica.gr/forum/viewtopic.php?f=109&t=30104

Επιμέλεια: xr.tsif Σελίδα 229

Λύση:

Αρχικά θα αποδείξω ότι: f (a) f (a 1)

1f (a 1) f (a 2)

.

Είναι:

2a 1 a 21 (a 3)(a 1) (a 2) (a 2)(a 3) ...

a 2 a 3

2 2a 3a 1 0 (a 1) a .

Αφού 2a 0 (a 1) 0 και a 0 , άρα η τελευταία ισχύει.

Τώρα μένει να αποδείξω ότι: f (a) f (a 1)

2f (a 1) f (a 2)

.

Μετά από κάποιες πράξεις ρουτίνας καταλήγουμε ότι 2a 5 0 .

Όμως είναι: a 0 2a 0 και 5 0 και έτσι η τελευταία ισχύει.

ΘΕΜΑ 262 (ΔΗΜΗΤΡΗΣ ΙΩΑΝΝΟΥ)

) Αν a,b,c,d,x,y,z,w 0 και αν a x,b y,c z,d w , να αποδείξετε ότι

α) (a x)(b y) 4ab .

β) (a x)(b y) 4xy .

γ) 1

xyzw (a x)(b y)(c z)(d w) abcd16

. (Β,Γ Γυμνασίου)

Λύση:

α) Πιο απλά: (a x)(b y) (a a)(b b) 4ab και

β) (a x)(b y) (x x)(y y) 4xy .

Με ακριβώς το ίδιο τρόπο η γ).

Page 230: Εισαγωγη σε διαγωνιστικα μαθηματικα για το γυμνασιο

http://www.mathematica.gr/forum/viewtopic.php?f=109&t=30104

Επιμέλεια: xr.tsif Σελίδα 230

ΘΕΜΑ 263 (ΔΗΜΗΤΡΗΣ ΙΩΑΝΝΟΥ)

) Έστω

1 2x ,x είναι οι ρίζες της εξίσωσης 2x bx b 1 0 , όπου b R . Να

αποδείξετε ότι: 2 2

1 2(1 x )(1 x ) 4(b 1) . (Γ Γυμνασίου)

Λύση:

Σύμφωνα με τους τύπους του Vieta έχουμε:

1 2

bx x b

a

(1) .

1 2

b 1x x b 1

a

(2) .

Αρκεί να αποδείξουμε ότι:

2 2 2 2 2 2

1 2 2 1 1 2(1 x )(1 x ) 4(b 1) 1 x x x ·x 4(b 1)

2 2 2 2 2 2 2

1 2 1 2 1 2 1 2x x x x 4b 3 (x ·x ) (x x ) 4b 3

(1),(2)2 2 2 2

1 2 1 2 1 2(x ·x ) [(x x ) 2x ·x ] 4b 3 (b 1) [( b) 2(b 1)] 4b 3

2 2b 2b 1 b 2b 2 4b 3 3 3 .

Η τελευταία όμως ισχύει και άρα και η ζητούμενη σχέση.

ΘΕΜΑ 264 (ΔΗΜΗΤΡΗΣ ΙΩΑΝΝΟΥ)

) Αν x,yείναι τα μήκη των καθέτων πλευρών ενός ορθογωνίου τριγώνου και αν

4 4(x y) (x y) 32xy , να υπολογίσετε το μήκος της υποτείνουσας.

Λύση:

Είναι:

4 4 2 2 2 2(x y) (x y) 32xy [(x y) ] [(x y) ] 32xy

Page 231: Εισαγωγη σε διαγωνιστικα μαθηματικα για το γυμνασιο

http://www.mathematica.gr/forum/viewtopic.php?f=109&t=30104

Επιμέλεια: xr.tsif Σελίδα 231

2 2 2 2[(x y) (x y) ][(x y) (x y) ] 32xy

2 2 2 2 2 24xy(2x 2y ) 32xy 2(x y ) 8 x y 4 .

Άρα, η υποτείνουσα είναι 4 2 .

ΘΕΜΑ 265 (ΔΗΜΗΤΡΗΣ ΙΩΑΝΝΟΥ)

) Δίνεται η εξίσωση: 2 3 2 29x 18x 10 (a 1)x (3a 1)x 4a 1 . Να

βρεθούν οι τιμές του a , για τις οποίες η πιο πάνω εξίσωση έχει ρίζα τον

αριθμό 1 . (Γ Γυμνασίου)

Λύση:

Με αντικατάσταση παίρνουμε:

3 2 3 2a 1 (3a 1) 4a 1 1 a 1 3a 1 4a 1 1

3 2 3 2 2a 3a 4a 2 0 a a 2a 4a 2 0

2 2 2 2a (a 1) 2(a 2a 1) 0 a (a 1) 2(a 1) 0

2 2(a 1)[a 2(a 1)] 0 (a 1)(a 2a 2) 0 .

Άρα: 2a 2a 2 0 ή a 1 0 .

Η πρώτη εξίσωση είναι αδύνατη στο R αφού έχει 4 0 .

Άρα a 1 0 a 1 , που είναι και η μοναδική λύση.

ΘΕΜΑ 266 (ΔΗΜΗΤΡΗΣ ΙΩΑΝΝΟΥ)

) Αν a,b,x,y 0 και a b 4,x y 9 , να αποδείξετε ότι: ax by 6 .

Λύση:

Θα χρησιμοποιήσουμε την ανισότητα:

Page 232: Εισαγωγη σε διαγωνιστικα μαθηματικα για το γυμνασιο

http://www.mathematica.gr/forum/viewtopic.php?f=109&t=30104

Επιμέλεια: xr.tsif Σελίδα 232

ax by a b x y (1) .

Πράγματι, η (1) γράφεται ισοδύναμα:

2

ax by a b x y ax by 2 abxy ax ay bx by

2

ay 2 abxy bx 0 ay bx 0 ,

που ισχύει, με το ίσον αν και μόνο αν a x

ay bxb y

.

Από την (1) έχουμε ότι ax by 4·9 36 6 , και το ζητούμενο έπεται.

Η ισότητα ισχύει για 4 4 9 9

a,b,x,y , , ,1 1 1 1

, όπου 0 .

Β τρόπος

ΑΝΙΣΟΤΗΤΑ: (a b)(x y) ax by .

ΑΠΟΔΕΙΞΗ

2(a b)(x y) ( ax by) ax ay bx by ax by 2 abxy

2ay bx 2 ay·bx ( ay bx) 0 , που ισχύει.

Στην περίπτωσή μας λοιπόν θα είναι ax by 4·9 6 .

Γ τρόπος

Και μία λύση με Cauchy – Schwarz:

Έχω:

2 2 2 2

ax by a x b y a b x y

a b x y 4 9 6 .

Page 233: Εισαγωγη σε διαγωνιστικα μαθηματικα για το γυμνασιο

http://www.mathematica.gr/forum/viewtopic.php?f=109&t=30104

Επιμέλεια: xr.tsif Σελίδα 233

ΘΕΜΑ 267 (ΔΗΜΗΤΡΗΣ ΙΩΑΝΝΟΥ)

) Για κάθε x,y,z 0 , να αποδείξετε ότι:

xy yzx y z zx2 2 2 2 2 2 .

Λύση:

Με την ανισότητα αριθμητικού – γεωμετρικού μέσου.

Έχω:

x yxy xyx y x y x y22 2 2 2 2 2·2 2·2 2 2 2·2

(1) .

y zyz yzy z y z y z22 2 2 2 2 2·2 2·2 2 2 2·2

(2) .

z xz x z x zx z x zx22 2 2 2 2 2·2 2·2 2 2 2·2

(3) .

Προσθέτοντας κατά μέλη τις (1) , (2) , (3) έχουμε το ζητούμενο.

ΘΕΜΑ 268 (ΔΗΜΗΤΡΗΣ ΙΩΑΝΝΟΥ)

)

Αν *x,y R , να αποδείξετε ότι:

2 2

2 2

x y x y4( ) 9( ) 10 0

y x y x .

Λύση:

Έχουμε,

2 2

2

2 2

x y x y x y x y4 9 10 4 9 2

y x y x y x y x( ) ( ) ( ) ( )

x y x y 14 2 0

y x y x 4( )( ) , το οποίο ισχύει αφού

x y2,xy 0

y x , άρα και

τα δύο θετικά και x y

2,xy 0y x , άρα και τα δύο αρνητικά συνεπώς το

γινόμενο θετικό και η απόδειξη ολοκληρώθηκε. Η ισότητα επιτυγχάνεται για

x y .

Page 234: Εισαγωγη σε διαγωνιστικα μαθηματικα για το γυμνασιο

http://www.mathematica.gr/forum/viewtopic.php?f=109&t=30104

Επιμέλεια: xr.tsif Σελίδα 234

Β τρόπος

Ως επίσης θα είχαμε από την θεωρία που αναφέρεται στο πρόσημο τριωνύμου:

Θέτουμε x y

t 2y x

, οπότε αρκεί να αποδείξουμε 24t 9t 2 0 ,

(λόγω της ύπαρξης του ( – ) μπροστά από το 9), που ισχύει αφού το τριώνυμο

24t 9t 2 έχει ρίζες τους αριθμούς 1

,24

.

ΘΕΜΑ 269 (ΔΗΜΗΤΡΗΣ ΙΩΑΝΝΟΥ)

)

Αν *a,b,c R και αν: 1 1 1 1 1 1

a( ) b( ) c( ) 0c b a c b a , να αποδείξετε ότι δύο

τουλάχιστον από τους αριθμούς a,b,c θα είναι ίσοι. (Γ Γυμνασίου)

Λύση:

Εκτελώντας τις πράξεις παίρνουμε:

a b c a b c0 ab(a b) bc(b c) ca(c a) 0

c b a

.

Χρησιμοποιώντας την ταυτότητα

ab(a b) bc(b c) ca(c a) (a b)(b c)(c a) έχουμε:

ab(a b) bc(b c) ca(c a) 0 (a b)(b c)(c a) 0

(a b)(b c)(c a) 0 .

Άρα:

a b 0 a b .

b c 0 b c .

c a 0 c a .

Έτσι, αποδείξαμε το ζητούμενο.

Page 235: Εισαγωγη σε διαγωνιστικα μαθηματικα για το γυμνασιο

http://www.mathematica.gr/forum/viewtopic.php?f=109&t=30104

Επιμέλεια: xr.tsif Σελίδα 235

ΘΕΜΑ 270 (ΔΗΜΗΤΡΗΣ ΙΩΑΝΝΟΥ)

) Αν

2 2 2 2a b a b 10(a b ab) 61 0 , να αποδείξετε ότι: 3 3a b 35 .

(Γ Γυμνασίου)

Λύση:

Είναι,

2 2 2 2a b a b 10 a b ab 61 0

2 2 2a b 2ab a b 10 a b 10ab 61 0

2 2 2a b 10 a b 25 a b 12ab 36 0

2 2 a b 5

a b 5 ab 6 0ab 6

.

Τετραγωνίζοντας την a b 5 , προκύπτει

2 2 2 2 2 2a b 2ab 25 a b 25 2ab a b 25 12 13 .

Επομένως, 3 3 2 2a b a b a ab b 5 13 6 5·7 35 .

Β τρόπος

Αφού συμπεράνουμε ότι a b 5 και ab 6 , μπορούμε να γράψουμε

23 3 2 2a b a b a ab b 5 a b 3ab 5 25 18 5·7 35

.

ΘΕΜΑ 271 (ΔΗΜΗΤΡΗΣ ΙΩΑΝΝΟΥ)

) α) Να αποδείξετε ότι: (x y)(y z)(z x) 8xyz , για κάθε x,y,z 0 .

β) Χρησιμοποιώντας το α) , (χωρίς αυτό να είναι απαραίτητο) , να αποδείξετε

ότι: Αν a,b,c είναι τα μήκη των πλευρών τριγώνου ABC, τότε:

2 2 2a (b c a) b (c a b) c (a b c) 3abc . (Γ Γυμνασίου)

Page 236: Εισαγωγη σε διαγωνιστικα μαθηματικα για το γυμνασιο

http://www.mathematica.gr/forum/viewtopic.php?f=109&t=30104

Επιμέλεια: xr.tsif Σελίδα 236

Λύση:

α) Με πολλαπλασιασμό κατά μέλη των παρακάτω ανισοτήτων καταλήγω στο

συμπέρασμα .

x y 2 xy .

y z 2 yz .

z x 2 zx .

β) Θέτω:

A b c a , B c a b , C a b c .

Προσθέτοντας κατά μέλη παίρνουμε: B C

a B C 2a2

.

Ομοίως παίρνουμε: C A 2b και A B 2c .

Χρησιμοποιώντας το (α) ερώτημα έχουμε:

(A B)(B C)(C A) 8ABC 8abc 8(b c a)(c a b)(a b c)

(b c a)(c a b)(a b c) 2abc 3abc .

Αρκεί τώρα να αποδείξουμε ότι:

2 2 2(b c a)(c a b)(a b c) 2abc a (b c a) b (c a b) c (a b c)

Εκτελώντας τις πράξεις παίρνουμε το ζητούμενο.

ΠΑΡΑΤΗΡΗΣΗ

Στην λύση του raf616 πιο πάνω χρησιμοποιήθηκε η τριγωνική ανισότητα για να

δειχθεί ότι τα A,B,C είναι θετικά.

Η ζητούμενη ανισότητα ισχύει για οποιαδήποτε a,b,c 0 ακόμη και αν δεν

αποτελούν πλευρές τριγώνου. Ισοδύναμα η ανισότητα γίνεται

3 3 3 2 2 2a b c 3abc a (b c) b (c a) c (a b) ,

Page 237: Εισαγωγη σε διαγωνιστικα μαθηματικα για το γυμνασιο

http://www.mathematica.gr/forum/viewtopic.php?f=109&t=30104

Επιμέλεια: xr.tsif Σελίδα 237

η οποία είναι γνωστή ως η http://en.wikipedia.org/wiki/Schur's_inequality

ανισότητα του Schur.

ΘΕΜΑ 272 (ΔΗΜΗΤΡΗΣ ΙΩΑΝΝΟΥ)

) Αν a,b,c είναι πλευρές τριγώνου ABC, να αποδείξετε ότι:

2 2 2 22(a b c ) (a b c) . (Γ Γυμνασίου)

Λύση:

2a b c a a ab ac a ab ac .

Με τον ίδιο τρόπο:

2b ba bc και 2c ca cb .

Προσθέτω κατά μέλη και παίρνω: 2 2 2a b c 2ab 2ac 2bc

2 2 2 2 2 2 2 2 2 22a 2b 2c a b c 2ab 2ac 2bc 2(a b c ) (a b c)

ΘΕΜΑ 273 (ΔΗΜΗΤΡΗΣ ΙΩΑΝΝΟΥ)

)

Αν x,y 0 και xy 1 , να αποδείξετε ότι: 2 2

x y1

x 1 y 1

. (Γ Γυμνασίου)

Λύση:

Αρκεί να δείξουμε ότι: 1 1

11 1

x yx y

.

Από ΑΜ – ΓΜ είναι: 1 1 1

x 21x 2xx

, όμοια : 1 1 1

y 21y 2yy

,

και προσθέτοντας τις δυο τελευταίες κατά μέλη παίρνουμε το ζητούμενο.

Page 238: Εισαγωγη σε διαγωνιστικα μαθηματικα για το γυμνασιο

http://www.mathematica.gr/forum/viewtopic.php?f=109&t=30104

Επιμέλεια: xr.tsif Σελίδα 238

ΘΕΜΑ 274 (ΔΗΜΗΤΡΗΣ ΙΩΑΝΝΟΥ)

)

Να αποδείξετε ότι: 2 6 12 1006 1007

... 5033 5 7 2013

. (Γ Γυμνασίου)

ΥΠΟΔΕΙΞΗ

Μπορείτε να χρησιμοποιήσετε κατάλληλα την ανισότητα AM – GM.

Λύση:

Αρκεί να δείξουμε ότι:

1006 1006 1006

n 1 n 1 n 1

n(n 1) 2 n(n 1) 2 n(n 1)1006503 1006

2n 1 2(2n 1) 2 2n 1

.

Στους αριθμητές του αθροίσματος από την AM GM (αφού n n 1 )

παίρνουμε: 2 n(n 1) 2n 1 .

Άρα:1006 1006 1006

n 1 n 1 n 1

2 n(n 1) 2n 11 1006

2n 1 2n 1

, και το ζητούμενο απεδείχθη.

Β τρόπος

Γράφω την ίδια στην ουσία λύση με αυτήν του Διονύση , με πιο κατανοητό

τρόπο

(για τον φάκελο αυτό)

2 1 2 12 2 1

2 3 2

.

Ομοίως: 2 3 1

5 2

.

3 4 1

7 2

.

...............................

Page 239: Εισαγωγη σε διαγωνιστικα μαθηματικα για το γυμνασιο

http://www.mathematica.gr/forum/viewtopic.php?f=109&t=30104

Επιμέλεια: xr.tsif Σελίδα 239

...............................

..............................

1006 1007 1

2013 2

.

Με πρόσθεση έχουμε:

2 6 1006 1007 1 1 1 1... ... 1006 503

3 5 2013 2 2 2 2

.

ΘΕΜΑ 275 (ΔΗΜΗΤΡΗΣ ΙΩΑΝΝΟΥ)

) Να αποδείξετε ότι για κάθε x Z η παράσταση

2 2A 4(x 3x 4)(x 3x 3) 1 , είναι τέλειο τετράγωνο ακεραίου.

(Γ Γυμνασίου)

Λύση:

Θέτουμε: 2x 3x 4 y Z και έχουμε:

2 2 2A 4y(y 1) 1 4y 4y 1 (2y) 2·2y 1 (2y 1) , δηλαδή αυτό που

θέλαμε!

ΘΕΜΑ 276 (ΔΗΜΗΤΡΗΣ ΙΩΑΝΝΟΥ)

) Να βρεθεί η ελάχιστη τιμή της παράστασης: 2 2A 4x 4x 10 y 4y 5 ,

όπου x,y R . (Γ Γυμνασίου)

Λύση:

2 2A (2x 1) 9 (y 2) 1 .

Ισχύει ότι: 2min (2x 1) 9 9 για

1x

2 και

Page 240: Εισαγωγη σε διαγωνιστικα μαθηματικα για το γυμνασιο

http://www.mathematica.gr/forum/viewtopic.php?f=109&t=30104

Επιμέλεια: xr.tsif Σελίδα 240

2min (y 2) 1 1 για y 2 .

Άρα minA 9 1 3 1 4 .

ΘΕΜΑ 277 (ΔΗΜΗΤΡΗΣ ΙΩΑΝΝΟΥ)

) Αν a,b Z και 2 2 2a 2ab 8b c 6bc 7 , να αποδείξετε ότι: a b 3 ,

(Δηλαδή a b 3 ή a b 3 ). (Γ Γυμνασίου)

Λύση:

Είναι:

2 2 2 2 2 2 2a 2ab 8b c 6bc 7 a 2ab b (c 6bc 9b ) 7 2 2(a b) (c 3b) 7 (a b c 3b)(a b c 3b) 7

(a 4b c)(a 2b c) 7 .

Άρα έχουμε τα συστήματα:

a 4b c 7

a 2b c 1

,

a 4b c 1

a 2b c 7

,

a 4b c 7

a 2b c 1

, a 4b c 1

a 2b c 7

.

Αντικαθιστώντας το c σε κάθε σύστημα ή προσθέτοντας κατά μέλη θα

πάρουμε a b 3 και a b 3 , δηλ. a b 3 , που είναι και το ζητούμενο.

ΘΕΜΑ 278 (ΔΗΜΗΤΡΗΣ ΙΩΑΝΝΟΥ)

)

Αν x y z a και 23(xy xz yz) a , να αποδείξετε ότι:

ax y z

3 .

(Γ Γυμνασίου)

Λύση:

Έχουμε:

2 2 2 23(xy xz yz) (x y z) x y z xy xz yz

Page 241: Εισαγωγη σε διαγωνιστικα μαθηματικα για το γυμνασιο

http://www.mathematica.gr/forum/viewtopic.php?f=109&t=30104

Επιμέλεια: xr.tsif Σελίδα 241

2

2 2 2 2 2 2 2ax y z (x y z) 3(x y z )

3 .

Όμως από C S θα είναι: 2 2 2 23(x y z ) (x y z) με την ισότητα να ισχύει

για a

x y z3

.

Β τρόπος

Από την υπόθεση παίρνουμε:

2 2 2 23(xy xz yz) (x y z) x y z xy xz yz

2 2 2 2 2 2x y z xy yz zx 0 2x 2y 2z 2xy 2yz 2zx 0

2 2 2(x y) (y z) (z x) 0 .

Από την τελευταία παίρνουμε:

x y 0 x y ,

y z 0 y z ,

z x 0 z x .

Αφού όμως x y z a και x y z , παίρνουμε:

ax y z a 3x a x

3 . Άρα,

ax y z

3 .

ΘΕΜΑ 279 (ΔΗΜΗΤΡΗΣ ΙΩΑΝΝΟΥ)

) Αν 2 2x y 2 2 x 4 5 y 22 0 , με x,y R , να αποδείξετε ότι ο αριθμός

1 5 5 x yA ( )( )

x y 2 10

είναι ρητός. (Γ Γυμνασίου)

Λύση:

Είναι:

Page 242: Εισαγωγη σε διαγωνιστικα μαθηματικα για το γυμνασιο

http://www.mathematica.gr/forum/viewtopic.php?f=109&t=30104

Επιμέλεια: xr.tsif Σελίδα 242

2 2 2 2 2 2x 2 2·x ( 2) y 4 5·y (2 5) 0 (x 2) (y 2 5) 0 .

Άρα θα είναι: x 2 y 2 5 .

Αντικαθιστώντας στη παράσταση παίρνουμε:

2 2

2 5 5 2 3A 5 2 A A Q

2 2 2 2

.

ΘΕΜΑ 280 (ΔΗΜΗΤΡΗΣ ΙΩΑΝΝΟΥ)

) Αν x,y,z Q και αν xy yz zx 2013 , να αποδείξετε ότι ο αριθμός

2 2(x z)(x 2013)(y z)(y 2013) , είναι ρητός. (Γ Γυμνασίου)

Λύση:

Είναι: 2x 2013 (x y)(x z) και όμοια:

2y 2013 (x y)(y z) .

Άρα:

2 2 2A (x z)(x y)(x z)(y z)(x y)(y z) A (x y) (x z) (y z)

A (x y)(x z)(y z) Q .

ΘΕΜΑ 281 (ΘΑΝΟΣ ΜΑΓΚΟΣ)

) Να βρεθούν οι ακέραιες λύσεις της εξίσωσης

2 25x 6xy 2y 1 .

Λύση:

Είναι:

2 2 2 2 2 2 2 25x 6xy 2y 1 x 2xy y 4x 4xy y 1 (x y) (2x y) 1

Γνωρίζουμε ότι κάθε τετράγωνο ακεραίου είναι μεγαλύτερο ή ίσο από το 0.

Page 243: Εισαγωγη σε διαγωνιστικα μαθηματικα για το γυμνασιο

http://www.mathematica.gr/forum/viewtopic.php?f=109&t=30104

Επιμέλεια: xr.tsif Σελίδα 243

Άρα έχουμε τα συστήματα:

x y 0

2x y 1

,

x y 0

2x y 1

,

x y 1

2x y 0

,

x y 1

2x y 0

.

Λύνοντας τα παραπάνω συστήματα παίρνουμε τις λύσεις:

(x,y) {(1, 1),( 1,1),( 1,2),(1, 2)} .

Β τρόπος

2 2 2 25x 6xy 2y 1 5x 6xy 2y 1 0 .

Θεωρούμε την εξίσωση β βαθμού ως προς x η οποία έχει διακρίνουσα

2 2 236y 20(2y 1) 4(5 y ) .

1η περίπτωση: Αν 0

δηλαδή y 5 όχι ακέραιος, απορρίπτεται.

2η περίπτωση: Αν 0

δηλαδή y 5 ή y 5 , ΑΔΥΝΑΤΗ.

Άρα πρέπει 0 δηλαδή 5 y 5 και αφού ο yείναι ακέραιος θα πάρει

τις τιμές y 2, 1,0,1,2 .

Από αυτές η τιμή y 0 δίνει 1

x5

, που απορρίπτεται και οι άλλες

(x,y) (1, 1),(1, 2),( 1,1),( 1,2) .

ΘΕΜΑ 282 (ΔΗΜΗΤΡΗΣ ΙΩΑΝΝΟΥ)

) Να λυθεί η εξίσωση: ( x 1 3)( x 1 9) x 80 . (Γ Γυμνασίου)

Page 244: Εισαγωγη σε διαγωνιστικα μαθηματικα για το γυμνασιο

http://www.mathematica.gr/forum/viewtopic.php?f=109&t=30104

Επιμέλεια: xr.tsif Σελίδα 244

Λύση:

Θέτω x 1 a , με a 0 . Έτσι, η εξίσωση γίνεται:

a 02( a 3)(a 9) a 1 80 ( a 3)(a 9) (a 9)(a 9) a 3 a 9

2 2a a 24a 144 a 25a 144 0 . Λύνοντας την παραπάνω

δευτεροβάθμια παίρνουμε τις λύσεις: a 16 και a 9 . Βλέπουμε ότι ισχύει

μόνο a 16 , που μας δίνει x 255 , που είναι και μοναδική λύση.

ΘΕΜΑ 283 (ΔΗΜΗΤΡΗΣ ΙΩΑΝΝΟΥ)

) Αν 2 2 2 2 2 2x y a 2ax 2y 10 x y b 2by 2x 5 5 , να

αποδείξετε ότι: 2013 2014a b 0 . (Γ Γυμνασίου)

Λύση:

Είναι:

2 2 2 2 2 2x y a 2ax 2y 10 x y b 2by 2x 5

2 2 2 2(x a) (y 1) 9 (x 1) (y b) 4 3 2 5 .

Με την ισότητα να ισχύει για x a 1 y b 1 και άρα πράγματι:

2013 2014 2013 2014a b ( 1) 1 1 1 0 .

Β τρόπος

Είναι:

2 2 2 2 2 2 2 2x y a 2ax 2y 10 x 2ax a y 2y 1 9 (x a) (y 1) 9 2 2 2 2 2 2 2 2x y b 2by 2x 5 x 2x 1 y 2by b 4 (x 1) (y b) 4

Αφού 5 3 2 9 4 έχουμε:

x a 0 x a ,

Page 245: Εισαγωγη σε διαγωνιστικα μαθηματικα για το γυμνασιο

http://www.mathematica.gr/forum/viewtopic.php?f=109&t=30104

Επιμέλεια: xr.tsif Σελίδα 245

y 1 0 y 1 ,

x 1 0 x 1 ,

y b 0 y b .

Άρα a x 1 και b y 1 .

Άρα 2013 2014 2013 2014a b ( 1) 1 1 1 0 .

Έτσι, το ζητούμενο αποδείχθηκε.

ΘΕΜΑ 284 (ΔΗΜΗΤΡΗΣ ΙΩΑΝΝΟΥ)

)

Να αποδειχθεί ότι: 40 20

10(2 3) (7 4 3)209

2

. (Γ Γυμνασίου)

Λύση:

Είναι 2

2 3 7 4 3 και έτσι το αριστερό μέλος είναι ίσο με

1020 2 10 10 107 4 3 7 4 3 97 56 3 97 56·2 209

.

ΘΕΜΑ 285 (ΔΗΜΗΤΡΗΣ ΙΩΑΝΝΟΥ)

) Αν 2 x 3 4 y 5 x 4y 15 , να βρεθούν οι αριθμοί x και y.

(Γ Γυμνασίου)

Λύση:

Είναι: 2 x 3 4 y 5 (x 3) 4(y 5) 2 .

Θέτουμε a x 3,b 2 y 5 και έχουμε:

2 2 2 22a 2b a b 2 (a 1) (b 1) 0 .

Page 246: Εισαγωγη σε διαγωνιστικα μαθηματικα για το γυμνασιο

http://www.mathematica.gr/forum/viewtopic.php?f=109&t=30104

Επιμέλεια: xr.tsif Σελίδα 246

Άρα θα πρέπει: 21

a 1 b 1 x 2 y4

.

ΘΕΜΑ 286 (ΔΗΜΗΤΡΗΣ ΙΩΑΝΝΟΥ)

) Αν x,y Z να λυθεί η εξίσωση:

2 2x 4y x 2 . (Γ Γυμνασίου)

Λύση:

'Έχουμε 2 2 2 2x 4y x 2 4x 4x 1 16y 7

2 2

2x 1 4y 7 2x 1 4y 2x 1 4y 7

ή

ή

2x 1 4y 1 2x 1 4y 7

2x 1 4y 1 2x 1 4y 7

2x 1 4y 7 2x 1

4y 1

2x 1 4y 7 2x 1

4y

ή

1

ή

ή

(x,y) ( 1, 1)

(x,y) (2,1)

(x,y) (2, 1)

(x,y)

( ,

ή

1 )

1

.

ΘΕΜΑ 287 (ΔΗΜΗΤΡΗΣ ΙΩΑΝΝΟΥ)

)

Να λυθεί η εξίσωση: x x

x

x x

3 52 3

5 6 . (Γ Γυμνασίου)

Λύση:

Απάντηση: x 0 .

Από ανισότητα ΑΜ – ΓΜ έχουμε x x x x

x x3

x x x x

3 5 3 53 2 3 2 · · 3

5 6 5 6 .

Άρα ισχύει x x

x

x x

3 52

5 6 . Η τελευταία ισότητα δίνει x x25 18 , οπότε x 0 ,

που ικανοποιεί την αρχική.

Page 247: Εισαγωγη σε διαγωνιστικα μαθηματικα για το γυμνασιο

http://www.mathematica.gr/forum/viewtopic.php?f=109&t=30104

Επιμέλεια: xr.tsif Σελίδα 247

Σχόλιο:

Υποθέτω ότι η ανισότητα ΑΜ – ΓΜ είναι "εντός φακέλου". Αλλιώς την

παρακάμπτουμε αν θέσουμε x x

3 x 3 3

x x

3 5a 2 ,b ,c

5 6 , οπότε η δοθείσα γράφεται

3 3 3a b c 3abc 0 , που με χρήση της ταυτότητας Euler δίνει a b c , και

λοιπά.

ΘΕΜΑ 288 (ΔΗΜΗΤΡΗΣ ΙΩΑΝΝΟΥ)

) Αν a,b,c,d είναι πραγματικοί αριθμοί και όλοι είναι μεγαλύτεροι από τον

επίσης πραγματικό αριθμό k , να αποδείξετε ότι: 2 2 2 2a b c d

16kb k c k d k a k

. (Γ Γυμνασίου)

Λύση:

Δεδομένου ότι σε αυτό το νήμα έχουμε αναφερθεί εκτενώς στην ανισότητα

Cauchy – Schwarz, μπορούμε να πούμε το εξής:

2 2 2 2 2a b c d (a b c d)

b k c k d k a k a b c d 4k

,

οπότε αρκεί 2(a b c d)

16ka b c d 4k

, η οποία είναι ισοδύναμη με την

2(a b c d 8k) 0 .

ΘΕΜΑ 289 (ΔΗΜΗΤΡΗΣ ΙΩΑΝΝΟΥ)

)

Αν2 2x y 1 , να αποδείξετε ότι:

4 2 4 2x 4y y 4x1

9 9

. (Γ Γυμνασίου)

(ΠΗΓΗ: Μαθηματικές Ολυμπιάδες - Γυμνάσιο, των Μ. Στεργίου και Ν

Σκομπρή) .

Page 248: Εισαγωγη σε διαγωνιστικα μαθηματικα για το γυμνασιο

http://www.mathematica.gr/forum/viewtopic.php?f=109&t=30104

Επιμέλεια: xr.tsif Σελίδα 248

Λύση:

Αρκεί να δείξουμε ότι 2

4 2 4 2x 4y y 4x 9 .

Πράγματι,

2

4 2 4 2 4 2 4 2 4 2 4 2x 4y y 4x x 4y y 4x 2 x 4y y 4x

22 2 2 2 2 2 4 4 6 6 2 2x y 2x y 4 x y 2 x y 4 x y 16x y

2 2 4 4 2 2 4 2 2 4 2 21 2x y 4 2 x y 4 x y x x y y 16x y

2 2 4 4 4 2 2 4 2 21 2x y 4 2 x y 4x 4x y 4y 16x y

22 2 2 2 2 2 4 4 2 21 2x y 4 2 4 x y 8x y x y 12x y

22 2 4 4 2 2 2 2 2 21 2x y 4 2 x y 4x y 4 1 2x y 4 2 x y 2

2 2 2 2 2 2 2 21 2x y 4 2 x y 2 1 2x y 4 2x y 4 9 .

Β τρόπος

Ποιο απλά

4 2 4 2 4 2 2 2 2 2x 4y x 4(1 x ) x 4x 4 (x 2) x 2 2 x ,

(διότι 2 2 2x x y 1 2 ) .

Όμοια το άλλο ριζικό. Άρα η δοθείσα ισούται 2 21(2 x 2 y ) 1

3 .

Γ τρόπος

Και άλλος ένας εναλλακτικός τρόπος:

4 2 4 2 2 2 2 2 2 2 2 2x 4y x 4y (x y ) (x 2y ) x 2y y 1

9 9 9 3 3

.

Ομοίως την άλλη ρίζα και με πρόσθεση έχουμε το ζητούμενο.

Page 249: Εισαγωγη σε διαγωνιστικα μαθηματικα για το γυμνασιο

http://www.mathematica.gr/forum/viewtopic.php?f=109&t=30104

Επιμέλεια: xr.tsif Σελίδα 249

ΘΕΜΑ 290 (ΔΗΜΗΤΡΗΣ ΙΩΑΝΝΟΥ)

) Αν

2 2 2 2a b x y 2 , να αποδείξετε ότι: ax by 2 . (Γ Γυμνασίου)

Λύση:

Είναι 2 2 2 2a x b y 4

ax by 22 2 2

.

ΘΕΜΑ 291 (ΣΕΜΑ ΔΙΟΝΥΣΗΣ)

) Να βρεθούν όλες οι τριάδες θετικών ακεραίων (x,y,z) τέτοιες ώστε:

3 3 3x y z 3xyz p , όπου pπρώτος αριθμός μεγαλύτερος του 3.

(Γ Γυμνασίου)

Λύση:

Από την ταυτότητα του Euler έχουμε: 3 3 3 2 2 2p x y z 3xyz (x y z)(x y z xy yz zx) .

Επειδή ο pείναι πρώτος και x y z 3 θα είναι αναγκαστικά

2 2 2 2 2 2x y z xy yz zx 1 (x y) (y z) (z x) 2

Από τη συμμετρία, έστω x y z .

Αν | x y |,| y z |,| z x | 1 έχουμε 2 2 2(x y) (y z) (z x) 3 , άτοπο.

Οπότε δύο από τους αριθμούς x,y,z είναι ίσοι.

Αν x z

τότε x y z , οπότε 2 2 2(x y) (y z) (z x) 0 , άτοπο.

Αν x y

τότε x y z 1 και p x y z 3z 2 , το οποίο είναι δυνατό αν

Page 250: Εισαγωγη σε διαγωνιστικα μαθηματικα για το γυμνασιο

http://www.mathematica.gr/forum/viewtopic.php?f=109&t=30104

Επιμέλεια: xr.tsif Σελίδα 250

p 1 mod3

Αν y z

τότε x 1 y z και p x y z 3z 1 το οποίο είναι δυνατό αν

p 1 mod3 .

Συνοψίζοντας, κάθε πρώτος p 1 mod3 γράφεται ως 3 3 3x y z 3xyz όπου

x,y,z θετικοί ακέραιοι με

p 1 p 1 p 2(x,y,z) ~ , ,

3 3 3

ή p 2 p 2 p 1

(x,y,z) ~ , ,3 3 3

.

ΘΕΜΑ 292 (ΔΗΜΗΤΡΗΣ ΙΩΑΝΝΟΥ)

) Δίνονται οι αριθμοί: 2a n 2,b n 2n 2 , *n N .

α) Να αποδείξετε ότι οι αριθμοί b a και b a , δεν είναι τέλεια τετράγωνα.

β) Να αποδείξετε ότι οι αριθμοί a και b , δεν μπορούν να είναι ταυτοχρόνως

τέλειοι κύβοι.

γ) Να βρεθούν οι τιμές του n , ώστε ο αριθμός b 4a 15 να είναι τέλειο

τετράγωνο. (Γ Γυμνασίου)

Λύση:

α) 2a b n 3n 4 .

Όμως 2 2 2(n 1) n 3n 4 (n 2) άρα ο αριθμός δεν μπορεί να είναι τέλειο

τετράγωνο.

2b a n n .

Όμως 2 2 2n n n (n 1) άρα ο αριθμός δεν μπορεί να είναι τέλειο

τετράγωνο.

β) Αν οι αριθμοί είναι τέλειοι κύβοι το γινόμενό τους θα είναι τέλειος κύβος.

Page 251: Εισαγωγη σε διαγωνιστικα μαθηματικα για το γυμνασιο

http://www.mathematica.gr/forum/viewtopic.php?f=109&t=30104

Επιμέλεια: xr.tsif Σελίδα 251

2 3 2(n 2)(n 2n 2) n 4n 6n 4 .

Όμως 3 3 2 3(n 1) n 4n 6n 4 (n 2) κι έτσι οι δύο αριθμοί δεν μπορούν

να είναι συγχρόνως τέλειοι κύβοι.

γ) 2 2 2 2 2 2b 4a 15 n 6n 15 k n 2 3n 3 3 15 k

2 2(n 3) k 24 .

Άρα (n 3 k)(n 3 k) 24 .

Από τα συστήματα που παίρνουμε, βρίσκουμε ότι τα μοναδικά που δεν είναι

αδύνατα είναι:

n 3 k 4

n 3 k 6

(από το οποίο βρίσκουμε ότι n 2 ) και

n 3 k 2

n 3 k 12

(από το οποίο βρίσκουμε n 4 ) και

n 3 k 6

n 3 k 4

(από το οποίο και πάλι βρίσκουμε n 2 ).

ΘΕΜΑ 293 (ΔΗΜΗΤΡΗΣ ΙΩΑΝΝΟΥ)

) Να αποδείξετε ότι ο αριθμός: n n 2n 2 na 16 8 2 3 2 3 , με *n N , δεν είναι

πρώτος. (Γ Γυμνασίου)

Λύση:

Θέτουμε nx 2 0 . Τότε

4 3 2 4 3 2 2 2 2a x x 4x 3x 3 (x x x ) (3x 3x 3) x 3 x x 1

με 2x 3 1 και 2x x 1 1 , άρα ο a δεν είναι ποτέ πρώτος.

ΘΕΜΑ 294 (ΜΠΑΜΠΗΣ ΣΤΕΡΓΙΟΥ)

) Να αποδείξετε ότι :

2 2 2 2 2a b c d e a(b c d e) για κάθε

Page 252: Εισαγωγη σε διαγωνιστικα μαθηματικα για το γυμνασιο

http://www.mathematica.gr/forum/viewtopic.php?f=109&t=30104

Επιμέλεια: xr.tsif Σελίδα 252

a,b,c,d,e R .

( Με εναλλακτικές λύσεις) (Γ Γυμνασίου)

Λύση:

Είναι

2 2

2 2 2a a a( b) 0 ab b 0 b ab2 4 4 .

2 2

2 2 2a a a( c) 0 ac c 0 c ac2 4 4 .

2 2

2 2 2a a a( d) 0 ad d 0 d ad2 4 4 .

2 2

2 2 2a a a( e) 0 ae e 0 e ae2 4 4 .

Με πρόσθεση των συμπερασμάτων κατά μέλη έχω το ζητούμενο.

Β τρόπος

Από την ανισότητα Cauchy – Schwarz

22 2 2 2 2 2 2 2(1 1 1 1 )(b c d e ) b c d e οπότε

2

2 2 2 2 2 2b c d e

a b c d e a a b c d e4

,

εφ όσον από την ανισότητα ΑΜ – ΓΜ

2

2b c d e b c d e

a 2a4 2

.

ΘΕΜΑ 295 (ΔΗΜΗΤΡΗΣ ΙΩΑΝΝΟΥ)

) Να βρεθούν οι ακέραιοι x,y οι οποίοι να ικανοποιούν την εξίσωση:

2 22y 3x 2xy(x 1) . (Γ Γυμνασίου)

Page 253: Εισαγωγη σε διαγωνιστικα μαθηματικα για το γυμνασιο

http://www.mathematica.gr/forum/viewtopic.php?f=109&t=30104

Επιμέλεια: xr.tsif Σελίδα 253

Λύση:

Είναι: 2 2 2 2 2 22y 3x 2x y 2xy 2y (2x 2x)y 3x 0 .

Το παραπάνω τριώνυμο ως προς y έχει διακρίνουσα:

2 2 2 2 2(2x 2x) 4·2·3x 4x (x 2x 5) .

Αν 0 , τότε πρέπει:

2 2 2 2x 2x 5 m (x 1) m 6 (x 1 m)(x 1 m) 6 .

Προκύπτουν κάποια απλά συστήματα τα οποία όμως δε δίνουν ακέραιες λύσεις.

Αν 0 , τότε:

24x 0 x 0 , που μας δίνει τη λύση (x,y) (0,0) ,

ή

2x 2x 5 0 , που δε δίνει ακέραιες λύσεις.

Άρα μοναδική λύση είναι η (x,y) (0,0) .

ΘΕΜΑ 296 (ΔΗΜΗΤΡΗΣ ΙΩΑΝΝΟΥ)

) Να αποδείξετε ότι οι αριθμοί: a 3n 1 και b 10n 3 , είναι πρώτοι μεταξύ

τους, για κάθε φυσικό αριθμό n . (Β Γυμνασίου)

Λύση:

Ας είναι (a,b) d (3n 1,10n 3) d .

Η παραπάνω σχέση μας λέει ότι: d|3n 1 και d|10n 3 .

Άρα ο d θα διαιρεί και:

d |10(3n 1) 3(10n 3) d |30n 10 30n 9 d |1 d 1 .

Άρα (3n 1,10n 3) 1 (a,b) 1 , δηλαδή οι a,b είναι πρώτοι μεταξύ τους.

Page 254: Εισαγωγη σε διαγωνιστικα μαθηματικα για το γυμνασιο

http://www.mathematica.gr/forum/viewtopic.php?f=109&t=30104

Επιμέλεια: xr.tsif Σελίδα 254

Β τρόπος

Αλλιώς:

Είναι 10a 3b 1 άρα (a,b) 1 .

ΘΕΜΑ 297 (ΔΗΜΗΤΡΗΣ ΙΩΑΝΝΟΥ)

) Αν m,n N , να αποδείξετε ότι ο αριθμός m na 3 3 1 , δεν είναι τέλειο

τετράγωνο ακεραίου. (Γ Γυμνασίου)

Λύση:

m na ( 1) ( 1) 1(mod4) m 0(mod2),n 1(mod2)

m n 1

2 2a 1 3·1 1(mod8) 5(mod8)

.

Το ζητούμενο έπεται.

ΘΕΜΑ 298 (ΔΗΜΗΤΡΗΣ ΙΩΑΝΝΟΥ)

) Να αποδείξετε ότι:

2 2 4 2 8 4 16 8(x x 1)(x x 1)(x x 1)(x x 1) x x 1 .

(Γ Γυμνασίου)

Λύση:

Έχουμε:

2 2 4 2 8 4(x x 1)(x x 1)(x x 1)(x x 1 )

2 2 2 4 2 8 4(x 1) x (x 1 x )(x x 1)

4 2 4 2 8 4 4 2 4 8 4(x 1 x )(x 1 x )(x x 1) (x 1) x (x x 1)

8 4 8 4 8 2 8 16 8(x 1 x )(x x 1) (x 1) x x x 1 .

Page 255: Εισαγωγη σε διαγωνιστικα μαθηματικα για το γυμνασιο

http://www.mathematica.gr/forum/viewtopic.php?f=109&t=30104

Επιμέλεια: xr.tsif Σελίδα 255

ΘΕΜΑ 299 (ΔΗΜΗΤΡΗΣ ΙΩΑΝΝΟΥ)

) Στο παρακάτω σχήμα, δίνεται ότι οι BE και CZ είναι διχοτόμοι των γωνιών

ABD και ACD αντιστοίχως. Επίσης, δίνεται ότι oBDC 145 και ότι

oBOC 95 . Να υπολογίσετε την A .

(Β Γυμνασίου)

Λύση:

Ονομάζω , τα μισά των ABD και ACD .

Προεκτείνω τη BD .... εξωτερική γωνία :

Στο OBS έχουμε o50 .

Στο τρίγωνο ABT : o o oA 180 2 2 35 45 .

ΘΕΜΑ 300 (ΔΗΜΗΤΡΗΣ ΙΩΑΝΝΟΥ)

)

Να λυθεί η εξίσωση: x 8 x 4 x 9 x 3

x 9 x 5 x 10 x 4

. (Γ Γυμνασίου)

Page 256: Εισαγωγη σε διαγωνιστικα μαθηματικα για το γυμνασιο

http://www.mathematica.gr/forum/viewtopic.php?f=109&t=30104

Επιμέλεια: xr.tsif Σελίδα 256

Λύση:

Για να ορίζεται η εξίσωση θα πρέπει να είναι: x 9, 5, 10, 4 .

x 8 x 4 x 9 x 3 1 1 1 11 1 1 1

x 9 x 5 x 10 x 4 x 9 x 5 x 10 x 4

1 1 1 1 x 5 x 9 x 10 x 4

x 9 x 5 x 10 x 4 (x 9)(x 5) (x 10)(x 4)

2 2(2x 14)(x 14x 45) (2x 14)(x 14x 40)

2 2(2x 14)(x 14x 45) (2x 14)(x 14x 40) 0

(2x 14)·5 0 10x 70 x 7 .

Η λύση αυτή δεν έρχεται σε αντίθεση με τους περιορισμούς και άρα αποτελεί

τη μόνη λύση της εξίσωσης.

Β τρόπος

Έχουμε : x 9 x 8 x 4 x 3

x 10 x 9 x 5 x 4

ή

2 2(x 9) (x 8)(x 10) (x 4) (x 3)(x 5)

(x 9)(x 10) (x 4)(x 5)

ή

2 2 2 2

2 2

(x 18x 81) (x 18x 80) (x 8x 16) (x 8x 15)

x 19x 90 x 9x 20

ή

2 2

1 1

x 19x 90 x 9x 20

ή x 7 .

ΘΕΜΑ 301 (ΔΗΜΗΤΡΗΣ ΙΩΑΝΝΟΥ)

)

Αν a,b,a b 0 και 4 4

2 2 x y 1x y 1,

a b a b

, να αποδείξετε ότι:

Page 257: Εισαγωγη σε διαγωνιστικα μαθηματικα για το γυμνασιο

http://www.mathematica.gr/forum/viewtopic.php?f=109&t=30104

Επιμέλεια: xr.tsif Σελίδα 257

8 8

3 3 3

x y 1

a b (a b)

. (Γ Γυμνασίου)

Λύση:

Είναι,

4 4 4 4 2 2

4 4 2 2x y 1 x y x yb a b x a a b y ab x y

a b a b a b a b

4 2 4 2 4 4 2 2abx b x a y aby abx aby 0

2 2 2 2 2 4 2 4abx x 1 aby y 1 b x a y 0

2

22 4 2 4 2 2 2 2 2 ayb x a y 2abx y 0 bx ay 0 x

b .

Έτσι,

2 2 2 2a bx y 1 y 1 1 y

b a b

και άρα 2 a

xa b

.

Σύμφωνα με τα παραπάνω, έχουμε

8 8 4 4

4 4 4 4 4 33 3 3 3

x y a b a b a b 1

a b a a b b a b a b a b a b a b

,

όπως θέλαμε.

ΘΕΜΑ 302 (ΔΗΜΗΤΡΗΣ ΙΩΑΝΝΟΥ)

) Αν a b c 0 , να αποδείξετε ότι:

2 2 2 2 2 2 2 2 2

1 1 10

a b c b c a c a b

. (Γ Γυμνασίου)

Λύση:

Είναι:

Page 258: Εισαγωγη σε διαγωνιστικα μαθηματικα για το γυμνασιο

http://www.mathematica.gr/forum/viewtopic.php?f=109&t=30104

Επιμέλεια: xr.tsif Σελίδα 258

2 2 2 2 2a b c (a b) c 2ab (a b c)(a b c) 2ab 2ab .

Ομοίως παίρνουμε: 2 2 2b c a 2bc και 2 2 2c a b 2ca .

Άρα: 2 2 2 2 2 2 2 2 2

1 1 1 1 1 1

a b c b c a c a b 2ab 2bc 2ca

c a b a b c0

2abc 2abc 2abc 2abc

.

Έτσι, το ζητούμενο αποδείχτηκε.

ΘΕΜΑ 303 (ΔΗΜΗΤΡΗΣ ΙΩΑΝΝΟΥ)

) Να αποδείξετε ότι για κάθε φυσικό αριθμό n , ο αριθμός n n 2a 3 4 2 1 ,

είναι σύνθετος.

Στη συνέχεια, να αποδείξετε ότι αν ο nείναι περιττός, τότε ο a είναι

πολλαπλάσιο του 3. (Γ Γυμνασίου)

ΣΗΜΕΙΩΣΗ (για το δεύτερο ερώτημα).

Πρέπει να γνωρίζουμε (δεν υπάρχει στο σχολικό βιβλίο της Γ Γυμνασίου) ότι:

n n n 1 n 2 n 3 2 n 1a b (a b)(a a b a b ... b ) , για κάθε *n N .

n n n 1 n 2 n 3 2 n 1a b (a b)(a a b a b ... b ) , για κάθε nάρτιο θετικό

ακέραιο.

n n n 1 n 2 n 3 2 n 1a b (a b)(a a b a b ... b ) , για κάθε nπεριττό θετικό

ακέραιο.

Λύση:

Έχουμε: n na 3·4 4·2 1 , και το θεωρούμε τριώνυμο ως προς το n2 .

Τότε: a

D 16 12 4 .

Page 259: Εισαγωγη σε διαγωνιστικα μαθηματικα για το γυμνασιο

http://www.mathematica.gr/forum/viewtopic.php?f=109&t=30104

Επιμέλεια: xr.tsif Σελίδα 259

Άρα: 1

1,2

2

a 14 2

a 16 a

3

και τελικά:

n n n n1a 3 2 2 1 a 3·2 1 2 1

3

για κάθε n N και άρα ο a είναι

σύνθετος.

Και φυσικά αν n 2k 1 (περιττός) θα ισχύει: n n n 1 n 2 n 32 1 3(2 2 2 ... 1) επομένως:

n 1 n 2 n 3 na 3(2 2 2 ... 1)(3·2 1) , δηλαδή: a o 3 .

ΘΕΜΑ 304 (ΔΗΜΗΤΡΗΣ ΙΩΑΝΝΟΥ)

) Να υπολογίσετε το άθροισμα:

2 2 2 2 21 2 3 ... 99 100 2(1 2 3 4 5 6 ... 99 100) . (Γ Γυμνασίου)

Λύση:

Έχουμε:

2 2 2 2 2

5050

2(1 2) (3 4) ... (99 100) 1 1 ... 1 50 .

ΘΕΜΑ 305 (ΔΗΜΗΤΡΗΣ ΙΩΑΝΝΟΥ)

) Να παραγοντοποιηθεί η παράσταση: (x 1)(x 2)(x 3)(x 4) 3 .

(Γ Γυμνασίου)

Λύση:

Βασιζόμαστε στην βασική παρατήρηση ότι

2 2(x 1)(x 2)(x 3)(x 4) 1 (x 5x 5) .

Τότε

Page 260: Εισαγωγη σε διαγωνιστικα μαθηματικα για το γυμνασιο

http://www.mathematica.gr/forum/viewtopic.php?f=109&t=30104

Επιμέλεια: xr.tsif Σελίδα 260

(x 1)(x 2)(x 3)(x 4) 3 (x 1)(x 2)(x 3)(x 4) 1 4

2 2 2 2 2 2 2(x 5x 5) 4 (x 5x 5) 2 (x 5x 7)(x 5x 3) .

Β τρόπος

Θέτω y x 1 , τότε η προς παραγοντοποίηση παράσταση γίνεται:

y(y 1)(y 2)(y 3) 3 .

Αν επιμείνω λίγο ακόμα γίνεται: 2 2(y 3y)(y 3y 2) 3 (κάνοντας γινόμενα

με πρώτη – τέταρτη παρένθεση και τρίτη – δεύτερη).

Τώρα είναι φανερό πως πρέπει να θέσω: 2w y 3y και τότε έχω:

2 2w(w 2) 3 w 2w 3 w 3w w 3 (w 1)(w 3) .

Πηγαίνοντας πίσω έχω:

2 22 2y 3y 1 y 3y 3 x 1 3 x 1 1 x 1 3(x 1) 3

2 2x 5x 3 x 5x 7 .

Γ τρόπος

Επίσης, αν ονομάζαμε Aτην προς παραγοντοποίηση παράσταση, θα

μπορούσαμε να θέσουμε 2t x 5x (αφού 4 1 3 2 5 ),

οπότε παραγοντοποιούμε ως εξής:

2 2A t 4 t 6 3 t 7 t 3 x 5x 7 x 5x 3 .

ΘΕΜΑ 306 (ΔΗΜΗΤΡΗΣ ΙΩΑΝΝΟΥ)

) Αν x y z 0 , να αποδείξετε ότι η παράσταση:

2 2x y 2yz είναι τέλειο

τετράγωνο. (Γ Γυμνασίου)

Λύση:

Είναι:

Page 261: Εισαγωγη σε διαγωνιστικα μαθηματικα για το γυμνασιο

http://www.mathematica.gr/forum/viewtopic.php?f=109&t=30104

Επιμέλεια: xr.tsif Σελίδα 261

2 2 2 2 2 2 2 2 2x y 2yz x (y 2yz z ) z x (y z) z

2 2(x y z)(x y z) z z .

Έτσι, το ζητούμενο αποδείχθηκε.

ΘΕΜΑ 307 (Παύλος Μαραγκουδάκης)

) Ο είναι ένας πραγματικός αριθμός για τον οποίο γνωρίζουμε ότι οι αριθμοί

34 ,6 ,21 είναι ακέραιοι. Να αποδειχθεί ότι:

α) ο 3 είναι ακέραιος.

β) ο 3 είναι ακέραιος.

γ) ο είναι ακέραιος. (Γ Γυμνασίου)

Λύση:

α) 6 Z,3 Z 6 3 18 Z .

21 Z,18 Z 21 18 3 Z .

β) 3 33 Z (3 ) Z 27 Z .

3 3 34 Z 7 4 Z 28 Z .

3 3 3 3 328 Z,27 Z 28 27 Z .

γ) Εφόσον 3 Z Z .

ΘΕΜΑ 308 (Παύλος Μαραγκουδάκης)

) Οι a,b,c είναι τρεις πραγματικοί μη μηδενικοί αριθμοί για τους οποίους

γνωρίζουμε ότι οι αριθμοί 25 13 10 6 4 3 37 18 14a b c ,a b c ,a b c είναι ρητοί. Να αποδειχθεί

ότι οι a,b,c είναι ρητοί. (Γ ΓΥΜΝΑΣΙΟΥ)

Page 262: Εισαγωγη σε διαγωνιστικα μαθηματικα για το γυμνασιο

http://www.mathematica.gr/forum/viewtopic.php?f=109&t=30104

Επιμέλεια: xr.tsif Σελίδα 262

Λύση:

Θέτουμε 25 13 10 6 4 3 37 18 14x a b c , y a b c , z a b c .

Τότε 2

2

xa

zy ,

27

20 15

xb

y z ,

31 22

40

y zc

x .

Άρα οι a,b,c είναι ρητοί.

Β τρόπος

(Χωρίς τις ανεξήγητες δυνάμεις των x,y,z από τις οποίες προκύπτει ότι οι

a,b,cείναι ρητοί και τις οποίες θα γράψω παρακάτω πως μπορούν να βρεθούν).

Μόλις βρούμε (όπως στον α τρόπο) ότι ο α είναι ρητός (σχετικά εύκολο να το

δεις) έπεται ότι οι αριθμοί 13 10

25

xab c

a και 4 3

6

yb c

a είναι ρητοί.

Άρα ο

13 10

34 3

b cbc

b c είναι επίσης ρητός δηλαδή ο

4 3

3

b cb

bc και τελικά και ο

bcc

b είναι ρητός.

Εξήγηση της εύρεσης των "περίεργων" δυνάμεων στον α τρόπο.

Θέλουμε να εξετάσουμε αν υπάρχουν ακέραιοι αριθμοί p,q,r ώστε

p q ra x y z , δηλαδή 25p 6q 37r 13p 4q 18r 10p 3q 14ra a b c .

Άρα είναι λογικό να προσπαθήσουμε να λύσουμε το σύστημα των παρακάτω

εξισώσεων:

25p 6q 37r 1

13p 4q 18r 0

10p 3q 14r 0

, που έχει λύση (p,q,r) (2, 2, 1) .

Όμοια θέλουμε p,q,r ώστε p q rb x y z και καταλήγουμε στο σύστημα

Page 263: Εισαγωγη σε διαγωνιστικα μαθηματικα για το γυμνασιο

http://www.mathematica.gr/forum/viewtopic.php?f=109&t=30104

Επιμέλεια: xr.tsif Σελίδα 263

25p 6q 37r 0

13p 4q 18r 1

10p 3q 14r 0

, που έχει λύση (p,q,r) (27, 20, 15) .

Τέλος θέλουμε p,q,r ώστε p q rc x y z και καταλήγουμε στο σύστημα

25p 6q 37r 0

13p 4q 18r 0

10p 3q 14r 1

που έχει λύση (p,q,r) ( 40,31,22) .

Γ τρόπος

Λύνοντας το σύστημα που προκύπτει από την εξίσωση των εκθετών των a,b,c

στα δύο μέλη της x y z25 13 10 6 4 3 37 18 14a a b c a b c a b c και κάνοντας τον σταυρό

μας με την ελπίδα να βρούμε ακέραιες λύσεις (πράγμα βέβαιο γιατί ο Παύλος

είναι καλός άνθρωπος) διαπιστώνουμε

225 13 10

26 4 3 37 18 14

a b ca

a b c a b c . που είναι ρητός ως γινόμενο και πηλίκο ρητών.

Όμοια

2725 13 10

20 156 4 3 37 18 14

a b cb Q

a b c a b c και

31 226 4 3 37 18 14

4025 13 10

a b c a b cc Q

a b c .

Δ τρόπος

Ρητός επί ρητό ή ρητός διά ρητό δίνει ρητό. Ας ''παίξουμε'' με αυτήν την

παρατήρηση και τις ιδιότητες των δυνάμεων.

6 4 3 12 8 6a b c Q a b c Q .

12 8 6a b c Q και 37 18 14 49 26 20a b c Q a b c Q .

25 13 10 50 26 20a b c Q a b c Q .

Διαιρώντας, προκύπτει ότι ο a είναι ρητός.

Άρα οι 13 10 4 3b c ,b c είναι ρητοί.

Page 264: Εισαγωγη σε διαγωνιστικα μαθηματικα για το γυμνασιο

http://www.mathematica.gr/forum/viewtopic.php?f=109&t=30104

Επιμέλεια: xr.tsif Σελίδα 264

Υψώνουμε τον δεύτερο στον κύβο και διαιρούμε οπότε ο 13 10

12 9

b cbc

b c είναι

ρητός.

Τότε ο 4 3

3

b cb

(bc) είναι ρητός και προφανώς και ο

bcc

b είναι ρητός.

ΘΕΜΑ 309 (Παύλος Μαραγκουδάκης)

) Να αποδειχθεί ότι o o o o1 · 2 ·...· 29 · 30 0,000.000.001 . (Γ Γυμνασίου)

Λύση:

Αφού o 130

2 άρα όλες οι γωνίες που εμφανίζονται παραπάνω έχουν

ημίτονο μικρότερο από 1

2.

Άρα

o o o o

330 3 310

1 1 1 11 · 2 ·...· 29 · 30 0,000.000.001

2 1024 10002 .

ΘΕΜΑ 310 (ΔΗΜΗΤΡΗΣ ΙΩΑΝΝΟΥ)

) Δίνεται ο αριθμός: 40 50 20A 3 4 7 9 4 .

α) Να αποδείξετε ότι ο αριθμός αυτός είναι θετικός.

β) Να βρείτε το ψηφίο των μονάδων του. (Α, Β , Γ Γυμνασίου)

Λύση:

α) 40 50 20 25 20 24 20 20 203 4 4 9 16 4 9 16 16 4 7 8 7 9 7 .

Δηλαδή αποδείξαμε ότι A 0 .

β) 40 203 9 ,

Page 265: Εισαγωγη σε διαγωνιστικα μαθηματικα για το γυμνασιο

http://www.mathematica.gr/forum/viewtopic.php?f=109&t=30104

Επιμέλεια: xr.tsif Σελίδα 265

20 409 1(mod10) 9 1(mod10) 3 1(mod10) .

Ακόμα 50 254 16 ,

25 5016 6(mod10) 16 6(mod10) 4 6(mod10) .

Ακόμα 4 6(mod10) ,

Άρα 40 503 4 4 1(mod10) .

Επίσης 20 107 49 ,

10 20 2049 1(mod10) 49 1(mod10) 7 1(mod10) 7 9 9(mod10) .

Άρα 40 50 203 4 4 7 9 8(mod10) 2(mod10) .

Άρα ο Aέχει τελευταίο ψηφίο το 2 .

Μπορούμε να λύσουμε το β) ερώτημα, και χωρίς να χρησιμοποιήσουμε τα mod.

Έχουμε: 40 4 10 103 (3 ) 81 και άρα λήγει σε 1.

Επίσης: 50 2 25 254 (4 ) 16 και άρα λήγει σε 6.

Συνεπώς ο αριθμός 40 503 4 4 , λήγει σε 1, (αφού 1 6 4 11 ).

Επίσης , 20 4 5 57 (7 ) (49 49) και άρα λήγει σε 1, οπότε ο αριθμός 207 9 , θα

λήγει σε 9.

Επειδή δείξαμε στο πρώτο ερώτημα ότι 40 50 203 4 4 7 9 , άρα ο αριθμός A

θα λήγει σε 2 .

ΘΕΜΑ 311 (ΔΗΜΗΤΡΗΣ ΙΩΑΝΝΟΥ)

) Να αποδείξετε ότι ο αριθμός: 3 2A 2n 3n n , με n N , είναι πολλαπλάσιο

του 6. (Γ Γυμνασίου)

Λύση:

Θα βάλω μια απόδειξη εκτός φακέλου για να ασχοληθούν κι άλλοι.

Page 266: Εισαγωγη σε διαγωνιστικα μαθηματικα για το γυμνασιο

http://www.mathematica.gr/forum/viewtopic.php?f=109&t=30104

Επιμέλεια: xr.tsif Σελίδα 266

Ο αριθμός ισούται με 2n(2n 3n 1) n[2n(n 1) (n 1)] n(n 1)(2n 1) .

Ξέρουμε ότι ισχύει 2 2 2 2 n(n 1)(2n 1)1 2 3 ... n

6

. Αφού το αριστερό

μέλος είναι ακέραιο θα ισχύει το ίδιο για το δεξί. Επομένως 6| n(n 1)(2n 1) .

Β τρόπος

Ας δούμε και μια άλλη:

3 2 3 3 2 3 2 3A 2n 3n n n n 3n n 2n (n 3n 2n) (n n)

2 2n(n 3n 2) n(n 1) n(n 1)(n 2) n(n 1)(n 1) (1) .

Ο αριθμός n(n 1)(n 2) είναι πολλαπλάσιο του 6, ως γινόμενο τριών

διαδοχικών ακεραίων. Το ίδιο συμβαίνει και με τον αριθμό n(n 1)(n 1) .

Άρα η σχέση (1) γράφεται: A 6k 6m 6(k m) , με k,m N .

Άρα ο Aείναι πολλαπλάσιο του 6.

ΣΗΜΕΙΩΣΗ

Χρησιμοποιήσαμε την πρόταση: "Το γινόμενο v διαδοχικών ακεραίων,

είναι πολλαπλάσιο του v!, όπου v! 1 2 2 ... (v 1) v .

Γ τρόπος

Μια λύση με επαγωγή:

Για n 1 έχουμε 1

A 6 οπότε 1

6| A .

Έστω n

6| A .

Είναι 3 2 3 2

n 1A 2(n 1) 3(n 1) (n 1) ... 2n 9n 13n 6

3 2 2(2n 3n n) 6(n 2n 1) , οπότε προφανώς n 1

6| A

και η απόδειξη

ολοκληρώνεται.

Page 267: Εισαγωγη σε διαγωνιστικα μαθηματικα για το γυμνασιο

http://www.mathematica.gr/forum/viewtopic.php?f=109&t=30104

Επιμέλεια: xr.tsif Σελίδα 267

ΘΕΜΑ 312 (Socrates)

) Βρείτε όλους τους ακεραίους mτέτοιους ώστε ο αριθμός 3 2m m 7 να

διαιρείται από τον 2m m 1 .

Λύση:

Είναι

2 3 2 2 2(m m 1)| (m m 7) (m m 1)| (m 2)(m m 1) m 5( )

2 2(m m 1)| m 5 m m 1 m 5 ή m 5 .

Για m 5 αυτή γράφεται 2m 2m 4 0 m (1 5,1 5) Z x 1,0,1,2,3 .

Επαληθεύουν μόνο οι 0,1.

Για m 5 , η ανισότητα γράφεται 2m 6 0 και είναι αδύνατη.

ΘΕΜΑ 313 (ΔΗΜΗΤΡΗΣ ΙΩΑΝΝΟΥ)

) Δίνονται οι άρτιοι φυσικοί αριθμοί a,b. Γνωρίζουμε ότι ο a διαιρούμενος με το

3 δίνει υπόλοιπο 2 , ενώ ο b διαιρούμενος με το 5δίνει υπόλοιπο 3 . Να βρεθεί

το υπόλοιπο της διαίρεσης του 2 22a 3b με το 12 . (Γ Γυμνασίου)

Λύση:

Από την υπόθεση έχουμε: a 3k 2 και b 5m 3 .

Αφού όμως ο a είναι άρτιος, θα πρέπει και ο kνα είναι άρτιος, δηλ. k 2n

Ομοίως, ο mθα πρέπει να είναι περιττός, δηλ. m 2 1 .

Τελικά, έχουμε: a 6n 2 και b 10 8 .

Άρα, μετά από κάποιες πράξεις έχουμε:

2 2 2 2 2 22a 3b 300 480 72n 48n 200 12(25 40 6n 4n 16) 8

Page 268: Εισαγωγη σε διαγωνιστικα μαθηματικα για το γυμνασιο

http://www.mathematica.gr/forum/viewtopic.php?f=109&t=30104

Επιμέλεια: xr.tsif Σελίδα 268

Άρα, το υπόλοιπο της διαίρεσης με το 12 είναι 8 .

Παρατήρηση

Για το bμας ενδιαφέρει μόνο ότι είναι άρτιος, γιατί 2 2 2 2b 2k b 4k 3b 12k , άρα αρκεί να βρούμε το υπόλοιπο της

διαίρεσης του 22a με το 12 .

Ομοίως a 6n 2 , άρα 2 2 22a 2(6n 2) 72n 72n 8 8 mod12 .

ΘΕΜΑ 314 (ΔΗΜΗΤΡΗΣ ΙΩΑΝΝΟΥ)

)

Αν a,b,c,x,y,z 0 και αν: a b c

x y z , να αποδείξετε ότι:

a a b c c

x x y z z

.

(Β, Γ Γυμνασίου)

Λύση:

Αρχικά, θα αποδείξω ότι a b c a

x y z x

.

Ισοδύναμα έχουμε, αφού a,b,c,x,y,z 0

a b c aax bx cx ax ay az bx xc ay az

x y z x

(1) .

Όμως, από την εκφώνηση έχουμε:

a bbx ay

x y .

a cxc az

x z .

Προσθέτοντας κατά μέλη τις παραπάνω σχέσεις παίρνουμε την (1) .

Ομοίως εργαζόμαστε και για τη σχέση a b c c

x y z z

.

Page 269: Εισαγωγη σε διαγωνιστικα μαθηματικα για το γυμνασιο

http://www.mathematica.gr/forum/viewtopic.php?f=109&t=30104

Επιμέλεια: xr.tsif Σελίδα 269

Β τρόπος

Ας γράψω μία φαινομενικά διαφορετική λύση αλλά στην πραγματικότητα η

ίδια. Την γράφω μόνο και μόνο για το ύφος, προς όφελος των μαθητών μας.

Έχουμε από υπόθεση cx cy

a ,bz z

, άρα

cx cy cza b c c x y z cz z z ·x y z x y z z x y z z

, όπως θέλαμε.

Όμοια η άλλη.

ΘΕΜΑ 315 (ΔΗΜΗΤΡΗΣ ΙΩΑΝΝΟΥ)

)

Να εξετάσετε αν ο αριθμός 5 4 5 4

a3 36 6

είναι φυσικός.

Λύση:

2 2

4 2 4 2 2 2 2 2a 2· 1 2· 1 1 1 1 1

6 66 6 6 6 6 6

Επειδή 6 2 θα είναι 2

16 και

2 2 21 0 1 1

6 6 6 .

Άρα 2 2

a 1 1 2 N6 6

.

Β τρόπος

Είναι:

2 5 4 5 4 (5 2 6)(5 2 6) 10 1 10 2 12a 2 2 4

3 3 9 3 9 3 3 36 6

.

Άρα a 4 2 N .

Page 270: Εισαγωγη σε διαγωνιστικα μαθηματικα για το γυμνασιο

http://www.mathematica.gr/forum/viewtopic.php?f=109&t=30104

Επιμέλεια: xr.tsif Σελίδα 270

ΘΕΜΑ 316 (ΔΗΜΗΤΡΗΣ ΙΩΑΝΝΟΥ)

)

Δίνεται ο αριθμός 169

A 3n 8n 1724

, όπου n N . Αν ο αριθμός 24A

είναι φυσικός, να αποδείξετε ότι ο αριθμός 6A , είναι ρητός. (Γ Γυμνασίου)

Λύση:

Αφού ο 24A είναι φυσικός, είναι 2k 17

8n 17 k N n8

.

Αντικαθιστώντας το παραπάνω στην παράσταση A , βρίσκουμε ότι

2 2(3k 4) (3k 4)A 6A

24 4

, άρα

3k 46A Q

2

.

ΘΕΜΑ 317 (Socrates)

) Σε καθεμιά κορυφή ενός τετραέδρου γράφουμε έναν ακέραιο. Αν το άθροισμα

των αριθμών στις κορυφές κάθε έδρας διαιρείται με το 5, να δείξετε ότι όλοι οι

αριθμοί στις κορυφές διαιρούνται με το 5.

Λύση:

Ας συμβολίσουμε με a,b,c,dτους αριθμούς στις κορυφές του τετραέδρου.

Τότε οι αριθμοί a b c, b c d, c d a, d a b είναι πολλαπλάσια του 5.

Άρα το άθροισμά τους δηλαδή ο 3(a b c d) είναι επίσης πολλαπλάσιος του

5, απ' όπου ο αριθμός a b c d είναι πολλαπλάσιος του 5 (αφού οι 3 και 5

δεν έχουν κοινούς διαιρέτες).

Άρα ο αριθμός a (a b c d) (b c d) είναι πολλαπλάσιος του 5 αφού

είναι διαφορά μεταξύ δύο αριθμών πολλαπλασίων του 5.

Όμοια και οι αριθμοί b (a b c d) (c d a) ,

c (a b c d) (d a b) και d (a b c d) (a b c) είναι

πολλαπλάσια του 5.

Page 271: Εισαγωγη σε διαγωνιστικα μαθηματικα για το γυμνασιο

http://www.mathematica.gr/forum/viewtopic.php?f=109&t=30104

Επιμέλεια: xr.tsif Σελίδα 271

ΘΕΜΑ 318 (Socrates)

) Βρείτε το μεγαλύτερο πενταψήφιο παλινδρομικό αριθμό που διαιρείται με το

101.

Λύση:

Ας είναι a xyzyx ο ζητούμενος αριθμός.

Είναι a 10000x 1000y 100z 10y x 10001x 1010y 100z

(9999x 101z 1010y) (2x z) . Η πρώτη παρένθεση είναι διαιρετή με το

101οπότε αρκεί να είναι και το 2x z

Επειδή τα x,y,z είναι ψηφία και θέλουμε ο αριθμός μας να είναι ο μέγιστος

δυνατός, βλέπουμε ότι πρέπει x 4,z 8,y 9

Άρα ο αριθμός είναι ο 49894 .

Από εδώ.

http://www.math.wisc.edu/talentsearch/sites/default/files/T13-1q_1.pdf.

ΘΕΜΑ 319 (ΔΗΜΗΤΡΗΣ ΙΩΑΝΝΟΥ)

) Αν x,y R και αν:

3 2 2x 3x y 3y 2x 6 και 3 2 2y 3xy 3x 2y 6xy , να

βρεθεί ο αριθμός x y . (Γ Γυμνασίου)

Λύση:

Με πρόσθεση κατά μέλη έχω (με λίγη παρατηρητικότητα) πως ισχύει:

a x y

3 2 3 2x y 3 x y 2 x y 6 0 a 3a 2a 6 0

2a 2 02 2a (a 3) 2(a 3) 0 (a 3)(a 2) 0 a 3 x y 3

.

Page 272: Εισαγωγη σε διαγωνιστικα μαθηματικα για το γυμνασιο

http://www.mathematica.gr/forum/viewtopic.php?f=109&t=30104

Επιμέλεια: xr.tsif Σελίδα 272

ΘΕΜΑ 320 (ΔΗΜΗΤΡΗΣ ΙΩΑΝΝΟΥ)

) Αν x,y N και αν ο αριθμός

4 4 2 2A x y x y είναι πρώτος, να βρεθούν οι

αριθμοί x,y . (Γ Γυμνασίου)

Λύση:

4 4 2 2 4 4 2 2 2 2 2 2 2A x y x y x y 2x y (xy) (x y ) (xy)

2 2 2 2(x y xy)(x y xy) 1·p ,

και επειδή 2 2 2 2x y xy x y xy και

2 2x y xy 0 θα πρέπει:

2 2

2 2

x y xy p

x y xy 1

.

2"

2"

2x 2y 1 p

(1) .

" "

2xy p 1

(2) .

2 2 2(1) (2) x y (x y) 2 .

Όμως: "

2

2 2 2"

2

2 2

x 1 x 1

y 1 y 1 x y (x y) 2

(x y) 0 (x y) 0

,

με την ισότητα να ισχύει για x y 1 .

ΘΕΜΑ 321 (Socrates)

) α) Να παραγοντοποιηθεί η παράσταση

2 2 2(1 x xy) y .

β) Να βρείτε το μεγαλύτερο τριψήφιο πρώτο pγια τον οποίο η εξίσωση

2 2 2(1 x xy) y p έχει ακέραια ρίζα. (Γ Γυμνασίου)

Page 273: Εισαγωγη σε διαγωνιστικα μαθηματικα για το γυμνασιο

http://www.mathematica.gr/forum/viewtopic.php?f=109&t=30104

Επιμέλεια: xr.tsif Σελίδα 273

Λύση:

α) 2 2 2 4 2 2 2 3 2(1 x xy) y x x y 1 2x 2x y 2xy y

4 2 2 2 2 3 2 2 2 2 2x 2x 1 y x y 2x y 2xy (x 1) y (x 1) 2xy(x 1)

2 2 2 2 2 2 2(x 1)(x 1 y ) 2xy(x 1) (x 1)(x 2xy y 1)

2 2(x 1)[(x y) 1] .

β) Από το (α) και την υπόθεση έχουμε: 2 2p (x 1)[(x y) 1] .

Έτσι έχουμε τα συστήματα: 1

( )2

2

x 1 1

(x y) 1 p

και

2( )

2

2

x 1 p

(x y) 1 1

.

Ξεκινάμε με το 1

( ) . Από την πρώτη εξίσωση παίρνουμε: 2x 1 1 x 0 .

Έτσι η δεύτερη γίνεται: 2 2(x y) 1 p y 1 p .

Αφού ο p είναι τριψήφιος θα είναι περιττός και άρα:

2 2y 1 2k 1 y 2k y 2m .

Ακόμα για να είναι τριψήφιος θα πρέπει να ισχύει:

10 y 31 10 2m 31 5 m 15 .

Αφού θέλουμε τον μεγαλύτερο εύκολα βρίσκουμε ότι

m 13 y 26 p 677 .

Έτσι παίρνουμε τις λύσεις: (x,y) (0,26) , (x,y) (0, 26) .

Ομοίως θα εργαστούμε στο σύστημα 2

( ) , όπου θα πάρουμε τον ίδιο πρώτο

όμως οι λύσεις θα είναι: (x,y) (26, 26) , (x,y) ( 26,26) .

ΘΕΜΑ 322 (ΔΗΜΗΤΡΗΣ ΙΩΑΝΝΟΥ)

) Αν

1 2 3 4 5a a a a a 2014 , όπου οι αριθμοί

1 2 5a ,a ,...,a είναι ακέραιοι και

Page 274: Εισαγωγη σε διαγωνιστικα μαθηματικα για το γυμνασιο

http://www.mathematica.gr/forum/viewtopic.php?f=109&t=30104

Επιμέλεια: xr.tsif Σελίδα 274

αν για τους πραγματικούς αριθμούς 1 2 3 4 5

x ,x ,x ,x ,x αληθεύουν οι σχέσεις:

1 2 1

2 3 2

3 4 3

4 5 4

5 1 5

x x a

x x a

x x a

x x a

x x a

, να αποδείξετε ότι οι αριθμοί 1 2 3 4 5

x ,x ,x ,x ,x είναι επίσης

ακέραιοι. (Β Γυμνασίου)

Λύση:

1 2 1

2 3 2

3 4 3

1 2 3 4 5

4 5 4

5 1 5

1 2 3 4 5

" "

x x a

x x a

x x ax x x x x 1007

x x a

x x a

a ,a ,a ,a ,a Z

.

Τώρα έχουμε:

1 2 3 4 5 1 2 5 5 1 2x x x x x 1007 a a x 1007 x 1007 a a .

Άρα 5

x Z .

Με όμοιο τρόπο αποδεικνύουμε ότι και οι υπόλοιποι είναι ακέραιοι.

ΘΕΜΑ 323 (ΘΑΝΟΣ ΜΑΓΚΟΣ)

) Δίνεται ο αριθμός 101010 10, στον οποίο εμφανίζονται 2013δεκάρικα.

α) Μπορεί να γραφεί ως άθροισμα τετραγώνων δύο ακεραίων;

β) Μπορεί να γραφεί ως διαφορά τετραγώνων δύο ακεραίων;

Λύση:

α) Το άθροισμα των ψηφίων του αριθμού είναι 2013 3 671 άρα ο αριθμός

διαιρείται από το 3.

Page 275: Εισαγωγη σε διαγωνιστικα μαθηματικα για το γυμνασιο

http://www.mathematica.gr/forum/viewtopic.php?f=109&t=30104

Επιμέλεια: xr.tsif Σελίδα 275

Άρα, αν γράψουμε 2 2A x y θα πρέπει, σύμφωνα με γνωστή ιδιότητα που

έχει αναφερθεί κι άλλες φορές στο συγκεκριμένο θέμα ,να είναι 3| x , 3| y .

Συνεπώς θα έχουμε x 3k , y 3m κι έτσι 2 2A 9(k m ) που είναι άτοπο

,αφού το 9 δε διαιρεί τον αριθμό μας.

Άρα ο αριθμός δε μπορεί να γραφεί ως άθροισμα δύο τέλειων τετραγώνων.

β) H απάντηση είναι όχι και στο δεύτερο ερώτημα.

Έστω Aο δοσμένος αριθμός. Αν 2 2A=a b με a,b Z τότε A= a b a+b .

Ο Aείναι άρτιος, επομένως ένας από τους a b,a+b είναι άρτιος. Όμως

a b a+b mod2 άρα και οι δύο είναι άρτιοι οπότε ο Aείναι πολλαπλάσιο

του 4 , άτοπο.

ΘΕΜΑ 324 (Παύλος Μαραγκουδάκης)

) Να βρείτε τον τετραψήφιο αριθμό abcd αν cda abc=297 και a+b+c=23.

(Β Γυμνασίου)

Λύση:

Καταρχήν είναι: a,b,c,d {1,2,3,4,5,6,7,8,9} .

Είναι:

cda abc 297 100c 10d a 100a 10b c 297

99c 99a 10d 10b 297 10(d b) 99(3 c a) (1) .

Από την τελευταία σχέση και από τον αρχικό περιορισμό παίρνουμε ότι:

d b 0 b d .

Άρα, η (1) γίνεται: 99(3 c a) 0 c a 3 .

Άρα, θα πρέπει a c .

Page 276: Εισαγωγη σε διαγωνιστικα μαθηματικα για το γυμνασιο

http://www.mathematica.gr/forum/viewtopic.php?f=109&t=30104

Επιμέλεια: xr.tsif Σελίδα 276

Από την άλλη σχέση που μας δίνεται είναι: a b c 23 2a b 20 .

Από την παραπάνω βλέπουμε ότι a 6 .

Εύκολα τώρα βρίσκουμε ότι a 6 και c 9 , γιατί αλλιώς θα ήταν a 6 .

Εύκολα τώρα βρίσκουμε ότι b d 8 . Άρα: abcd 6898 .

ΘΕΜΑ 325 (Παύλος Μαραγκουδάκης)

) Αν a,b,c είναι πραγματικοί αριθμοί με 2 2 2a b c 1 να αποδείξετε ότι

1ab bc ca 1

2 . (Γ Γυμνασίου)

Λύση:

Το ότι 2 2 2a b c ab bc ca είναι γνωστό, άρα ab bc ca 1 .

Αν 1

ab bc ca2

θα είναι

2 2 2 2a b c 2ab 2bc 2ca 0 (a b c) 0 , άτοπο, άρα αποδείξαμε το

ζητούμενο.

ΘΕΜΑ 326 (Παύλος Μαραγκουδάκης)

) Οι πραγματικοί αριθμοί a,b,c,A,B,C ικανοποιούν τις σχέσεις

ac 2bB CA 0 και 2ac b 0 . Να αποδείξετε ότι 2AC B 0 .

(Γ Γυμνασίου)

Λύση:

Εικάζω ότι είναι όπως παραπάνω.

Έχουμε 2 2 24b B (ac AC) 4acAC οπότε 2 2b B acAC .

Είναι 2 2 2 2 2 2 2ac b 0 B (ac b ) 0 B ac b B acAC B AC , αφού

2ac b 0 ac 0 .

Page 277: Εισαγωγη σε διαγωνιστικα μαθηματικα για το γυμνασιο

http://www.mathematica.gr/forum/viewtopic.php?f=109&t=30104

Επιμέλεια: xr.tsif Σελίδα 277

Αν ισχύει η ισότητα, τότε B 0 και ac AC . Όμως, τότε είναι και ac AC 0

οπότε ac 0 , άτοπο αφού 2ac b 0 ac 0 .

ΘΕΜΑ 327 (Socrates)

) Να βρείτε όλους τους πενταψήφιους αριθμούς abcde τέτοιους ώστε

abcde 3·ab·cde . (Γ Γυμνασίου)

Λύση:

Θέτοντας x ab και y cde έχουμε να λύσουμε στους ακεραίους την εξίσωση

1000x y 3xy με 10 x 99 και 100 y 999 .

Η εξίσωση έχει λύση μόνο όταν x | y. Για y kx είναι 999

k 10010

και

1000 k 3x 1 .

Είναι 1000 1(mod3) οπότε k 2(mod3) .

Οι μόνοι διαιρέτες του 1000που δεν υπερβαίνουν το 99 και είναι της μορφής

3k 2 είναι στο σύνολο 2,5,8,20,50 .

Για k 2 είναι x 167 που απορρίπτεται.

Για k 5 είναι x 67 και y 335 δηλαδή μία λύση είναι ο 67335.

Για k 8 είναι x 42 και y 336 δηλαδή μία λύση είναι ο 42336 .

Για k 20 είναι x 17 και y 340 δηλαδή μία λύση είναι ο 17340.

Για k 50 είναι x 7 που απορρίπτεται.

ΘΕΜΑ 328 (Socrates)

) Να λυθεί ως προς xη εξίσωση

2 2

2 2

1 1 1 1(a b x) (a ab b )

a x b x a b

όπου a,b 0,a b .

(Γ Γυμνασίου)

Page 278: Εισαγωγη σε διαγωνιστικα μαθηματικα για το γυμνασιο

http://www.mathematica.gr/forum/viewtopic.php?f=109&t=30104

Επιμέλεια: xr.tsif Σελίδα 278

Υπόδειξη

Το πρώτο μέλος γράφεται b a

2...a x b x

Κάνουμε το ίδιο και στο δεύτερο και συνεχίζουμε με παραγοντοποίηση.

Λύση:

Η εξίσωση έχει περιορισμούς x a και x b Είναι ισοδύναμη με τις

ακόλουθες:

2 2

2 2

b a ab b a ab2 2

a x b x a b

2 2

1 a b 1 a bb a 0

a x a b x b

2 2

ab x(a b) ab x(a b)b a 0

(a x)a (b x)b

3 3ab x(a b) b b x a a x 0 .

Προκύπτουν οι ab

xa b

ή 4 4

3 3

a bx

a b

.

Και οι δύο λύσεις ικανοποιούν τους περιορισμούς αφού από υπόθεση a b .

ΘΕΜΑ 329 (ΔΗΜΗΤΡΗΣ ΙΩΑΝΝΟΥ)

) Δύο τρίγωνα και , έχουν ίσες τις περιμέτρους , A Z και ˆˆ Z .

α) Προεκτείνουμε τις πλευρές και και επί των προεκτάσεων αυτών,

παίρνουμε τα σημεία και αντιστοίχως, έτσι ώστε να είναι BH AB και

E E . Να αποδείξετε ότι τα τρίγωνα AHκαι Z είναι ίσα.

β) Να αποδείξετε ότι και τα τρίγωνα και είναι επίσης ίσα.

(Γ Γυμνασίου)

Page 279: Εισαγωγη σε διαγωνιστικα μαθηματικα για το γυμνασιο

http://www.mathematica.gr/forum/viewtopic.php?f=109&t=30104

Επιμέλεια: xr.tsif Σελίδα 279

Λύση:

α) Αφού έχουν ίσες περιμέτρους έχουμε:

AC DZ

AB BC AC EZ DZ DE

AB BC DE EZ

HB BC E EZ HC Z .

Τα AHC και DZ είναι ίσα από το

, αφού ˆ ˆHC Z,DZ AC,C Z .

β) Από την παραπάνω ισότητα παίρνουμε:

HAC D , CHA Z D .

Όμως, τα HBA και E D είναι ισοσκελή και άρα CHA B HA και

Z D ED .

Επομένως: BAH DE .

Είναι όμως: AC D HAB BAC DZ EDZ BAC EDZ .

Έτσι, τα ABC και EDZ είναι ίσα από το διότι:

AC DZ , BAC EDZ , ˆ ˆC Z .

Page 280: Εισαγωγη σε διαγωνιστικα μαθηματικα για το γυμνασιο

http://www.mathematica.gr/forum/viewtopic.php?f=109&t=30104

Επιμέλεια: xr.tsif Σελίδα 280

ΘΕΜΑ 330 (ΔΗΜΗΤΡΗΣ ΙΩΑΝΝΟΥ)

) Δίνεται τρίγωνο AB και έστω ότι οι διχοτόμοι των

εξωτερικών γωνιών B και , τέμνονται στο O . Στην

προέκταση της AB προς το μέρος του B , παίρνουμε

τμήμα B B A και στην προέκταση της A

προς το μέρος του , παίρνουμε σημείο ώστε να

είναι E B . Να αποδείξετε ότι τα τρίγωνα BOE και

OA είναι ισοσκελή.

(Γ Γυμνασίου)

Λύση:

Στο ισοσκελές τρίγωνο BCE, η CO θα είναι και ύψος και διάμεσος.

Έτσι τα ορθογώνια τρίγωνα BOM

και OEMείναι ίσα (αφού η

είναι κοινή) και άρα παίρνουμε το

πρώτο ζητούμενο.

Είναι γνωστό ότι AˆCOB 902

.

Επομένως,

ˆ ˆEOB 2COB 180 A

.

Άρα:

2 2

A2E 180 180 A E

2

.

Ακόμα: 1 1

180 180 C CE E

2 2

.

Page 281: Εισαγωγη σε διαγωνιστικα μαθηματικα για το γυμνασιο

http://www.mathematica.gr/forum/viewtopic.php?f=109&t=30104

Επιμέλεια: xr.tsif Σελίδα 281

Επομένως: 21

CE E

2

.

Είναι: 1

180 C

2 2

.

Ακόμα, από το πρώτο ζητούμενο έχουμε ότι BO OE .

Τέλος, αφού το τρίγωνο BCE είναι ισοσκελές θα είναι BC CE . Άρα, η

δοθείσα σχέση γίνεται: BD AC BC BD AC CE BD AE .

Από τα παραπάνω, παίρνουμε ότι τα τρίγωνα BODκαι AOEείναι ίσα και από

εκεί παίρνουμε το δεύτερο ζητούμενο.

ΘΕΜΑ 331 (ΔΗΜΗΤΡΗΣ ΙΩΑΝΝΟΥ)

) Να υπολογίσετε την τιμή της παράστασης

2 2 2 2

1 1 1 1A (1 )(1 )(1 )...(1 )

2 3 4 2013 . (Γ Γυμνασίου)

Λύση:

Σύμφωνα με την ταυτότητα 2 2a b (a b)(a b) είναι:

2 2 2 2

1 1 1 1A 1 1 1 ··· 1

2 3 4 2013

1 1 1 1 1 1 1 11 1 1 1 1 1 ······ 1 1

2 2 3 3 4 4 2013 2013

1 3 2 4 3 5 2012 2014 1 2014 2014 1007· · · · · ··· · ·

2 2 3 3 4 4 2013 2013 2 2013 4026 2013 .

Διότι οι όροι απλοποιούνται εκτός από τον πρώτο και τον τελευταίο.

Β τρόπος

2 2 2 2

2 2 2 2 2 2 2 2

1 1 1 1 2 1 3 1 4 1 2013 1A (1 )(1 )(1 )...(1 ) ( )( )( )...( )

2 3 4 2013 2 3 4 2013

Page 282: Εισαγωγη σε διαγωνιστικα μαθηματικα για το γυμνασιο

http://www.mathematica.gr/forum/viewtopic.php?f=109&t=30104

Επιμέλεια: xr.tsif Σελίδα 282

2 2 2 2 2

(2 1)(2 1)(3 1)(3 1)(4 1)(4 1)...(2013 1)(2013 1)

2 ·3 ·4 ·5 ... 2013

.

Παρατηρούμε ότι στα γινόμενα στον αριθμητή εμφανίζεται ο αριθμοί x 1 και

x 1 με ελάχιστο το x 2 και μέγιστο το x 2013 .

Άρα, εμφανίζονται οι όροι 2 2 2 22,3 ,4 ,5 ...,2012 ,2013,2014 , άρα έχουμε:

2 2 2 2

2 2 2 2 2 2

2·3 ·4 ·5 ...·2012 ·2013·2014

2 ·3 ·4 ·5 ...·2012 ·2013.

Απλοποιούμε τον αριθμητή με τον παρανομαστή και έχουμε: 2014 2014

2013·2 4026 .

ΘΕΜΑ 332 (ΔΗΜΗΤΡΗΣ ΙΩΑΝΝΟΥ)

)

Αν x,y 0 και αν x x 1 x 2 x 2013

... 2014y y 1 y 2 y 2013

, να αποδείξετε ότι

x y . (Γ Γυμνασίου)

Λύση:

Παρατηρούμε ότι το LHS αποτελείτε από 2014όρους.

Διακρίνουμε τις περιπτώσεις:

Αν x y ,

τότε x x 1 x 2 x 2013

1, 1, 1,..., 1y y 1 y 2 y 2013

.

Με πρόσθεση κατά μέλη λαμβάνουμε: LHS 2014 , άτοπο.

Αν x y ,

τότε x x 1 x 2 x 2013

1, 1, 1,..., 1y y 1 y 2 y 2013

.

Με πρόσθεση κατά μέλη λαμβάνουμε: LHS 2014 , πάλι άτοπο.

Page 283: Εισαγωγη σε διαγωνιστικα μαθηματικα για το γυμνασιο

http://www.mathematica.gr/forum/viewtopic.php?f=109&t=30104

Επιμέλεια: xr.tsif Σελίδα 283

Αν x y ,

Τότε x x 1 x 2 x 2013

1, 1, 1,..., 1y y 1 y 2 y 2013

.

Με πρόσθεση κατά μέλη λαμβάνουμε: LHS 2014 , που ισχύει.

Άρα, αποδείξαμε ότι x y .

Β τρόπος

x x 1 x 2 x 2013.... 2014

y y 1 y 2 y 2013

x y y x (y 1) y x (y 2013) y... 2014

y y 1 y 2013

.

Απλοποιούμε και έχουμε

x y x y x y x y x y x y1 1 .... 1 2014 ... 0

y y 1 y 2013 y y 1 y 2013

Αν x y x y 0 ,

επειδή x,y 0 όλα τα κλάσματα θα είναι θετικά όπως και το άθροισμά τους,

επομένως έχουμε άτοπο.

Αν x y x y 0 ,

επειδή x,y 0 όλα τα κλάσματα θα είναι αρνητικά όπως και το άθροισμά τους,

επομένως καταλήγουμε πάλι σε άτοπο.

Συνεπώς x y το οποίο ικανοποιεί και τη δοθείσα ισότητα.

ΘΕΜΑ 333 (Παύλος Μαραγκουδάκης)

) Αν a,b,c είναι πραγματικοί αριθμοί με a b,b c,c a να αποδείξετε ότι

4 4 4a b c b c a c a b 0 . (Γ Γυμνασίου)

Page 284: Εισαγωγη σε διαγωνιστικα μαθηματικα για το γυμνασιο

http://www.mathematica.gr/forum/viewtopic.php?f=109&t=30104

Επιμέλεια: xr.tsif Σελίδα 284

Λύση:

Ας παραγοντοποιήσουμε την παράσταση.

Έχουμε 4 4 4a b c b c a c a b ...

2 2 2 2 4(a c)[b(a c )(a c) ac(a ac b ) b ] .

Αφού a c , πρέπει να αποδείξουμε ότι 2 2 2 2 4b(a c )(a c) ac(a ac b ) b 0 .

Έχουμε

2 2 2 2 4b(a c )(a c) ac(a ac b ) b ...

2 2 2 2 3(a b)[b(a ab b ) a c ac c ] .

Αφού a b , ομοίως 2 2 2 2 3b(a ab b ) a c ac c ...

2 2 2(b c)[a a(b c) b bc c ] .

Αφού c b , τότε πρέπει να αποδείξω ότι

2 2 2 2 2 2a a(b c) b bc c a ab ac b bc c 0

2 2 22(a ab ac b bc c ) 0

2 2 2 2 2 2(a 2ab b ) (b 2bc c ) (c 2ca a ) 0

2 2 2(a b) (b c) (c a) 0 .

Ας υποθέσουμε ότι ισχύει, δηλαδή a b,b c,c a ,

όμως, με αντικατάσταση έχουμε ότι a ( c) c , που είναι άτοπο.

Άρα, αποδείξαμε το ζητούμενο.

Page 285: Εισαγωγη σε διαγωνιστικα μαθηματικα για το γυμνασιο

http://www.mathematica.gr/forum/viewtopic.php?f=109&t=30104

Επιμέλεια: xr.tsif Σελίδα 285

Μετά τις λύσεις που προηγήθηκαν, το ηθικό δίδαγμα είναι

4 4 4 2 2 2a (b c)+b (c a)+c (a b)= (a b)(b c)(c a)(a +b +c +ab+bc+ca) .

ΘΕΜΑ 334 (Παύλος Μαραγκουδάκης)

) Αν ax by cz να αποδείξετε ότι a y b z c x x b y c z a .

(Γ Γυμνασίου)

Λύση:

Θέτουμε ax by cz k .

Επομένως: k

xa

, k

yb

, k

zc

.

Άρα, το πρώτο μέλος της αποδεικτέας γράφεται:

k k k ab k bc k ac k

a y b z c x a b c · ·b c a b c a

(ab k)(bc k)(ac k)

abc

(1) .

Το δεύτερο μέλος είναι:

k k k k ab k bc k ac

x b y c z a b c a · ·a b c a b c

(ab k)(bc k)(ac k)

abc

(2) .

Από τις (1) και (2) προκύπτει το ζητούμενο.

ΘΕΜΑ 335 (Παύλος Μαραγκουδάκης)

) Αν

2 2 2x yz a,y zx b,z xy c να αποδείξετε ότι

ax by cz a b c x y z . (Γ Γυμνασίου)

Page 286: Εισαγωγη σε διαγωνιστικα μαθηματικα για το γυμνασιο

http://www.mathematica.gr/forum/viewtopic.php?f=109&t=30104

Επιμέλεια: xr.tsif Σελίδα 286

Λύση:

Έχουμε: 3 3 3x xyz ax,y xyz by,z xyz cz .

Και με πρόσθεση κατά μέλη, λαμβάνουμε:

3 3 3x y z 3xyz ax by cz .

Από την ταυτότητα του Euler, γνωρίζουμε ότι:

3 3 3 2 2 2x y z 3xyz (x y z)(x y z xy xz yz) .

Άρα:

2 2 2ax by cz (x y z)[(x yz) (y zx) (z xy)] (x y z)(a b c)

ΘΕΜΑ 335β (ΔΗΜΗΤΡΗΣ ΙΩΑΝΝΟΥ)

)

Αν 4 3 a

a b c , με *a,b,c N , να αποδείξετε ότι ο αριθμός A c ab , είναι

τέλειο τετράγωνο. (Β Γυμνασίου)

Λύση:

Θέτουμε: 4 3 a

ka b c .

Άρα: 4

ak

, 3

bk

, 2

4a 4kck k k

.

Επομένως:

2

2 2 2

4 12 16 4A c ab

k k k k

.

Έτσι, το ζητούμενο αποδείχθηκε.

Page 287: Εισαγωγη σε διαγωνιστικα μαθηματικα για το γυμνασιο

http://www.mathematica.gr/forum/viewtopic.php?f=109&t=30104

Επιμέλεια: xr.tsif Σελίδα 287

ΘΕΜΑ 336 (Παύλος Μαραγκουδάκης)

) Για τους αριθμούς x,y ισχύουν οι σχέσεις

2 2x 3xy 2y x y 0 και

2 2x 2xy y 5x 7y 0 . Να αποδείξετε ότι xy 12x 15y 0 .

(Γ Γυμνασίου)

Λύση:

Η πρώτη με κατάλληλο..πείραγμα δίνει:

22 2 2x 2xy y y xy x y 0 x y y x y x y 0

x y x 2y 1 0 x y x 2y 1 .

Αν x y ,

τότε η δεύτερη δίνει: 2 2 2x 2x x 5x 7x 0 x 0 y 0 .

Οι συγκεκριμένες τιμές ικανοποιούν το ζητούμενο.

Αν x 2y 1 ,

τότε η δεύτερη δίνει (μετά τις πράξεις): 2y 5y 6 0 y 2 y 3 .

Αν y 2 ,

τότε x 3 ικανοποιεί τη ζητούμενη σχέση.

Αν y 3 ,

τότε x 5 παρομοίως δεκτό ζεύγος αφού ικανοποιεί τη ζητούμενη σχέση.

Επομένως για κάθε ζεύγος (x,y) που επαληθεύει τις δύο αρχικές συμπεράναμε

πως επαληθεύεται και η τρίτη σχέση.

ΘΕΜΑ 337 (ΔΗΜΗΤΡΗΣ ΙΩΑΝΝΟΥ)

)

Αν:1 1 1 1

... 91 2 2 3 3 4 n n 1

, να βρεθεί ο φυσικός

Page 288: Εισαγωγη σε διαγωνιστικα μαθηματικα για το γυμνασιο

http://www.mathematica.gr/forum/viewtopic.php?f=109&t=30104

Επιμέλεια: xr.tsif Σελίδα 288

αριθμός n . (Γ Γυμνασίου)

Λύση:

Είναι: 1 1 1 1

... 91 2 2 3 3 4 n n 1

2 1 3 2 4 3...

( 2 1)( 2 1) ( 3 2)( 3 2) ( 4 3)( 4 3)

n 1 n9

( n 1 n)( n 1 n)

2 1 3 2 4 3 ... n 1 n 9

n 1 1 9 n 1 100 n 99 .

ΘΕΜΑ 338 (ΔΗΜΗΤΡΗΣ ΙΩΑΝΝΟΥ)

) Να αποδείξετε ότι ο αριθμός a 2013! 2013 δεν είναι τέλειο τετράγωνο.

( 2013! 1 2 3 ... 2012 2013 ). (Α Γυμνασίου)

Λύση:

Παρατηρούμε ότι το 2013! είναι πολλαπλάσιο του 10, άρα (2013!) 0 (όπου

με () συμβολίζουμε το τελευταίο ψηφίο). Επίσης, (2013) 3 , άρα

(2013! 2013) 3 . Όμως, είτε χρησιμοποιώντας την παρατήρηση του κ.

Δημήτρη, είτε το παρατηρούμε μόνοι μας, κανένα τέλειο τετράγωνο δεν

τελειώνει σε 3, όμως αυτός ο αριθμός τελειώνει σε 3, άρα δεν είναι τέλειο

τετράγωνο.

ΘΕΜΑ 339 (Παύλος Μαραγκουδάκης)

) α) Βρείτε το άθροισμα των ψηφίων του αριθμού 201310 2013 .

β) Διαιρείται ο αριθμός αυτός με το 9;

Page 289: Εισαγωγη σε διαγωνιστικα μαθηματικα για το γυμνασιο

http://www.mathematica.gr/forum/viewtopic.php?f=109&t=30104

Επιμέλεια: xr.tsif Σελίδα 289

γ) Ποιο είναι το υπόλοιπο της διαίρεσης του αριθμού αυτού με το 125;

δ) Είναι τέλειο τετράγωνο;

ε) Είναι τέλειος κύβος; (Α Γυμνασίου)

Λύση:

α) Αν κάνουμε την αφαίρεση, βρίσκουμε ότι a 999...997987 (με 2009

εννιάρια). Άρα S(a) 9·2010 22 18090 22 18112 (όπου S(a) είναι το

άθροισμα των ψηφίων).

β) Έχουμε S[S(a)] S(18112) 1 8 2 2 13 , αφού ο 13 δεν διαιρεί τον 9 ,

τότε ο a δεν είναι πολλαπλάσιό του.

γ) Αν κάνουμε προσεκτικά την διαίρεση στα εννιάρια, βρίσκουμε μηδενικό

υπόλοιπο και έτσι στα τρία τελευταία ψηφία έχουμε 187 1·125 62 , άρα το

υπόλοιπο της διαίρεσης είναι 62 .

δ) Όχι, αφού το τελευταίο ψηφίο του αριθμού είναι το 7και γνωρίζουμε (ή το

παρατηρούμε μόνοι μας) ότι όταν ένας αριθμός τελειώνει σε 7δεν είναι ποτέ

τέλειο τετράγωνο.

ε) Οι τέλειοι κύβοι είναι οι αριθμοί 3 3 3 3 31 ,2 ,3 ,4 ,5 ,...

Ας σχηματίσουμε τις διαφορές δύο διαδοχικών κύβων.

3 32 1 7 ,

3 33 2 21 ,

3 34 3 37 ,

3 35 4 61 .

Παρατηρούμε ότι οι διαφορές αυτές μεγαλώνουν διαρκώς και γρήγορα

ξεπερνούν το 2013.

Για παράδειγμα 3 331 30 2791 .

Page 290: Εισαγωγη σε διαγωνιστικα μαθηματικα για το γυμνασιο

http://www.mathematica.gr/forum/viewtopic.php?f=109&t=30104

Επιμέλεια: xr.tsif Σελίδα 290

Με άλλα λόγια οι αριθμοί 30και 31 ενώ διαφέρουν κατά 1οι κύβοι τους

απέχουν πολύ.

Τώρα ο αριθμός που μας ενδιαφέρει είναι πολύ κοντά σε έναν τέλειο κύβο,

είναι μόλις 2013 μικρότερος από τον τέλειο κύβο 3

32013 671

671

10 10 100...0

.

Ο αμέσως προηγούμενος κύβος είναι ο

3

671

99...9

.

Είναι λοιπόν

3 3

2013

671 671

99...9 10 2013 100...0

.

Άρα ο αριθμός 201310 2013 δεν είναι τέλειος κύβος.

Ας δούμε και μια ακόμα λύση για το (ε), αλλά όχι για μαθητές Α Γυμνασίου

(μόνο για Γ Γυμνασίου).

Παρατηρούμε ότι 671 3 671 3 671 3 671 3 2013 671 3(10 1) (10 ) 2013 (10 ) (10 1) 10 2013 (10 ) .

Άρα ο αριθμός 201310 2013 δεν μπορεί να είναι τέλειος κύβος, αφού είναι

ανάμεσα σε δύο διαδοχικούς τέλειους κύβους.

ΘΕΜΑ 340 (ΔΗΜΗΤΡΗΣ ΙΩΑΝΝΟΥ)

) Αν ένας φυσικός αριθμός είναι μεγαλύτερος από τον 1202 και μικρότερος από

τον 15 2(16 1) , να αποδείξετε ότι δεν μπορεί να είναι τέλειο τετράγωνο.

(Γ Γυμνασίου)

Λύση:

Έστω a ο αριθμός που δεν μπορεί να είναι τέλειο τετράγωνο.

Τότε, δημιουργείτε η ανισότητα:

Page 291: Εισαγωγη σε διαγωνιστικα μαθηματικα για το γυμνασιο

http://www.mathematica.gr/forum/viewtopic.php?f=109&t=30104

Επιμέλεια: xr.tsif Σελίδα 291

15 2 120 4 15 60 2 60 60 2(16 1) a 2 [(2 ) 1] a (2 ) (2 1) a (2 ) .

Όμως, επειδή ο αριθμός a βρίσκεται ανάμεσα σε δύο διαδοχικά τέλεια

τετράγωνα, έπεται ότι δεν είναι τέλειο τετράγωνο. Άρα, αποδείξαμε το

ζητούμενο.

ΘΕΜΑ 341 (ΔΗΜΗΤΡΗΣ ΙΩΑΝΝΟΥ)

) Αν οι ακέραιοι αριθμοί a,b δεν είναι και οι δύο άρτιοι, να αποδείξετε ότι ο

αριθμός 4 2 2 4 2 2A a b a b a b δεν είναι τέλειο τετράγωνο. (Γ Γυμνασίου)

Λύση:

Έστω ότι 4 2 2 4 2 2A a b a b ya b για ακέραιο y.

Τότε 2 2 2 2 2a b (a b 1) y .

Θα πρέπει 2 2 2a b | y οπότε ab | y και έστω ότι abx y , οπότε θα γίνει

2 2 2a b 1 x . Ξέρουμε ότι οι a,b δεν είναι ταυτόχρονα άρτιοι οπότε έχουμε

2επιλογές.

Αν ο ένας είναι άρτιος και ο άλλος περιττός τότε

2 2a b 1 2(mod4) άρα 2y mod4 που είναι αδύνατον.

Αν a,b περιττοί τότε

2 2a b 1 3(mod4) άρα 2y 3(mod4) αδύνατον.

Η εξίσωση δεν έχει λύσεις αν τουλάχιστον ένας εκ των a,b είναι περιττός.

Κάποιες ιδέες

Αν a,b και οι δυο άρτιοι τότε 2 2 2a b 1 y .

2 2a b 1 1(mod4) άρα 2y 1(mod4) . Οπότε yπεριττός.

Η παραπάνω εξίσωση έχει άπειρες λύσεις για a,b άρτιοι και ένας τρόπος απλός

για να το αποδείξεις είναι ο εξής.

Page 292: Εισαγωγη σε διαγωνιστικα μαθηματικα για το γυμνασιο

http://www.mathematica.gr/forum/viewtopic.php?f=109&t=30104

Επιμέλεια: xr.tsif Σελίδα 292

2 2 2a 1 y b .

Δεδομένου ότι κάθε σύνθετος n με n 0,1,3(mod4) παριστάνεται ως διαφορά

δυο τετραγώνων και εφόσον ο 2a 1 είναι σύνθετος για άπειρες τιμές του a

τότε και η εξίσωση 2 2 2a 1 y b επαληθεύεται για άπειρες τριάδες.

Είπα κάθε σύνθετος n με n 0,1,3(mod4) παριστάνεται ως διαφορά

τετραγώνων και αυτό προκύπτει από την ταυτότητα: 2 2p q p qpq ( ) ( )

2 2

Πως θα βρω μια τέτοια τριάδα.

Έστω a 2k τότε 2 2 2 2 2 24k 1 (4k 1) 1 (2k 1) (2k ) ,

2 2(a,b,y) (2k,2k ,2k 1) .

ΘΕΜΑ 342 (Παύλος Μαραγκουδάκης)

) Τα A,Bείναι δύο ψηφία, δηλαδή ακέραιοι από το 0 έως το 9. Αν το γινόμενο

των τριψήφιων αριθμών 2A5 και 13B είναι πολλαπλάσιο του 36, να βρείτε

όλα τα δυνατά ζεύγη (A,B) .

Λύση:

Πρέπει A,B 0,1,2,3,4,5,6,7,8,9 .

2A5 2 100 10 A 5 205 10A και 13B 100 1 3 10 B 130 B .

Άρα 2A5 13B (205 10A) (130 B) 5(41 2A) (130 B) 36 .

Άρα 5(41 2A) (130 B) 36k με k Z,k 5p .

Αφού ο 2A5τελειώνει σε 5, πρέπει ο 13Bνα τελειώνει σε άρτιο.

Άρα το B 0,2,4,6,8 και διακρίνουμε περιπτώσεις:

B 0 ,

τότε 5(41 2A) 130 36 5p (41 2A) 65 18 p , αδύνατη.

Page 293: Εισαγωγη σε διαγωνιστικα μαθηματικα για το γυμνασιο

http://www.mathematica.gr/forum/viewtopic.php?f=109&t=30104

Επιμέλεια: xr.tsif Σελίδα 293

B 2 ,

Τότε 5(41 2A) 132 36 5p (41 2A) 11 3 p και αφού (11,3) 1 πρέπει

το

(41 2A) 3 , το οποίο συμβαίνει για 2,5,8 .

B 4 ,

τότε 5(41 2A) 134 36 5p (41 2A) 67 18 p , αδύνατη.

B 6 ,

τότε 5(41 2A) 136 36 5p (41 2A) 34 9 p , και αφού (34,9) 1

πρέπει το (41 2A) 9 , το οποίο συμβαίνει για 2 .

B 8 ,

τότε 5(41 2A) 138 36 5p (41 2A) 23 6 p , αδύνατη.

ΘΕΜΑ 343 (Παύλος Μαραγκουδάκης)

) Αν m,n είναι δύο μη αρνητικοί ακέραιοι με m n ορίζουμε m n να είναι το

άθροισμα όλων των ακεραίων από το mέως το n .

Να βρείτε τον θετικό ακέραιο xστις επόμενες περιπτώσεις:

α)

3x 1 3x 1 9

2x 5 2x 7 7

.

β)

3x 1 3x 1Z

2x 5 2x 7

.

Λύση:

α) Έχουμε (3x 1) (3x 1) 9 3x 1 3x 3x 1 9

(2x 5) (2x 7) 7 2x 5 2x 6 2x 7 7

Page 294: Εισαγωγη σε διαγωνιστικα μαθηματικα για το γυμνασιο

http://www.mathematica.gr/forum/viewtopic.php?f=109&t=30104

Επιμέλεια: xr.tsif Σελίδα 294

9x 9 3·3x 9 3x 9

6x 18 7 3(2x 6) 7 2x 6 7

5421x 18x 54 3x 54 x 18

3 .

β)

(3x 1) (3x 1) 3x 1 3x 3x 1 9x 3x 3 9

(2x 5) (2x 7) 2x 5 2x 6 2x 7 6x 18 2x 6 2 2(x 3)

.

Πρέπει το x 3 να διαιρεί το 9.

Άρα x 3 1 ή x 3 3 ή x 3 9 από όπου προκύπτουν οι x 2,0,6 . Από

αυτές κάνουμε δεκτή την τιμή x 6 (γιατί;).

ΘΕΜΑ 344 (Παύλος Μαραγκουδάκης)

) H ακολουθία Fibonacci είναι μια σειρά αριθμών που ξεκινά ως εξής:

1,1,2,3,5,8,13,21,... .

(κάθε αριθμός μετά τον δεύτερο είναι το άθροισμα των δύο προηγούμενων

αριθμών).

Θα συμβολίσουμε n

f τον n– οστό όρο. Για παράδειγμα 1 4 7

f 1,f 3, f 13 .

α) Ποιοι από τους παρακάτω αριθμούς38 500 2013

f , f , f είναι περιττοί;

Δικαιολογήστε την απάντησή σας.

β) Ποιοι από τους παρακάτω αριθμούς 48 1000 2014

f , f , f είναι πολλαπλάσια του 3;

Δικαιολογήστε την απάντησή σας..

Λύση:

α) Παρατηρούμε ότι κάθε όρος που είναι πολλαπλάσιο του 3, ο αριθμός που

αντιστοιχεί σε αυτόν είναι άρτιος.

Άρα, S(38) 8 3 11,S(500) 5,S(2013) 3 2 1 6 .

Page 295: Εισαγωγη σε διαγωνιστικα μαθηματικα για το γυμνασιο

http://www.mathematica.gr/forum/viewtopic.php?f=109&t=30104

Επιμέλεια: xr.tsif Σελίδα 295

Παρατηρούμε ότι ο μοναδικός όρος που είναι πολλαπλάσιο του 3είναι ο 2013.

Άρα έχουμε:

38f 2n 1 (περιττός).

500f 2m 1 (περιττός).

2013f 2l (άρτιος).

β) Παρατηρούμε ότι κάθε όρος που είναι πολλαπλάσιο του 4 , ο αριθμός που

αντιστοιχεί σε αυτόν είναι πολλαπλάσιο του 3. Τελικά βρίσκουμε ότι

πολλαπλάσιο του 4είναι οι αριθμοί 48 και 1000. Άρα έχουμε:

48f 3n (είναι πολλ. του 3).

1000f 3m (είναι πολλ. του 3).

2014f 3 u,u {1,2} (δεν είναι πολλ. του 3).

ΘΕΜΑ 345 (ΔΗΜΗΤΡΗΣ ΙΩΑΝΝΟΥ)

) Να αποδείξετε ότι ο αριθμός: 20122 2012 , είναι άρρητος. (Β Γυμνασίου)

Λύση:

Έχουμε στην παραπάνω μαθηματική σχέση: 2012 5032 2012 16 2012 . Άρα

έχουμε: 503 503(16 2012) (6 2012) (6 2) 8 .

Άρα το τελευταίο ψηφίο είναι το 8 . Όμως, το 8ποτέ δεν είναι τελευταίο ψηφίο

ενός τέλειο τετραγώνου. Επομένως, ο αριθμός πιο πάνω δεν θα είναι ποτέ

τέλειο τετράγωνο και έτσι αποδείξαμε το ζητούμενο.

Β τρόπος

Έστω ότι είναι ρητός.

Τότε θα πρέπει να υπάρχει ρητός k ώστε 20122 2012 k .

Page 296: Εισαγωγη σε διαγωνιστικα μαθηματικα για το γυμνασιο

http://www.mathematica.gr/forum/viewtopic.php?f=109&t=30104

Επιμέλεια: xr.tsif Σελίδα 296

Άρα θα ισχύει ότι 2012 22 2012 k , εδώ καταλαβαίνουμε ότι ο k είναι θετικός

ακέραιος.

Βρίσκω κατάλληλα διαδοχικά τετράγωνα ώστε ο 20122 2012 να είναι ανάμεσα

στα τετράγωνα αυτά.

Ισχύει ότι 1006 2 2012 1006 2(2 ) 2 2012 (2 1) .

Άρα ο αριθμός 20122 2012 δεν είναι τετράγωνο φυσικού γιατί είναι ανάμεσα

σε 2 διαδοχικά τετράγωνα φυσικών αριθμών οπότε καταλήγουμε σε άτοπο

γιατί δεχτήκαμε την πρόταση « 20122 2012 είναι ρητός » ως αληθής.

Επομένως ο αριθμός 20122 2012 είναι άρρητος.

ΘΕΜΑ 346 (Νίκος Αϊνστάιν)

) α) Να αποδείξετε ότι το άθροισμα 8 περιττών και 3άρτιων αριθμών είναι

άρτιος. Τι παρατηρείτε;

β) Να αποδείξετε ότι το άθροισμα 5 περιττών και 6 άρτιων αριθμών είναι

περιττός. Τι παρατηρείτε; (Β Γυμνασίου)

Λύση:

α) Αν προσθέσουμε άρτιο πλήθος περιττών αριθμών, το άθροισμα είναι άρτιος,

ενώ αν προσθέσουμε περιττό πλήθος περιττών , τότε το άθροισμα είναι

περιττός. Ενώ το άθροισμα οσονδήποτε άρτιων, είναι άρτιος.

Τέλος, είναι : ΑΡΤΙΟΣ + ΠΕΡΙΤΤΟΣ= ΠΕΡΙΤΤΟΣ.

Έτσι: Το άθροισμα 8 περιττών είναι ένας αριθμός a άρτιος και το άθροισμα 3

άρτιων είναι ένας αριθμός bάρτιος. Άρα a b άρτιος.

β) Όμοια: Το άθροισμα 5 περιττών είναι x περιττός και το άθροισμα 6 άρτιων

είναι y άρτιος. Άρα x y περιττός.

ΘΕΜΑ 347 (ΔΗΜΗΤΡΗΣ ΙΩΑΝΝΟΥ)

) Να αποδείξετε ότι για κάθε n N , ο αριθμός n n 1A 3 7 , δεν είναι τέλειο

Page 297: Εισαγωγη σε διαγωνιστικα μαθηματικα για το γυμνασιο

http://www.mathematica.gr/forum/viewtopic.php?f=109&t=30104

Επιμέλεια: xr.tsif Σελίδα 297

τετράγωνο.

Λύση:

Μία ιδέα με invariance.

Για 0

n 0 A 1 7 8 , που δεν είναι τέλειο τετράγωνο.

Αρχικά παίρνουμε όλα τα ζευγάρια φυσικών αριθμών που εκφράζονται από την

ακολουθία: n

a (1 4n,2 4n) με n N {0} .

Παρατηρούμε ότι το Aδεν αλλάζει mod 5. Αρχικά είναι A 2(mod 5) και άρα

και στο τέλος θα κρατήσει την ίδια ιδιότητα. Όμως το 2 δεν είναι τετραγωνικό

υπόλοιπο του mod 5 και άρα ο A δεν είναι τέλειο τετράγωνο γι αυτά τα n .

Τώρα παίρνουμε όλα τα ζευγάρια φυσικών αριθμών που εκφράζονται από την

ακολουθία: n

b (3 4n,4 4n) με n N {0} .

Και πάλι παρατηρούμε ότι το A δεν αλλάζει mod 5. Αρχικά είναι

A 3(mod 5) και άρα και στο τέλος θα κρατήσει την ίδια ιδιότητα. Όμως το 3

δεν είναι τετραγωνικό υπόλοιπο του mod 5 και άρα ο A δεν είναι τέλειο

τετράγωνο γι αυτά τα n .

Μένει τώρα να δείξουμε ότι η ένωση των στοιχείων των δυο ακολουθιών δίνει

το σύνολο των φυσικών αριθμών.

Πράγματι παίρνοντας mod 4 βλέπουμε ότι οι δυο ακολουθίες

επαναλαμβάνονται με περίοδο T 4 . Δηλαδή:

1 4n 1(mod 4) , 2 4n 2(mod 4) , 3 4n 3(mod 4) , 4 4n 0(mod 4) και

τελικά: n n

a b N .

ΘΕΜΑ 348 (ΔΗΜΗΤΡΗΣ ΙΩΑΝΝΟΥ)

) Να αποδείξετε ότι ο αριθμός 2012n 2012 , είναι άρρητος, για κάθε n N .

Page 298: Εισαγωγη σε διαγωνιστικα μαθηματικα για το γυμνασιο

http://www.mathematica.gr/forum/viewtopic.php?f=109&t=30104

Επιμέλεια: xr.tsif Σελίδα 298

Λύση:

Για n 1 φανερό.

Για n 2 δείχνουμε ότι η ρίζα είναι γνήσια μεταξύ δύο διαδοχικών ακεραίων,

1006 2012 1006n n 2012 n 1 , οπότε η υπόριζη ποσότητα δεν μπορεί να είναι

τέλειο τετράγωνο.

Η αριστερή ανισότητα είναι άμεση. Για την δεξιά έχουμε

2012 2012 2012 11 2012 1006n 2012 n 2048 n 2 n 2·2 1

22012 1006 1006n 2n 1 n 1 , και λοιπά.

Β τρόπος

Ο αριθμός 4n λήγει σε 0 ή 1 ή 5 ή 6 και άρα και ο αριθμός 4 503(n ) λήγει

επίσης σε 0 ή 1 ή 5 ή 6. Άρα ο αριθμός 4 503(n ) 2012 , θα λήγει σε 2 ή 3 ή 7

ή 8 ,

και άρα δεν μπορεί να είναι τέλειο τετράγωνο ακεραίου, και το ζητούμενο

εδείχθη.

Γ τρόπος

Για n 1 έχουμε: 2012n 1 1 1 2 , που δεν είναι τέλειο τετράγωνο.

Για n 2 αρκεί να αποδείξουμε ότι: 1006 2 1006 2 1006 2(n ) (n ) 2012 (n 1) .

Η πρώτη ανισότητα είναι προφανής, αφού 1006 2(n ) 0 .

Το τρίτο μέλος της σχέσης γίνεται: 1006 2 1006 2 1006(n 1) (n ) 2n 1 . Όμως, από

την αρχική υπόθεση έχουμε ότι n 2 , έτσι 10062n 1 2012 , άρα αποδείξαμε το

ζητούμενο.

ΘΕΜΑ 349 (ΔΗΜΗΤΡΗΣ ΙΩΑΝΝΟΥ)

) Να βρεθεί ο θετικός ακέραιος x , αν είναι γνωστό, ότι ο αριθμός 29x 5x 4

είναι τέλειο τετράγωνο. (Γ Γυμνασίου)

Page 299: Εισαγωγη σε διαγωνιστικα μαθηματικα για το γυμνασιο

http://www.mathematica.gr/forum/viewtopic.php?f=109&t=30104

Επιμέλεια: xr.tsif Σελίδα 299

Λύση:

Θα διακρίνουμε περιπτώσεις:

x 3 .

Τότε: 2 2(3x) 9x 5x 4 και

2 29x 5x 4 (3x 1) .

Άρα: 2 2 2(3x) 9x 5x 4 (3x 1) .

Επομένως, ο αριθμός θα βρίσκεται σε δύο διαδοχικά τέλεια τετράγωνα και άρα

δεν μπορεί να είναι τέλειο τετράγωνο.

x 3 .

Τότε: 2 2(3x 1) 9x 5x 4 και

2 29x 5x 4 (3x 2) .

Άρα: 2 2 2(3x 1) 9x 5x 4 (3x 2) .

Επομένως, ο αριθμός βρίσκεται ανάμεσα σε δύο διαδοχικά τέλεια τετράγωνα

και άρα δεν μπορεί να είναι τέλειο τετράγωνο.

x 3 .

Τότε: 2 29x 5x 4 81 15 4 100 10 .

Επομένως, ο αριθμός είναι τέλειο τετράγωνο και άρα x 3 .

Β τρόπος

Μια άλλη λύση, που βασίζεται στη μέθοδο συμπλήρωσης τετραγώνου:

Το πρόβλημα είναι ισοδύναμο με την επίλυση της διοφαντικής εξίσωσης 2 29x 5x 4 k στο σύνολο των θετικών ακεραίων.

Η εξίσωση είναι ισοδύναμη με τις ακόλουθες:

2 2

2 2 25 5 25 5 1193x 2·3x· 4 k 3x k

6 6 36 6 36

2 218x 5 119 36k 6k 18x 5 6k 18x 5 119.

Page 300: Εισαγωγη σε διαγωνιστικα μαθηματικα για το γυμνασιο

http://www.mathematica.gr/forum/viewtopic.php?f=109&t=30104

Επιμέλεια: xr.tsif Σελίδα 300

Ο δεύτερος παράγοντας του πρώτου μέλους είναι σίγουρα θετικός, οπότε και ο

πρώτος είναι θετικός. Από την άλλη, το 119γράφεται ως γινόμενο δύο θετικών

παραγόντων μόνο με δύο τρόπους: 119 1·119 7·17 .

Είναι 6k 18x 5 6k 18x 5 . (Πράγματι είναι ισοδύναμη με 5

x18

που

ισχύει.)

Υπάρχουν λοιπόν δύο περιπτώσεις:

6k 18x 5 1 και 6k 18x 5 119 .

Εύκολα βρίσκουμε k,x 10,3 .

6k 18x 5 7 και 6k 18x 5 17 .

Εύκολα βλέπουμε ότι δίνει x 0 , που απορρίπτεται.

ΘΕΜΑ 350 (ΔΗΜΗΤΡΗΣ ΙΩΑΝΝΟΥ)

) Να λυθεί η εξίσωση:

2x 1 4x 2 9x 3 16x 4 2012 x 2012... 1006 2013

2 3 4 5 2013

.

(Β Γυμνασίου)

Λύση:

Ισχύει: n(n 1)

1 2 3 ... n2

.

Εύκολα βρίσκουμε ότι 1003·2013 1 2 3 ... 2012 .

Επομένως:

2x 1 4x 2 9x 3 16x 4 2012 x 2012... 1006·2013

2 3 4 5 2013

Page 301: Εισαγωγη σε διαγωνιστικα μαθηματικα για το γυμνασιο

http://www.mathematica.gr/forum/viewtopic.php?f=109&t=30104

Επιμέλεια: xr.tsif Σελίδα 301

2x 1 4x 2 9x 3 16x 4 2012 x 20121 2 3 4 ... 2012 0

2 3 4 5 2013

2 2x 1 4x 4 9x 9 16x 16 2012 x 2012... 0

2 3 4 5 2013

21 4 3 4 2012(x 1) ... 0

2 3 4 5 2013

.

Επομένως: x 1 0 x 1 .

Β τρόπος

Παρατηρούμε ότι 2012·2013

1006·20132

.

Όμως, 2012·2013

1 2 3 4 ... 20122

.

Άρα, θα ισχύει και ότι 2x 1 4x 2 9x 3 16x 4 2012 x 2012

... 1 2 3 4 ... 20122 3 4 5 2013

.

Αφού και τα δύο μέλη της εξίσωσης αποτελούνται από 2012 όρους, έπεται ότι

x 11 x 1 2 x 1

2

.

Αν αντικαταστήσουμε, η εξίσωση επαληθεύεται.

Καλύτερα θα μπορούσαμε να πούμε ότι επειδή η εξίσωση είναι πρωτοβάθμια

και εύκολα φαίνεται ότι δεν είναι ταυτότητα, άρα θα έχει μια το πολύ λύση.

Και επειδή παρατηρούμε ότι για x 1 επαληθεύεται, άρα αυτή είναι και η

μοναδική της λύση.

ΘΕΜΑ 351 (ΔΗΜΗΤΡΗΣ ΙΩΑΝΝΟΥ)

) Αν

1 2 3 2013a ,a ,a ,...,a είναι περιττοί αριθμοί, να αποδείξετε ότι ο αριθμός

2 2 2 2

1 2 3 2013A a a a ... a 2 είναι άρρητος. (Γ Γυμνασίου)

Page 302: Εισαγωγη σε διαγωνιστικα μαθηματικα για το γυμνασιο

http://www.mathematica.gr/forum/viewtopic.php?f=109&t=30104

Επιμέλεια: xr.tsif Σελίδα 302

Λύση:

Όλα τα περιττά τετράγωνα είναι 1 mod4 Άρα το υπόριζο είναι

2011 mod4 3 mod4 άρα δεν είναι τέλειο τετράγωνο κι έτσι ο αριθμός

είναι άρρητος.

Β τρόπος

Θα δώσω μια πιο απλή λύση, χωρίς την χρήση της θεωρίας Modulo. Έχουμε ότι

1 1 2 2 2013 2013a 2k 1,a 2k 1,...,a 2k .

Έτσι, έχουμε στην παράσταση:

2 2 2 2 2 2 2

1 2 3 2013 1 2 2013A a a a ... a 2 (2k 1) (2k 1) ... (2k 1) 2

2 2 2 2

1 1 2 2 3 3 2013 20134k 4k 1 4k 4k 1 4k 4k 1 ... 4k 4k 1

2 2 2

1 1 2 2 2013 20134(k k k k ... k k ) 2013 2

4m 2011 4m 2008 3 4m 4·502 3 4n 3 .

Όμως, εμείς γνωρίζουμε (δεν γράφω απόδειξη, διότι υπάρχει σε προηγούμενες

ασκήσεις) ότι τα τέλεια τετράγωνα δεν παίρνουν την παραπάνω μορφή. Έτσι, η

ρίζα δεν απλοποιείτε και επομένως ο αριθμός είναι άρρητος. Άρα, αποδείξαμε

το ζητούμενο.

ΘΕΜΑ 352 (ΔΗΜΗΤΡΗΣ ΙΩΑΝΝΟΥ)

)

Αν *m,n N , και αν οι αριθμοί: 2 2

2 2

m 2n n 2m,

n 2m m 2n

είναι ακέραιοι, να

αποδειχθεί ότι: 2 m n 2 , (δηλαδή m n 2 ). (Γ Γυμνασίου)

Λύση:

Είναι 2 2 2m 2n n 2m n 2m οπότε (m n)(m n 2) 0 ή m n 2 .

Page 303: Εισαγωγη σε διαγωνιστικα μαθηματικα για το γυμνασιο

http://www.mathematica.gr/forum/viewtopic.php?f=109&t=30104

Επιμέλεια: xr.tsif Σελίδα 303

Είναι 2 2 2n 2m m 2n m 2n οπότε (m n)(n m 2) 0 ή m n 2 .

ΘΕΜΑ 353 (ΔΗΜΗΤΡΗΣ ΙΩΑΝΝΟΥ)

) Αν a,b,c Z και αν: 2 2 2a b c 1,b c a 1,c a b , να βρεθούν όλες οι

δυνατές τιμές των a,b,c. (Γ Γυμνασίου)

Λύση:

Προσθέτουμε και έχουμε 2 2 2(a 1) (b 1) (c 1) 1 οπότε έχουμε τις

δυνατότητες:

2 2 2(a 1) (b 1) (c 1) 0 ,

οπότε a b c 1 , που είναι λύση.

2 2 2(a 1) (b 1) (c 1) 1 ,

οπότε προκύπτουν οι τριάδες (a,b,c) (1,1,0), (1,1,2) και οι μεταθέσεις τους,

που όμως δεν είναι λύσεις.

ΘΕΜΑ 354 (Παύλος Μαραγκουδάκης)

)

Να λυθεί στο σύνολο των ακεραίων η εξίσωση 1 1

x y 55 5

.

(Γ Γυμνασίου)

Λύση:

Αφού οι αριθμοί x,y είναι ακέραιοι, τότε πρέπει x,y 1 .

H δοσμένη εξίσωση γράφεται: 5x 1 5y 1 5 .

Θέτοντας A 5x 1,B 5y 1 , (όπου A,B 0 ), έχουμε:

A B 5 A B 2 AB 25 2 AB 25 A B και επομένως

πρέπει

Page 304: Εισαγωγη σε διαγωνιστικα μαθηματικα για το γυμνασιο

http://www.mathematica.gr/forum/viewtopic.php?f=109&t=30104

Επιμέλεια: xr.tsif Σελίδα 304

25 A B 0 A B 25 5x 1 5y 1 25 5x 5y 27 x y 5 .

Άρα (και με δεδομένο ότι όπως είδαμε είναι x,y 1 ), θα πρέπει

(x,y) {(1,1),(1,2),(1,3),(1,4),(2,1),(2,2),(2,3),(3,1),(3,2),(4,1)} και η δοσμένη

εξίσωση επαληθεύεται όταν x 1,y 2 ή x 2,y 1 .

ΣΗΜΕΙΩΣΗ:

Υπάρχουν και διάφοροι άλλοι τρόποι λύσης της άσκησης αυτής

π.χ 5x 1 5 5y 1 5x 1 25 5y 1 10 5y 1

5x 5y 27 10 5y 15 .

Άρα πρέπει 2y 1 m , m . Όμοια πρέπει 25x 1 n , n. Τότε έχουμε :

m n 5 , όπου m,n 1 . Άρα (m,n) {(1,4),(2,3),(3,2),(4,1)} , από όπου

δεκτές είναι μόνο οι τιμές m 2,n 3 ή m 3,n 2 , από όπου και βρίσκουμε

τις λύσεις x 2,y 1 και x 1,y 2 .

Β τρόπος

Προφανώς πρέπει να ισχύει 1 x,1 y . Θεωρούμε x y 1 , οπότε

χρησιμοποιώντας συζυγή παράσταση παίρνουμε:

5x 1 5y 1 5 5x 1 5y 1 x y .

Αφαιρώντας κατά μέλη έχουμε: 5 x y 2 5y 1 4 0 x y 1 .

Αν 29

x y 0 x y x Z20

.

Αν x y 1 x y 1 , παίρνουμε: 1 5y 1 5y 4 y 1, x 2 .

Τελικά λόγω της συμμετρίας το σύνολο λύσεων είναι το σύνολο 1,2 , 2,1 .

Page 305: Εισαγωγη σε διαγωνιστικα μαθηματικα για το γυμνασιο

http://www.mathematica.gr/forum/viewtopic.php?f=109&t=30104

Επιμέλεια: xr.tsif Σελίδα 305

ΘΕΜΑ 355 (Παύλος Μαραγκουδάκης)

) Να λυθεί στο σύνολο των ακεραίων η εξίσωση

2 2 2 2x xy y x y .

(Γ Γυμνασίου)

Υπόδειξη

1) 2(x y) xy(xy 1) και μετά d (xy,xy 1) κλπ.

2) Μια άλλη ιδέα, πιο κοντά σε αυτά που συζητήθηκαν πρόσφατα:

Προσθέτουμε και στα δύο μέλη το xy .

Κατόπιν πολλαπλασιάζουμε κατά μέλη με 4και κάνουμε συμπλήρωση

τετραγώνου στο δεύτερο μέλος.

Οδηγούμαστε σε ωραία παραγοντοποίηση.

Λύση:

2(x y) xy(xy 1) και μετά d (xy,xy 1) .

Τότε d | xy και d | xy 1 , οπότε d | xy 1 xy d |1 d 1 .

Άρα οι αριθμοί xy,xy 1 είναι πρώτοι προς αλλήλους και από εδώ έπεται ότι 2xy m και 2xy 1 και η συνέχεια είναι απλή.

Χρησιμοποιήθηκε ένα 'λήμμα' παραπάνω. Εντοπίστε το και αποδείξτε το!

Β τρόπος

Η δοσμένη εξίσωση γράφεται: 2 2 2(1 y )x yx y 0 .

1η Περίπτωση: 2 0 y 11 y .

Τότε παίρνουμε τις λύσεις (x,y) (1, 1) και (x,y) ( 1,1) .

2η Περίπτωση: y 1 .

Page 306: Εισαγωγη σε διαγωνιστικα μαθηματικα για το γυμνασιο

http://www.mathematica.gr/forum/viewtopic.php?f=109&t=30104

Επιμέλεια: xr.tsif Σελίδα 306

Τότε για να έχει το παραπάνω τριώνυμο ρίζες πραγματικές, θα πρέπει 2 2 2 2 20 y 4y (1 y ) 0 y (4y 3) 0 .

Αν y 0 ,

τότε έχουμε την επί πλέον λύση (x,y) (0,0) .

Αν y 0 ,

τότε πρέπει 2 24y 3 k , όπου k Z .

Άρα: (2y k)(2y k) 3 , από όπου προκύπτει ότι: 2y k 3,2y k 1 ή

2y k 1,2y k 3 ή 2y k 3,2y k 1 ή 2y 3 1,2y k 3 .

Στις παραπάνω περιπτώσεις, με πρόσθεση κατά μέλη παίρνουμε y 1 , που

είναι άτοπο. Τελικά έχουμε: (x,y) {(0,0),(1, 1),( 1,1)} .

Γ τρόπος

Παρατηρούμε ότι x 0 y 0 , οπότε μία προφανής λύση είναι η (0,0) .

x 0 και y x x 0 , άτοπο.

Αν x,y 0 και x y , από την αρχική και με την υπόθεση ότι έχει ακέραια

λύση παίρνουμε 3 3 3 2 2 3 3 2 3 2x y x y x y x 1 y y 1 x .

Από εδώ οδηγούμαστε στην

2 21 x 0 1 y 0 , οπότε δύο πιθανές λύσεις είναι οι (1, 1),( 1,1) ή

2 21 x 0, 1 y 0 και 3 2 3 2x ,x 1 y ,y 1 1 , που είναι άτοπο.

Τελικά επαληθεύοντας εύκολα διαπιστώνουμε ότι το σύνολο των λύσεων είναι

το 0,0 , 1,1 , 1, 1 .

ΘΕΜΑ 356 (Παύλος Μαραγκουδάκης)

) Υποθέτουμε ότι οι

1 2 1990x ,x ,...,x είναι θετικοί ακέραιοι αριθμοί ώστε

2 2 2 2

1 2 1989 1990x x ... x x .

Page 307: Εισαγωγη σε διαγωνιστικα μαθηματικα για το γυμνασιο

http://www.mathematica.gr/forum/viewtopic.php?f=109&t=30104

Επιμέλεια: xr.tsif Σελίδα 307

α) Να αποδείξετε ότι τουλάχιστον δύο από αυτούς είναι άρτιοι.

β) Να βρείτε παράδειγμα τέτοιων αριθμών. (Γ Γυμνασίου)

Λύση:

Θα βάλω το (α) ερώτημα μόνο αν και δεν είμαι σίγουρος για την ορθότητά

του...

Θα αποδείξουμε ότι δεν ισχύει όταν όλοι οι αριθμοί είναι περιττοί ή όταν ένας

μόνο είναι άρτιος. Διακρίνουμε τις περιπτώσεις:

Όλοι οι αριθμοί είναι περιττοί.

Γράφουμε τότε: 1 1 2 2 1990 1990

x 2k 1,x 2k 1,...,x 2k 1 . Είναι:

2 2 2 2 2 2 2

1 2 1989 1990 1 1 2 2 1989 1989x x ... x x 4(k k k k ... k k ) 1989

2

1990 19904k 4k 1

1 1 2 2 1989 1989 1990 1990k (k 1) k (k 1) ... k (k 1) 497 k (k 1) .

Το πρώτο μέλος όμως είναι περιττό ενώ το άλλο άρτιο, άτοπο.

Ένας μόνο αριθμός είναι άρτιος.

Γράφουμε τότε: 1 1 2 2

x 2k ,x 2k 1 , κτλ. Είναι:

2 2 2 2 2 2 2

1 2 1989 1990 1 2 2 1989 1989x x ... x x 4(k k k ... k k ) 1988

2

2 2 2 2 1990

1990 1990 1 2 2 1989 1989

2k 14k 4k 1 k k k ... k k 497

2

.

Το πρώτο μέλος όμως είναι ακέραιος ενώ το άλλο δεν είναι, άτοπο.

Υ.Γ. Δεν ξέρω αν πρέπει να πάρουμε και την περίπτωση ο 1990

x να είναι άρτιος.

Πάντως, έχω απόδειξη και γι' αυτό.

Β τρόπος

α) Έστω κανένας άρτιος. Παίρνοντας mod8 έχουμε 1989 1 mod8 ,

άτοπο. (είναι 2a 1 mod8 για περιττό a ).

Page 308: Εισαγωγη σε διαγωνιστικα μαθηματικα για το γυμνασιο

http://www.mathematica.gr/forum/viewtopic.php?f=109&t=30104

Επιμέλεια: xr.tsif Σελίδα 308

Έστω ένας είναι άρτιος.

Τότε εύκολα το ένα μέλος θα είναι άρτιος και το άλλο περιττός. Άτοπο.

Άρα δύο τουλάχιστον είναι άρτιοι.

β) Βάζουμε 1 2 1988

x x ... x 1 , οπότε 1990 1989 1990 1989

1988 (x x )(x x ) και μια

λύση είναι 1990 1989

x 498,x 496 .

ΘΕΜΑ 357 (ΔΗΜΗΤΡΗΣ ΙΩΑΝΝΟΥ)

) Αν

1 2 2013a ,a ,...,a R , με

1 2a 3,a 4 και γενικά

na (n 2) ( 2) na n ,

για κάθε n 1,2,3,...,2013 και αν: 1 2 3 2013

1 1 1 1A ...

a 3 a 4 a 5 a 2015

και 1 2 3 2013

1 2 3 2013

a 2 a 3 a 4 a 2014 2013...

a 3 a 4 a 5 a 2015 2014

, να αποδείξετε ότι ο

αριθμός 2014A, είναι τέλειο τετράγωνο. (Β Γυμνασίου)

ΣΗΜΕΙΩΣΗ (Εισαγωγή στην έννοια του αθροίσματος)

Η παραπάνω άσκηση, μπορεί να διατυπωθεί και με τους εξής συμβολισμούς:

Αν n

a R με n

a (n 2) , για κάθε n 1,2,3,...,2013 και αν 2013

n 1n

1A

a n 2

και 2013

n

n 1n

a n 1 2013

a n 2 2014

, να αποδείξετε ότι ο αριθμός 2014A είναι τέλειο

τετράγωνο.

Λύση:

Είναι:

2013 2013n n

n 1 n 1n n

a n 1 2013 a n 2 1 2013

a n 2 2014 a n 2 2014

Page 309: Εισαγωγη σε διαγωνιστικα μαθηματικα για το γυμνασιο

http://www.mathematica.gr/forum/viewtopic.php?f=109&t=30104

Επιμέλεια: xr.tsif Σελίδα 309

22013 2013

n 1 n 1n n

1 2013 1 20132013

a n 2 2014 a n 2 2014

.

Επομένως:2013

2 2

n 1n

12014 2013 2014A 2013

a n 2

.

Έτσι, αποδείξαμε το ζητούμενο.

ΘΕΜΑ 358 (ΔΗΜΗΤΡΗΣ ΙΩΑΝΝΟΥ)

) Αν

*x,y,z N και αν: 2013 2013x y z xyz , να βρεθούν οι αριθμοί x,y,z .

(Γ Γυμνασίου)

ΣΗΜΕΙΩΣΗ:

Αν A,B φυσικοί αριθμοί τότε για να είναι ο αριθμός A

B φυσικός, είναι

απαραίτητο να ισχύει A 0 ή B A .

Γενικότερα, ανA,B ακέραιοι, τότε για να είναι ο αριθμός A

B ακέραιος, είναι

απαραίτητο να ισχύει A 0 ή B A .

Λύση:

Είναι: 2013 2013 2013 2013 x y

x y z xyz x y z (xy 1) zxy 1

.

Αφού z N θα είναι και 2013z N . Επομένως, σύμφωνα με την παρατήρηση

έχουμε: x y 0 x y , άτοπο αφού x,y N ,

ή xy 1 x y xy x y 1 2 (x 1)(y 1) 2 .

Στη συνέχεια, δεδομένου ότι *x,y N , θα πρέπει (x 1)(y 1) 0 ή

(x 1)(y 1) 1 ή (x 1)(y 1) 2 .

Τελικά πρέπει, ελέγχοντας τις περιπτώσεις να πάρουμε τις τριάδες:

Page 310: Εισαγωγη σε διαγωνιστικα μαθηματικα για το γυμνασιο

http://www.mathematica.gr/forum/viewtopic.php?f=109&t=30104

Επιμέλεια: xr.tsif Σελίδα 310

(x,y,z) (2,3,1) , (x,y,z) (3,2,1) .

ΘΕΜΑ 359 (Παύλος Μαραγκουδάκης)

)

Να λυθεί στους θετικούς ακεραίους η εξίσωση 3x 1 2 y

2(x 1) 3 y 1

.

(Γ Γυμνασίου)

Λύση:

Είναι:

3x 1 2 y 3(x 1) 2 2 y 3 1 2 y

2(x 1) 3 y 1 2(x 1) 3 y 1 2 x 1 3 y 1

y 1 5 xy y y 1 56xy 12y 6 5xy 5y 5x 5

y 1 x 1 6 xy y x 1 6

xy 7y 5x 1 0 xy 5x 7y 1 0 xy y 5x 5 6y 6 0

y(x 1) 5(x 1) 6y 6 0 (5 y)(x 1) 6(5 y) 30 6 0

(5 y)(x 7) 36 .

Είναι όμως: x 0 x 7 7 και y 0 y 0 5 y 5 .

Επομένως παίρνουμε τα συστήματα(τα αντίστοιχα και με αρνητικούς που

δίνουν όμως τις ίδιες λύσεις):

5 y 1

x 7 36

,

5 y 2

x 7 18

,

5 y 3

x 7 12

,

5 y 4

x 7 9

Εύκολα τώρα παίρνουμε τις λύσεις: (x,y) {(29,4),(11,3),(5,2),(2,1)} .

ΘΕΜΑ 360 (Παύλος Μαραγκουδάκης)

) Υπάρχουν ακέραιοι x,y,z ώστε 2 2 2x 3y 4z 2014 ; (Γ Γυμνασίου)

Page 311: Εισαγωγη σε διαγωνιστικα μαθηματικα για το γυμνασιο

http://www.mathematica.gr/forum/viewtopic.php?f=109&t=30104

Επιμέλεια: xr.tsif Σελίδα 311

Λύση:

Ο 24z θα είναι άρτιος ανεξάρτητα από το z .

Επομένως, θα διακρίνουμε περιπτώσεις για το x . Είναι:

Αν x 2k .

Τότε αναγκαστικά θα είναι και y 2m . Έχουμε:

2 2 2 2 2 2(2k) 3(2m) 4z 2014 4k 12m 4z 2014

2 2 24(k 3m z ) 2014 .

Από την παραπάνω παίρνουμε ότι 4| 2014, άτοπο.

Αν x 2k 1 .

Τότε αναγκαστικά θα είναι και y 2m 1 . Έχουμε:

2 2 2 2 2 2(2k 1) 3(2m 1) 4z 2014 4k 4k 1 3(4m 4m 1) 4z 2014 2 2 2 2 2 24k 4k 4m 4m 4z 2010 4(k k m m z ) 2010 .

Από την παραπάνω παίρνουμε ότι 4| 2010, άτοπο.

Επομένως, δεν υπάρχουν ακέραιοι που να ικανοποιούν αυτήν την εξίσωση.

ΘΕΜΑ 361 (Παύλος Μαραγκουδάκης)

) Βρείτε τους φυσικούς x y z ώστε x y z3 3 3 21897 . (Γ Γυμνασίου)

Λύση:

Αφού x y z , θέτουμε y x a και z x a b για κάποιους a,b N .

Τότε:

x y z x x a x a b x a b3 3 3 21897 3 3 3 21897 3 [3 (3 1) 1] 21897 .

Διαιρώντας διαδοχικά με το 3 θα πάρουμε: x 3 a b3 [3 (3 1) 1] 811 .

Page 312: Εισαγωγη σε διαγωνιστικα μαθηματικα για το γυμνασιο

http://www.mathematica.gr/forum/viewtopic.php?f=109&t=30104

Επιμέλεια: xr.tsif Σελίδα 312

Επομένως, θα πρέπει να ισχύει x 3 0 x 3 .

Έτσι, η εξίσωση γίνεται: a b a b3 (3 1) 1 811 3 (3 1) 810 .

Διαιρώντας ξανά με το 3 θα φτάσουμε: a 4 b3 (3 1) 10 .

Έτσι, a 4 y 7 .

Τώρα: b b3 1 10 3 9 b 2 z 9 .

Επομένως (x,y,z) (3,7,9) που επαληθεύουν την αρχική.

ΘΕΜΑ 362 (ΔΗΜΗΤΡΗΣ ΙΩΑΝΝΟΥ)

) Να δείξετε ότι δεν υπάρχουν ακέραιοι x,y , τέτοιοι ώστε να επαληθεύουν την

εξίσωση: 2 2x 2y 8z 3 0 . (Γ Γυμνασίου)

Λύση:

Η εξίσωση γίνεται 2 2 2 2x 2y 8z 3 0 x 8z 2y 3 .

Το δεύτερο μέλος είναι περιττός, άρα αναγκαστικά x 2k 1 .

Επομένως, έχουμε 2 2 2 2 2 2x 8z 2y 3 (2k 1) 8z 2y 3 4k 4k 8z 1 2y 3

2 2 2 24k 4k 8z 2y 2 2k 2k 4z y 1 .

Αφού το πρώτο μέλος είναι άρτιος, πρέπει y 2 1 .

Και έτσι η εξίσωση γίνεται 2 2 2 22k 2k 4z y 1 2k 2k 4z (2 1) 1

2 2 2 22k 2k 4z 4 4 2 k k 2z 2 2 1

2k(k 1) 2z 2( ) 1 .

Προφανώς,

• k(k 1) 2m , ως γινόμενο δύο διαδοχικών αριθμών.

Page 313: Εισαγωγη σε διαγωνιστικα μαθηματικα για το γυμνασιο

http://www.mathematica.gr/forum/viewtopic.php?f=109&t=30104

Επιμέλεια: xr.tsif Σελίδα 313

• 2n .

Άρα, η εξίσωση παίρνει την μορφή 2k(k 1) 2z 2( ) 1 2m 2z 2n 1 2(m z) 2n 1 .

Όμως, το πρώτο μέλος είναι άρτιος και το δεύτερο περιττός (άτοπο). Άρα, δεν

υπάρχουν αριθμοί που να ικανοποιούν την δοσμένη εξίσωση.

ΘΕΜΑ 363 (ΔΗΜΗΤΡΗΣ ΙΩΑΝΝΟΥ)

) Αν x,y Z , να λυθεί η εξίσωση: 2 2x y 4 x . (Β,Γ Γυμνασίου)

Λύση:

Πρέπει 2 24 x 0 x 4 x 2 2 x 2 .

Διακρίνουμε τις περιπτώσεις για το x :

Για x 0 :

2 2x y 4 x 0 y 4 0 y 4 y 16 y 16 .

Για x 1 ή x 1 :

2 2x y 4 x 1 y 4 1 1 y 3 1 y 9 y 9 1 8 .

Για x 2 ή x 2 :

2 2x y 4 x 4 y 4 4 4 y 0 4 y 0 y 4 .

Για να ισχύει η ρίζα, πρέπει 2x y και τελικά όλες οι 5 λύσεις είναι αποδεκτές.

ΘΕΜΑ 364 (ΔΗΜΗΤΡΗΣ ΙΩΑΝΝΟΥ)

) Αν x,y,z R με (x y)(y z)(z x) 0 , και αν ισχύει ότι :

x y z1

x y y z z x

, τότε να αποδείξετε ότι:

y z x2

x y y z z x

.

Page 314: Εισαγωγη σε διαγωνιστικα μαθηματικα για το γυμνασιο

http://www.mathematica.gr/forum/viewtopic.php?f=109&t=30104

Επιμέλεια: xr.tsif Σελίδα 314

(Β, Γ Γυμνασίου)

Λύση:

Ας υποθέσουμε ότι η αποδεικτέα σχέση ισχύει.

Τότε, αν την προσθέσουμε κατά με την συνθήκη, λαμβάνουμε:

x y z y z x3

x y y z z x x y y z z x

x y y z z x3 1 1 1 3 3 3

x y y z z x

, που ισχύει.

Άρα, θα ισχύει και η αποδεικτέα σχέση.

ΘΕΜΑ 365 (Παύλος Μαραγκουδάκης)

)

Να βρεθούν οι θετικοί ακέραιοι x,y,z ώστε 2x 4y x z

2x 1 2y 3 z 4

.

Λύση:

Έστω ότι 2x 4y x z

k2x 1 2y 3 z 4

, όπου kένας θετικός πραγματικός

αριθμός.

Ας υποθέσουμε ότι k 1 .

Τότε, πρέπει, 2x 2x 1 0 1 , άτοπο.

Ας υποθέσουμε τώρα ότι k 1 .

Τότε, πρέπει 3

2y 3 4y 3 2y y y 12

, αφού είναι θετικός

ακέραιος.

Page 315: Εισαγωγη σε διαγωνιστικα μαθηματικα για το γυμνασιο

http://www.mathematica.gr/forum/viewtopic.php?f=109&t=30104

Επιμέλεια: xr.tsif Σελίδα 315

Για y 1 , έχουμε ότι

2x 4y 2x 410x 8x 4 2x 4 x 2

2x 1 2y 3 2x 1 5

.

Για y 1 και x 2 , έχουμε ότι

x z 4y 2 z 44(z 4) 5(z 2)

z 4 2y 3 z 4 5

4z 16 5z 10 z 6 .

Άρα έχουμε την λύση (x,y,z) (1,2,6) , που επαληθεύει την αρχική εξίσωση.

ΘΕΜΑ 366 (ΔΗΜΗΤΡΗΣ ΙΩΑΝΝΟΥ)

)

α) Να αποδείξετε ότι για κάθε *n N , ισχύει ότι: 1 1 1

n(n 1) n n 1

.

β) Να αποδείξετε (χρησιμοποιώντας το (α) ή με άλλο τρόπο) ότι:

2 2 2 2

1 1 1 1 11...

1 2 3 6 6 . (Γ Γυμνασίου)

Λύση:

α) Στο δεύτερο μέλος της εξίσωσης έχουμε

1 1 n 1 n 1RHS

n n 1 n(n 1) n(n 1)

, που ισχύει.

β) Παρατηρούμε ότι 2 2 2 2

1 1 1 1 1 1 1... 1 ...

1 2 3 6 4 9 6

1 1 1 1 1 1 1 3 1 1 1 2 11 1 1 1

4 9 12 12 12 4 9 12 4 9 4 4 9

18 9 2 29

18 18

.

Page 316: Εισαγωγη σε διαγωνιστικα μαθηματικα για το γυμνασιο

http://www.mathematica.gr/forum/viewtopic.php?f=109&t=30104

Επιμέλεια: xr.tsif Σελίδα 316

Αρκεί να αποδείξουμε ότι 11 29 33 29

6 18 18 18 , που ισχύει.

ΘΕΜΑ 367 (Παύλος Μαραγκουδάκης)

) Ένας τριψήφιος αριθμός συμβολίζεται συνήθως ως abc όπου a,b,c είναι

αντιστοίχως τα ψηφία των εκατοντάδων, των δεκάδων και των μονάδων. Να

βρεθούν τα ψηφία a,b,c ώστε 10 6

abc a b c a b c 1 .

Λύση:

Αν θέσουμε k a b c , τότε η δοσμένη σχέση γίνεται:

10 6 6 4abc k k 1 abc k (k 1) 1 , παρατηρούμε ότι 10k 0 καθώς και

6k 0 , άρα θα θεωρήσουμε τον kθετικό ακέραιο ή μηδέν.

Διακρίνουμε τις περιπτώσεις για το k :

Αν k 3 ,

παρατηρούμε ότι το αποτέλεσμα δεν είναι τριψήφιος.

Αν k 2 ,

η σχέση γράφεται 64 5 1 961 , άρα μια λύση είναι η (a,b,c) (9,6,1) .

Αν k 1 ,

τότε η παράσταση γράφεται 1 0 1 1 , άτοπο.

Αν k 0 ,

τότε έχουμε 0 ( 1) 1 1 , άτοπο.

ΘΕΜΑ 368 (Παύλος Μαραγκουδάκης)

)

Page 317: Εισαγωγη σε διαγωνιστικα μαθηματικα για το γυμνασιο

http://www.mathematica.gr/forum/viewtopic.php?f=109&t=30104

Επιμέλεια: xr.tsif Σελίδα 317

Βρείτε τους διψήφιους αριθμούς ab,cd,ef ώστε ab ef 4cd και

2

ab ef 100cd .

Λύση:

Στην πρώτη σχέση έχουμε

10a b 10e f 4(10c d) 10a 10e b f 40c 4d

10(a e 4c) (b f 4d) 0 .

Αφού τα ψηφία είναι θετικά ή μηδέν, τότε αναγκαστικά

a e 4c 0 a e 4c

b f 4d 0 b f 4d

. Άρα πρέπει c 1,2 και d 0,1,2 .

Στην δεύτερη σχέση έχουμε 2 2

ab ef 100cd ab 100cd ef .

Παρατηρούμε ότι αν c 1 , τότε a 3 και αν c 2 , τότε a 4 , αλλιώς

καταλήγουμε σε άτοπο.

Επομένως, βρίσκουμε ότι e 2 σε κάθε περίπτωση.

Διακρίνουμε τις περιπτώσεις για τα c,d , χρησιμοποιώντας τον περιορισμό

e 2 και b f 4d .

ΘΕΜΑ 369 (Socrates)

)

Να λυθεί το σύστημα

ab c 6

bc a 6

ca b 6

.

Υπόδειξη:

Αφαιρέστε κατά μέλη...

Page 318: Εισαγωγη σε διαγωνιστικα μαθηματικα για το γυμνασιο

http://www.mathematica.gr/forum/viewtopic.php?f=109&t=30104

Επιμέλεια: xr.tsif Σελίδα 318

Λύση:

Αφαιρώντας την πρώτη από την δεύτερη παίρνουμε (a c)(1 b) 0 .

Αν b 1

καταλήγουμε στο σύστημα a c 6 και ac 5 και λύνοντας την αντίστοιχη

δευτεροβάθμια παίρνουμε τις λύσεις (1,1,5) και (5,1,1) .

Αν a c

καταλήγουμε στο σύστημα a(b 1) 6 και 2a b 6 . Αυτό δίνει

3a 7a 6 0 . Παρατηρούμε ότι το a 1 είναι λύση οπότε μπορούμε να

παραγοντοποιήσουμε ως (a 1)(a 2)(a 3) 0 .

Άρα παίρνουμε επιπλέον τις λύσεις (1,5,1),(2,2,2) και ( 3, 3, 3) .

ΘΕΜΑ 370 (Socrates)

)

Να βρεθεί η τιμή της παράστασης 4 4 4 4

2 2 2 2

2014 4 2013 2012 4 2013

2013 4027 2013 4025

.

Λύση:

Θέτουμε a 2013 για ευκολία.

Έστω Aη δοσμένη παράσταση.

Έχουμε 22 2 2 2 2 2 24 4 4 4

2 2 2 2 2

(a 1) (2a ) 4a (a 1) 4a (a 1)(a 1) 4a (a 1) 4aA

a (2a 1) a (2a 1) 5a 4a 1

22 2 2 2 2 2 2

2

(a 1) (2a ) 4a (a 1) 4a (a 1)

5a 4a 1

2 22 22 2 2 2

2 2

(a 1) 2a 2a(a 1) (a 1) 2a 2a(a 1)

5a 4a 1 5a 4a 1

Page 319: Εισαγωγη σε διαγωνιστικα μαθηματικα για το γυμνασιο

http://www.mathematica.gr/forum/viewtopic.php?f=109&t=30104

Επιμέλεια: xr.tsif Σελίδα 319

2 2 2 2

2 2

2 2

(5a 4a 1)(a 1) (5a 4a 1)(a 1)a 1 a 1 2

5a 4a 1 5a 4a 1

.

ΘΕΜΑ 371 (Socrates)

) Βρείτε τους πρώτους p,q,r ώστε pq r 1 και

2 2 22(p q ) r 1 .

Λύση:

Παρατηρούμε ότι δεν είναι δυνατόν να είναι όλοι περιττοί. Διακρίνουμε τις

περιπτώσεις:

p 2 (ομοίως και για q 2 ).

Είναι: 2q r 1 και 2 2r 2q 7 .

Από την πρώτη εξίσωση έχουμε r 2q 1 . Επομένως, η δεύτερη εξίσωση

γίνεται:

2 2 2 2 2 2 2r 2q 7 (2q 1) 2q 7 4q 4q 1 2q 7 0 2q 4q 6 0 2q 2q 3 0 q 1 ή q 3 .

Η περίπτωση q 1 απορρίπτεται και άρα q 3 που μας δίνει τις λύσεις

(p,q,r) (2,3,5),(3,2,5) .

r 2 .

Είναι: pq 3 και 2 22p 2q 5 .

Λύνοντας την πρώτη ως προς p και αντικαθιστώντας στη δεύτερη θα

καταλήξουμε στη διτετράγωνη εξίσωση: 4 22q 5q 18 0 .

Η παραπάνω εξίσωση έχει 119 0 και άρα είναι αδύνατη.

Άρα τελικά: (p,q,r) (2,3,5),(3,2,5) .

Page 320: Εισαγωγη σε διαγωνιστικα μαθηματικα για το γυμνασιο

http://www.mathematica.gr/forum/viewtopic.php?f=109&t=30104

Επιμέλεια: xr.tsif Σελίδα 320

ΘΕΜΑ 372 (Socrates)

) Δίνονται πέντε θετικοί διαιρέτες του αριθμού 200810 . Δείξτε ότι το γινόμενο δύο

από αυτούς είναι τέλειο τετράγωνο.

Λύση:

Κάθε διαιρέτης είναι της μορφής k m2 5 . Το ζεύγος των εκθετών (k,m) μπορεί

να έχει μια από τις ακόλουθες μορφές:

α) άρτιος – άρτιος

β) άρτιος – περιττός

γ) περιττός – άρτιος

δ) περιττός – περιττός

Αν έχουμε πέντε τέτοιους διαιρέτες, τουλάχιστον δύο είναι της ίδιας μορφής

οπότε το γινόμενό τους θα είναι της μορφής k m2 5 με τους εκθέτες άρτιους, άρα

είναι τέλειο τετράγωνο.

Από εδώ: http://www.mathematica.gr/forum/viewtopic.php?f=111&t=39113

ΘΕΜΑ 373 (Socrates)

) Να βρεθεί η τιμή της παράστασης

1 2010 1 2011 1 2012 1 2013 2015 .

Λύση:

Είναι 2 21 a(a 2) a 2a 1 (a 1) , οπότε έχουμε

1 2010 1 2011 1 2012 1 2013 2015

21 2010 1 2011 1 2012 2014

Page 321: Εισαγωγη σε διαγωνιστικα μαθηματικα για το γυμνασιο

http://www.mathematica.gr/forum/viewtopic.php?f=109&t=30104

Επιμέλεια: xr.tsif Σελίδα 321

21 2010 1 2011 1 2012 2014 1 2010 1 2011 2013

21 2010 1 2011 2013 1 2010 2012 1 2010 2012

22011 2011 .

ΘΕΜΑ 374 (ΔΗΜΗΤΡΗΣ ΙΩΑΝΝΟΥ)

) Αν

1 1 1

z(x y) x(y z) xyz

όπου x,y,z 0 , να αποδείξετε ότι:

x y z . (Γ Γυμνασίου)

Λύση:

Έχοντας ως οδηγό τη γνωστή ανισότητα: 2 2a b 2ab αρκεί να αποδείξουμε

ότι:

xy yz1 x y z

(x y) (y z)

(για a x ,b y , xy 0 ).

Γενικότερα λόγω της προαναφερθείσας ανισότητας έχουμε: xy 1

x y 2

,

yz 1

y z 2

με τις ισότητες να ισχύουν αν x y και y z .

Προσθέτοντας κατά μέλη έχω: xy yz

1x y y z

με την ισότητα που

επιθυμούμε να ισχύει τελικά αν x y z .

ΘΕΜΑ 375 (Socrates)

)

Αν 2 2 2a b 2c , να βρεθεί η τιμή της παράστασης 2 2 2(a b 2c)(2a b c )

(a b)(a c)(b c)

.

Page 322: Εισαγωγη σε διαγωνιστικα μαθηματικα για το γυμνασιο

http://www.mathematica.gr/forum/viewtopic.php?f=109&t=30104

Επιμέλεια: xr.tsif Σελίδα 322

Λύση:

2 2 2 2 2 2(a b 2c)(2a b c ) (a b 2c)(4a 2b 2c )

(a b)(a c)(b c) 2(a b)(a c)(b c)

2

(a b 2c)(a b)(a b) (a b 2c)(a b) (a b 2c)(a b)3 3 3

2(a b)(a c)(b c) 2(a c)(b c) 2ab 2ac 2bc 2c

2 2 2

(a b 2c)(a b) (a b 2c)(a b) (a b 2c)(a b)3 3 3 3

2ab 2c(a b) a b (a b) 2c(a b) (a b)(a b 2c)

.

ΘΕΜΑ 376 (Socrates)

)

Να δείξετε ότι ανa b c τότε 3 3a c a b b c

abc3 c a

.

Λύση:

3 3

2 2a c a b b cabc (a c)(a ac c ) 3b[a(a b) c(b c)]

3 c a

2 2(a c)(a ac c ) 3b(a c)(a c b)

2 2(a c)(a ac c ) 3b(a c)(a c b) 0

2 2 2(a c)(a ac c 3ba 3bc 3b ) 0

2 2 2(a c)[c (a 3b)c a 3ab 3b ] 0 ,

που ισχύει διότι a c και για το τριώνυμο ως προς c είναι

2 2 2 2(a 3b) 4(a 3b 3ab) 3(a b) 0 .

ΘΕΜΑ 377 (ΘΑΝΟΣ ΜΑΓΚΟΣ)

) Να αποδείξετε ότι ο αριθμός 6465 64 είναι σύνθετος.

Page 323: Εισαγωγη σε διαγωνιστικα μαθηματικα για το γυμνασιο

http://www.mathematica.gr/forum/viewtopic.php?f=109&t=30104

Επιμέλεια: xr.tsif Σελίδα 323

Λύση:

64 32 2 2 32 2 32 32 2 2 3265 64 (65 ) 8 (65 8) 2 65 8 (65 8) 4 65

32 2 16 2 32 16 32 16(65 8) (4 65 ) (65 8 4 65 )(65 8 4 65 ) και άρα είναι

σύνθετος.

ΘΕΜΑ 378 (ΔΗΜΗΤΡΗΣ ΙΩΑΝΝΟΥ)

) Αν a,b,c 0 και αν 2 2 2a b c και 3 3 3a b c , να βρεθούν οι αριθμοί a,b,c.

(Β,Γ Γυμνασίου)

Λύση:

2 2 2a b c (1) και 3 3 3a b c (2) .

Έχουμε 2 2 3 3 3 2 6 4 2 2 4 6 6 3 3 6(a b ) (a b ) a 3a b 3a b b a 2a b b

2 2 2 2 3 3 23a b (a b ) 2a b (a 0ήb 0ή3c 2ab) .

Έστω ότι είναι a,b 0 , τότε θα είναι c 0 και 23c 2ab (3) .

Έχουμε όμως

(3)2 2 2 2 2 2 2 2a b c (a b) 2ab c (a b) 3c c a b 2c (4) ,

οπότε (2)

3 3 3 2 33(2) (a b) 3ab(a b) c (2c) 3 c 2c c c 0

2

,

άτοπο!!!

Άρα είναι a 0 ή b 0 .

Αν είναι a 0 ,τότε οι (1) και (2) δίνουν b c .

Ομοίως αν είναι b 0 ,τότε έχουμε a c .

Page 324: Εισαγωγη σε διαγωνιστικα μαθηματικα για το γυμνασιο

http://www.mathematica.gr/forum/viewtopic.php?f=109&t=30104

Επιμέλεια: xr.tsif Σελίδα 324

Από τα παραπάνω προκύπτει ότι

2 2 2

3 3 3

a b c a 0,b c 0

a b c ή

a,b,c 0 b 0,a c 0

.

Β τρόπος

3 3 3 2 2 2 2a b c c(a b ) a (a c) b (b c) 0 (1) .

όμως: c a ,c b , οπότε 2a (a c) 0 ,

2a (b c) 0 .

Άρα

a 0ήa c (a,b,c) (0,c,c)

(1) ή

b 0ήb c (a,b,c) (c,0,c)

.

ΘΕΜΑ 379 (ΔΗΜΗΤΡΗΣ ΙΩΑΝΝΟΥ)

)

Να αποδείξετε ότι το κλάσμα 22014! 2013

2013! 2012

είναι ανάγωγο. (Γ Γυμνασίου)

Λύση:

Έστω πως δεν είναι ανάγωγο. Τότε υπάρχει φυσικός n 1 ώστε να διαιρεί και

τους δύο όρους του κλάσματος.

Δηλαδή 2n 2014!| 2013 και n 2013!| 2012 .

Τότε: n 2013! 2012 n 2014(2013! 2012) n 2014! 2012 201| | | 4 .

Δηλαδή 2n 2014! (2013 1) (2013 1) n 2014! 2| | 013 1 .

Επομένως: 2 2n 2014! 2013 (2014! 2013 1) n| 1| . Άτοπο. Άρα το κλάσμα

είναι ανάγωγο.

Β τρόπος

Ας υποθέσουμε ότι δεν είναι ανάγωγο το κλάσμα.

Τότε, πρέπει να αποδείξουμε ότι:

Page 325: Εισαγωγη σε διαγωνιστικα μαθηματικα για το γυμνασιο

http://www.mathematica.gr/forum/viewtopic.php?f=109&t=30104

Επιμέλεια: xr.tsif Σελίδα 325

2 22013! 2012 2014! 2013 1 2 3 ... 2013 2012 1 2 3 ... 2013 2014 3| 20| 1 2012(1 2 3 ... 2011 2013 1) 2013(1 2 3 ... 2012 20| 14 2013)

2012(1 2 3 ... 2011 2013 1) | 2013k (*) .

Άρα, αναγκαστικά, πρέπει

|2012 2013 και (1 2 3 ... 2011 2013 1) | 2013k .

Πράγμα άτοπο, αφού (1 2 3 ... 2011 2013 1) 1(mod2013) , ενώ

2013k 0(mod2013) .

Άρα, το κλάσμα είναι ανάγωγο.

ΘΕΜΑ 380 (ΔΗΜΗΤΡΗΣ ΙΩΑΝΝΟΥ)

)

Αν 0 x y να αποδείξετε ότι: 2 1

1y x(y x)

. (Γ Γυμνασίου)

Λύση:

Είναι:

1 1 1 10 x y 0 x y y y x 0

y x y x(y x) xy

.

Επομένως αρκεί να δείξω ότι: 1 2

1xy y

.

Όμως: 2

1 1 x 1 21 2

xy xy y y y , αφού 0 x y .

ΘΕΜΑ 381 (Μπάμπης Στεργίου)

) Να αποδείξετε ότι :

2 2 4 4 1024 1024 2048 2048(9 8)(9 8 )(9 8 )...(9 8 ) 9 8 .

Page 326: Εισαγωγη σε διαγωνιστικα μαθηματικα για το γυμνασιο

http://www.mathematica.gr/forum/viewtopic.php?f=109&t=30104

Επιμέλεια: xr.tsif Σελίδα 326

Λύση:

Θα ξεκινήσουμε από το β' μέλος και θα παραγοντοποιήσουμε με διαφορά

τετραγώνων. Είναι:

2048 2048 1024 1024 1024 1024 512 512 512 512 1024 10249 8 (9 8 )(9 8 ) (9 8 )(9 8 )(9 8 ) ... 4 4 4 4 512 512 1024 1024(9 8 )(9 8 )...(9 8 )(9 8 ) 2 2 2 2 4 4 512 512 1024 1024(9 8 )(9 8 )(9 8 )...(9 8 )(9 8 )

2 2 4 4 512 512 1024 1024(9 8)(9 8 )(9 8 )...(9 8 )(9 8 ) .

Έτσι, αποδείξαμε το ζητούμενο.

Β τρόπος

Έχουμε λοιπόν:2 2 4 4 1024 1024 2048 2048A 1 A (9 8) A (9 8) (9 8)(9 8 )(9 8 )...(9 8 ) 9 8 .

ΘΕΜΑ 382 (ΔΗΜΗΤΡΗΣ ΙΩΑΝΝΟΥ)

) Αν x,y N , να αποδείξετε ότι ο αριθμός :

2014 2014 1007 1007 1007 1007x y 2x y 3x 3y 2 ,δεν είναι τέλειο τετράγωνο.

(Γ Γυμνασίου)

Λύση:

Θέτουμε 1007a x και 1007b y .Τότε η παράσταση γίνεται:

2 2 2a b 2ab 3a 3b 2 (a b) 3(a b) 2 (a b 3)(a b) 2 .

Τώρα, θέτουμε k a b .

Έτσι: 2(a b 3)(a b) 2 k(k 3) 2 k 3k 2 .

Είναι όμως:2 2 2(k 1) k 3k 2 (k 2) .

Άρα, η παράσταση δεν είναι ποτέ τέλειο τετράγωνο αφού βρίσκεται ανάμεσα

σε δύο διαδοχικά τέλεια τετράγωνα.

Page 327: Εισαγωγη σε διαγωνιστικα μαθηματικα για το γυμνασιο

http://www.mathematica.gr/forum/viewtopic.php?f=109&t=30104

Επιμέλεια: xr.tsif Σελίδα 327

ΘΕΜΑ 383 (ΔΗΜΗΤΡΗΣ ΙΩΑΝΝΟΥ)

) Αν *a,b N με a b να αποδείξετε ότι: b a (a,b) .

Λύση:

Αφού (a,b) | a τότε υπάρχει *x N ώστε a x (a,b) .

Αφού (a,b) | b τότε υπάρχει *y N ώστε b y (a,b) .

Τότε, a b y (a,b) x (a,b) y x y x 1 .

Επομένως, b a (a,b) y (a,b) x (a,b) (a,b) (y x 1) (a,b) 0 ,

όπως θέλαμε.

ΘΕΜΑ 384 (ΔΗΜΗΤΡΗΣ ΙΩΑΝΝΟΥ)

) Αν για κάθε a,b,c 0 ισχύει ότι:

n n n n 1 n 1 n 1a b c 3abc (a b c)(a b c ab ac bc) , να βρεθεί ο

φυσικός αριθμός n .

Λύση:

Αφού η σχέση ισχύει για κάθε αριθμό a,b,c 0 , προφανώς θα ισχύει και για

a b c .

Άρα, η σχέση γίνεται: n n n n 1 n 1 n 1a b c 3abc (a b c)(a b c ab ac bc)

n n n n 1 n 1 n 1 2 2 2a a a 3aaa (a a a)(a a a a a a )

n 3 n 1 2 n 3 n 1 2 n 3 n 33a 3a 3a(3a 3a ) a a 3a(a a ) a a 3a 3a

n 32a 2a n 3 .

Επομένως, η σχέση ισχύει για n 3 και παρατηρούμε, αν κάνουμε

αντικατάσταση, ότι είναι η ταυτότητα του Euler.

Page 328: Εισαγωγη σε διαγωνιστικα μαθηματικα για το γυμνασιο

http://www.mathematica.gr/forum/viewtopic.php?f=109&t=30104

Επιμέλεια: xr.tsif Σελίδα 328

ΘΕΜΑ 385 (ΔΗΜΗΤΡΗΣ ΙΩΑΝΝΟΥ)

)

Να αποδείξετε ότι: 2 6 12 20 30 5

3 5 7 9 11 2 . (Γ Γυμνασίου)

Λύση:

Ωραία εφαρμογή της ανισότητας αριθμητικού-γεωμετρικού μέσου.

Ισχύει:

1 2 2 12

2 3 2

.

2 3 6 16

2 5 2

.

3 7 12 112

2 7 2

.

4 5 20 120

2 9 2

.

5 6 30 130

2 11 2

.

Με πρόσθεση κατά μέλη παίρνουμε το ζητούμενο. Οι ισότητες δεν ισχύουν για

προφανείς λόγους.

ΘΕΜΑ 386 (ΔΗΜΗΤΡΗΣ ΙΩΑΝΝΟΥ)

)

Αν m,n N και m n 1 , και αν επί πλέον ισχύει ότι: mn 1

2m n

, να

αποδείξετε ότι ο αριθμόςmn 1

A 2m n

, είναι τέλειο τετράγωνο φυσικού

αριθμού. (Γ Γυμνασίου)

Page 329: Εισαγωγη σε διαγωνιστικα μαθηματικα για το γυμνασιο

http://www.mathematica.gr/forum/viewtopic.php?f=109&t=30104

Επιμέλεια: xr.tsif Σελίδα 329

Λύση:

Από τη δοθείσα σχέση λαμβάνουμε:

mn 12 mn 2m 2n 1 0 m(n 2) 2(n 2) 0 (m 2)(n 2) 3

m n

Άρα, αφού m,n N έχουμε ότι: m 2 1 και n 2 3 ή m 2 3 και

n 2 1 .

Μόνο το δεύτερο σύστημα μας δίνει λύση που ικανοποιεί τις συνθήκες τις

εκφώνησης, δηλ. m 5,n 3 .

Επομένως, η παράσταση γίνεται: 216A 2 16 4

2 .

Έτσι, το ζητούμενο αποδείχτηκε.

ΘΕΜΑ 387 (ΧΡΗΣΤΟΣ ΚΥΡΙΑΖΗΣ)

) Να συγκρίνετε τους αριθμούς: 8 8! , 9 9! , όπου n! 1 2 3 ... (n 1) n .

Λύση:

Με ύψωση στην 72αρκεί να συγκρίνουμε τους 9(8!) ,

8(9!) .

Είναι 8 8 8 8

9 8

(9!) (8!) 9 91

(8!) (8!) 8! 8!

.

ΘΕΜΑ 388 (ΔΗΜΗΤΡΗΣ ΙΩΑΝΝΟΥ)

) Να βρείτε το υπόλοιπο της διαίρεσης του αριθμού 20122002 με το 15.

(εισαγωγή στα modulo) (Γ Γυμνασίου)

Λύση:

Είναι: 42002 8(mod15) 2002 4096 1(mod15) .

Άρα: 20122002 1(mod15) . Έτσι, το ζητούμενο υπόλοιπο είναι 1.

Page 330: Εισαγωγη σε διαγωνιστικα μαθηματικα για το γυμνασιο

http://www.mathematica.gr/forum/viewtopic.php?f=109&t=30104

Επιμέλεια: xr.tsif Σελίδα 330

ΘΕΜΑ 389 (ΔΗΜΗΤΡΗΣ ΙΩΑΝΝΟΥ)

Έστω x (2 3)( 6 2) 2 3 .

α) Να αποδείξετε ότι ο αριθμός x είναι φυσικός.

β) Αν 2011 2010 2m x x ... x x 1 και 2012n x , να αποδείξετε ότι οι αριθμοί

m,n είναι διαδοχικοί φυσικοί

γ) Να αποδείξετε ότι ο αριθμός 1 mn , δεν είναι ρητός. (Γ Γυμνασίου)

Λύση:

Θα απαντήσω μόνο τα πρώτα δύο... Για το τρίτο έχω λύση με τη μέθοδο του

τελευταίου ψηφίου... Ψάχνω όμως μία απάντηση με ισοτιμίες αλλά δεν μπορώ

να βρω ποιο mod να χρησιμοποιήσω...

α) Είναι:

2 2 2 2x (2 3) ( 6 2) ( 2 3) (2 3)(2 3)(8 4 3)(2 3)

4(2 3)(2 3) 4 .

Παρατηρούμε όμως ότι: 2 3 0 ,

6 2 6 2 0 ,

2 3 0 .

Έτσι, ο αριθμός είναι θετικός και άρα x 2 .

β) Είναι:

2 2012 2 2011 2 3 2011m 1 2 2 ... 2 m 1 2 2 2 ... 2 2·2 2 2 ... 2

2 2 3 2011 2 3 2011 20122 2 2 ... 2 2·2 2 ... 2 ... 2 n .

γ) Τελικά, βρήκα μία διαφορετική λύση...

Page 331: Εισαγωγη σε διαγωνιστικα μαθηματικα για το γυμνασιο

http://www.mathematica.gr/forum/viewtopic.php?f=109&t=30104

Επιμέλεια: xr.tsif Σελίδα 331

Από το β) είναι n m 1 . Επομένως: 21 mn 1 m(m 1) m m 1 .

Όμως: 2 2 2m m m 1 (m 1) .

Άρα, δεν μπορεί ποτέ να είναι τέλειο τετράγωνο αφού βρίσκεται ανάμεσα σε

δύο διαδοχικά τέλεια τετράγωνα και άρα ο αριθμός είναι άρρητος.

Επαναφέρω όμως το ερώτημα μου για το πως θα μπορούσε να λυθεί με

ισοτιμίες...

Εν συντομία ...

2 2m m 1 n .

Α ΤΡΌΠΟΣ

Για m,nθετικούς .

Απλά θέσε n m k για θετικό k .

Β ΤΡΟΠΟΣ

(n m)(n m) m 1 , οπότε m n| m 1 .

Γ ΤΡΟΠΟΣ

2 2(2m 1) 3 (2n) .

Β τρόπος

α) x (2 3)( 6 2) 2 3 2(2 3)( 3 1) 2 3

22( 3 1) (2 3) (2 3) 2( 3 1) 2 3 ( 3 1) 4 2 3

2( 3 1) ( 3 1) ( 3 1)( 3 1) 3 1 2 .

β) 2011 2010 2011 2010 2011 2010m x x ... x 1 2 2 ... 2 1 (2 1)(2 2 ... 2 1)

Page 332: Εισαγωγη σε διαγωνιστικα μαθηματικα για το γυμνασιο

http://www.mathematica.gr/forum/viewtopic.php?f=109&t=30104

Επιμέλεια: xr.tsif Σελίδα 332

2012 20122 1 x 1 n 1 .

γ) Ας υποθέσουμε ότι ο αριθμός είναι ρητός. Άρα, το υπόρριζο πρέπει να είναι

τέλειο τετράγωνο.

Όμως, 2mn 1 n(n 1) 1 n n 1 .

Ωστόσο, 2 2 2 2(n 1) n 2n 1 n n 1 n , που ισχύει, αφού n N . Άρα,

δεν είναι τέλειο τετράγωνο (άτοπο) και έτσι θα είναι άρρητος αριθμός.

ΘΕΜΑ 390 (ΓΙΩΡΓΟΣ ΒΑΣΔΕΚΗΣ)

)

Αν x,y,z 0 και x y z 3xyz τότε 1 1 1

x y z 8x y z

.

Λύση:

Είναι 33xyz x y z 3 xyz xyz 27 .

Άρα 3x y z 3 xyz 9 .

Άρα αρκεί να αποδειχθεί ότι 1 1 1

1 xyz xy yz zxx y z .

Πράγματι, είναι 2(x y z)

xy yz zx xyz3

.

ΘΕΜΑ 391 (ΜΠΑΜΠΗΣ ΣΤΕΡΓΙΟΥ)

) Γράφουμε τον αριθμό:

2011

a 122223222422225222226....2011222...2

.

Ποιο ψηφίο βρίσκεται στη θέση 2005 και στη θέση 2014 ;

Ας σημειώσουμε ότι ο αριθμός είναι ο

έ 2, ύ 22, ί 222, έ 2222, έ 22222 και αρκεί να

γράψετε τους αριθμούς ένα , δύο , τρία , κλπ με ψηφία, κάτι που μαρτυράει το

τελευταίο ψηφίο του αριθμού , όπως αυτός δόθηκε στην εκφώνηση.

Page 333: Εισαγωγη σε διαγωνιστικα μαθηματικα για το γυμνασιο

http://www.mathematica.gr/forum/viewtopic.php?f=109&t=30104

Επιμέλεια: xr.tsif Σελίδα 333

Λύση:

Παρατηρούμε ότι:

Μέχρι τον αριθμό 1, υπάρχουν 1αριθμός.

Μέχρι τον 2 , υπάρχουν 3 1 2 αριθμοί.

Μέχρι τον 3, υπάρχουν 6 1 2 3 αριθμοί.

Μέχρι τον 4 , υπάρχουν 10 1 2 3 4 αριθμοί

Γενικά, μέχρι τον αριθμό n , υπάρχουν 1 2 3 ... n αριθμοί.

Εκτός από τους αριθμούς 1 2 3 ... n , που είναι στις 1,1 2 , 1 2 3 , … ,

1 2 3 ... n θέσεις αντιστοίχως, στις υπόλοιπες θέσεις, βρίσκονται δυάρια.

Άρα για να είναι ένας αριθμός που βρίσκεται στην kθέση διαφορετικός από το

2 , θα πρέπει να υπάρχει n , ώστε να είναι 1 2 3 ... n .

Εξετάζουμε λοιπόν αν ο αριθμός που βρίσκεται στην 2005 θέση, είναι δυάρι ή

κάποιος αριθμός n .

Θα πρέπει η εξίσωση 1 2 3 ... n 2005k , να έχει λύση (με n N )

Δηλαδή η εξίσωση: n(n 1)

20052

, να έχει λύση στο N . Δηλαδή η εξίσωση:

2n n 4010 0 , να έχει λύση στο N .

Η διακρίνουσα όμως του τριωνύμου, είναι 16041 , η οποία δεν είναι τέλειο

τετράγωνο και άρα η εξίσωση είναι αδύνατη. Άρα ο αριθμός που κατέχει την

2005 θέση, είναι το 2 .

Όμοια βρίσκουμε ότι και η εξίσωση n(n 1)

20142

, είναι αδύνατη στο N και

άρα και ο αριθμός που κατέχει την 2014 θέση, είναι και πάλι το 2 .

Page 334: Εισαγωγη σε διαγωνιστικα μαθηματικα για το γυμνασιο

http://www.mathematica.gr/forum/viewtopic.php?f=109&t=30104

Επιμέλεια: xr.tsif Σελίδα 334

ΘΕΜΑ 392 (ΔΗΜΗΤΡΗΣ ΙΩΑΝΝΟΥ)

)

Να βρεθούν οι τιμές του ακέραιου αριθμού x , ώστε ο αριθμός x 2010

x 1

, να

είναι ρητός. (Β,Γ Γυμνασίου)

Λύση:

Θέτουμε: 2

2

x 2010 a

x 1 b

με (a,b) 1 .

Είναι:

2 2

2 2 2 2 2 2 2 2

2 2

a 2010bb x 2010b a x a x(b a ) a 2010b x

b a

2

2 2

2011bx 1

b a

.

Θα πρέπει 2 2 2b a | 2011b .

Θέτουμε: 2 2 2(b ,b a ) d .

Τότε: 2d | b και 2 2d| b a .

Θα είναι όμως: 2 2 2 2d | b (b a ) d | a .

Τελικά 2d | a και 2d | b , δηλαδή 2 2(a ,b ) d .

Είναι όμως: 2 2(a,b) 1 (a ,b ) 1 . Άρα d|1 d 1 .

Επομένως, θα πρέπει 2 2b a | 2011 .

Προκύπτουν κάποια συστήματα (δεν τα γράφω λόγω έλλειψης χρόνου) από τα

οποία παίρνουμε τις τιμές: x 2010 και x 1012035 .

ΘΕΜΑ 393 (ΔΗΜΗΤΡΗΣ ΙΩΑΝΝΟΥ)

) Αν ο φυσικός αριθμός n N είναι πρώτος, τότε:

Page 335: Εισαγωγη σε διαγωνιστικα μαθηματικα για το γυμνασιο

http://www.mathematica.gr/forum/viewtopic.php?f=109&t=30104

Επιμέλεια: xr.tsif Σελίδα 335

α) Αν ο αριθμός 2012n 2012 είναι φυσικός, να τον υπολογίσετε.

β) Ο αριθμός 2012n 2012 λήγει σε 25, όταν ο αριθμός 2012n 2012 είναι

φυσικός. (Γ Γυμνασίου)

Λύση:

α) Πρέπει 2012 2012 2 2 2012n 2012 m n 2012 m n m 2012

1006 1006(m 2012 )(m 2012 ) .

Αφού m,n N και ο αριθμός n πρέπει να είναι πρώτος, έπεται ότι 1006 1006m 2012 1 m 2012 1 .

Επομένως, 1006 1006 1006 1006n m 2012 2012 1 2012 2·2012 1 .

β) Είναι: 2012 1006 2012 1006 2n 2012 1 2·2012 2012 (2012 1) .

Όμως: 1006 1006 2(2012 ) (2 ) (2 ) 4 .

Άρα: 1006 2 2((2012 1) ) ((4 1) ) 25 .

Έτσι, το ζητούμενο απεδείχθη.

ΘΕΜΑ 394 (ΔΗΜΗΤΡΗΣ ΙΩΑΝΝΟΥ)

) Αν x {0,1,2,...,7} και αν x,4 1,(x 2) 1 , να βρεθεί ο x .

(Οι αριθμοί μέσα στις τετραγωνικές ρίζες είναι δεκαδικοί περιοδικοί).

(Γ Γυμνασίου)

Λύση:

Προσωπικά, μου άρεσε πολύ αυτή η άσκηση και με έκανε να θυμηθώ την

ξεχασμένη έννοια του περιοδικού αριθμού και τη διαδικασία ρητοποίησης του...

Έστω y x,4 x,444444... . Τότε: 10y x4,4444444...

Page 336: Εισαγωγη σε διαγωνιστικα μαθηματικα για το γυμνασιο

http://www.mathematica.gr/forum/viewtopic.php?f=109&t=30104

Επιμέλεια: xr.tsif Σελίδα 336

Με αφαίρεση κατά μέλη λαμβάνουμε:

9x 49y 10x 4 x y

9

.

Εντελώς όμοια θα λάβουμε ότι: x 11

1,(x 2)9

.

Επομένως, η εξίσωση γίνεται:

9x 4 x 111 9x 4 x 11 3

9 9

9x 4 11 x 9 6 11 x 3 6 11 x 8x 16 2 23 11 x 4x 8 99 9x 16x 64x 64 16x 73x 35 0 .

Η παραπάνω εξίσωση έχει ρίζες τους αριθμούς 7

5,8

. Λόγω του περιορισμού

της εκφώνησης έχουμε τελικά ότι x 5 .

ΘΕΜΑ 395 (ΔΗΜΗΤΡΗΣ ΙΩΑΝΝΟΥ)

)

Αν x,y,z 0 , να αποδείξετε ότι: x

xy xz xy xz 3y 3z2

.

(Γ Γυμνασίου)

Λύση:

Από την ΑΜ – ΓΜ παίρνουμε:

x x4y 2 ·4y 2 xy

4 4 .

x x4z 2 ·4z 2 xz

4 4 .

x x(2y 2z) 2 ·(2y 2z) 2 xy xz

2 2 .

Page 337: Εισαγωγη σε διαγωνιστικα μαθηματικα για το γυμνασιο

http://www.mathematica.gr/forum/viewtopic.php?f=109&t=30104

Επιμέλεια: xr.tsif Σελίδα 337

Με πρόσθεση κατά μέλη παίρνουμε:

x x x4y 4z 2y 2z 2 xy 2 xz 2 xy xz

4 4 2

x 6y 6z 2 xy 2 xz 2 xy xz

xxy xz xy xz 3y 3z

2 .

Έτσι, το ζητούμενο απεδείχθη.

ΘΕΜΑ 396 (ΔΗΜΗΤΡΗΣ ΙΩΑΝΝΟΥ)

) Η διχοτόμος της ορθής γωνίας oA 90

ενός ορθογωνίου τριγώνου ABC τέμνει

τον περιγεγραμμένο κύκλο στο σημείο D . Αν K,Lείναι οι προβολές του D

πάνω στις ευθείες AC και AB αντιστοίχως, να αποδείξετε ότι:

AB AC DL DK . (Γ Γυμνασίου)

ΣΗΜΕΙΩΣΗ

(Ορθή) προβολή ενός σημείου πάνω σε μια ευθεία είναι το σημείο που η

κάθετη στην ευθεία, η οποία άγεται (διέρχεται) από το σημείο τέμνει (συναντά)

την ευθεία

με απλά λόγια είναι το σημείο τομής της δοσμένης ευθείας και της κάθετης (σε

αυτήν) ευθείας που περνά από το σημείο.

Λύση:

Επειδή το ABC είναι ορθογώνιο, η BC είναι διάμετρος του κύκλου.

Το AKDLείναι παραλληλόγραμμο.

Page 338: Εισαγωγη σε διαγωνιστικα μαθηματικα για το γυμνασιο

http://www.mathematica.gr/forum/viewtopic.php?f=109&t=30104

Επιμέλεια: xr.tsif Σελίδα 338

Όμως AC AD AD L D L και άρα

AL DL . Επομένως το AKDLείναι

τετράγωνο.

Φέρουμε τις BDκαι DC. Ισχύει DK DL

από το ADLK .

H ADείναι διχοτόμος το D είναι το μέσο

του τόξου BC . Άρα, αν Oτο κέντρο του

κύκλου, η DO είναι μεσοκάθετος της BC

και άρα DC BK .

Επομένως, τα ορθογώνια τρίγωνα BLDκαι

DCKείναι ίσα και άρα CK BL .

Άρα: DL DK AL AK AB BL AC CK AB AC .

Έτσι, το ζητούμενο εδείχθη.

ΘΕΜΑ 397 (ΔΗΜΗΤΡΗΣ ΙΩΑΝΝΟΥ)

) Αν x,y R και αν 3(x y) 28 8 3x 5 6 3y 2 , να αποδείξετε ότι:

2014(2x y 12) 1 .

Λύση:

3(x y) 28 8 3x 5 6 3y 2 3x 3y 8 3x 5 6 3y 2 28 0

(3x 5) 5 4·2 3x 5 16 16 (3y 2) 2 2·3 3y 2 9 9 28 0

2 2 2 2[( 3x 5) 4·2 3x 5 4 ] [( 3y 2) 2·3 3y 2 3 ] 28 28 0

2 2( 3x 5 4) ( 3y 2 3) 0 .

Πρέπει 2 2( 3x 5 4) 0 ( 3y 2 3) 0

2 2( 3x 5 4) 0 ( 3y 2 3) 0

Page 339: Εισαγωγη σε διαγωνιστικα μαθηματικα για το γυμνασιο

http://www.mathematica.gr/forum/viewtopic.php?f=109&t=30104

Επιμέλεια: xr.tsif Σελίδα 339

2 2( 3x 5 4) 0 ( 3y 2 3) 0

3x 5 4 0 3y 2 3 0 3x 5 4 3y 2 3

11 11 113x 5 16 3y 2 9 3x 11 3y 11 x y x y

3 3 3 .

Άρα, το πρώτο μέλος της σχέσης προς απόδειξη γίνεται: 2014 2014 2014(2x y 12) (2x x 12) (3x 12)

2014 2014 201411(3 12) (11 12) ( 1) 1

3 , που είναι ίσο με το δεύτερο μέλος

και έτσι αποδείξαμε το ζητούμενο.

ΘΕΜΑ 398 (ΔΗΜΗΤΡΗΣ ΙΩΑΝΝΟΥ)

) Έστω ότι *x R και 18 11x ,x Q . Να αποδείξετε ότι x Q . (B,Γ Γυμνασίου)

Λύση:

Θα χρησιμοποιήσουμε το γεγονός ότι το πηλίκο δύο ρητών αριθμών είναι

ρητός.

Προφανώς, 18

7

11

xx Q

x (1) .

Επίσης, 11

4

7

xx Q

x (2) .

(1) : (2) έχουμε 7

3

4

xx Q

x (3) .

(2) : (3)έχουμε 4

3

xx Q

x και άρα αποδείξαμε το ζητούμενο.

ΘΕΜΑ 399 (ΔΗΜΗΤΡΗΣ ΙΩΑΝΝΟΥ)

)

Page 340: Εισαγωγη σε διαγωνιστικα μαθηματικα για το γυμνασιο

http://www.mathematica.gr/forum/viewtopic.php?f=109&t=30104

Επιμέλεια: xr.tsif Σελίδα 340

Αν x,y,z 0 και αν x y z 3xyz να αποδείξετε ότι: 1 1 1

1x y z .

(Γ Γυμνασίου)

Λύση:

Ισχύει

2 1 1 13xyz (x y z) 3(xy yz zx) xyz xy yz zx 1

x y z .

ΘΕΜΑ 400 (ΔΗΜΗΤΡΗΣ ΙΩΑΝΝΟΥ)

) Αν x,y,z 0 και αν

8 27 1252 x 3 y 5 z 76

x y z , να βρεθούν οι

αριθμοί x,y,z . (Γ Γυμνασίου)

Λύση:

Θέτοντας x a, y b, z c , η εξίσωση γίνεται:

2 2 28 27 125 2a 8 3b 27 5c 1252a 3b 5z 76 8 18 50 0

a b c a b c

2 2 22(a 2) 3(b 3) 5(c 5)0

a b c

.

Άρα θα είναι:

2(a 2) 0 a 2 x 4 .

2(b 3) 0 b 3 y 9 .

2(c 5) 0 c 5 z 25 .